Download as pdf or txt
Download as pdf or txt
You are on page 1of 105

TOPNOTCH MEDICAL BOARD PREP PEDIATRICS MAIN DIGITAL HANDOUT BY DR. PUNONGBAYAN AND DR.

DE VERA
For inquiries visit www.topnotchboardprep.com.ph or https://www.facebook.com/topnotchmedicalboardprep/
This handout is only valid for the March 2021 PLE batch. This will be rendered obsolete for the next batch since we update our handouts regularly.

Important Legal Information NEONATOLOGY


The handouts, videos and other review materials, provided by Topnotch Medical Board ✔ GUIDE QUESTIONS
Preparation Incorporated are duly protected by RA 8293 otherwise known as the The significance of delayed cord clamping is to:
Intellectual Property Code of the Philippines, and shall only be for the sole use of the person:
a) whose name appear on the handout or review material, b) person subscribed to Topnotch
A. Prevent hypothermia
Medical Board Preparation Incorporated Program or c) is the recipient of this electronic B. Decrease the incidence of anemia
communication. No part of the handout, video or other review material may be reproduced, C. Remove the crawling reflex
shared, sold and distributed through any printed form, audio or video recording, electronic D. To allow good bacteria from the mother to penetrate the newly
medium or machine-readable form, in whole or in part without the written consent of
born
Topnotch Medical Board Preparation Incorporated. Any violation and or infringement,
whether intended or otherwise shall be subject to legal action and prosecution to the full The rationale of this intervention in the Essential Newborn Care is to
extent guaranteed by law. prevent neonatal hypothermia and to increase colonization with
protective family bacteria:
A. Immediate drying
DISCLOSURE B. Delayed cord clamping
The handouts/review materials must be treated with utmost confidentiality. It shall be the
responsibility of the person, whose name appears therein, that the handouts/review C. Uninterrupted skin-to-skin contact
materials are not photocopied or in any way reproduced, shared or lent to any person or D. Breastfeeding
disposed in any manner. Any handout/review material found in the possession of another
person whose name does not appear therein shall be prima facie evidence of violation of RA
8293. Topnotch review materials are updated every six (6) months based on the current ESSENTIAL NEWBORN CARE
trends and feedback. Please buy all recommended review books and other materials listed
below. • Series of time bound chronologically ordered, standard
THIS HANDOUT IS NOT FOR SALE! procedures that a baby receives at birth
• Immediate drying → prevents hypothermia
INSTRUCTIONS • Uninterrupted skin-to-skin contact → prevents hypothermia,
To scan QR codes on iPhone and iPad
1. Launch the Camera app on your IOS device
increases colonization with protective family bacteria and
2. Point it at the QR code you want to scan improves breastfeeding initiation and exclusivity.
3. Look for the notification banner at the top • Delayed cord clamping after 1 to 3 minutes → decreases
of the screen and tap
To scan QR codes on Android
anemia in 1 out of 3 premature babies and prevents brain
1. Install QR code reader from Play Store hemorrhage in 1 out of 2; prevents anemia in 1 out of 7 term
2. Launch QR code app on your device babies.
3. Point it at the QR code you want to scan • Non-separation of mother and baby. Breastfeeding within
4. Tap browse website
first hour of life prevents 19.1% of all neonatal deaths.
Please take note of the chronological order and importance of each step.
Approach to Topnotch Pediatrics
A common mistake of most students is to answer cord clamping as the 2nd
• Please have the following Topnotch materials at hand:
step. Remember this is the 3rd step. Think “delayed”
o (1)Topnotch Main Handout will serve as your main reference Dr. De Vera
material
✔ GUIDE QUESTIONS
• Please buy the following:
o Nelson Textbook of Pediatrics, 20th / 21st ed After catching a newborn full term male, and placing him in the
o Pedia Platinum incubator, you noticed that the air-conditioning system was set to full
and that the radiant warmer was not working. You are worried that the

:
baby might develop hypothermia which might lead to following except?
A. Acidosis
B. Hypoglycemia
INTRODUCTION TO PEDIATRICS C. Hypoxemia
MEDICAL BOARD EXAM D. Increased renal excretion of water and solutes
https://qrs.ly/lnbj83z E. NOTA
The target temperature for newborns is?
A. 36 - 37°C C. 36.5 - 37.5°C
B. 36 - 37.5°C D. 36.7 - 37°C
The following are the most common mechanisms of heat loss in the
PEDIATRICS immediate newborn period EXCEPT?
A. Convection
By Adrian Salvador M. De Vera, MD, DPPS B. Conduction
Ruby Ann L. Punongbayan, MD, FPPS, MA C. Heat Radiation
D. Evaporation
TOPIC PAGE E. NOTA
Neonatology 1 THERMOREGULATION
Nutrition 12 • Newborns are prone to heat loss and hypothermia
Preventive Pediatrics 18 • body surface area of a newborn infant is approximately 3 times
IMCI 22 that of an adult
Growth and Development 22 • Mechanisms
Gastroenterology 26 (1) Convection of heat energy to the cooler surrounding air
Nephrology 34 (2) Conduction of heat to the colder materials touching the
Hematology/Oncology 39 infant
Neurology 46 (3) Heat radiation from the infant to other nearby cooler
Pulmonology 58 objects
Rheumatology 67 (4) Evaporation from skin and lungs
Cardiology 74 • Optimal method for maintaining temperature in a stable
Infectious Diseases 84 neonate?
Endocrinology 94 o Skin-skin contact
Immunology/ Allergology 100 Here are some examples of heat loss for you to better remember them.
Convection: This is the reason why room temperature should be set at 25-
28°C.
This handout is only valid for the March 2021 PLE batch. This will
Conduction: Important to pre-heat radiant warmer.
be rendered obsolete for the next batch since we update our Radiation: Cold metal cabinets inside the operating room are sources of
handouts regularly. heat radiation.
Dr. De Vera

✔ GUIDE QUESTION
All of the ff. is an effect of early skin to skin contact in newborn care

±
except?
A. Bonding
B. Prevents hypothermia
C. Prevents anemia
D. Increases colonization with protective bacterial flora
TOPNOTCH MEDICAL BOARD PREP PEDIATRICS MAIN DIGITAL HANDOUT BY DR. PUNONGBAYAN AND DR. DE VERA Page 1 of 105
For inquiries visit www.topnotchboardprep.com.ph or https://www.facebook.com/topnotchmedicalboardprep/
This handout is only valid for the March 2021 PLE batch. This will be rendered obsolete for the next batch since we update our handouts regularly.
TOPNOTCH MEDICAL BOARD PREP PEDIATRICS MAIN DIGITAL HANDOUT BY DR. PUNONGBAYAN AND DR. DE VERA
For inquiries visit www.topnotchboardprep.com.ph or https://www.facebook.com/topnotchmedicalboardprep/
This handout is only valid for the March 2021 PLE batch. This will be rendered obsolete for the next batch since we update our handouts regularly.
Classification of Prematurity Based on Birth Weight and • At increased risk of:
Gestational Age: o respiratory distress syndrome
CLASSIFICATION METRIC o congenital cardiac defects
Birth weight o lumbosacral agenesis
• Low birth weight LBW <2,500 grams o Hyperbilirubinemia
• Very low birth weight VLBW <1,500 grams o birth injuries
• Extremely low birth weight Evan <1,000 grams
Gestational age
• Late preterm birth LPB Equal/>34 and <37 weeks IMMEDIATE NEWBORN CARE
• Very preterm birth VPB <32 weeks https://qrs.ly/19bj8nw
• Extremely preterm birth EPB <28 weeks See diagram on next page

Please do not get confused with SGA and low birth weight.
Low birth weight pertains to weight alone, regardless of gestational age. ✔ GUIDE QUESTION
SGA compares the birth weight to the gestational age using the All of the ff. are components of essential newborn care except?
Lubchenco chart. A. Early skin to skin contact
Dr. De Vera
B. Immediate thorough drying
SMALL FOR GESTATIONAL AGE C. Properly timed clamping and cutting of the cord
D. Timely vitamin k and eye prophylaxis
• also known as intra-uterine growth retardation (IUGR)
• BW is < 3rd percentile for calculated gestational age
SUCTIONING
• growth of the fetus affected by 000 fetal, uterine, placental, and
• Routine suctioning of nose and mouth has no proven benefit
0
maternal factors
• increased morbidity and mortality • No benefit of routine suction for babies born with meconium
stained amniotic fluid
IUGR • Oropharyngeal suctioning is not indicated for a healthy, active
ASYMMETRIC SYMMETRIC baby
• Associated with poor • Associated with genetic and
✔ GUIDE QUESTIONS
maternal nutrition or with metabolic conditions
A child was delivered preterm via emergency CS to a 37-year-old
late onset or exacerbation • Associated with diseases primigravida because of fetal distress. Upon delivery, he was noted to
of maternal vascular that seriously affect fetal be cyanotic on the extremities, HR 120/min, coughed when suctioned,
disease cell number with good tone and with regular respiration.
•0Weight affected > length • 0
Weight, height, HC equally What is the APGAR score at 1 minute?
• Head continues to grow affected Answer: APGAR 9
• Fetus affected in late • Fetus affected early in
gestation gestation <18 weeks The most important component of the APGAR score is the:
• Postnatal catch-up growth • Poor brain growth pre- and A. Heart rate
B. Activity
is good postnatally
C. Respiration
• High morbidity and D. Grimace
mortality The best description of the APGAR score is that it:
A. Accurately predicts who will develop cerebral palsy
LARGE FOR GESTATIONAL AGE B. Assesses neonates in need of resuscitation
C. Accurately predicts a low umbilical cord pH
• Birth weight of >90th percentile for gestational age
D. Accurately predicts neonates who will die in the neonatal period
• maternal diabetes & obesity are predisposing factors
• At risk for hypoglycemia, polycythemia

8 ⇐

EINC TIME-BOUND INTERVENTIONS


UNANG YAKAP-DOH EINC

TOPNOTCH MEDICAL BOARD PREP PEDIATRICS MAIN DIGITAL HANDOUT BY DR. PUNONGBAYAN AND DR. DE VERA Page 2 of 105
For inquiries visit www.topnotchboardprep.com.ph or https://www.facebook.com/topnotchmedicalboardprep/
This handout is only valid for the March 2021 PLE batch. This will be rendered obsolete for the next batch since we update our handouts regularly.
TOPNOTCH MEDICAL BOARD PREP PEDIATRICS MAIN DIGITAL HANDOUT BY DR. PUNONGBAYAN AND DR. DE VERA
For inquiries visit www.topnotchboardprep.com.ph or https://www.facebook.com/topnotchmedicalboardprep/
This handout is only valid for the March 2021 PLE batch. This will be rendered obsolete for the next batch since we update our handouts regularly.
IMMEDIATE NEWBORN CARE

NEWBORN REUSCITATION: FROM PHILIPPINE SOCIETY OF NEWBORN MEDICINE, NRPH+

TOPNOTCH MEDICAL BOARD PREP PEDIATRICS MAIN DIGITAL HANDOUT BY DR. PUNONGBAYAN AND DR. DE VERA Page 3 of 105
For inquiries visit www.topnotchboardprep.com.ph or https://www.facebook.com/topnotchmedicalboardprep/
This handout is only valid for the March 2021 PLE batch. This will be rendered obsolete for the next batch since we update our handouts regularly.
TOPNOTCH MEDICAL BOARD PREP PEDIATRICS MAIN DIGITAL HANDOUT BY DR. PUNONGBAYAN AND DR. DE VERA
For inquiries visit www.topnotchboardprep.com.ph or https://www.facebook.com/topnotchmedicalboardprep/
This handout is only valid for the March 2021 PLE batch. This will be rendered obsolete for the next batch since we update our handouts regularly.

APGAR CONGENITAL HYPOTHYROIDISM


• Normal birth weight & length
• Delayed physical, mental & sexual development
• Sluggish, feeding difficulties, hypothermia
• Edema of scrotum / genitals
• Prolonged physiologic jaundice
• Most common cause is Thyroid Dysgenesis

CONGENITAL ADRENAL HYPERPLASIA


• Autosomal recessive disorders of cortisol biosynthesis
• Deficiency in cortisol → ↑corticotropin (ACTH) → adrenocortical
hyperplasia and overproduction of intermediate metabolites
• Depending on pathway
o Mineralocorticoid deficiency or excess
o Incomplete virilization or precocious puberty in males
o Virilization or sexual infantilism in females
• Deficiency of 21-hydroxylase enzyme (90%): deficiency of
cortisol
• Normal at birth but signs of sexual & somatic precocity appear
within the 1st 6 months of life
• Vomiting, failure to thrive
http://www.ijrhs.org/article/2015/3/1-38 • Lab findings: Hyponatremia, hyperkalemia, hypoglycemia
This should be an easy point but believe me, even residents and (usually 10th-14th day of life)
consultants get this wrong. Please memorize this table. Remember when • Marked elevation of 17-hydroxyprogesterone
given the word “grimace” the score for that is 1. o Normally elevated in first 2-3 DOL and in sick pre-term infants
Dr. De Vera
• Treatment
APGAR o Glucocorticoid replacement
• rapidly assess the need to resuscitate § 15-20 mg/m2/24 hr of hydrocortisone daily administered
orally in 3 divided doses.
• some value in predicting neonatal mortality and cerebral palsy,
§ Stress dose (2-3x usual dose)
it has a poor positive predictive value.
§ Goal is to achieve normal growth and development.
• Most children with cerebral palsy have had normal APGAR
o Mineralocorticoid replacement for salt-wasting type
scores
§ Fludrocortisone
• Neonates with low APGAR scores do not universally get cerebral
o Surgical management
palsy.
o Prenatal treatment
§ Dexamethasone – suppresses androgens and prevents
Take note that PPV- TP/FP+TP (probability that patient with positive test
truly has disease. If the test is more specific, the greater the PPV.)
virilization if given by 6weeks AOG
Dr. De Vera
Take note that prenatal treatment of Dexamethasone is done for females
ADDITIONAL FACTS ABOUT APGAR SCORE only
Dr. De Vera
• It should NOT be used to determine the need for resuscitation
or to guide the steps of resuscitation CONGENITAL ADRENAL
• Changes in the score MAY reflect how well the infant is HYPERPLASIA
responding to resuscitation
https://qrs.ly/qcbj8d0
• Several factors may affect the APGAR score such as prematurity, Se diagram on next page
sedative drugs, congenital anomalies, and neuropathies.
• There is NO consistent data on the significance of APGAR score
on preterm infants GALACTOSEMIA
• It is NOT used to established asphyxia • 3 distinct enzyme deficiencies:
• It does NOT predict specific neurologic outcomes 1. Galactose-1-phosphate uridyltransferase deficiency
• A low 1-minute score does not correlate with outcome. (GALT) – classic form
However, the 5-minute score is a valid predictor of neonatal § Classic galactosemia – complete or near complete deficiency
mortality. § Partial Transferase Deficiency
2. Galactokinase deficiency (GALK)
NEWBORN SCREENING § presents with cataracts
• Congenital Hypothyroidism (CH) 3. Galactose-4-epimerase deficiency (GALE)
§ Benign form – healthy individuals. Enzyme deficiency limited
• Congenital Adrenal Hyperplasia
to leukocytes and erythrocytes
• Galactosemia
§ Severe form – similar to transferase deficiency
• Phenylketonuria
• Without the enzyme, unable to convert galactose to galactose-1-
• Glucose-6-Phosphate Dehydrogenase Deficiency
phosphate and uridine diphosphate galactose
• MSUD
• → accumulation & injury to parenchymal cells of the kidney,
liver, & brain (may begin in utero)
✔ GUIDE QUESTIONS
• Classic form may manifest within weeks after birth
On what day of life is NBS done?
• Injury to parenchymal cells of the kidneys, liver & brain
Answer: 24-48 hours of life • Feeding intolerance, vomiting, jaundice, convulsions, lethargy,
hypotonia, mental retardation
• Patients with galactosemia are at increased risk for E. coli
NEWBORN SCREENING TEST neonatal sepsis
• RA #9288 • Tx: Lactose-free milk
• Ideally done at 48 hours of life • Long term sequelae (even with treatment): ovarian failure,
• If blood was collected <24 hours old, repeat at 2 weeks old. developmental delay, learning disability
• For preterm: ideal time for NBS should be at 5-7 days old
• Can be done until 1 month old (for sick babies)
• Expanded NBS covers 28 diseases

TOPNOTCH MEDICAL BOARD PREP PEDIATRICS MAIN DIGITAL HANDOUT BY DR. PUNONGBAYAN AND DR. DE VERA Page 4 of 105
For inquiries visit www.topnotchboardprep.com.ph or https://www.facebook.com/topnotchmedicalboardprep/
This handout is only valid for the March 2021 PLE batch. This will be rendered obsolete for the next batch since we update our handouts regularly.
TOPNOTCH MEDICAL BOARD PREP PEDIATRICS MAIN DIGITAL HANDOUT BY DR. PUNONGBAYAN AND DR. DE VERA
For inquiries visit www.topnotchboardprep.com.ph or https://www.facebook.com/topnotchmedicalboardprep/
This handout is only valid for the March 2021 PLE batch. This will be rendered obsolete for the next batch since we update our handouts regularly.

Figure for Congenital Adrenal Hyperplasia


Trevor AJ, Katzung BG, Masters SB: Pharmacology Examination and Board Review 9th ed.

G6PD DEFICIENCY QUICK SHEET


• Episodic or chronic hemolytic anemia Galactosemia: CNS + kidney + liver
• Episodic: symptoms develop 1-2 days after exposure to a PKU: CNS + delayed presentation + musty odor
substance with oxidant properties: MSUD: CNS + more abrupt presentation + maple syrup odor
o sulfonamides, nalidixic acid, nitrofurantoin, chloramphenicol,
antimalarials, vitamin K analogs, ASA, benzene, naphthalene PHYSICAL FINDINGS DIAGNOSIS
Slate blue, well demarcated areas
PHENYLKETONURIA of pigmentation over the buttocks Mongolian Spots
• What is the enzyme that is deficient in phenylketonuria? and back
o Phenylalanine hydroxylase --- hyperphenylalaninemia Small, white occasionally
• What are the effects of excessive phenylalanine? vesiculopustular papules on an
Erythema toxicum
o Excess phenylalanine is transaminated to phenylpyruvic acid erythematous base develop after
or decarboxylated to phenylethylamine & disrupts normal 1-3 days. Contains eosinophils.
metabolism & causes brain damage Vesiculopustular eruption over a
• Affected infant is normal at birth dark macular base around the
Pustular melanosis
• Most common manifestation without treatment is chin, neck, back, and soles.
developmental delay Contains neutrophils.
• MR develop gradually Cysts appearing on the hard
• Infant: severe vomiting, hypertonic, hyperactive DTRs, seizures; palate which is composed of Epstein pearls
Older: hyperactive with purposeless movements, rhythmic accumulations of epithelial cells.
rocking, & athetosis Pearly white papules seen mostly
• unpleasant musty odor on the chins and around the Milia
cheeks
MAPLE SYRUP URINE DISEASE Open and closed comedones or
• Genetic metabolic disorder resulting from the defective activity inflammatory pustules and
of the enzyme branched chain alpha-keto acid papules on the cheeks of the baby Neonatal acne
dehydrogenase complex usually after a week from
• Accumulation of branched chain keto acid is toxic to the brain delivery
• Irritability, poor feeding, maple syrup odor, encephalopathy, Pink macular lesions on the nape,
central respiratory failure glabella, upper eyelids, or Nevus simplex
• Treatment: removal of sources of EAA of leucine, isoleucine, or nasolabial region
valine in the diet Hair that covers the skin of
✔ GUIDE QUESTIONS preterm infants. Seen on term Lanugo
The cheese-like material that covers the normal term infant in varying infants around the shoulders.
amounts: Thick, white creamy material
A. Lanugo Vernix caseosa
usually absent in post term infants
B. Vellus hair Mottling of the skin with venous
C. Vernix caseosa
prominence. Cobblestone, lacy Cutis marmorata
D. Milia
The purplish reticulated pattern noted on the skin of a neonate when
appearance
exposed to cold:
A. Mongolian spots
B. Acrocyanosis
C. Cutis marmorata
D. Harlequin color change
TOPNOTCH MEDICAL BOARD PREP PEDIATRICS MAIN DIGITAL HANDOUT BY DR. PUNONGBAYAN AND DR. DE VERA Page 5 of 105
For inquiries visit www.topnotchboardprep.com.ph or https://www.facebook.com/topnotchmedicalboardprep/
This handout is only valid for the March 2021 PLE batch. This will be rendered obsolete for the next batch since we update our handouts regularly.
TOPNOTCH MEDICAL BOARD PREP PEDIATRICS MAIN DIGITAL HANDOUT BY DR. PUNONGBAYAN AND DR. DE VERA
For inquiries visit www.topnotchboardprep.com.ph or https://www.facebook.com/topnotchmedicalboardprep/
This handout is only valid for the March 2021 PLE batch. This will be rendered obsolete for the next batch since we update our handouts regularly.
PHYSICAL ✔ GUIDE QUESTION
DIAGNOSIS INTERVENTION
EXAMINATION A 12 hour-old newborn born NSD to a G1P1 28-year-old mother was
• Abnormal noted to have frothing at the mouth of secretions. When he was given
milk, he regurgitated it. PE: crying, CR 155/min, RR=68/min, harsh
crepitus
breath sounds, good cardiac tone, soft abdomen, cyanotic lips and
palpated nailbeds. What is your impression?
around the A. GER
Clavicular
clavicle Immobilization B. GERD
fracture
• Clavicle not C. EA with TEF
clearly D. Diaphragmatic Hernia
delineated in
ABSENT the skin ESOPHAGEAL ATRESIA
MORO • Arm abducted
REFLEX • Pronated
• Internally
rotated
• History of Brachial
Observe
excessive plexopathy
traction on
the head
• Intact hand
grasp reflex

http://misc.medscape.com/

TEF MANIFESTATIONS:
• Frothing & bubbling at the mouth & nose, cough, cyanosis,
respiratory distress
• Feeding exacerbates the symptoms, causes regurgitation &
precipitate aspiration
• Diagnosis: inability to pass an NGT or OGT in the newborn is
suggestive

MANAGEMENT OF TEF
• Management: maintain a patent airway
• Prone position
• Esophageal suctioning
• Surgical ligation of TEF & primary end-to-end anastomosis of
the esophagus
CAPUT SUCCEDANEUM
vs. CEPHALHEMATOMA Type C (esophagus ends in blind pouch + distal TEF) is the most common.
vs. SUBGALEAL HEMORRHAGE If someone tells you that it is type A, you have to read more books J
Dr. De Vera
https://qrs.ly/xnbj8cw
✔ GUIDE QUESTIONS
✔ GUIDE QUESTIONS
A newborn infant develops tachypnea, grunting, and use of accessory
A. Cephalohematoma
muscles together with cyanosis at the 2nd hour of life. He also has a
B. Caput succedaneum
scaphoid abdomen, (+) decreased breath sounds bilaterally, the point
Caput
____________1. Cross the midline and suture lines of maximal impulse is shifted near the sternal side, what is the most
cepnal Subperiosteal hemorrhage
____________2. likely diagnosis?
____________3.
cepnal Limited to the surface of one cranial bone A. Bochdalek
____________4.
cephal No discoloration of overlying scalp B. Morgagni
____________5. Edema, ecchymosis, and swelling apparent right after
caput birth
C. Congenital cystic adenomatous malformation
D. Pulmonary Sequestration
____________6. May cause jaundice A term infant with severe respiratory disease. Scaphoid abdomen on
Answers: B A A A B A PE.
Diagnosis: Congenital Diaphragmatic Hernia
Most common type: Bochdalek hernia
SURGICAL CONDITIONS IN NEWBORN
• Esophageal atresia with TEF
• Congenital diaphragmatic hernia
• Abdominal wall defects
• Intestinal obstruction
• Necrotizing Enterocolitis (NEC)

TOPNOTCH MEDICAL BOARD PREP PEDIATRICS MAIN DIGITAL HANDOUT BY DR. PUNONGBAYAN AND DR. DE VERA Page 6 of 105
For inquiries visit www.topnotchboardprep.com.ph or https://www.facebook.com/topnotchmedicalboardprep/
This handout is only valid for the March 2021 PLE batch. This will be rendered obsolete for the next batch since we update our handouts regularly.
TOPNOTCH MEDICAL BOARD PREP PEDIATRICS MAIN DIGITAL HANDOUT BY DR. PUNONGBAYAN AND DR. DE VERA
For inquiries visit www.topnotchboardprep.com.ph or https://www.facebook.com/topnotchmedicalboardprep/
This handout is only valid for the March 2021 PLE batch. This will be rendered obsolete for the next batch since we update our handouts regularly.
A term infant is born with extrusion of the abdominal viscera In questions like this, always stay calm. There are several ways of
Diagnosis? answering this. First would be to memorize the diagnostic
Omphalocele versus Gastroschisis finding. If you know your anatomy from the ultrasound you will
• What is your immediate intervention? know that the answer is malrotation, hence the management is
o Decompression and wrapping detorsion. If you are not able to memorize the ultrasound finding,
• Which one is associated with other congenital anomalies? the examiner is kind enough to give you another test which is the
o Omphalocele (OEIS complex) upper GI series. From UGIS findings, one can deduce that this is
malrotation (cecum did not migrate to the RLQ).
• Gastroschisis is more common on left side or right side of umbilicus? Dr. De Vera
o Right A 28-year-old primigravida went into labor at 33 weeks age of
gestation. The obstetrician was concerned regarding the fetal lung
OMPHALOCELE VS GASTROSCHISIS maturity and requested for amniotic fluid L/S ratio. Which should be?
OMPHALOCELE GASTROSCHISIS A. >2 C. >4
B. >3 D. >5
SAC + -
Below the Lateral to the
LOCATION
umbilicus umbilicus, Right RESPIRATORY CONDITIONS
UMBILICAL Center of SURFACTANT
Left of the defect
CORD membrane • Surfactant is present in high concentrations in fetal lung
ASSOCIATED homogenates by 20 wk of gestation
60% 10%
DEFECTS • It appears in amniotic fluid between 28 and 32 wk of gestation
BOWEL Normal Inflamed • Mature levels of pulmonary surfactant are present usually after
ALIMENTATION Normal Delayed 35 wk of gestation
Surgical, TPN, Surgical, TPN,
MANAGEMENT
Hydration Hydration

✔ GUIDE QUESTIONS SURFACTANT


A preterm baby won’t stop crying. He then developed abdominal https://qrs.ly/2sbj8ap
distention with abdominal erythema. The baby cries more when
touched. What is your diagnosis?
NEC
• Coagulation necrosis – histologic finding
• Thickened bowel walls and air in the bowel wall:
o PNEUMATOSIS INTESTINALIS
What will be your intervention?
Supportive
Two days later, the baby developed pneumoperitoneum. What will be
your intervention?
Surgery

COMPOSITION OF SURFACTANT RECOVERED BY ALVEOLAR


WASH
Nelson Textbook of Pediatrics, 20th ed.
All of the ff. are accepted interventions for Necrotizing enterocolitis
except? ✔ GUIDE QUESTION
A. Broad Spectrum Antibiotic Therapy After catching a full-term baby boy, 2600g, AGA, on the 5th minute of life
B. Umbilical Catheterization you decided to check the oxygen saturation of the patient. A maximum
C. IV Volume Replacement saturation of 90% was read. The baby has good APGAR score, is
D. Surgery comfortable, and there are no other abnormal physical findings. What
is the next step in managing this patient?
The classic story of NEC is a preterm neonate with a stormy A. Provide O2 support C. Observe and monitor
clinical course given formula milk. The neonate then deteriorates B. Stimulate the patient D. Suction secretions
with signs of bowel compromise. This is in line with the triad of
the pathophysiology of NEC:
MINUTE(S) OF LIFE TARGET PRE DUCTAL O2 SATS
1. Intestinal ischemia (GI tract of toxic neonates are ischemic)
2. Enteral nutrition (serves as substrate for organism) 1 min 60-65%
3. Pathologic organisms (colonic bacteria like Clostridium, E. 2 min 65-70%
coli, Klebsiella)
Dr. De Vera 3 min 70-75%
A 1-day old male neonate is being referred for vomiting. The vomitus
4 min 75-80%
is greenish yellowish in color according to the mother. PE revealed a
non-distended, non-tender abdomen. What is the most likely finding on 5min 80-85%
workup? 10min 85-95%
A. Double Bubble Sign
B. Increased pyloric thickness ✔ GUIDE QUESTION
C. Donut Ring Sign A preterm 34-week neonate was referred by the nurse due to cessation
D. Absence of parasympathetic postganglionic cell bodies of breathing lasting 10 seconds followed by rapid respiration of 50-
A 1-week old neonate presents with vomiting characterized as bilious, 60bpms. Vital signs are stable, no cyanosis. If another episode occurs
last passage of stool was 2 days ago. Ultrasound shows that the what will you advise the nurse?
superior mesenteric vein is to the left of the artery, upper GI series A. Stimulate patient
shows absence of the cecum at the RLQ, what is the best next step in the B. Positive Pressure Ventilation
management of this patient? C. CPR
A. Detorsion C. Watchful Waiting D. NOTA
B. Barium Enema D. PEG Insertion

TOPNOTCH MEDICAL BOARD PREP PEDIATRICS MAIN DIGITAL HANDOUT BY DR. PUNONGBAYAN AND DR. DE VERA Page 7 of 105
For inquiries visit www.topnotchboardprep.com.ph or https://www.facebook.com/topnotchmedicalboardprep/
This handout is only valid for the March 2021 PLE batch. This will be rendered obsolete for the next batch since we update our handouts regularly.
TOPNOTCH MEDICAL BOARD PREP PEDIATRICS MAIN DIGITAL HANDOUT BY DR. PUNONGBAYAN AND DR. DE VERA
For inquiries visit www.topnotchboardprep.com.ph or https://www.facebook.com/topnotchmedicalboardprep/
This handout is only valid for the March 2021 PLE batch. This will be rendered obsolete for the next batch since we update our handouts regularly.

Nelson Textbook of Pediatrics, 20th ed.

This image simply shows the huge gamut of possible differential diagnoses for a neonate that presents with respiratory distress. Remember that the problem may
not be limited to the lungs (e.g. anemia can present with respiratory distress).
Dr. De Vera

APNEA A 23 y/o primigravid gave birth at 38wk AOG via CS to male newborn.
Several minutes after birth, the newborn developed tachypnea,
• defined as cessation of breathing for longer than 20 seconds or retractions, expiratory grunting. Upon auscultation, the lungs are clear
for any duration if accompanied by cyanosis and bradycardia without crackles or wheeze. Which of the following is the initial
• More common in pre-term infants diagnosis?
• Causes A. Respiratory distress syndrome
o MCC is idiopathic apnea of prematurity B. Transient tachypnea of the newborn
o direct depression of the central nervous system’s control of C. Bronchopulmonary dysplasia
D. PPHN
respiration (hypoglycemia, meningitis, drugs, hemorrhage,
What is the primary cause of the above case?
seizures) A. Slow absorption of fetal lung fluid
o disturbances in oxygen delivery (shock, sepsis, anemia) B. Surfactant deficiency
o ventilation defects (obstruction of the airway, pneumonia, C. Persistence of the fetal circulatory pattern of right-to-left
muscle weakness). shunting through the PDA and foramen ovale after birth
• Apnea Management D. AOTA
o immediate management What is the expected chest x-ray finding of the above case?
§ Stimulation + O2 for 30 seconds, if it does not work → A. Fine reticular granularity of the parenchyma and air
bronchograms
§ PPV for 30 seconds, if it does not work →
B. Prominent pulmonary vascular markings, fluid in the
§ Intubate intralobar fissures, overaeration, flat diaphragms
§ CPR anytime if heart rate falls <60bpm C. Normal findings
o Identify and correct causes D. Patchy infiltrates, coarse streaking of both lung fields, increased
o Methylxanthines (Caffeine or theophylline) anteroposterior diameter, and flattening of the diaphragm
§ increase central respiratory drive by lowering the threshold E. NOTA
of response to hypercapnia A 32 y/o G4P3 gave birth to a 42wk male neonate via SVD after 18
§ enhances contractility of the diaphragm and preventing hours of labor. The neonate was noted to have aspirated meconium.
diaphragmatic fatigue Twelve hours after birth, the neonate was noted to have grunting, nasal
flaring, and intercostal retractions. He was also tachycardic and was
hypoxemic at 80% O2 sats. After drawing blood from the right radial
✔ GUIDE QUESTIONS artery and umbilical artery, a PaO2 gradient was noted. Which of the
A 23 year-old primigravid gave birth at 33 week AOG to male newborn following is the initial diagnosis?
via SVD. Several minutes after birth, the newborn developed A. Respiratory distress syndrome
tachypnea, prominent grunting, intercostal and subcostal retractions, B. Transient tachypnea of the newborn
nasal flaring which progressed to cyanosis. Upon auscultation, breath C. Bronchopulmonary dysplasia
sounds were diminished with a harsh tubular quality and on deep D. PPHN
inspiration, fine crackles were heard. Which of the following is your What is the primary cause of the above case?
initial diagnosis? A. Surfactant deficiency
A. Respiratory distress syndrome B. Slow absorption of fetal lung fluid
B. Transient tachypnea of the newborn C. Persistence of the fetal circulatory pattern of right-to-left
C. Bronchopulmonary dysplasia shunting through the PDA and foramen ovale after birth
D. PPHN D. AOTA
What is the primary cause of the above case? What is the expected chest x-ray finding of the above case?
A. Persistence of the fetal circulatory pattern of right-to-left A. Fine reticular granularity of the parenchyma and air
shunting through the PDA and foramen ovale after birth bronchograms
B. Slow absorption of fetal lung fluid B. Prominent pulmonary vascular markings, fluid in the intralobar
C. Surfactant deficiency fissures, overaeration, flat diaphragms
D. AOTA C. Normal findings
What is the expected chest x-ray finding of the above case? D. Patchy infiltrates, coarse streaking of both lung fields, increased
A. Fine reticular granularity of the parenchyma and air anteroposterior diameter, and flattening of the diaphragm
bronchograms Which of the following is very useful in evaluating a patient with the
B. Prominent pulmonary vascular markings, fluid in the intralobar above case?
fissures, overaeration, flat diaphragms A. Chest x-ray
C. Normal findings B. Chest CT with IV contrast
D. Patchy infiltrates, coarse streaking of both lung fields, increased C. 12 lead ECG
anteroposterior diameter, and flattening of the diaphragm D. Real-time echocardiography with Doppler flow
E. NOTA E. AOTA
TOPNOTCH MEDICAL BOARD PREP PEDIATRICS MAIN DIGITAL HANDOUT BY DR. PUNONGBAYAN AND DR. DE VERA Page 8 of 105
For inquiries visit www.topnotchboardprep.com.ph or https://www.facebook.com/topnotchmedicalboardprep/
This handout is only valid for the March 2021 PLE batch. This will be rendered obsolete for the next batch since we update our handouts regularly.
TOPNOTCH MEDICAL BOARD PREP PEDIATRICS MAIN DIGITAL HANDOUT BY DR. PUNONGBAYAN AND DR. DE VERA
For inquiries visit www.topnotchboardprep.com.ph or https://www.facebook.com/topnotchmedicalboardprep/
This handout is only valid for the March 2021 PLE batch. This will be rendered obsolete for the next batch since we update our handouts regularly.
A 30-weeker premature baby was treated for severe RDS. The patient The following are acceptable management for this patient except?
was intubated on the first day of life, given surfactant therapy, and was A. Fluid restriction
on assisted ventilation for two weeks due to complications. Patient was B. Nutritional Support
subsequently extubated; however, oxygen support was never C. Treatment of Infection
completely weaned off. At 36 weeks, he could tolerate oxygen support D. NOTA
at 1 lpm via nasal cannula. What is the pathophysiology of this disease
condition? CONDITION CXR FINDINGS
A. Chronic lung injury due to hypoxia
Hyaline Membrane Finely granular lungs
B. Generation of free radicals from supplemental oxygenation
C. Inflammation Disease Ground glass appearance
D. All of the above Prominent pulmonary vascular
What is the classification of the case above? TTN markings
A. Mild Fluid lines in fissure
B. Moderate BPD “bubbly lungs” (cystic lucencies)
C. Severe Meconium Aspiration Coarse streaking granular pattern
D. NOTA
Syndrome of both lungs fields
What is the expected chest x-ray finding in the case above?
A. interstitial emphysema, wandering atelectasis with con- Neonatal Pneumonia Perihilar streaking
comitant hyperinflation, and cyst formation
B. Ground glass opacity with underinflation
C. Increased pulmonary vascular marking
D. All of the above

Nelson Textbook of Pediatrics, 20th ed.


GROUND GLASS OPACITY Pathogenesis
• Maladaptation from acute injury
• the result of increased pulmonary artery medial muscle
thickness and extension of smooth muscle layers into the usually
non-muscular, more peripheral pulmonary arterioles in
response to chronic fetal hypoxia
• a consequence of pulmonary hypoplasia (diaphragmatic hernia,
Potter syndrome)
• obstructive (ex TAPVR, Polycythemia)

Other Important Facts About PPHN


• PaO2 or oxygen saturation gradient between a preductal (right
radial artery) and a post-ductal (umbilical artery) site of blood
sampling suggests right-to-left shunting through the ductus
arteriosus.
• 2D echo with Doppler is helpful

PERSISTENT PULMONARY HYPERTENSION


• Persistence of the fetal circulatory pattern of right-to-left
shunting through the PDA and foramen ovale after birth is a Take a look at the flow of blood through a PDA. PDA is generally
result of excessively high PVR acyanotic because the flow is left to right. However, in certain instances
where PDA is large and/or the pulmonary vascular resistance is high
• PVR normally declines rapidly as a consequence of vasodilation
(such as in PPHN), shunt becomes right to left and patient therefore
secondary to lung inflation, a rise in postnatal PaO2, a reduction becomes CYANOTIC!
in PaCO2, increased pH, and release of vasoactive substances Dr. De Vera

TOPNOTCH MEDICAL BOARD PREP PEDIATRICS MAIN DIGITAL HANDOUT BY DR. PUNONGBAYAN AND DR. DE VERA Page 9 of 105
For inquiries visit www.topnotchboardprep.com.ph or https://www.facebook.com/topnotchmedicalboardprep/
This handout is only valid for the March 2021 PLE batch. This will be rendered obsolete for the next batch since we update our handouts regularly.
TOPNOTCH MEDICAL BOARD PREP PEDIATRICS MAIN DIGITAL HANDOUT BY DR. PUNONGBAYAN AND DR. DE VERA
For inquiries visit www.topnotchboardprep.com.ph or https://www.facebook.com/topnotchmedicalboardprep/
This handout is only valid for the March 2021 PLE batch. This will be rendered obsolete for the next batch since we update our handouts regularly.
RDS TTN PPHN
DEMOGRAPHIC • Pre-Term • Pre-term or term, CS • Term or post term, MAS
• Ground Glass opacities, Under- • Hyper-aerated, Prominent • May be normal OR depending on
X-RAY
aerated, atelectasis Vascular markings co-morbid condition
• Early onset but relieved with • Within first 12 hours of birth.
ONSET OF SIGNS AND • Within minutes of birth
minimal oxygen • Cyanosis
SYMPTOMS • Grunting
supplementation • Oxygen gradient
• Progressive worsening of
cyanosis and dyspnea
NATURAL COURSE • Recovers rapidly within 3 days • Unpredictable course
• Symptoms peak within 3 days
then improves
• Prevent with antenatal steroids
• Supportive
TREATMENT • Surfactant replacement • Supplemental O2
• Treat underlying cause
• PEEP

In relation to the case above, which among the ff. will most likely be
seen in this patient?
LUNG PATHOLOGY A. Coombs Test Positive
IN NEONATES B. Triangular Cord Sign
https://qrs.ly/tobj8ar C. (+) blood culture growth of gram-positive cocci
D. Intracranial Calcifications
The most serious complication of hyperbilirubinemia in the newborn
✔ GUIDE QUESTIONS is:
A newborn infant born at 38 weeks AOG developed jaundice at the 12th A. Severe anemia
hour of life. Within the next 8 hours, the jaundice spread up to the mid B. Heart failure
abdomen. Careful examination revealed some hepatosplenomegaly, C. Respiratory distress
chorioretinitis, and mild hydrocephalus. Based on the given data, what D. Encephalopathy
is the estimated level of bilirubin in this neonate? Jaundice appearing between the second and third day after birth in full-
A. 5mg/dL terms infants is likely due to:
B. 10mg/dL A. Normal changes
C. 15mg/dL B. Acute hemolysis
D. 20mg/dL C. Sepsis neonatorum
When you have newborns with chorioretinitis and other features D. Erythroblastosis fetalis
like hydrocephalus and organomegaly, TORCHes should always The most common cause of jaundice in neonates is:
be considered. A. Physiologic
Dr. De Vera B. Acute hemolysis
In relation to the above case, if a blood test was performed and is C. Sepsis neonatorum
consistent with the bilirubin levels estimated against the physical D. Erythroblastosis fetalis
exam, what is this child’s risk classification based on the Bhutani chart?
A. High Risk NEONATAL JAUNDICE
B. High Intermediate Risk
C. Low Intermediate Risk JAUNDICE RISK FACTORS IN NEONATAL
D. Low Risk HYPERBILIRUBINEMIA
• Jaundice visible on the 1st day of life
• A sibling with neonatal jaundice or anemia
• Unrecognized hemolysis (ABO, Rh, other blood group,
incompatibility); UDP-glucuronyl transferase deficiency
(Crigler-Najjar, Gilbert disease)
• Non-optimal feeding (formula or breast-feeding)
• Deficiency of glucose-6-phosphate dehydrogenase
• Infection (viral, bacterial). Infant of diabetic mother. Immaturity
(prematurity)
• Cephalohematoma or bruising. Central hematocrit >65%
(polycythemia)
• East Asian, Mediterranean, Native American heritage

PHYSIOLOGIC PATHOLOGIC
Jaundice visible only Appears on the
on the 2nd-3rd day first 24-36 hours of life
Serum bilirubin is
Peaks at 5-6 mg/dL
rising at a rate faster than
on the 2nd-4th day
5 mg/dL/24 hours
Decrease to below 2 mg/dL Serum bilirubin >12 mg/dL
between 5-7 days of life or 10-14 mg/dL in preterm
TB increases not TB increases
> 5mg/dL/day > 0.5 mg/dL/hour
Nelson Textbook of Pediatrics, 20th ed.
Decline to adult levels by Jaundice persist after
In relation to the above case, what is the best step in the management
of the child’s hyperbilirubinemia? 10-14 days of life 10-14 days
A. Evaluate for phototherapy Direct reacting bilirubin is
B. Administer phenobarbital >2 mg/dL at any time
C. Follow up within 2 days
D. Exchange transfusion Physiologic jaundice is a DIAGNOSIS OF EXCLUSION. JUST ONE FEATURE
In relation to the above case, which diagnostic modality can help of pathologic makes it pathologic. Students often fail to remember this.
confirm the diagnosis? Dr. De Vera
A. CT Scan
B. Blood Culture
C. Indirect Coomb Test
D. Newborn Screening

TOPNOTCH MEDICAL BOARD PREP PEDIATRICS MAIN DIGITAL HANDOUT BY DR. PUNONGBAYAN AND DR. DE VERA Page 10 of 105
For inquiries visit www.topnotchboardprep.com.ph or https://www.facebook.com/topnotchmedicalboardprep/
This handout is only valid for the March 2021 PLE batch. This will be rendered obsolete for the next batch since we update our handouts regularly.
TOPNOTCH MEDICAL BOARD PREP PEDIATRICS MAIN DIGITAL HANDOUT BY DR. PUNONGBAYAN AND DR. DE VERA
For inquiries visit www.topnotchboardprep.com.ph or https://www.facebook.com/topnotchmedicalboardprep/
This handout is only valid for the March 2021 PLE batch. This will be rendered obsolete for the next batch since we update our handouts regularly.
JAUNDICE WITHIN 24 H 2. Extrahepatic Cholestasis
KEY CLUE MOST LIKELY ETIOLOGY o choledochal cyst
First born child ABO-incompatibility o biliary atresia
Second born child Rh-incompatibility § paucity of bile ducts
History of prolonged second
stage of labor Sepsis Neonatorum COOMBS’ TEST
No prenatal check up • Direct Coombs test – used to detect antibodies that are bound
History of maternal infection to the surface of RBCs
TORCH infection
during pregnancy • Indirect Coombs test – detects antibodies against RBCs that are
JAUNDICE AFTER 24 H present unbound to the patient’s serum
ONSET OF MOST LIKELY • Clinical uses:
KEY CLUE 1. Blood transfusion preparation
JAUNDICE ETIOLOGY
Baby otherwise 2. Antenatal antibody screening
2-3 days Physiologic
normal
Mother supplements INDIRECT HYPERBILIRUBINEMIA
3-4 days feeding with sugar Breastfeeding COOMBS’ TEST POSITIVE = ISOIMMUNIZATION
water • Rh / ABO incompatibility (before treatment)
Baby is purely
>1 week Breast Milk
breastfed ABO INCOMPATIBILITY
COMPARISON OF JAUNDICE RELATED TO BREASTFEEDING • Most common cause of hemolytic disease of the newborn
BREAST FEEDING BREAST MILK • Occurs in 20-25% of pregnancies but hemolysis develops in only
JAUNDICE JAUNDICE 10% of such offspring
Onset DOL End of the 1st week • Mother is type O and baby is either A or B
Inadequate nursing, Substance in • Most cases are mild; jaundice
Factors decreased caloric breastmilk: • Mild hepatosplenomegaly
intake Glucuronidase • Phototherapy; if severe, IVIG or exchange transfusion
Duration Few days 3 weeks – 3 mos • ABO incompatibility lab test results:
Continue 1. Weakly to moderately (+) direct Coombs test
Treatment Stop for 2 days 2. Spherocytes in blood smear
breastfeeding 10h/day
3. Hemoglobin is usually normal but maybe as low as 10-12
An easy way to remember which comes first. It’s alphabetical, F comes g/dL
before M. BFeeding jaundice occurs earlier than BMilk jaundice.
Dr. De Vera
4. Increased reticulocyte count in 10-15%
5. Increased B1 (may reach 20 mg/dL in 10-20%)
✔ GUIDE QUESTION
A 3-day old infant is jaundiced from the head down to the upper trunk. RH INCOMPATIBILITY
His serum bilirubin level is probably between:
A. 6-8 mg/dL
• Rh antigenic determinants are genetically transmitted from each
B. 9-12 mg/dL parent & direct the production of blood group factors
C. 12-14 mg/dL (C, c, D, d, E, e)
D. 15-18 mg/dL • Each factor can elicit a specific antibody response where 90%
are due to D antigen.
• Conditions when Rh incompatibility occurs:
1.When Rh+ blood is infused into a Rh- woman by error, or;
2.When Rh+ fetal blood with D Ag inherited from a Rh+ father
enter the maternal circulation during pregnancy, with
spontaneous or induced abortion, at delivery
• Ab formation against D Ag may be induced in the unsensitized
Rh- recipient mother → rise in IgM initially then rise in IgG
crossing the placenta
• Rarely occurs during the 1st pregnancy because transfusion of Rh+
fetal blood into a Rh- mother occurs near the time of delivery, too
late for the mother to become sensitized & transmit antibody to
her infant before delivery.
• Injection of anti-D gamma globulin (RhoGAM) into the mother
immediately after the delivery of each Rh+ infant reduces Rh
hemolytic disease
Rh incompatibility lab test results:
KERNICTERUS • Before treatment:
1. Direct Coombs test is +
• Results from deposition of unconjugated bilirubin in the basal
2. Anemia
ganglia and brainstem
3. Increased reticulocyte count
• Kernicterus is rare in healthy infants if the serum level is less 4. B1 rises rapidly in the 1st 6 hours of life
than 25 mg/dL 5. B2 may also be elevated
• Clinical manifestation
o Phase 1 – poor sucking, stupor, hypotonia, seizure COOMBS TEST NEGATIVE
o Phase 2 – hypertonia, opisthotonos, fever
Coombs test negative with ↑ Hb and ↑B1
o Phase 3 – hypertonia
• polycythemia
HYPERBILIRUBINEMIA • infant of diabetic mother
DIRECT HYPERBILIRUBINEMIA • SGA
(ALWAYS PATHOLOGIC) • delayed cord clamping
• twin transfusion / maternal-fetal transfusion
1. Intrahepatic Cholestasis
o sepsis / TORCHeS Coombs test negative with normal / ↓ Hb and normal
o prolonged TPN reticulocyte count
o hypothyroidism • enclosed hemorrhage
o galactosemia • increased enterohepatic circulation
o cystic fibrosis • decreased calories (e.g. breastfeeding jaundice)
o alpha-1-antitrypsin deficiency • disorders of conjugation (e.g. breastmilk jaundice)
TOPNOTCH MEDICAL BOARD PREP PEDIATRICS MAIN DIGITAL HANDOUT BY DR. PUNONGBAYAN AND DR. DE VERA Page 11 of 105
For inquiries visit www.topnotchboardprep.com.ph or https://www.facebook.com/topnotchmedicalboardprep/
This handout is only valid for the March 2021 PLE batch. This will be rendered obsolete for the next batch since we update our handouts regularly.
TOPNOTCH MEDICAL BOARD PREP PEDIATRICS MAIN DIGITAL HANDOUT BY DR. PUNONGBAYAN AND DR. DE VERA
For inquiries visit www.topnotchboardprep.com.ph or https://www.facebook.com/topnotchmedicalboardprep/
This handout is only valid for the March 2021 PLE batch. This will be rendered obsolete for the next batch since we update our handouts regularly.
Coombs test negative with normal / ↓Hb & ↑reticulocyte count ✔ GUIDE QUESTION
• with characteristic RBC morphology A one-month old male was brought to your clinic due to scrotal
o Spherocytosis swelling. Mother noticed this to be present since birth. There were no
o Elliptocytosis other associated symptoms. No fever, no tenderness. Baby was well,
with good suck and activity. On physical examination you see this
• with non-characteristic RBC morphology
o G6PD deficiency
o Pyruvate kinase deficiency

✔ GUIDE QUESTIONS
An infant is born premature at 36 weeks to a mother with prolonged
preterm rupture of membranes. He developed jaundice on the 3rd day
of life and has poor suck, is lethargic, and has retractions. You suspect
him as having Gram-negative sepsis. Which antibiotic therapy will
most likely benefit this child?
A. Ampicillin + amikacin
B. Trimethoprim + sulfamethoxazole
C. Ciprofloxacin What is your diagnosis?
D. Imipenem + Cilastatin
The most important risk factor that predisposes a neonate to sepsis is: Answer: Hydrocele
A. History of maternal infection
B. Prematurity
C. Route of delivery GENITOURINARY ANOMALIES
D. Unestablished normal flora
• Hydrocele - accumulation of fluid in the tunica vaginalis (1-2%
of neonates); majority are noncommunicating; resolves by 12
EVALUATION OF HYPERBILIRUBINEMIA months old
• Hernias – usually indirect inguinal hernias; presents as a
reducible scrotal swelling
Important points to remember:
Hydrocoele – may observe up to 1 year of age
Undescended testes – may observe up to 3-4months of age
Inguinal hernia – needs to be repaired surgically
Dr. De Vera

EVALUATION OF
HYPERBILIRUBINEMIA
https://qrs.ly/y2bj8bc

Please study this flowchart as it is a guide to evaluate


jaundice/hyperbilirubinemia. Some key points.
1. Direct hyperbilirubinemia = pathologic = cholestasis
2. In neonates, it is mostly indirect hyperbilirubinemia
3. First thing to check is Coombs if positive think RH vs ABO
4. Coombs (-) with increase Hgb = polycythemia
5. Coombs neg with normal Hgb and normal retic = physiologic • Undescended testes
Dr. De Vera
o About 4.5% of boys at birth
o Majority descend simultaneously during the 1st 3 months of
CONGENITAL INFECTIONS life
CASE KEY CLUES DIAGNOSIS o By 6 mos old, the incidence decreases to 0.8%
Vesicular lesions on o If the testes has not descended by 4 months, it will remain
HSV undescended.
the face and mouth
Purpuric o Treated surgically not later than 9-15 months old
hemorrhagic lesions Rubella
all over the body PEDIATRIC NUTRITION
Maculopapular rash
✔ GUIDE QUESTIONS
Imaging: Periostitis Syphilis The most practical and pertinent guide in evaluating nutritional status
A baby with of the bone in children is:
IUGR born to a Chorioretinitis A. Biochemical studies of food and determination of vitamin levels
mother with a Imaging: B. Regular or periodic follow up of weight and height
CMV
history of Periventricular C. X-ray studies of bone
infection calcifications D. Dietary history
during Chorioretinitis Colostrum is produced over the first 2 days of lactation. The secretion
pregnancy has higher protein and lower fat and lactose than mature human milk.
Microcephaly
Likewise, it is especially rich in:
Hepatosplenomegaly A. IgM
Toxoplasmosis
Imaging: B. IgG
Intracerebral C. IgD
calcifications D. IgA
Cutaneous scars The whey-to-casein ratio in mature human milk is about:
Imaging: Cortical Varicella A. 1:1
atrophy B. 1:4
C. 3:2
D. 4:1
Whey is better than Casein. Think bodybuilders.
Dr. De Vera

TOPNOTCH MEDICAL BOARD PREP PEDIATRICS MAIN DIGITAL HANDOUT BY DR. PUNONGBAYAN AND DR. DE VERA Page 12 of 105
For inquiries visit www.topnotchboardprep.com.ph or https://www.facebook.com/topnotchmedicalboardprep/
This handout is only valid for the March 2021 PLE batch. This will be rendered obsolete for the next batch since we update our handouts regularly.
TOPNOTCH MEDICAL BOARD PREP PEDIATRICS MAIN DIGITAL HANDOUT BY DR. PUNONGBAYAN AND DR. DE VERA
For inquiries visit www.topnotchboardprep.com.ph or https://www.facebook.com/topnotchmedicalboardprep/
This handout is only valid for the March 2021 PLE batch. This will be rendered obsolete for the next batch since we update our handouts regularly.
Until when can you feed breastmilk solely to a child? GERD
A. 3 months
B. 6 months • Epidemiology: infants’ condition peak at 4 mos & resolve mostly
C. 12 months at 12 mos of age & nearly all at 24 mos old
D. 2 years • Genetic predisposition: AD (Chromosome 13q,14 & 19)
Exclusive up to 6 mos. Then complimentary feeding. • Clinical manifestations:
Breastfeeding is best for baby up to 2yo and BEYOND… o infants: postprandial regurgitation, signs of esophagitis
Dr. De Vera
(irritability, arching, choking, gagging, feeding aversion),
failure to thrive
FEEDING: 1ST 6 MONTHS OF LIFE
o Older children: regurgitation, abdominal & chest pain;
• Initiated as soon as after birth respiratory manifestations related to asthma or sinusitis
o Within 1-4 hrs
• Diagnosis: Barium study of esophagus & upper GIT (poor
• Schedule: “Self-regulation” by infant sensitivity and specificity)
o 1st wk 60-90mL/feeding: 6-9 feedings/24hrs
• Definitive test: esophageal pH probe
o Middle of the night feedings: from birth to 6-8wks, beyond
• Esophageal pH monitoring of distal esophagus: document acidic
o By 9-12 mos: 3 meals/day + snacks
reflux episodes (normal value of distal esophageal acid exposure
o There is no need to feed infants every time they cry.
<5-8% of total monitored time)

BREAST MILK MANAGEMENT OF GERD


BREAST TIMING OF • Dietary measures: thicken the formula; avoid acidic/spicy foods,
CONSISTENCY CONTENT
MILK RELEASE juices, alcohol, caffeinated & carbonated drinks
At the start of High lactose, • Awake: prone or upright carried position
Foremilk Watery • Antacids, H2RAs, PPIs, prokinetic agents
feeding high protein
As feeding • Fundoplication – for intractable GERD, refractory esophagitis &
High fat (5x strictures, with morbidities from chronic pulmonary disease
progresses
Hind the fat (recurrence rate of 14%)
until towards Creamy
milk content of
the end of
foremilk)
feeding

STAGES OF LACTATION
FIRST 0-7 DAYS: COLOSTRUM
• 37-84 mL/day; D1-2 of life
• Watery (>80% water)
• protein-rich
• High in Ig/protective factors: lactoferrin, lysozyme
• Na, vit A/K & growth factors
• Low levels: fat & carbohydrates

TRANSITIONAL MILK
• Between colostrum & mature milk
• Rising levels of macronutrients

MATURE MILK
• D 10-14 of life
• Colostrum content + high fat & lactose

INVOLUTIONARY MILK
• produced when breastfeeding frequency decreases
• reverts to being more like colostrum

BREAST MILK COMPOSITION


✔ GUIDE QUESTION
• casein to whey ratio - low Which of the following is an absolute contraindication to
o 10:90 in early milk breastfeeding?
o 40:60 in mature milk A. An infant with galactosemia
o 50:50 in late lactation B. A mother with postpartum blues
• fat globules bound by membranes rich in: C. A mother with active pulmonary tuberculosis
o phospholipids – cell growth & brain development D. An infant with breastfeeding jaundice
o cholesterol – facilitates myelination of the CNS
• Breast milk has MORE compared to Formula milk CONTRAINDICATIONS TO BREASTFEEDING
o Exceptions • Galactosemia
§ Iron • Septicemia
§ Vit D • Active TB
§ Vit K
• Breast Cancer
• Malaria
✔ GUIDE QUESTION
• Substance Abuse
Overall, the 3-month-old male infant is apparently well but he
regurgitates 10-20 mL of milk usually after feeding, what is the best • Severe neurosis or Psychosis
management for this condition? • HIV
A. Advise Thick Feeding Important points to remember:
B. Endoscopy TB – after two weeks of treatment, mother no longer considered
C. Observe and Reassurance infectious, may do direct breastfeeding. Prior to this, expressed mother’s
D. Upper GI Endoscopy milk can be given
HIV – not absolutely contraindicated. In times where no safer alternative
is available, HIV mother can breastfeed provided that:
1. Shortest duration possible
2. Exclusive breastfeeding (mixed feeding increases risk of AGE which
increases transmission rate because of damaged intestinal villi)
Dr. De Vera

TOPNOTCH MEDICAL BOARD PREP PEDIATRICS MAIN DIGITAL HANDOUT BY DR. PUNONGBAYAN AND DR. DE VERA Page 13 of 105
For inquiries visit www.topnotchboardprep.com.ph or https://www.facebook.com/topnotchmedicalboardprep/
This handout is only valid for the March 2021 PLE batch. This will be rendered obsolete for the next batch since we update our handouts regularly.
TOPNOTCH MEDICAL BOARD PREP PEDIATRICS MAIN DIGITAL HANDOUT BY DR. PUNONGBAYAN AND DR. DE VERA
For inquiries visit www.topnotchboardprep.com.ph or https://www.facebook.com/topnotchmedicalboardprep/
This handout is only valid for the March 2021 PLE batch. This will be rendered obsolete for the next batch since we update our handouts regularly.

This table simply shows presentation of GERD on


different age group. For children please take note
that GERD should be a differential diagnosis for
chronic cough and/or respiratory symptoms.
Dr. De Vera

Nelson Textbook of Pediatrics, 20th ed.

MICRONUTRIENT DEFICIENCIES
Vesiculobullous, eczematous, dry scaly or psoriasiform lesions Chronic diarrhea, Stomatitis, Glossitis,
Zinc
symmetric perioral, sacral, and perianal areas Irritability, superinfection
Alopecia , Thrombocytopenia, Failure to Essential Fatty
Generalized scaly dermatitis
thrive Acids
Edema, erythema and burning of sun exposed skin on the face neck Diarrhea Niacin (Pellagra)
hands on butterfly distribution dermatitis around the neck dementia / Vit B3

Alopecia with neurological symptoms Biotin / Vit B7

Xerosis, Night blindness Vit A


Follicular hyperkeratosis Perifollicular erythema and hemorrhage
Vit C
Bleeding swollen gums

Tips on studying micronutrient deficiency. When answering the exam, think first of micronutrients with disease entities such as Rickets, Scurvy, Zinc
(acrodermatitis-enterohepatica), etc. Because as you will see these micronutrient deficiencies have overlapping presentation such as eczema, diarrhea, dermatitis
and others.
Dr. De Vera

FAT SOLUBLE VITAMINS IN BREAST MILK • Increase in the circumference of the growth plate and the
VITAMIN A metaphysis → widening of the wrists & ankles
• General softening of the bones
• Bioavailability of Vit A in breast milk is more than 90%
• Craniotabes or softening of the cranial bones
• Preferentially transferred to breast milk from maternal stores
• Widening of the costochondral junctions leads to “rachitic
and dietary intake
rosary”
• Premature babies – born with smaller stores of Vit A → more
• “Harrison groove” is a horizontal depression along the lower
vulnerable to deficiency
anterior chest due to pulling of the softened ribs by the
• Component of retinal pigments; for bone & tooth development;
diaphragm during inspiration
formation & maturation of epithelia
Vitamin A deficiency
• Nyctalopia (reduced ability to see in decreased illumination)
• Photophobia
• Xerophthalmia (abnormal dryness of the conjunctiva & cornea)
• Keratomalacia (softening & drying of the cornea)
• Faulty epiphyseal bone formation
• Defective tooth enamel
• Impaired resistance to infection
• Bitot spot – oval or triangular spot of keratinized epithelium in
the conjunctiva
• Windswept deformity (valgus deformity
VITAMIN D of 1 leg with varus deformity of the other
• content in human milk depends on maternal vitamin D status leg)
(maternal sunlight exposure) • Treatment of Rickets:
• The amount of sunlight needed to produce sufficient vitamin D • Vitamin D 300,000-600,000 IU orally or
depends on skin color, latitude, and season. IM as 2-4 doses over 1 day (Stoss
• Facial exposure to sunlight of 2 hours at least once a week or 30 therapy)
minutes with bare arms and legs is sufficient. • Either strategy should be followed by
daily Vit D intake of 400 IU/day as a
Rickets
multivitamin
• Disease of growing bone which occurs in children only before
• Adequate dietary calcium & phosphorus
fusion of the epiphyses
• Due to unmineralized matrix at the growth plates
TOPNOTCH MEDICAL BOARD PREP PEDIATRICS MAIN DIGITAL HANDOUT BY DR. PUNONGBAYAN AND DR. DE VERA Page 14 of 105
For inquiries visit www.topnotchboardprep.com.ph or https://www.facebook.com/topnotchmedicalboardprep/
This handout is only valid for the March 2021 PLE batch. This will be rendered obsolete for the next batch since we update our handouts regularly.
TOPNOTCH MEDICAL BOARD PREP PEDIATRICS MAIN DIGITAL HANDOUT BY DR. PUNONGBAYAN AND DR. DE VERA
For inquiries visit www.topnotchboardprep.com.ph or https://www.facebook.com/topnotchmedicalboardprep/
This handout is only valid for the March 2021 PLE batch. This will be rendered obsolete for the next batch since we update our handouts regularly.
VITAMIN E BIOTIN (VITAMIN B7)
• Antioxidant; protects PUFA in cell membranes from oxidative • Cofactor in carboxylase reactions of amino acids
damage • Alopecia, dermatitis, hypotonia
• constant in mature milk at 0.32 mg tocopherol • Sources: meats, yeast
equivalents/100mL
• If deficient: VITAMIN B12 AND FOLATE
o RBC hemolysis in premature infants • bound to proteins in the whey fraction of breast milk (enhanced
o loss of neural integrity bioavailability)
o posterior column and cerebellar dysfunction • breastfed infants of strict vegetarians or malnourished mothers
o pigmentary retinopathy can exhibit clinical Vitamin B12 deficiency leading to severe and
persisting neurological damage
VITAMIN K • Coenzyme for 5-methyl-tetrahydrofolate formation
• Prothrombin formation, coagulation factors II, VII, IX, X • Megaloblastic anemia, peripheral neuropathy
• The vitamin K content of breast milk is between 0.1 – 0.4
µg/100mL and is higher in colostrum than mature milk. VITAMIN C
• deficiency leads to hemorrhagic disease of the newborn or • Infants of well-nourished mothers ingest around 20mg/day of
vitamin K deficiency bleeding (VKDB) Vitamin C from breast milk compared with 8mg required to
• VKDB with breastfeeding is due to low overall intakes of vitamin prevent scurvy
K in the first week of life, rather than low vitamin K content of • Integrity & maintenance of intercellular material; facilitates
breast milk. absorption of iron & conversion of folic acid to folinic acid;
metabolism of tyrosine & phenylalanine
VKDB OF THE NEWBORN
SCURVY
• Early VKDB
• Vitamin C deficiency early
o called classic hemorrhagic disease
symptoms: low-grade fever,
o occurs at 1-14 days of age
irritability, tachypnea, anorexia,
o low stores of vitamin K at birth
generalized tenderness esp. in
§ poor transfer of vitamin K across the placenta
the legs
§ Inadequate intake during the 1st few days of life
§ In addition, there is no intestinal synthesis of vitamin K2 • Pseudoparalysis with hips &
because the newborn gut is sterile knees semi-flexed & the feet
rotated outward
• Late VKDB
• “scorbutic rosary” at the
o 2-12 weeks of life
costochondral junction &
o Some up to 6 months of age
depression of the sternum
o Mostly breastfed infants (low vit K content)
o Malabsorption syndromes • Angulation of the scorbutic beads is sharper than the rachitic
• Third form rosary
o At birth or shortly after birth • Bluish, purple spongy swellings of the mucous membranes esp.
o Maternal medications (warfarin, phenobarbital, phenytoin) over the upper incisors
• Other symptoms: swollen joints, purpura and ecchymoses, poor
wound & fracture healing
WATER SOLUBLE VITAMINS IN BREAST MILK
• perifollicular hemorrhages, hyperkeratosis of hair follicles,
THIAMINE / B1 “corkscrew hair”
• Coenzyme in ketoacid decarboxylation • X ray changes: distal ends of long bones with a ground-glass
• Beri-beri, polyneuropathy, calf tenderness, heart failure appearance
RIBOFLAVIN / B2
• FAD coenzyme in oxidation-reduction reactions Treatment of scurvy:
• mucositis, cheilosis, anemia, nasolabial seborrhea • Daily intake of 3-4 oz of orange or tomato juice
o Angular cheilosis (perleche) - begins with pallor at the angles • Vitamin C supplements of 100-200 mg orally or parenterally are
of the mouth; fissures covered by yellow crusts preferable to ensure more rapid and complete cure.
o Glossitis (magenta tongue) - painful tongue inflammation • Larger doses (>2 grams) may produce abdominal pain and
osmotic diarrhea
Manifestations of B2 deficiency is non-specific. It is associated with
perleche and glossitis but please take note these are also present in
Scurvy, Zinc deficiency, and other diseases MINERALS IN BREAST MILK
Dr. De Vera
IRON
• The iron content of breast milk is unaffected by maternal iron
NIACIN / B3 status, maternal iron deficiency, or supplementation
• NAD coenzyme in • Infants’ iron requirements are largely met from body stores built
oxidation-reduction up in utero
reactions • Combined with breastfeeding, these stores are usually sufficient
• Manifestations: pellagra: to meet infants iron needs for 6 – 12 months.
dermatitis, diarrhea, • Lesser in breast milk but more bioavailable
dementia
• Sources: meat, fish, liver, ZINC
whole grains, green leafy
Casal Necklace • Infant’s requirement in the first 6 months are largely met by fetal
vegetables http://mkginbc.blogspot.com/2007/06/
stores accumulated in the last trimester of pregnancy →
thereafter should be met by appropriate complementary foods
VITAMIN B6
• In well-nourished populations, breast milk provides sufficient IODINE
B6 to meet requirements for up to 6 months
• Iodine accumulates in the mammary gland and levels in breast
• Mothers who have been long term users of high estrogen- milk reflect maternal diet.
containing contraceptives may have lower levels of B6 and need
• In areas where iodine deficiency is common, maternal
supplements of 20mg/day
supplementation is necessary.
• Cofactor in amino acid metabolism
• Microcytic anemia, cheilosis, glossitis, dermatitis Please remember this. Most micronutrients in breastmilk are not affected
by maternal diet/status. Vitamin D and Iodine are affected by maternal
Vitamin B6 deficiency is also non-specific and not common. But take note status.
of the mother who takes in estrogen pills and the infant presenting with For Iron and Zinc, please remember that full-term neonates are born with
microcytic anemia + other findings stated above. sufficient stores that are enough for the first 6 months of life.
Dr. De Vera Dr. De Vera

TOPNOTCH MEDICAL BOARD PREP PEDIATRICS MAIN DIGITAL HANDOUT BY DR. PUNONGBAYAN AND DR. DE VERA Page 15 of 105
For inquiries visit www.topnotchboardprep.com.ph or https://www.facebook.com/topnotchmedicalboardprep/
This handout is only valid for the March 2021 PLE batch. This will be rendered obsolete for the next batch since we update our handouts regularly.
TOPNOTCH MEDICAL BOARD PREP PEDIATRICS MAIN DIGITAL HANDOUT BY DR. PUNONGBAYAN AND DR. DE VERA
For inquiries visit www.topnotchboardprep.com.ph or https://www.facebook.com/topnotchmedicalboardprep/
This handout is only valid for the March 2021 PLE batch. This will be rendered obsolete for the next batch since we update our handouts regularly.

B VITAMINS o Weight for height


§ Wasting
B1 Thiamine
§ Acute malnutrition
B2 Riboflavin
o Weight for age
B3 Niacin
§ Low value
B5 Pantothenic Acid
§ Limited clinical significance
B6 Pyridoxine o Mid upper arm circumference
B7 Biotin § 11.5-12.5 cm = wasting
B9 Folic Acid § <11.5 cm = severe wasting
B12 Cobalamin
For malnutrition, the term we are looking for is WASTING. Only three
parameters can be used depending on age of patient: Weight for
Bowlegs or knock knees length/height, MUAC, BMI
Caput quadratum, Periosteal osteoid Dr. De Vera

Craniotabes KWASHIORKOR VS MARASMUS


Vit D FEATURE KWASHIORKOR MARASMUS
Ping pong ball sensation
Rachitic rosary, Pigeon chest Growth Failure Present Present
Harrison groove Wasting Present Present, marked
“White line” on end of shafts Present
Vit C Edema Absent
Scorbutic rosary/beads (sometimes mild)
Pellagra Hair changes Common Less common
Casal necklace Mental changes Very common Uncommon
Niacin (Vit B3) Dermatosis,
Pellagrous glove and boots
Common Does not occur
Glove and boots lesion flaky-paint
Perleche Vit B2 Appetite Poor Good
Reduced but
Bitot spots Subcutaneous fat Absent
present
Xerosis conjunctivae Vit A Deficiency
Drawn in,
Xerosis cornea Face May be edematous
monkey-like
Hyperostosis Fatty infiltration
Vit A Intoxication Present Absent
Absence of metaphyseal changes of liver
✔ GUIDE QUESTIONS Having problem differentiating the two? Super easy! Kwashiorkor is
A deficiency of this trace element is associated with skin ulcers, reduced more complicated (spelling pa lang mahirap na! hehe). If the finding in
immune response and hypogonadal dwarfism: questions is negative in nature, probably that is Kwashiorkor. Don’t
A. Zinc believe me? Look at the table above. Marasmus is more benign among
B. Chromium the two. J
Dr. De Vera
C. Cobalt
D. Copper
Irritability, pruritus, painful extremities, with brawny swelling, coarse
hair, dry skin, seborrhea and increased intracranial pressure is seen in:
A. Hypervitaminosis A
B. Hypervitaminosis D
C. Hypovitaminosis A
D. Hypovitaminosis D
In a Southwestern town in Mindanao, children are fed mostly with corn
as staple. The most common vitamin deficiency encountered in these
children is:
A. Niacin
B. Folate http://www.differencebetween.info/difference-between-kwashiorkor-and-marasmus
C. Thiamine
D. Riboflavin
In children with malnutrition, the most seriously compromised SEVERE ACUTE MALNUTRITION
immunologic function is: • defined as severe wasting and/or bilateral Edema
A. Antibody production • Severe wasting is extreme thinness diagnosed by a weight-for-
B. Phagocytosis length (or height) below −3 SD of the WHO Child Growth
C. Cell-mediated immunity
D. Complement activation
Standards
• children ages 6-59 mos, a mid-upper arm circumference
One of the macronutrients severely affected in malnutrition are <115mm also denotes extreme thinness
proteins (think Kwashiorkor). Therefore, problem in proteins • Differentiates wasted from stunted
→ problem in antibodies. o Stunted not priority in acute clinical care
Dr. De Vera
A 5 year old male child is brought to the clinic for being weak and wants
• Pathophysiology
her child to be dewormed. It has been going on for the past 6 months o When intake cannot meet daily needs → physiologic and
according to the mother. She says that the child usually is prone to metabolic changes in an orderly progression to compensate =
having diarrhea and usually he has episodes every month. On Reductive Adaptation
examination, the child looks apathetic, he is underweight, there is some 1. Fat stores mobilized for energy
edema of the bilateral lower extremities, the abdomen is protuberant, 2. Protein is mobilized
the hair is sparse thin with reddish streaks, what is the most likely 3. Energy conserved by reducing activity and growth, basal
diagnosis in this patient? metabolism, functional reserve of organs, inflammatory
A. Marasmus
B. Non-edematous protein energy malnutrition
and immune response
C. Kwashiorkor o Liver makes less glucose = hypoglycemia
D. Pellagra o Liver makes less albumin, transferrin and other proteins
o Heat production is less = hypothermia
o Kidneys excrete less = fluid overload
UNDERNUTRITION o Heart is smaller and weaker = cardiac failure
• Greatest risk occurs in the 1st 1000 days (0-24months) o Sodium builds up due to leaky membranes and inactive pumps
• Term malnutrition covers undernutrition to over-weight = edema
• International standards of determining anthropometry using o Potassium leaks out of cells
WHO charts o Loss of muscle protein
o Height for age (length for age if <2)
Remember the words REDUCTIVE ADAPTATION. This explain all the
§ Measures linear growth features of severe malnutrition.
§ Represents cumulative impact of adverse events Dr. De Vera

TOPNOTCH MEDICAL BOARD PREP PEDIATRICS MAIN DIGITAL HANDOUT BY DR. PUNONGBAYAN AND DR. DE VERA Page 16 of 105
For inquiries visit www.topnotchboardprep.com.ph or https://www.facebook.com/topnotchmedicalboardprep/
This handout is only valid for the March 2021 PLE batch. This will be rendered obsolete for the next batch since we update our handouts regularly.
TOPNOTCH MEDICAL BOARD PREP PEDIATRICS MAIN DIGITAL HANDOUT BY DR. PUNONGBAYAN AND DR. DE VERA
For inquiries visit www.topnotchboardprep.com.ph or https://www.facebook.com/topnotchmedicalboardprep/
This handout is only valid for the March 2021 PLE batch. This will be rendered obsolete for the next batch since we update our handouts regularly.
IN PATIENT MANAGEMENT OF MALNUTRITION (WHO)

Nelson Textbook of Pediatrics, 20th ed.

• Management PATHOGENESIS
• Dysregulation of caloric intake and energy expenditure
• Environmental changes
o Fast food diets
o Snacking in between meals
o Increased sedentary activities
o Lack of exercise
• Genetics
o Genes determine resting energy expenditure and the weight
set point – level of stored fuel that satisfies an individual
o Heredity: influences fatness, regional fat distribution,
response to overfeeding

• BMI – Body Mass Index


%&'(ℎ*("#)
!"# = &
+&'(ℎ*(%)
BMI PERCENTILE FOR AGE WEIGHT STATUS
< 5th percentile Underweight
5th – 84th Normal weight
85th –94th At risk for overweight
≥ 95th Overweight

OBESITY
• Excessive accumulation of body fat
o boys: TBW>25% fat
Nelson Textbook of Pediatrics, 20th ed. o girls: TBW>32% fat
✔ GUIDE QUESTION • Skinfold of triceps + calf
A 4-year-old malnourished girl was admitted for diarrhea with severe o boys: N 10-25mm
dehydration. Patient was resuscitated successfully and is completing o Girls: N 16-30mm
antibiotic treatment. She was feeding well and is able to finish the • Appears most frequently in the 1st yr of life, at 5-6 yrs of age &
regular serving of food given. On the 4th hospital day, patient was
during adolescence
suddenly developed seizure and cardiac arrest. What is the possible
cause of the patient’s demise? • Disproportionately fine facial features, adiposity in the
A. Inadequate antibiotic coverage for CNSI mammary regions, pendulous abdomen, disproportionately
B. Excessive feeding syndrome small external genitalia for boys
C. Dehydration
D. Refeeding Syndrome OBESITY IN CHILDHOOD
• ↑ risk for obesity in adulthood
REFEEDING SYNDROME • Leading cause of pediatric hypertension
• Malnutrition usually has normal serum electrolytes but is • Associated with Type 2 DM
associated with intracellular electrolyte depletion. • ↑ risk of coronary heart disease
• Excessive carbohydrates → surge of insulin → hypokalemia, • ↑ stress on weight-bearing joints
hypophosphatemia, hypomagnesemia • Low self-esteem
• Hallmark is severe hypophosphatemia
• Usually during the 1st week of refeeding EVALUATION:
• S/Sx: Serum phosphate levels of ≤0.5 mmol/L can produce • Screening includes:
weakness, rhabdomyolysis, neutrophil dysfunction, o (+) family history (CV disease, parental elevated total
cardiorespiratory failure, arrhythmias, seizures, altered level of cholesterol level, DM, parental obesity)
consciousness, or sudden death o BP
o Total cholesterol level (>200 mg/dL)
OBESITY o Large annual incremental increase in BMI (increase over the
previous year of 2 BMI units)
• First predictors o Concern about weight (assessment of personal concerns,
o High birth weight emotional or psychological, related to overweight or the
§ Maternal obesity perception of overweight)
§ Maternal diabetes
o Overweight children – overweight adults
o Parental obesity
§ Doubles risk for adult obesity
§ Parental modeling of eating and exercise behaviors
TOPNOTCH MEDICAL BOARD PREP PEDIATRICS MAIN DIGITAL HANDOUT BY DR. PUNONGBAYAN AND DR. DE VERA Page 17 of 105
For inquiries visit www.topnotchboardprep.com.ph or https://www.facebook.com/topnotchmedicalboardprep/
This handout is only valid for the March 2021 PLE batch. This will be rendered obsolete for the next batch since we update our handouts regularly.
TOPNOTCH MEDICAL BOARD PREP PEDIATRICS MAIN DIGITAL HANDOUT BY DR. PUNONGBAYAN AND DR. DE VERA
For inquiries visit www.topnotchboardprep.com.ph or https://www.facebook.com/topnotchmedicalboardprep/
This handout is only valid for the March 2021 PLE batch. This will be rendered obsolete for the next batch since we update our handouts regularly.
COMORBIDITIES PREVENTIVE PEDIATRICS
• Harvard Growth Study:
o Doubling of cardiovascular disease esp. in male adolescents
• Bogalusa Health Study: IMMUNIZATION
o BMI >85th percentile https://qrs.ly/8mbj8b4
§ Hypercholesterolemia, hypertriglyceridemia, hypertension
• Metabolic syndrome
o Hypertension, glucose intolerance, hypertriglyceridemia, low
HDL, abdominal central obesity IMMUNIZATION
• Type 2 DM
• Ultimate goal: eradication of disease
• Gallstones
• Immediate goal: prevention of disease
• Asthma
• Two types:
• Obstructive sleep apnea - snoring
1. Active immunization
• Polycystic ovary syndrome
2. Passive immunization
o hirsutism, male pattern baldness, severe acne
• Fatty liver disease – non-alcoholic
ACTIVE IMMUNIZATION
• Psychosocial disorders
o Anxiety, depression • Efficacy is assessed by the evidence of protection against the
particular disease.
PICKWICKIAN SYNDROME o Antibody formation – indirect measure
• Rare complication of extreme exogenous obesity in which • Types of antigen for active immunization:
patients have severe cardiorespiratory distress with alveolar 1. Live attenuated virus or bacteria (weakened)
hypoventilation & a decrease in pulmonary, tidal & expiratory 2. Inactivated vaccine (killed microorganisms)
reserve volumes
INACTIVATED VACCINES LIVE VACCINES
• Polycythemia, hypoxemia, cyanosis, cardiac enlargement, CHF,
somnolence • Hepatitis B vaccine • BCG vaccine
• DPT vaccine • Measles vaccine
• Weight reduction as rapidly as possible
• H. influenzae b vaccine • MMR vaccine
TREATMENT OF OBESITY • Pneumococcal vaccine • Varicella vaccine
• Hepatitis A vaccine • Rotavirus vaccine
• Aim: not weight loss but to slow or halt weight gain – weight
• Meningococcal vaccine • Influenza attenuated
maintenance
• For every 20% excess of ideal BW, the child needs 1 ½ years of • Influenza trivalent vaccine vaccine (intranasal)
• Human papillomavirus • Typhoid fever (oral)
wt maintenance to attain ideal BW
vaccine vaccine
• Wt loss – for skeletally mature with severe complications
• Typhoid fever vaccine (IM) • Oral polio vaccine
o 0.5kg or less/wk to 10% wt reduction
o Interventions: lifestyle changes • Rabies vaccine • Japanese Encephalitis
o Physical activity, diet management, behavior modification • Inactivated polio vaccine
(IPV)
TRAFFIC LIGHT / STOPLIGHT DIET: Memorize all the live vaccines. They are the only ones which will
GREEN LIGHT YELLOW LIGHT RED LIGHT complicate scheduling. Very easy to memorize, if it is NOT intramuscular
• Low Calorie (IM), probably it is live!
• Nutrient Dense • High Calorie Dr. De Vera
• High Fiber
• High Calorie • High Fat
• Low Fat
• High Fat • High Sugar PRINCIPLES OF IMMUNIZATION:
• Nutrient Dense
1. Simultaneous administration of multiple vaccines
• Lean Meat
• Fatty Meat (for missed doses):
• Fruits • Dairy
• Sugar • most vaccines can be safely and effectively given simultaneously
• Veggies • Starches
• Fried Food • use separate syringes and sites
• Grains
• Unlimited • Limited • Infrequent Vaccine spacing and intervals:
• Quantities • Quantities • Avoided ANTIGEN RECOMMENDED MINIMUM
COMBINATION INTERVAL BETWEEN DOSES
ROLE OF MEDICATIONS IN CHILDHOOD OBESITY • 2 or more
• Meds for child with severe complications • None; can be given simultaneously
inactivated
o The use of pharmacologic treatment for overweight and obese or at any interval between doses
vaccines
children is of marginal value with potential and unclear risks. • Inactivated & live • None; can be given simultaneously
§ i.e. Orlistat: intestinal lipase inhibitor (decreases fat vaccines or at any interval between doses
absorption) for >12 yrs old
§ WOF diarrhea, flatulence, bloating, dyspepsia This is very important. The DEFAULT ANSWER IS YES if asked whether
two vaccines can be given simultaneously on the same visit. There’s is
only one exception: yellow fever and cholera.
PLEASE READ MORE ON: Question: can chickenpox and pneumococcal vaccine be given
• Components of breast milk (bioactive, vitamins, etc.) simultaneously? Bakit nag-isip ka pa? YES NGA!
• Macronutrients vs Micronutrients Dr. De Vera

• Essential Fatty Acids


SUPPLEMENT: QUICK SHEET
SUPPLEMENT: QUICK SHEET EXCEPTIONS!!!
1. Yellow fever vaccine (live) and Cholera vaccine
Important notes on obesity
(inactivated) – separate by at least 3 weeks because of
1. As bad as undernutrition
diminished Ab response if given simultaneously
2. Lots of complications
3. Management is mostly dietary and activity modification 2. Live oral vaccines can be given simultaneously and
regardless of interval from other live vaccines
4. Pharmacologic regimen for severe cases
5. Goal most of the time is weight maintenance (as the child
grows taller while maintaining weight, BMI decreases. Vaccine spacing and intervals:
Remember the formula for BMI) ANTIGEN RECOMMENDED MINIMUM INTERVAL
COMBINATION BETWEEN DOSES
• 2 parenteral • May be given simultaneously at the
live vaccines same visit
TOPNOTCH MEDICAL BOARD PREP PEDIATRICS MAIN DIGITAL HANDOUT BY DR. PUNONGBAYAN AND DR. DE VERA Page 18 of 105
For inquiries visit www.topnotchboardprep.com.ph or https://www.facebook.com/topnotchmedicalboardprep/
This handout is only valid for the March 2021 PLE batch. This will be rendered obsolete for the next batch since we update our handouts regularly.
TOPNOTCH MEDICAL BOARD PREP PEDIATRICS MAIN DIGITAL HANDOUT BY DR. PUNONGBAYAN AND DR. DE VERA
For inquiries visit www.topnotchboardprep.com.ph or https://www.facebook.com/topnotchmedicalboardprep/
This handout is only valid for the March 2021 PLE batch. This will be rendered obsolete for the next batch since we update our handouts regularly.
As said earlier, only live-live poses a problem in scheduling. ✔ GUIDE QUESTION
Can two live vaccines be given together? YES! (sabi ko wag na mag-isip Which of the following is an example of naturally acquired passive
diba). immunity?
Can live-inactivated be given together? YES! A. Immunization with HPV vaccine
Can I give a live vaccine today followed by an inactivated tomorrow? YES B. Administration of HRIG
Can I give an inactivated today then inactivated tomorrow? YES C. Immunity from a natural infection
How about inactivated today and live tomorrow? YES D. Transplacental transfer of maternal antibodies
Live today and another live tomorrow? NO
If live + live vaccine, either give it SIMULTANEOUSLY, otherwise you
have to wait 4 weeks. PASSIVE IMMUNIZATION
Dr. De Vera • For IM administration: peaks at 48-72 hrs not to exceed 5 ml
2. Lapsed Immunizations • Indications:
• a lapse does not require reinstitution of the entire series o Congenital or acquired B-lymphocyte cell defects alone or in
• subsequent immunizations should be given at the next visit as if combination with other immunodeficiencies
the usual interval had elapsed o When time does not permit adequate protection by active
immunization alone (hepatitis B, measles, rabies, varicella)
3. Unknown or uncertain immunization status o When a person susceptible to a disease is exposed to or has a
• In general, when in doubt, immunizations should be initiated high likelihood of exposure to that infection or has a high risk
without delay on a schedule commensurate with the person’s of complications from the disease (leukemic child exposed to a
current age. person with varicella or measles)
• No evidence that giving vaccines to already immune recipients is o When a disease is already present and antibody may aid in
harmful. suppressing its toxin effects (tetanus) or the inflammatory
response (Kawasaki disease)
CONTRAINDICATIONS TO VACCINATION
• 2 permanent contraindications: ✔ GUIDE QUESTION
1. Anaphylactic reaction What is the earliest time you can give the measles vaccine?
2. Encephalopathy not due to another identifiable cause A. at birth C. 6 months
B. 3 months D. 9 months
occurring within 7 days after pertussis vaccination
• 2 temporary contraindications to live vaccines but not with
inactivated vaccines: PPS RECOMMENDED VACCINES
1. Pregnancy 1. BCG 9. Measles
2. immunosuppression 2. Hep B 10. Japanese Encephalitis
3. DPT 11. MMR
4. HiB 12. Varicella
5. Polio 13. MMRV
6. Pneumococcal 14. Hepatitis A
7. Rotavirus 15. Td/Tdap
8. Influenza 16. HPV

PPS RECOMMENDED VACCINES


PRIMARY
VACCINE MINIMUM AGE BOOSTER NOTES
SERIES
• At birth,
preferably • Intradermal. 0.05ml < 12months.
BCG --- ---
within first 0.1ml >12months
2mo
• At birth w/in • 6-10-
Hep B • depends on titers
12hours of life 14weeks
• Interval between 3rd and 4th dose is
• 6-10- • Usually at 1y/o 6months (4months is acceptable).
DPT • 6weeks
14weeks and 4yo 5th dose not needed if 4th dose given
at 4y/o or older
• 6-10- • Booster given 6mo from 3rd dose
Hib • 6weeks • 12-15mo
14weeks • Not routinely given >5y/o
• Oral or IM
• 6-10- • On or after 4th
Polio • 6weeks • Booster 6months from the previous
14weeks birthday
dose
• Healthy children 2-5y/o without
previous vaccine may be given 1
• 6-10- • 6months after 3rd
Pneumococcal • 6weeks dose PCV 13 or 2 doses PCV 10 8
14weeks dose
weeks apart
• Not recommended for > 5y/o
• Oral
• Monovalent RV1 – 2 doses
• 6-10- • *Pentavalent RV5 – 3 doses
Rotavirus • 6weeks
*14weeks • Do not start in infants >15weeks
• Not given later than 32weeks due to increased risk of
intussusception
• Initial of 2
doses 4
• 0.25ml for 6mo-35mo
Influenza • 6months weeks apart • Yearly
• 0.5ml for 36mo-18y/o
for 6months
to 8y/o
• Subcutaneous
Measles • 9months • 9months --- • May be given at 6months if with
outbreak

TOPNOTCH MEDICAL BOARD PREP PEDIATRICS MAIN DIGITAL HANDOUT BY DR. PUNONGBAYAN AND DR. DE VERA Page 19 of 105
For inquiries visit www.topnotchboardprep.com.ph or https://www.facebook.com/topnotchmedicalboardprep/
This handout is only valid for the March 2021 PLE batch. This will be rendered obsolete for the next batch since we update our handouts regularly.
TOPNOTCH MEDICAL BOARD PREP PEDIATRICS MAIN DIGITAL HANDOUT BY DR. PUNONGBAYAN AND DR. DE VERA
For inquiries visit www.topnotchboardprep.com.ph or https://www.facebook.com/topnotchmedicalboardprep/
This handout is only valid for the March 2021 PLE batch. This will be rendered obsolete for the next batch since we update our handouts regularly.
PRIMARY
VACCINE MINIMUM AGE BOOSTER NOTES
SERIES
• Subcutaneous
• 9months of
• 9-17yo give 1 booster 12-24 months from primary series
Japanese Encephalitis • 9months age and
• 18y/o and above total 1 dose
above
• <18y/o total of 2 doses
• 2 doses at
least 4 weeks • Subcutaneous
apart • Children <12mo given any measles
MMR • 12months ---
• 2nd dose containing vaccine should receive
usually at 4- additional 2 doses of MMR
6y/o
• <13yo – interval between 2 doses at
• 2 doses
least 3months
Varicella • 12months • 2nd dose at ---
• (*4 weeks is acceptable)
4-6y/o
• 13 and above- 4 weeks interval
• 2 doses 6
Hepatitis A • 12months --- ---
months apart
• Fully immunized children, defined as 5 doses of DTP or 4 doses if 4th dose given on or after 4th
Tetanus and diphtheria toxoid birthday, Td booster given every 10 years.
(Td) / Tetanus and diphtheria • Single Tdap can be given to replace due dose of Td
toxoid and acellular pertussis • Fully immunized pregnant adolescent give 1 dose Tdap any time after 20 weeks AOG
(TDaP) • Unimmunized pregnant adolescent give 3 dose td containing vaccine (Td/Tdap) following 0-1-
6mo schedule. Tdap should replace Td preferably after 20weeks AOG
• Bivalent 0-1- • Interval between:
6months • 1st and 2nd dose is 1 month
Human Papilloma Virus • 9y/o ---
• Quadrivalent • 2nd and 3rd dose is 3months
0-2-6months • 3rd and 1st dose is 6months
Please do memorize this table. This is arranged according to sequence from birth. Most common values in case you forget to memorize. Schedule usually is 6-
10-14 weeks, boosters usually 1 year old and 4 years old. You need total of 2 doses of chickenpox and MMR.
Dr. De Vera

4TH DOSE OF HEP B VACCINE § One dose PCV 13 if completely immunized with PCV 7
• If 3rd dose given at age less than 24 weeks (6 months) § PPSV at least 8 weeks after the most recent PCV 13
• Pre-term infants less than 2kgs, whose 1st dose was given at o 6-18yo
birth § 1 dose of PCV 13 then PPSV after 8weeks
o Pre-term, HBsAg (-) mother, clinically stable o Single revaccination of PPSV after 5 years
o 1st dose given at 30days of life (this can be continued as part of
the 3-dose vaccination regimen) ✔ GUIDE QUESTIONS
Persistent inconsolable crying, seizures and, rarely, acute neurologic
✔ GUIDE QUESTIONS illness may follow administration of whole cell vaccine of:
A. Pertussis
A male neonate was born from a HbsAg (+) mother, which among the
B. Diphtheria
ff. should be given to the neonate?
C. Tetanus
A. Hep B Vaccine D. A +B
D. BCG
B. Hep B Immunoglobulin E. A+B+C
A grade schoolboy sustained a superficial scratch on his right leg while
C. Ribavirin
playing with his pet Pomeranian. There was apparently neither break
The above regimen should be given within how many hours after birth?
in the skin nor bleeding. His mother brings him to you for advice. You
A. 12 hrs C. 48 hrs
will wash the wound with soap and water and:
B. 24 hrs D. 72 hrs
A. Not vaccinate
B. Give rabies immunization
MATERNAL HEPATITIS B INFECTION C. Give rabies immunoglobulin
• HBsAg (+) D. Give rabies immunization and immunoglobulin
o HBV plus HBIG (0.5ml) within 12 hours of life
• Unknown status RABIES PEP
o HBV within 12 hours of life • Category I – touching or feeding animals, licks on the skin (no
o Determine status exposure)
o If HBsAg (+) • Category II – nibbling of uncovered skin, minor scratches or
§ Give HBIg no later than 7 days of life abrasions without bleeding, licks on broken skin (exposure)
• Category III – single or multiple transdermal bites or scratches,
VACCINES FOR HIGH RISK/SPECIAL GROUPS contamination of mucous membrane with saliva from licks;
• Typhoid exposure to bat bites or scratches (severe exposure)
o 2 years old • Member of the family Rhabdoviridae
o Every 2-3 years • From saliva of rabid dogs, cats, bats, and cattle: bite of rabid
• Rabies animal or by licking the mucosa or open wound
o Pre-exposure – IM or ID days 0, 7, 21or 28 (old guidelines) • Variable period of communicability
o WHO 2018 • Incubation period: 20-180 days (9 days/7 years)
§ 2 site ID at Day 0 and 7 Madali lang tandaan. Category I – nag-inarte lang.
§ 1 site IM at day 0 and 7 Category III – SPONTANEUS BLEEDING or Cat II in head and neck
• Pneumococcal for High Risk (PCV + PPSV) Dr. De Vera

o IM MANAGEMENT OF BITES:
o High Risk: Chronic heart, lung, kidney disease, DM, CSF leak, • Active anti-rabies vaccine – 0.5 ml IM on days 0, 3, 7, 14, 28
cochlear implant, sickle cell disease and other (5th dose is optional)
hemoglobinopathies, anatomic and functional asplenia, HIV • Passive rabies vaccine –
and immunodeficiency 1. HRIG – 20 U/kg (1/2 of the dose IM & ½ of the dose
o 2-5y/o infiltrated around the wound), or;
§ One dose PCV 13 if incomplete PCV was given 2. ERIG – 40 U/kg (needs skin testing)
§ 2 doses of PCV 13 (8weeks apart) if unimmunized
TOPNOTCH MEDICAL BOARD PREP PEDIATRICS MAIN DIGITAL HANDOUT BY DR. PUNONGBAYAN AND DR. DE VERA Page 20 of 105
For inquiries visit www.topnotchboardprep.com.ph or https://www.facebook.com/topnotchmedicalboardprep/
This handout is only valid for the March 2021 PLE batch. This will be rendered obsolete for the next batch since we update our handouts regularly.
TOPNOTCH MEDICAL BOARD PREP PEDIATRICS MAIN DIGITAL HANDOUT BY DR. PUNONGBAYAN AND DR. DE VERA
For inquiries visit www.topnotchboardprep.com.ph or https://www.facebook.com/topnotchmedicalboardprep/
This handout is only valid for the March 2021 PLE batch. This will be rendered obsolete for the next batch since we update our handouts regularly.
WHO 2018 UPDATE ON RABIES IMMUNIZATION
Category I Category II Category III
• Washing of wound PLUS
• 2-sites ID on day 0,3,7 • Cat II management PLUS
OR • RIG
• Washing of wound • 1-site IM day 0,3,7 and • Infiltrate as much as possible
Immunologically Naïve
• No PEP between 14-28 to wound and area around it
OR • No need to give the remaining
• 2-sites IM day 0 and 1-site IM via IM
day 7, 21
• Washing of wound
PLUS
Previously Immunized • 1-site ID on days 0 and 3
Those who received PreP or PEP • Washing of wound • Same as Cat II
OR
within 3 months requires NO • No PEP • No RIG
Vaccination • 4-sites ID on day 0
OR
• 1-site IM days 0 and 3
Take note that this is the latest guidelines. In the 20th edition of nelson’s the much older one is used (5 doses of vaccine, now only 4. IG remaining dose is infiltrated
on the deltoid which now is no longer recommended)
Dr. De Vera

Before sending the patient home…. POST EXPOSURE PROPHYLAXIS


• Check tetanus immunization status • Who receives: Household, Playmate, Hospital workers
• Who needs antibiotic prophylaxis? Category III bites • Varicella
• Drug of choice? o VZIG within 96 hours 1 vial (125 U/1.25ml) per 10 kg IM
o Co-Amoxiclav at 40 mg/kg/day for 7 days (never IV!!) max of 5 vials
o Vaccine: healthy children within 3-5 days
o Newborn: give if onset of varicella <5 d pre delivery or within
48 hours
o Acyclovir?
• Meningococcemia
o Rifampicin, Ceftriaxone or Ciprofloxacin (adults)
• Diphtheria
o All close contact regardless of immune status
o Give booster if none within last 5 years
o Erythromycin 40-50mg/kg/d or Benzathine Pen G
Important points to remember:
• For IM injections, the choice of site is based on the volume of the
injected material and the size of the muscle.
• In children younger than 1 year of age, the anterolateral aspect of
the thigh provides the largest muscle and is the preferred site
• In older children, the deltoid muscle is usually large enough for
IM injection.
• The upper, outer aspect of the buttocks should not be used for
active immunization because the gluteal region is covered by a
significant layer of subcutaneous fat and because of the
possibility of damaging the sciatic nerve.
IRON SUPPLEMENTATION
AGE PREPARATION DOSE
• 0.3ml once a day
• Drops: 15mg
• Low birth to start at 2mo of
elemental
weight age until
iron/0.6ml
6months
• Infants 6- • 0.6ml OD for
• Drops
Nelson Textbook of Pediatrics, 20th ed.
11months 3months
• Children 1- • 5ml OD for 3
• Syrup: 30mg/5ml
SUPPLEMENT: QUICK SHEET 5yo months
IMPORTANT POINTS: • Tablet with 60mg
• Simultaneous administration of Vaccines • Adolescents iron and 400mcg • OD
o No contraindication except for folic acid
1. MCV and DPT, and • Notes
2. Yellow fever and Cholera o Preferably taken on empty stomach
• Lapsed immunization schedules o Preferably not mixed with milk
o Does not require reinstitution of series DEWORMING
(DTP, IPV, Hep B, & Hib) • Recommended for all children 1-12yo
• Unknown Immunization status • Albendazole
o Assume not immunized and proceed with regular schedule o 12mos to 23mos – 200mg single dose Q6mo
o 24mos and up – 400mg single dose Q6mo
✔ GUIDE QUESTION • Mebendazole
A 10-month-old infant had measles, what will you give in an exposed o 12months and above – 500mg single dose Q6mo
unvaccinated child to prevent measles? • Contraindications to Deworming
A. Vitamin A o Severe malnutrition
B. Vaccinate immediately
o High grade fever
C. Ribavirin within the first 72 hours
D. Amoxicillin clavulanate
o Profuse diarrhea
o Abdominal Pain
o Serious illness
o Hypersensitivity to anti-helminthic drug
TOPNOTCH MEDICAL BOARD PREP PEDIATRICS MAIN DIGITAL HANDOUT BY DR. PUNONGBAYAN AND DR. DE VERA Page 21 of 105
For inquiries visit www.topnotchboardprep.com.ph or https://www.facebook.com/topnotchmedicalboardprep/
This handout is only valid for the March 2021 PLE batch. This will be rendered obsolete for the next batch since we update our handouts regularly.
TOPNOTCH MEDICAL BOARD PREP PEDIATRICS MAIN DIGITAL HANDOUT BY DR. PUNONGBAYAN AND DR. DE VERA
For inquiries visit www.topnotchboardprep.com.ph or https://www.facebook.com/topnotchmedicalboardprep/
This handout is only valid for the March 2021 PLE batch. This will be rendered obsolete for the next batch since we update our handouts regularly.
✔ GUIDE QUESTION
INTEGRATED MANAGEMENT OF CHILDHOOD A patient is admitted for diarrhea and was diagnosed to have severe
ILLNESSES dehydration, what is the appropriate fluid to use for this patient
EXCEPT?
LETHARGY A. ORS
UNCONSCIOUSNESS B. Plain Lactated Ringer’s Solution
C. Plain NaCl
D. D5 0.9 NaCl

COMPOSITION OF THE OLD AND REFORMULATED ORS:


VOMITING
DANGER CONVULSIONS OLD WHO- Reformulated

SIGNS ORS (meq ORS (meq or


or mmol/l) mmol/l)
Glucose 111 75
Sodium 90 75
Chloride 80 65
INABILITY TO DRINK Potassium 20 20
OR BREASTFEED Citrate 10 10
Osmolarity 311 245
This is very important. ORS is one of the most important managements
DEHYDRATION for AGE. Most diarrhea in children is viral. It will resolve spontaneously.
Reformulated ORS has lower osmolarity thus less chance of actually
https://qrs.ly/zmbj8ch pulling water out into the lumen. The glucose:sodium ratio is also
modified 1:1. This is to utilize the Na-Gluc co-transporter. J
Dr. De Vera

✔ GUIDE QUESTION
A toddler was brought to you for multiple bouts of watery diarrhea TREATMENT OF > 6 MONTHS OLD
without vomiting. He is tachycardic, with weak pulses, reduced skin WITH ACUTE DIARRHEA:
turgor, and sunken eyes. He is lethargic but irritable to touch. He gulps • Duration: MD-10.44 hrs (-21.13 to 0.25)
vigorously the oral rehydration solution offered to him. What is your
o modestly decreased the duration by half a day
assessment?
A. No dehydration • Acute diarrhea persisting beyond 7 days: RR 0.73 (0.61 to 0.88)
B. Some dehydration o likelihood of prolonged diarrhea is decreased by 30%
C. Severe dehydration • Duration if with Moderate Malnutrition: MD -26.98 hrs (-14.92
D. Dehydration with danger signs to -39.34)
o decreased the duration by more than a day
SIGNS CLASSIFICATION TREATMENT
Two of the ROLE OF SUPPLEMENTAL ZINC THERAPY:
following signs: • Plan C • Reduction in the severity and duration of diarrhea:
• abnormally • Refer urgently to o 25% reduction of diarrhea
sleep or difficult the hospital o 30% reduction in stool volume
to awaken Severe • If child is 2 years • Prevention of subsequent episodes of diarrhea:
• Sunken eyes dehydration old and there is o 10-20 mg of Zn per day for 10-14 days reduces the number of
• Not able to drink cholera in the diarrheal episodes in the 2-3 months after supplementation
or drinks poorly area, give
✔ GUIDE QUESTION
• Skin pinch goes Tetracycline
A 4-year-old child is currently being treated with Cefuroxime for
back slowly Pneumonia but is not improving. If you suspect that he has Mycoplasma
Two of the infection, you expect him to improve with:
following signs: • Plan B A. Cefuroxime for 1 more week
• Restless irritable Some • Continue feeding B. Azithromycin
• Sunken eyes dehydration • Follow up in 5 C. Ceftriaxone
D. Vancomycin
• Drinks eagerly, days
thirsty PLEASE READ MORE ON:
• Give fluid and • Updates on IMCI
Not enough signs food
to classify as some • Plan A
or severe
No dehydration
• Follow up in 5
GROWTH AND DEVELOPMENT
dehydration days if not
improving
• Treat for 5 days
with Co-
Blood in stool Dysentery trimoxazole and
return after 2
days
Amount given Amount of ORS to
PLAN A after each loose provide for use at
stool home
< 24 mo old 50-100mL 500/day
2-10 y/o 100-200mL 1000/day
As much as Much like immunizations, G&D entails memorization. Understanding the
> 10 y/o 2000/day principles below is VERY IMPORTANT!
wanted Dr. De Vera

Amount needed = PRINCIPLES IN GROWTH AND DEVELOPMENT


PLAN B • Weight (g) x 0.075 • a continuous process
• Weight (kg) x 75 • Progresses in a cephalocaudal manner & proximodistal
pattern
FIRST, give THEN 70mL/kg
PLAN C • Responses proceed from generalized reflexes to discrete
30mL/kg for: for:
voluntary actions
< 12 mo old 1 hr 5 hrs
• Developmental sequence is orderly and predictable
> 12 mo old 30 mins 2 ½ hrs
• Rate varies from child to child
TOPNOTCH MEDICAL BOARD PREP PEDIATRICS MAIN DIGITAL HANDOUT BY DR. PUNONGBAYAN AND DR. DE VERA Page 22 of 105
For inquiries visit www.topnotchboardprep.com.ph or https://www.facebook.com/topnotchmedicalboardprep/
This handout is only valid for the March 2021 PLE batch. This will be rendered obsolete for the next batch since we update our handouts regularly.
TOPNOTCH MEDICAL BOARD PREP PEDIATRICS MAIN DIGITAL HANDOUT BY DR. PUNONGBAYAN AND DR. DE VERA
For inquiries visit www.topnotchboardprep.com.ph or https://www.facebook.com/topnotchmedicalboardprep/
This handout is only valid for the March 2021 PLE batch. This will be rendered obsolete for the next batch since we update our handouts regularly.
DEVELOPMENTAL MILESTONES ✔ GUIDE QUESTION
• Though an age level is assigned to each milestone, the actual age A mother brought in her 8-month old daughter and was concerned that
at which a milestone is achieved by a normal child may range her child has no erupted teeth yet. Her daughter had unremarkable
birth and maternal history and is otherwise well. What will you advise
within 2 months of this age level in the first year of life to 4
the mother?
months from the second year on. A. work up for congenital hypothyroidism as this may manifest
with delayed tooth eruption
DELAY B. This is definitely abnormal because at 8months the child should
• Child’s development is not as advanced as it should be have at least 2 teeth
• the rate of development has been slower than what is usually C. Request for a panoramic x-ray
D. Observe and re-assure the mother
acceptable
✔ GUIDE QUESTION ALL ABOUT THE TEETH
A 5-month-old baby was brought in for consult. The child was born full • # 3G *&&*ℎ = 2(& '- 03-*ℎ1 − 6
term and his birth and maternal history was unremarkable. His birth • Suspect thyroid disorder if no teeth by 13 mos
weight is 2900g. Based on your knowledge in growth and development, • Usually begins at 6 months
you expect the child’s weight at present to be? = b- Mo ✗ 6001-21900 • Central mandibular incisors
0A. 5,800 C. 4,000g
• All 20 primary teeth have erupted by 3 years old
B. 7,000g D. 9,000g 3000 t 2,900
=

=
5,900
ANTHROPOMETRICS FIRST YEAR OF LIFE
Weight GROWTH
* 1-6 months • Objective assessment: weight, length, head circumference
%&'(ℎ* '- (./01 = 2(& '- 03-*ℎ1 × 566 + !% • Average newborn: 3 kilos or 6 lbs.
• Babies double their birth weight during the 4th-5th months and
7-11 months triple at 1 year old
%&'(ℎ* '- (./01 = 2(& '- 03-*ℎ1 × 866 + !% • Average birth length: 50 cms
• Average length at 1 year old: 75 cms.
1-6 years old • Average head circumference at birth: 33-35 cms.
%&'(ℎ* '- 9: = 2(& '- ;<=>? × 2 + 8 • Average HC at 1 year old: 45 cms.
Take note that BW is doubled at 4 months and tripled at 1 year DEVELOPMENTAL MILESTONES
Dr. De Vera

Length and Height


0-3 mos 9 cm
4-6 mos 8 cm
6-9 mos 5 cm
10-12 mos 3 cm
Ave gain in length during
25 cm
1st year

GROWTH PARAMETERS
• Mnemonics for length / height:
o +* '- B01 = 2(& '- ;<=>? × 5 + 80
o +* '- '-Bℎ&1 = 2(& '- ;<=>? × 2 + 32
DEVELOPMENTAL
SUPPLEMENT: QUICK SHEET MILESTONES
• At 2 yrs old: 1/2 mature height for boys https://qrs.ly/fgbj8bz
• At 3 yrs old: 3 feet tall
• At 4 yrs old: 2x the birth length GROSS & FINE MOTOR SKILLS: 1 YEAR OLD
• At 13 yrs old: 3x the birth length • 10 mos: sits up alone and • 8 mos: pincer grasp
indefinitely without • 12 mos: turns pages of
HEAD CIRCUMFERENCE support with back straight; book; releases object to
• at birth 33-35 cm pulls to stand another person on request
• taken up to 3 years old • 12 mos: rises • 13 mos: scribbles
• Attained 90% of adult head size by age 2 independently; takes • 15 mos: builds tower of 2-3
• approximates adult head at 6 years old several steps cubes; makes a line with
Remember! You get a big head first! • 15 mos: walks alone; crawls crayon
Dr. De Vera up stairs • 18 mos: imitates vertical
• 18 mos: runs stiffly; walks line; tower of 4 cubes
upstairs with one hand held
RECEPTIVE & EXPRESSIVE LANGUAGE: 1 YEAR OLD
• 1.5 mos: smiles in response • 1 mo: throaty gurgling
to face & voice sounds
• 2 mos: smiles on social • 2 mos: vocalizes, coos
contact • 3 mos: says “aah”
• 3 mos: turns head to • 4 mos: laughs aloud
sound/voice • 5 mos: “ah-goo”
• 6 mos: looks to see where • 6 mos: imitates sounds;
dropped toy is babbles: “da-da, ba-ba, pa-
• 7 mos: responds to name- pa”
Nelson Textbook of Pediatrics, 20 ed.th calling; inhibits to “no”; • 10 mos: says “mama”,
Changes in body proportions. Approximate changes in body proportions follows 1-step command “dada”
from fetal life through adulthood are shown. with gesture • 12 mos: speaks first real
• 10 mos: follows 1-step word; 2-word vocabulary
command w/o gesture • 15 mos: jargon
• 18 mos: points to body • 18 mos: speaks 10-15
parts words; names pictures
TOPNOTCH MEDICAL BOARD PREP PEDIATRICS MAIN DIGITAL HANDOUT BY DR. PUNONGBAYAN AND DR. DE VERA Page 23 of 105
For inquiries visit www.topnotchboardprep.com.ph or https://www.facebook.com/topnotchmedicalboardprep/
This handout is only valid for the March 2021 PLE batch. This will be rendered obsolete for the next batch since we update our handouts regularly.
TOPNOTCH MEDICAL BOARD PREP PEDIATRICS MAIN DIGITAL HANDOUT BY DR. PUNONGBAYAN AND DR. DE VERA
For inquiries visit www.topnotchboardprep.com.ph or https://www.facebook.com/topnotchmedicalboardprep/
This handout is only valid for the March 2021 PLE batch. This will be rendered obsolete for the next batch since we update our handouts regularly.

2-5 YEARS OLD

Mnemonics above shows what a child can draw. Circle at 3 y/o, cross at
4, square at 5, triangle at 6yo.
Dr. De Vera

Nelson Textbook of Pediatrics, 20th ed.

SUPPLEMENT: QUICK SHEET FULLY


REFLEX ONSET DURATION
OTHER IMPORTANT STUFF TO REMEMBER DEVELOPED
• Handedness usually at 3 years old Palmar 28 wk 32 wk 2-3 mo
• Bed wetting grasp gestation gestation postnatal
o Up to 4 y/o in Girls Less prominent
32 wk 36 wk
o Up to 5 y/o in Boys Rooting after 1 mo
gestation gestation
• Ties shoes at 6 y/o postnatal
• Language explosion at 2 y/o (20-50 words by end of 2nd year) 28-32 wk 37 wk 5-6 mo
• Interest in Basic sexuality at 4-5 years old Moro
gestation gestation postnatal

✔ GUIDE QUESTION 35 wk 1 mo 6-7 mo


Tonic Neck
Among the following reflexes, which normally persist throughout life? gestation postnatal postnatal
A. Moro reflex
B. Rooting 7-8 mo 10-11 mo Remains
Parachute
C. Tonic Neck reflex postnatal postnatal throughout life
D. Parachute
E. Sucking reflex

TOPNOTCH MEDICAL BOARD PREP PEDIATRICS MAIN DIGITAL HANDOUT BY DR. PUNONGBAYAN AND DR. DE VERA Page 24 of 105
For inquiries visit www.topnotchboardprep.com.ph or https://www.facebook.com/topnotchmedicalboardprep/
This handout is only valid for the March 2021 PLE batch. This will be rendered obsolete for the next batch since we update our handouts regularly.
TOPNOTCH MEDICAL BOARD PREP PEDIATRICS MAIN DIGITAL HANDOUT BY DR. PUNONGBAYAN AND DR. DE VERA
For inquiries visit www.topnotchboardprep.com.ph or https://www.facebook.com/topnotchmedicalboardprep/
This handout is only valid for the March 2021 PLE batch. This will be rendered obsolete for the next batch since we update our handouts regularly.

REFLEXES
• Moro reflex is elicited by supporting the infant in a semi-erect
position and then allowing the infant’s head to fall backwards
onto the examiner’s hand
o normal = symmetric extension and abduction of the fingers
and upper extremities, followed by flexion of the upper
extremities and an audible cry
o asymmetric response = fractured clavicle, brachial plexus
injury, or hemiparesis
o No response = significant CNS dysfunction
• Grasp response is elicited by placing a finger in the open palm
of each hand
o 37 wk of gestation, strong enough that the examiner can lift the
infant from the bed with gentle traction
• Tonic neck reflex is produced by manually rotating the infant’s
head to 1 side and observing for the characteristic fencing
posture
• Parachute reflex
o slightly older infants
o evoked by holding the infant’s trunk and then suddenly
lowering the infant as if he or she were falling
o The arms will spontaneously extend to break the infant’s fall
o this reflex a prerequisite to walking Nelson Textbook of Pediatrics, 20th ed.

AGE OF AGE OF
REFLEX
APPEARANCE DISAPPEARANCE AUTISM SPECTRUM DISORDER (DSM V)
Rooting 32 weeks AOG 1 month • Encompasses the following:
Palmar 28 weeks AOG 2-3 months 1. Autistic disorder (autism)
Placing/Stepping 37 weeks AOG 4-5 months 2. Asperger disorder
28-32 weeks 3. Childhood disintegrative disorder
Moro 5-6 months 4. Rett disorder
AOG
Tonic neck 5. Pervasive developmental disorder NOS
35 weeks AOG 6-7 months • Characterized by deficits in 2 core domains:
reflex
Plantar 11 weeks AOG 7-9 months 1. Deficits in social communication and social interaction
Covered up by 2. Restricted repetitive patterns of behavior, interests, and
Landau 3 months activities
voluntary action
Covered up by These are the 2 most important things to remember in ASD. Please
Parachute reflex 4-9 months remember this 2 core domain deficits.
voluntary action Dr. De Vera
Please memorize this table. This shows at what age primitive reflexes are
expected to disappear. Remember, primitive reflexes should disappear in AUTISM SPECTRUM DISORDER
order for voluntary actions to happen. For patients with cerebral palsy,
it is common to find persistence of these reflexes. A. Persistent deficits in social communication & social interaction
Dr. De Vera 1. Deficits in social-emotional reciprocity (failure of normal
back-and-forth conversation; reduced sharing of emotions,
interests, or affect; failure to initiate or respond to social
interactions)
2. Deficits in nonverbal communicative behaviors used for
social interaction (abnormal eye contact and body
language, deficit in understanding and use of gestures; total
lack of facial expressions)
3. Deficits in developing, maintaining, and understanding
relationships (absence of interest in peers, difficulties in
making friends)
B. Restricted, repetitive patterns of behavior, interests, or
activities as manifested by at least 2 of the following currently
or by history:
1. Stereotyped or repetitive motor movements, use of
objects, or speech (lining up of toys, echolalia, flipping
objects)
2. Insistence on sameness, inflexible adherence to routines,
or ritualized patterns of verbal or nonverbal behavior
(extreme distress at small changes, rigid thinking patterns,
difficulties with transition, need to take same route or eat
same food every day)
✔ GUIDE QUESTIONS
3. Highly restricted, fixated interests that are abnormal in
A 5-year-old male was brought in for consult due to poor school
intensity or focus (preoccupation with unusual objects,
performance. The child was noted to have poor participation in
group activities. Most of the time the child would not look at you perseverative interests)
when he/she is being talked to. He was noted to be mostly busy 4. Hyper- or hypo- reactivity to sensory input or unusual
arranging his favorite toy cars. Upon further history there was note interest in sensory aspects of the environment (apparent
of developmental delay in verbal and cognitive domains. This is indifference to pain/temperature, excessive smelling or
probably a case of? touching of objects, visual fascination with lights or
A. ADHD movement, adverse reaction to sounds/textures)
B. Global developmental delay C. Symptoms must be present in the early developmental period
C. Autism Spectrum Disorder
D. Symptoms cause clinically significant impairment in social,
D. Generalized Epilepsy
The average hour of sleep of a 3-month-old baby is?
occupational, or other important areas of current functioning.
A. 15 E. These disturbances are not better explained by intellectual
B. 15.5 disability or global developmental delay.
C. 16
D. 16.5
TOPNOTCH MEDICAL BOARD PREP PEDIATRICS MAIN DIGITAL HANDOUT BY DR. PUNONGBAYAN AND DR. DE VERA Page 25 of 105
For inquiries visit www.topnotchboardprep.com.ph or https://www.facebook.com/topnotchmedicalboardprep/
This handout is only valid for the March 2021 PLE batch. This will be rendered obsolete for the next batch since we update our handouts regularly.
TOPNOTCH MEDICAL BOARD PREP PEDIATRICS MAIN DIGITAL HANDOUT BY DR. PUNONGBAYAN AND DR. DE VERA
For inquiries visit www.topnotchboardprep.com.ph or https://www.facebook.com/topnotchmedicalboardprep/
This handout is only valid for the March 2021 PLE batch. This will be rendered obsolete for the next batch since we update our handouts regularly.
The best-established prognostic factors for individual GASTROENTEROLOGY
outcome are:
✔ GUIDE QUESTIONS
• presence or absence of associated intellectual disability
Cholelithiasis in children is usually due to:
• language impairment A. Abnormal lipid metabolism
o Currently, as many as 15% of ASD are associated with a known B. Cholesterolosis
genetic mutation; males > females (4:1) C. Hemolytic dyscrasia
D. Abnormal calcium deposition
ATTENTION-DEFICIT / HYPERACTIVITY DISORDER A 3-month old baby boy was brought for jaundice. Patient was noted
to develop jaundice on the 10th day of life. He was exclusively breast
(ADHD) fed and daily sun exposure was done. Mother reported the dark
• DSM V changes: staining urine in the diaper and whitish stool which she thought were
o Age at onset: several inattentive or hyperactive-impulsive normal. The child is well, no fever, has good appetite and activity.
symptoms were present prior to age 12 Family history is unremarkable, patient is full term. On physical
o a comorbid diagnosis with ASD is now allowed examination there is generalized jaundice and hepatomegaly. Bilirubin
was done Total 15mg/dl, direct 10mg/dl, indirect is 5mg/dl. The
• A neurodevelopmental disorder defined by impairing levels of
following are possible differential diagnosis except?
inattention, disorganization, and/or hyperactivity-impulsivity; A. Neonatal Hepatitis
males > females (2:1) B. Biliary Atresia
• Inattention and disorganization: unable to stay on task, seeming C. Alagille Syndrome
not to listen, losing materials at levels that are inconsistent with D. Breastfeeding Jaundice
age or developmental level If you are suspecting biliary atresia, what is the gold standard for
• Hyperactivity-impulsivity: overactivity, fidgeting, inability to diagnosis of this disease?
stay seated, intruding into other people’s activities, inability to A. HIDA Scan
B. Ultrasound
wait
C. Biopsy
• Symptoms are excessive for age and developmental level D. Direct Cholangiography
• Frequently overlaps with oppositional defiant disorder and
conduct disorder
If ASD is characterized by deficit in social communication / interaction + NEONATAL CHOLESTASIS
restricted repetitive behavior, ADHD also has 2 dominant features to • prolonged elevation of the serum levels of conjugated bilirubin
remember, Inattention / disorganization + hyperactivity-impulsivity. beyond the 1st 14 days of life
Dr. De Vera
• Mechanical Obstruction of bile flow
A. Inattention (6 or more of the ff:) o Biliary Atresia
1. Often fails to give close attention to details or makes • Functional impairment of hepatic excretory function and bile
careless mistakes in school secretion
2. Often has difficulty sustaining attention in tasks or play
activities
3. Often does not seem to listen when spoken to directly
4. Often does not follow through on instructions and fails to
finish schoolwork
5. Often has difficulty in organizing tasks
6. Often avoids, dislikes or is reluctant to engage in tasks
that require sustained mental effort
7. Often loses things necessary for tasks
8. Often easily distracted by external stimuli
9. Often forgetful in daily activities
B. Hyperactivity and impulsivity (6 or more of the ff:)
1. Often fidgets with hands or feet or squirms in seat
2. Often leaves seat in classroom
3. Often runs about or climbs excessively
4. Often has difficulty playing or engaging in leisure
activities quietly
5. Often “on the go”
6. Often talks excessively
7. Often blurts out answers before questions have been
Nelson Textbook of Pediatrics, 20th ed.
completed
8. Often has difficulty awaiting turn
9. Often interrupts or intrudes on others BILIARY ATRESIA
*** SEVERAL symptoms were present before 12 years of age. • a.k.a. Non-cystic Obliterative Cholangiopathy
C. Several inattentive or hyperactive-impulsive symptoms are • 2 types: Fetal and Perinatal
present in 2 or more settings (ex: at home, school, or work; • Most common form (85%) – obliteration of the entire
with friends or relatives; in other activities) extrahepatic biliary tree at or above the porta hepatis
D. There is clear evidence that symptoms interfere with, or • Most are normal at birth with postnatal progressive obliteration
reduce the quality of, social, academic, or occupational of bile ducts
functioning. • Post-natal onset is usually immune or infection mediated
E. The symptoms do not occur exclusively during the course of
schizophrenia or another psychotic disorder and are not BILIARY ATRESIA VS. NEONATAL HEPATITIS
better explained by another mental disorder.
• Difficult to differentiate
MANAGEMENT • No single biochemical test
• Therapy provided by: • Ultrasound
1. Pediatrician o Not diagnostic
2. Clinical psychologist o Important to rule out other surgical causes of cholestasis (e.g.
3. Occupational therapist Choledochal cyst)
4. Parents o Triangular Cord sign – cone-shaped fibrotic mass cranial to the
5. Other members of the family (siblings, grandparents, bifurcation of the portal vein, may be seen in patients with
caregivers) biliary atresia
• Hepatobiliary scintigraphy – sensitive but not specific
PLEASE READ MORE ON: • Liver biopsy – important to discriminate between the various
causes of cholestasis
• Specific developmental milestones especially during the first
• Cholangiography – gold standard. To determine the presence
and second years of life – Memory work
and site of obstruction
TOPNOTCH MEDICAL BOARD PREP PEDIATRICS MAIN DIGITAL HANDOUT BY DR. PUNONGBAYAN AND DR. DE VERA Page 26 of 105
For inquiries visit www.topnotchboardprep.com.ph or https://www.facebook.com/topnotchmedicalboardprep/
This handout is only valid for the March 2021 PLE batch. This will be rendered obsolete for the next batch since we update our handouts regularly.
TOPNOTCH MEDICAL BOARD PREP PEDIATRICS MAIN DIGITAL HANDOUT BY DR. PUNONGBAYAN AND DR. DE VERA
For inquiries visit www.topnotchboardprep.com.ph or https://www.facebook.com/topnotchmedicalboardprep/
This handout is only valid for the March 2021 PLE batch. This will be rendered obsolete for the next batch since we update our handouts regularly.

BILIARY TREATMENT
NEONATAL HEPATITIS • Fluid & electrolyte replacement
ATRESIA
Location • Extrahepatic • Intrahepatic • When do you use antibiotics?
1. To shorten the duration of illness
Familiar
• Unlikely • 20% 2. To reduce period of excretion of the organism
Incidence
3. To decrease the requirements for fluid therapy
Premature
• Uncommon • Common
and SGA
DRUGS USED IN CHOLERA
Persistent
• Tetracycline 12.5 mg/kg/dose 4x day PO qid for 3 days (max 2
acholic • Yes • +/-
g/day); not for <9 yrs old; or
stool
• Doxycycline 5 mg/kg PO single dose (max 200 mg/day)
• Bile duct • Severe, diffuse
• Alternatives: TMP-SMX 8-10 mg/kg/day bid PO; Erythromycin
proliferation, hepatocellular disease,
40 mg/kg/day every 6 hours max. 2g/day; or Furazolidone 5-8
bile plugs, distortion of lobular
mg/kg/day max 400 mg/day
portal or architecture, marked
perilobular infiltration with
Biopsy ✔ GUIDE QUESTIONS
edema and inflammatory cells and
What is the mechanism behind the bleeding diarrhea in shigella?
fibrosis, focal hepatocellular A. exotoxin
INTACT basic necrosis. B. invasion of the intestinal epithelium
hepatic lobular • BILE DUCT SHOWS LESS C. increased cGMP
architecture ALTERATION D. ADP ribosylation
A 10-year-old boy presents with 9 days intermittent fever with Tmax =
✔ GUIDE QUESTIONS 39.2°C, malaise, and soft stools. 3 days PTC, he had on & off abdominal
The most common cause of viral gastroenteritis in infants is: pain which increased in intensity. Coherent, tachycardic, febrile, no
A. Rotavirus C. Respiratory syncytial virus TPC, clear breath sounds, flat abdomen, hyperactive bowel sounds,
B. Norwalk virus D. Parvovirus B19 tenderness in the periumbilical area, full pulses. If you want to isolate
By definition acute diarrhea DOES NOT last longer than ___days? the causative organism, the best specimen for sample collection would
A. 11 C. 13 be?
B. 12 D. 14 A. Blood C. Stool
B. Urine D. All of the above

ACUTE DIARRHEA ENTERIC FEVER


• excessive loss of fluid and electrolyte in the Stool • Ingestion of food or water contaminated with S. typhi from human
• sudden onset of excessively loose stools of >10 mL/kg/day in feces – most common mode of transmission
infants and >200 g/24 hr in older children, which lasts <14 days
• Period of communicability: throughout the duration of fecal
• > 14 days = chronic or persistent diarrhea excretion
• Incubation period: 7-14 days
o More dramatic presentation in <5 years old
DIARRHEA
WHAT ARE THE SIGNS/SYMPTOMS OF TYPHOID FEVER?
https://qrs.ly/xvbj8da • High-grade fever, malaise, myalgia, cough, abdominal pain,
hepatosplenomegaly, anorexia, diarrhea / constipation
• Maculopapular rashes (rose spots) in 25% - visible on day 7-10
Patients presenting with non-inflammatory diarrhea of illness on the lower chest or abdomen and lasts 2-3 days
Day care center • If no complications occur, the symptoms & physical findings
Rotavirus, ETEC gradually resolve within 2-4 weeks
Infants and Toddlers
Profuse diarrhea and Typhoid is one of your great mimickers. A clue would be prolonged fever
vomiting + weird signs and symptoms (may present with UTI, pneumonia, GI
Cholera symptoms, myocarditis etc.)
Flecks of mucous on Dr. De Vera
voluminous diarrhea
Watery Profuse diarrhea after WHAT ARE THE COMPLICATIONS?
Vibrio
diarrhea, eating raw oysters or
parahaemolyticus • Intestinal hemorrhage (<1%) and perforation (0.5-1%) is
vomiting undercooked shellfish infrequent in children.
Greasy stool after • If with perforation: RLQ pain, tenderness, vomiting, features of
camping Giardiasis
peritonitis
Gas
• Rare: toxic myocarditis, delirium, increased ICP, pyelonephritis,
Water diarrhea and
meningitis, endocarditis
vomiting after history ETEC
of travel PROGNOSIS
• Despite appropriate Tx, 2-4% may relapse after initial clinical
CHOLERA response to treatment.
PATHOGENESIS • Those who excrete S. typhi for more than 3 months are chronic
• Colonization of upper small intestine → produces enterotoxin → carriers.
promotes secretion of fluids & electrolytes into the lumen of the • Risk for becoming a carrier is <2% for all infected children.
SI → enterotoxin’s B subunit binds to GM1 ganglioside receptor
→ allows A subunit to enter cells → activates adenylate cyclase
→ ↑ cAMP → blocks NaCl absorption → promotes secretion of Cl
& H2O by crypt cells → massive outpouring of isotonic fluid into
the SI → overwhelms the absorptive capacity of the LI → severe
diarrhea

TOPNOTCH MEDICAL BOARD PREP PEDIATRICS MAIN DIGITAL HANDOUT BY DR. PUNONGBAYAN AND DR. DE VERA Page 27 of 105
For inquiries visit www.topnotchboardprep.com.ph or https://www.facebook.com/topnotchmedicalboardprep/
This handout is only valid for the March 2021 PLE batch. This will be rendered obsolete for the next batch since we update our handouts regularly.
TOPNOTCH MEDICAL BOARD PREP PEDIATRICS MAIN DIGITAL HANDOUT BY DR. PUNONGBAYAN AND DR. DE VERA
For inquiries visit www.topnotchboardprep.com.ph or https://www.facebook.com/topnotchmedicalboardprep/
This handout is only valid for the March 2021 PLE batch. This will be rendered obsolete for the next batch since we update our handouts regularly.
TREATMENT OF TYPHOID FEVER

Nelson Textbook of Pediatrics, 20th ed.

INFLAMMATORY DIARRHEA/DYSENTERY SHIGELLOSIS


Patients presenting with PATHOGENESIS
inflammatory diarrhea/dysentery • Basic virulence trait: ability to invade intestinal epithelial cells
Trophozoites with ingested • Pathologic changes occur in the colon
Amoebiasis
RBC’s • Shigella cross the colonic epithelium through M cells overlying
Abdominal cramps, systemic Clostridium the Peyer patches
toxicity after antibiotic use difficile • S. dysenteriae serotype 1 produce Shiga toxin, a potent protein
Bloody Abdominal cramps, tenesmus, synthesis-inhibiting exotoxin causes HUS.
Shigella
diarrhea abundant pus and WBC in stool
High fever, headache, FACTS ABOUT SHIGELLA
Pus and drowsiness, confusion, • Incubation period: 12 hrs to several days before symptoms ensue
WBC in meningismus, seizures, • Period of communicability: as long as the person excretes the
Salmonella
stool abdominal distention bacteria (1-4 wks)
History of eating eggs, poultry, • Symptoms: severe abdominal pain, high fever, emesis, anorexia,
unpasteurized milk generalized toxicity, painful defecation, urgency
Diarrhea with blood after • PE findings: abdominal distention & tenderness, hyperactive
EHEC
eating hamburger bowel sounds, tender rectum on digital exam

Patients presenting with WHAT TO EXPECT IN SHIGELLOSIS:


inflammatory diarrhea/dysentery • Watery voluminous diarrhea initially evolving into frequent
Ham, potato salad, cream small-volume, bloody mucoid stools → significant dehydration
Staphylococcus
Abdominal pastries • Neurologic manifestations (40%) but the cause is not
cramps Reheated fried rice Bacillus cereus understood
Diarrhea Home-canned food • Mortality rate is high (20%) when sepsis occurs
Sweating Muscle weakness
Botulism
No fever Diplopia DIFFERENTIAL DIAGNOSIS:
Blurring of vision • Campylobacter jejuni
• Salmonella
WHAT ARE THE 5 MAJOR GROUPS OF E. COLI? • Enteroinvasive E.coli (EIEC)
• ETEC - infantile explosive diarrhea with dehydration; few or no • Entamoeba histolytica
structural changes in the gut mucosa • Clostridium difficile
• EIEC – cause colonic lesions like dysentery • Yersinia enterocolitica
• EPEC – non-bloody diarrhea with mucus; prolonged
• EAEC- significant dehydration; prolonged diarrhea DIAGNOSTIC TESTS:
• EHEC - shiga-toxin producing E.coli; bloody diarrhea • Presumptive dx: fecal leukocytes (>50-100 PMNS/hpf) & fecal
blood; ↑ WBC in CBC
✔ GUIDE QUESTION • Definitive dx: culture of stool & rectal swab specimen
An 8 year old male child was brought to the clinic for diarrhea with (MacConkey agar, xylose-lysine deoxycholate, SS agar)
bloody watery stools. He had 3 episodes of diarrhea today which o use clinical judgment in the management of clinical
started suddenly accompanied by fever which was checked at 38.5°C, syndromes consistent with shigellosis
he had 2 episodes of vomiting and intermittent abdominal pain, the boy
also complains of painful defecation, the stools are characterized as
TREATMENT
initially watery then becoming bloody with mucus, no family members
or close friends with the same illness was noted, which of the ff. is the • Empirical therapy:
most likely diagnosis? o Cefixime 8 mg/kg/day po every 12 hrs for 5 days;
A. Rotavirus C. Vibrio cholera o Ceftriaxone 50 mg/kg/day IV or IM once a day for 5 days; or
B. Shigella D. lactose intolerance o Azithromycin 12 mg/kg/day po 1st day followed by 6
mg/kg/day for the next 4 days

TOPNOTCH MEDICAL BOARD PREP PEDIATRICS MAIN DIGITAL HANDOUT BY DR. PUNONGBAYAN AND DR. DE VERA Page 28 of 105
For inquiries visit www.topnotchboardprep.com.ph or https://www.facebook.com/topnotchmedicalboardprep/
This handout is only valid for the March 2021 PLE batch. This will be rendered obsolete for the next batch since we update our handouts regularly.
TOPNOTCH MEDICAL BOARD PREP PEDIATRICS MAIN DIGITAL HANDOUT BY DR. PUNONGBAYAN AND DR. DE VERA
For inquiries visit www.topnotchboardprep.com.ph or https://www.facebook.com/topnotchmedicalboardprep/
This handout is only valid for the March 2021 PLE batch. This will be rendered obsolete for the next batch since we update our handouts regularly.
• WHO recommendation for all patients with bloody diarrhea DIAGNOSIS
irrespective of age: Ciprofloxacin 30 mg/kg/day in 2 divided • Plain abdominal X-
doses ray: (+) density
• Zinc 20 mg/day for 14 days improves immune response to • Barium enema:
Shigella coiled-spring sign
Again, let’s make things simple. (thin rim of barium
Acute diarrhea in children = virus unless proven otherwise trapped around the
Acute bloody diarrhea = shigella NOT amoebiasis (<30%, look for E. invaginating part
histolytica TROPHOZOITES, NOT cyst) within the
Dr. De Vera
intussuscipiens;
edematous mucosal
✔ GUIDE QUESTIONS folds of the
A 1-and-a-half-year-old male baby was brought into the clinic due to returning limb of
crying spells accompanied by knee flexion. There are intermittent
intussusceptum
episodes that he feels well. The patient reportedly has vomited once
during an episode of abdominal pain, vomitus consists of previously
outlined by the
ingested food and sometimes clear watery fluid. There is poor appetite. contrast)
Which among the ff. should be asked? • Ultrasound: tubular
A. any recent change in diet mass & a doughnut
B. any illness in the family or target sign (98-
C. any blood in the stool 100% sensitivity) Ultrasound finding: target sign
D. any bulging of the umbilicus
In relation to the case above, what is the initial diagnostic test of choice
for this patient? MODALITY ADVANTAGES DISADVANTAGES
A. Barium enema
B. Ultrasound of the abdomen
• Success rate under • Recurrent cases
C. Upper GI endoscopy Hydrostatic fluoroscopic or noted in 5-8%
D. Abdominal CT scan (saline ultrasonic (more common in
In relation to the case above, what is the most likely finding on workup? reduction) guidance in 80- hydrostatic
A. Double bubble sign 85% reduction)
B. Increased pyloric thickness
C. Donut ring sign
• Fewer
D. Absence of parasympathetic postganglionic cell bodies Air complications and • Perforation rate (0-
reduction lower radiation 1-0.2%)
exposure
PRESENTING AS INTESTINAL OBSTRUCTION
Barium and
• Bowel perforation
hydrostatic
INTESTINAL reduction
at 0.5-2.5%
OBSTRUCTION
qrs.ly/vabj8b1 ✔ GUIDE QUESTION
A 2 month-old boy was noted to have vomiting after every feeding for
the past week. There were no other symptoms noted. PE: irritable,
SUPPLEMENT: QUICK SHEET slightly tachycardic, no retractions, clear breath sounds, (+) firm,
THINGS TO ASK TO RULE OUT OBSTRUCTION: movable 2x2 cm mass on the abdomen. What is your impression?
A. Duodenal Atresia C. Malrotation
1. Onset and duration – early onset vomiting in the first few
B. Pyloric Stenosis D. Intussusception
days of life is almost always obstructive (i.e. duodenal
atresia)
2. Bilious or non-bilious? – bilious is commonly associated PYLORIC STENOSIS
with obstruction
3. Abdominal distention?
4. BM or flatus present? – vomiting without BM may point to
obstruction. AGE can initially present with vomiting
followed by LBM. However, if patient has been vomiting for
several days without BM, think obstruction

INTUSSUSCEPTION
• Occurs when a portion of the alimentary tract is telescoped into
an adjacent segment
• Most common cause of intestinal obstruction between 3 months
- 6yrs.old; M > F
• Unknown cause in most cases
• Correlation with Adenovirus

CLINICAL MANIFESTATIONS
• Severe paroxysmal colicky pain that recurs at frequent intervals DIAGNOSTIC TESTS
with straining efforts • Ultrasound is confirmatory: pyloric thickness >4mm or length
• legs & knees are flexed with loud crying >14mm
• 60% of infants pass currant jelly stool • Barium: elongated pyloric channel, a bulge of the pyloric muscle
into the antrum (shoulder sign) & streaks of barium in the
PE FINDINGS narrowed channel (double tract sign)
• Palpation of abdomen: slightly tender sausage-shaped mass in
the RUQ which may increase in size & firmness during a ✔ GUIDE QUESTION
paroxysm of pain A 48-hr old boy born 34 wks AOG was noted to vomit greenish material
• Child looks well in between the paroxysms of pain after every feeding. PE: awake, alert, no respiratory distress, clear
• If not reduced → shock-like state breath sounds, distinct heart sounds, non-distended abdomen with
hyperactive bowel sounds. What is your impression?
A. Volvulus C. Pyloric Stenosis
B. Malrotation D. Duodenal Atresia

TOPNOTCH MEDICAL BOARD PREP PEDIATRICS MAIN DIGITAL HANDOUT BY DR. PUNONGBAYAN AND DR. DE VERA Page 29 of 105
For inquiries visit www.topnotchboardprep.com.ph or https://www.facebook.com/topnotchmedicalboardprep/
This handout is only valid for the March 2021 PLE batch. This will be rendered obsolete for the next batch since we update our handouts regularly.
TOPNOTCH MEDICAL BOARD PREP PEDIATRICS MAIN DIGITAL HANDOUT BY DR. PUNONGBAYAN AND DR. DE VERA
For inquiries visit www.topnotchboardprep.com.ph or https://www.facebook.com/topnotchmedicalboardprep/
This handout is only valid for the March 2021 PLE batch. This will be rendered obsolete for the next batch since we update our handouts regularly.

DUODENAL ATRESIA
FACTS ABOUT DUODENAL ATRESIA: HIRSCHSPRUNG DISEASE
• Due to failure to recanalize the lumen during the 4th-5th week of FACTS ABOUT HIRSCHSPRUNG DISEASE
gestation • Congenital aganglionic megacolon
• 25-40% of all intestinal atresias • Most common cause of lower intestinal obstruction in neonates;
• 50% are premature M>F
• obstruction usually distal to the ampulla of Vater • Arrest of neuroblast migration from the proximal to distal bowel
• may have other congenital anomalies – absence of ganglion cells in the bowel wall beginning in the
internal anal sphincter
OTHER ESSENTIAL MANIFESTATIONS
• Hallmark: bilious vomiting without abdominal distention
usually noted on the 1st day of life
• Polyhydramnios in 50% due to failure of absorption of amniotic
fluid in the distal intestine
• Jaundice in 1/3 of patients

DUODENAL ATRESIA
• Dx: double-bubble sign on plain abdominal X-ray due to
distended & gas-filled stomach & proximal duodenum
• Tx: NGT/OGT decompression with IV fluid replacement; surgery
– duodenoduodenostomy with gastrostomy tube

https://www.cancercarewny.com/content.aspx?chunkiid=22828
HIRSCHSPRUNG DISEASE
• Dominant & recessive patterns seen
• RET genes on chr 10q11 & the EDNRB gene on chr13q22
• Aganglionic segment limited to the rectosigmoid in 80%
• Absence of Meissner & Auerbach plexus & hypertrophied bundles
with high concentrations of acetylcholinesterase between the
muscular & submucosa layers
o Decreased motility in the affected bowel segment
o Lack of propagation of peristaltic waves into the aganglionic
colon
o Abnormal or absent relaxation of this segment and of the
internal anal sphincter
✔ GUIDE QUESTIONS
A 3 year-old boy presents with chronic constipation since he was an PATHOGENESIS
infant. You find a boy who looks active and playful. His abdomen was
globular, non-tender and rectal exam revealed a normal sphincteric
• Symptoms usually begin at birth with the delayed passage of
tone and no feces on the examining finger. What is your most likely meconium
impression? • Some present with history of chronic constipation
A. Hirschsprung Disease C. Meckel Diverticulum • Failure to pass stool → dilatation of the proximal bowel &
B. Functional Constipation D. Duodenal Atresia abdominal distention → ↑ intraluminal pressure → ↓ blood flow
From the case above, what part of the digestive tract is most probably & deterioration of mucosal barrier → bacteria proliferates →
affected? enterocolitis
A. Duodenum
B. Large Intestine
C. Small Intestine DIAGNOSIS
D. Recto-sigmoid • Dx: rectal suction biopsy – gold standard (easy and reliable)
What is the definitive diagnostic tool for this disease? • Abdominal X-ray:
A. Upper GI Series o transition zone between normal dilated proximal colon
B. Barium Enema o a smaller caliber obstructed distal colon due to nonrelaxation
C. Biopsy of the aganglionic bowel
D. Clinical
MANAGEMENT
• Treatment:
o a temporary colostomy & wait until the infant is 6-12 mos old
to perform definitive repair
o definitive procedure (Swenson, Duhamel, Boley endorectal
pull-through via laparoscopy)
Patients presenting as obstruction
• Can’t pass NG tube • Omega sign or Coffee bean sign on
Volvulus
• Severe pain and emesis AXR
• Sausage-shaped RUQ mass /
• Bloody Currant jelly stools absence of bowel sounds on RLQ Intussusception
Abdominal • Coiled-spring sign
pain • Post-prandial vomiting, non-bilious • Olive-shaped mass
Vomiting • Abdominal distention • Barium studies: Shoulder sign, Pyloric stenosis
± Distention • Down’s syndrome Double Tract Sign
• Normal history or recurrent obstructive
symptoms • Scintigraphy scan to detect
Meckel Diverticulum
• Painless rectal bleeding gastric tissue
• Intermittent pain

SUPPLEMENT: QUICK SHEET • 2 times more males are affected


MECKEL DIVERTICULUM RULE OF 2 • 2 inches long
• 2% of the population is affected • 2 feet from the ileocecal valve
• 2 yrs old on presentation (usually) • 2 types of common ectopic tissue (gastric and pancreatic)
• Does not apply 2 everyone!
TOPNOTCH MEDICAL BOARD PREP PEDIATRICS MAIN DIGITAL HANDOUT BY DR. PUNONGBAYAN AND DR. DE VERA Page 30 of 105
For inquiries visit www.topnotchboardprep.com.ph or https://www.facebook.com/topnotchmedicalboardprep/
This handout is only valid for the March 2021 PLE batch. This will be rendered obsolete for the next batch since we update our handouts regularly.
TOPNOTCH MEDICAL BOARD PREP PEDIATRICS MAIN DIGITAL HANDOUT BY DR. PUNONGBAYAN AND DR. DE VERA
For inquiries visit www.topnotchboardprep.com.ph or https://www.facebook.com/topnotchmedicalboardprep/
This handout is only valid for the March 2021 PLE batch. This will be rendered obsolete for the next batch since we update our handouts regularly.
TREATMENT
• Patient education
o Toilet training
o Dietary modification
• Relief of impaction
• Stool softener
o Polyethylene glycol
o Lactulose
o Mineral oil
o Prolonged stimulants (Senna or Bisacodyl) should be
AVOIDED
✔ GUIDE QUESTION
✔ GUIDE QUESTION
A three-year old male was brought to clinic for a chief complaint of
A 14-year-old female started having foul smelling watery soft stools in
constipation. Mother noted child to be defecating only once a week in
the morning. She noticed this after she started taking breakfast cereal
the last 4 months. Patient was noted to be irritable at times with
with milk. She says that she usually drinks coffee in the mornings and
decrease in appetite. There was no weight loss, failure to thrive, nor
is not accustomed to drinking milk. What is the most likely mechanism
abdominal distention. The child cries when he passes stool which was
of diarrhea in this case?
observed to be seldomly blood streaked. Mother was also concerned
A. Secretory
that his child was already toilet trained and yet there would be times
B. Osmotic
that the boy would stool in his underwear while attending daycare.
C. Exudative
According to the Rome criteria for functional constipation in children
D. Inflammatory
less than 4 years of age, how many criteria must be met in order to
diagnose functional constipation?
A. 1 DIFFERENCE BETWEEN OSMOTIC & SECRETORY DIARRHEA:
B. 2 OSMOTIC SECRETORY
C. 3
D. 4
Volume of stool <200 ml/day >200 ml/day
Response to Diarrhea
FUNCTIONAL CONSTIPATION Diarrhea stops
fasting continues
• Constipation – delay or difficulty in defecation present for 2 wk
Stool Na <70 mEq/L >70 mEq/L
or longer and significant enough to cause distress to the patient.
• Differentiate from organic causes with a GOOD history and Reducing
positive negative
physical examination substances
• Usually starts AFTER the neonatal period Stool pH <5 >6
• Vicious cycle See Table on Mechanisms of Diarrhea on the next page
• Hard stool → anal irritation → stool retention → Hard Stool
• Encopresis is common ✔ GUIDE QUESTION
o voluntary or involuntary passage of feces into inappropriate A 3 yr-old girl presents with sudden onset of dysphagia. No other
places at least once a month for 3 consecutive months once a symptoms noted. Her older brother witnessed her to have accidentally
chronologic or developmental age of 4 yr has been reached swallowed a round object 1 hr PTC. PE: well-nourished, coherent,
fearful-looking, slightly tachycardic, clear & equal breath sounds, soft &
o Retentive encopresis – with constipation and overflow
non-tender abdomen? What is your impression?
incontinence, 65-95% A. Asthma
o Non-retentive encopresis – without constipation and B. Foreign body Aspiration
incontinence C. Foreign body Ingestion
To simplify functional constipation. It’s a behavioral disorder. Where the D. None of the above
child thinks that it is painful to poo and therefore avoid defecating. This
causes problem of impaction, hard stools thus a vicious cycle. To address FOREIGN BODY INGESTION
this the patient should be toilet trained, this will only be possible if pain is
removed. Thus, decompaction and stool softeners are mainstay. OTHER IMPORTANT POINTS:
Lactulose is given for around 4-6 months. Common pitfall of some • 30% may be asymptomatic
practitioners is to give stool softeners as PRN basis, this will not break the • Initial sx: choking, gagging, coughing followed by excessive
cycle. salivation, dysphagia, food refusal, emesis, pain in the neck or
Dr. De Vera
throat
• If oropharynx or proximal esophagus is perforated: cervical
swelling, erythema, subcutaneous crepitations
WHEN IS ENDOSCOPY INDICATED?
• Sharp objects, disc button batteries, or FBs with respiratory
symptoms mandate urgent removal
• Failure to visualize object + symptomatic = URGENT endoscopy
Additional reminder…
• Asymptomatic: blunt objects may be observed up to 24 hrs in
anticipation of passage into the stomach

CAUSTIC INGESTIONS
• Ingestion results in esophagitis, necrosis, perforation & stricture
formation
• Liquid alkali produces liquefaction necrosis
• Acidic agents are bitter so less may be ingested – coagulation
necrosis
• Vomiting, drooling, refusal to drink, oral burns, dysphagia,
abdominal pain
• Circumferential ulcers, white plaques, sloughing of mucosa

Nelson Textbook of Pediatrics, 20th ed.

TOPNOTCH MEDICAL BOARD PREP PEDIATRICS MAIN DIGITAL HANDOUT BY DR. PUNONGBAYAN AND DR. DE VERA Page 31 of 105
For inquiries visit www.topnotchboardprep.com.ph or https://www.facebook.com/topnotchmedicalboardprep/
This handout is only valid for the March 2021 PLE batch. This will be rendered obsolete for the next batch since we update our handouts regularly.
TOPNOTCH MEDICAL BOARD PREP PEDIATRICS MAIN DIGITAL HANDOUT BY DR. PUNONGBAYAN AND DR. DE VERA
For inquiries visit www.topnotchboardprep.com.ph or https://www.facebook.com/topnotchmedicalboardprep/
This handout is only valid for the March 2021 PLE batch. This will be rendered obsolete for the next batch since we update our handouts regularly.

• Absence of oropharyngeal MANAGEMENT


lesions does not exclude the • Upper endoscopy – for all symptomatic patients to rapidly
possibility of significant identify tissue damage
esophagogastric injury! • Dilution by water/milk is recommended.
• Circumferential ulcers, white • Neutralization, induced emesis & gastric lavage are
plaques, sloughing of mucosa contraindicated.
→ increased risk of strictures • Surgical resection: dilation of strictures, SI or colon interposition
Oral lesions (plaques) after caustic ingestion
✔ GUIDE QUESTION
A 10-year-old girl was diagnosed to have systemic viral infection 1
Severity of esophageal injury post-caustic ingestion as graded week PTC. She was managed conservatively. 2 days PTC, she had
by endoscopic appearance of esophageal mucosa epigastric pain that increased in intensity associated with 6 vomiting
• Grade 0: normal mucosa episodes for the past few hours. PE: febrile, toxic-looking, flat, tensed
• Grade 1: erythema abdomen with tenderness radiating to the back. What is your treatment
• Grade 2: erythema, sloughing, ulceration, and exudates (not of choice for this patient?
circumferential) A. Exploratory Laparotomy with appendectomy
B. NPO, TPN, Antibiotics, Analgesics
• Grade 3: deep mucosal sloughing and ulceration C. Analgesics Alone
(circumferential) D. Barium decompression
• Grade 4: eschar, full thickness injury, and perforation

Nelson Textbook of Pediatrics, 20th ed.

ACUTE PANCREATITIS
• Most common pancreatic disorder in children
• Common causes:
o blunt abdominal injury
o mumps & other viral illnesses
o multisystem disease
o congenital anomalies
o biliary microlithiasis
o drugs & toxins

PATHOGENESIS
• Theory: following an insult, lysosomal hydrolase co-localizes
with pancreatic proenzymes within the acinar cell →
pancreastasis with continued synthesis of enzymes occur →
proenzymes are activated by cathepsin leading to auto-digestion
→ further activation & release of active proteases → lecithin is
activated by Phospholipase A2 into the toxic lysolecithin

CLINICAL MANIFESTATIONS
• Abdominal pain (epigastric and may radiate to the back, steady,
appears acutely ill), persistent vomiting, fever
• Pain increases in intensity for 24-48 hrs with vomiting epomedicine.com

• Severe acute type: rare in children; severe nausea, vomiting,


shock, jaundice, high fever, Cullen sign, Grey Turner sign, Criteria for the diagnosis of pancreatitis in children
necrotic pancreas; mortality rate of 50% • 2 of 3 of the following:
o abdominal pain
o serum amylase and/or lipase activity at least 3 times greater
than the upper limit of normal
o and imaging findings characteristic of, or compatible with,
acute pancreatitis.
TOPNOTCH MEDICAL BOARD PREP PEDIATRICS MAIN DIGITAL HANDOUT BY DR. PUNONGBAYAN AND DR. DE VERA Page 32 of 105
For inquiries visit www.topnotchboardprep.com.ph or https://www.facebook.com/topnotchmedicalboardprep/
This handout is only valid for the March 2021 PLE batch. This will be rendered obsolete for the next batch since we update our handouts regularly.
TOPNOTCH MEDICAL BOARD PREP PEDIATRICS MAIN DIGITAL HANDOUT BY DR. PUNONGBAYAN AND DR. DE VERA
For inquiries visit www.topnotchboardprep.com.ph or https://www.facebook.com/topnotchmedicalboardprep/
This handout is only valid for the March 2021 PLE batch. This will be rendered obsolete for the next batch since we update our handouts regularly.
AMYLASE VS LIPASE? ✔ GUIDE QUESTION
• Serum amylase level typically elevated for up to 4 days A 15 year old sexually active male came in due to history of fever, chills
• Serum lipase is more specific than amylase for acute and right upper quadrant abdominal pain. Upon physical examination
inflammatory pancreatic disease you noted icteric sclera, hepatomegaly and jaundice. You suspect an
acute hepatitis B infection. What test would you expect to find?
o Rises by 4-8 hrs, peaks at 24-48 hrs, remains elevated 8-14
A. HBsAg (+), Anti-HBc (+), IgM Anti-HBc (+), Anti-HBs (–)
days LONGER than serum amylase B. HBsAg (+), Anti-HBc (+), IgM Anti-HBc (-), Anti-HBs (–)
C. HBsAg (-), Anti-HBc (+), Anti-HBs (+)
OTHER LABORATORY FINDINGS: D. HBsAg (-), anti-HBc (-), Anti-HBs (+)
• Hemoconcentration
• Coagulopathy VIRAL HEPATITIS
• Leukocytosis • Identified Antibodies:
• Hyperglycemia o Anti-HAV, IgM anti-HAV
• Hyperbilirubinemia o Anti-HBsAg, IgM anti-HBsAg, anti-HBcAg, anti-HBeAg
• High glutamyl transpeptidase o Anti-HCV
o Anti-HDV
IMAGING STUDIES: o Anti-HEV, IgM anti-HEV
• Abdominal x ray: o Anti-HGV
o Sentinel loop HEPATITIS B ANTIGENS & ANTIBODIES
o Dilated transverse colon (cutoff sign) • First clinical evidence of HBV infection is elevation of ALT levels
o Ileus which occurs about 6-7 wks after exposure
o Blurring of the left psoas margin • Anti-HBcAg – most valuable single serologic marker of acute
o Peripancreatic extraluminal gas bubbles HBV infection because it is present as early as HbsAg and
continues to be present later in the course of the disease when
HBsAg has disappeared
• HBcAg – inner portion of the virion that encodes the viral DNA
• HBeAg – serves as a marker of active viral replication;
identification of infected people at increased risk of transmitting
HBV
• HBsAg – first serologic marker to appear & its rise coincides
with the onset of symptoms; detection of acutely or chronically
infected people; antigen used in hepatitis B vaccine
• Anti-HBs – identification of people who have resolved infections
with HBV; determination of immunity after immunization
• Anti-HBe – identification of infected people with lower risk of
transmitting HBV
• Anti-HBc – identification of people with acute, resolved, or
chronic HBV infection
• IgM anti-HBc – identification of people with acute or recent HBV
infections (including HBsAg-negative people during the
“window” phase of infection)

• Abdominal ultrasound and CT scan:


o Pancreatic enlargement
o Hypoechoic, sonolucent, edematous
o Pancreatic masses
o Fluid collections
o Abscess
*20% of children may have normal findings

TREATMENT
• Medical mgt: to relieve pain and to restore metabolic
homeostasis
• Analgesia
• Fluid electrolyte & mineral balance restored & maintained
• Prophylactic antibiotics useful in severe cases to prevent
infected pancreatic necrosis TIME
• Refeed when vomiting has resolved, serum amylase is falling, HBsAg Anti-HBS Anti-HBc HBeAg
PERIOD
and clinical symptoms are resolving Incubation
• Endoscopic therapy – if due to anatomic abnormalities + - - +
Period
(strictures or stones) Acute
+ - + IgM +
• Severe acute type: enteral or TPN, antibiotics, gastric acid infection
suppression, peritoneal lavage Window
Period
- - +IgM -
PROGNOSIS Complete
Recovery
- + +IgG -
• Uncomplicated cases: recover over 2-5 days
• If associated with trauma or systemic disease: prognosis is Chronic
Carrier
+ - +IgG -
related to the associated medical conditions
Chronic
• Ranson criteria and APACHE score – prognostic systems used in + - +IgG +
Active
adults are inappropriate for use in children
Vaccinated - + - -

TOPNOTCH MEDICAL BOARD PREP PEDIATRICS MAIN DIGITAL HANDOUT BY DR. PUNONGBAYAN AND DR. DE VERA Page 33 of 105
For inquiries visit www.topnotchboardprep.com.ph or https://www.facebook.com/topnotchmedicalboardprep/
This handout is only valid for the March 2021 PLE batch. This will be rendered obsolete for the next batch since we update our handouts regularly.
TOPNOTCH MEDICAL BOARD PREP PEDIATRICS MAIN DIGITAL HANDOUT BY DR. PUNONGBAYAN AND DR. DE VERA
For inquiries visit www.topnotchboardprep.com.ph or https://www.facebook.com/topnotchmedicalboardprep/
This handout is only valid for the March 2021 PLE batch. This will be rendered obsolete for the next batch since we update our handouts regularly.

NEPHROLOGY • Cloudy urine – can be normal; due to crystal formation at room


temp.
✔ GUIDE QUESTIONS
• Uric acid crystals form in acidic urine
A 10 year old male patient comes to the clinic for swelling. His mother
noticed that he had been having swelling especially of the lower • Phosphate crystals form in alkaline urine
extremities. She also noticed him to be weak and sometimes lethargic. • Normal specific gravity – 1.015-1.025
He had an infected skin infection a month ago which resolved with • Falsely high SG if with glucose, abundant protein and with
unrecalled antibiotics. Physical exam revealed BP130/90, HR 102, iodine-containing contrast materials
RR24, T 36.5°C, (+) bipedal edema, clear breath sounds bilaterally, non-
hyperemic posterior pharynx, tonsils not enlarged, Urinalysis: revealed SOURCES OF HEMATURIA?
RBC 25-30, protein +1, bacteria +1, epithelial cells +1, with RBC casts,
• Glomerular – brown, cola, tea, burgundy colored urine.
serum creatinine is within normal limits, what is the most likely
diagnosis in this patient? Proteinuria >100mg/dl on dipstick, RBC casts, dysmorphic RBCs
A. Minimal change disease (acanthocytes)
B. Post-streptococcal glomerulonephritis • Tubular system – leukocytes or renal tubular cast
C. Membranoproliferative glomerulonephritis • Lower urinary tract – gross hematuria, terminal hematuria,
D. Membranous glomerulonephritis blood clots, normal morphology, minimal proteinuria
Ask yourself, is this nephrotic or nephritic? This is definitely <100mg/dl on dipstick
nephritic. Hypertension and a history of pyoderma. Classic PSGN. Three things are very useful when comparing lower tract vs glomerular
Some might be misled by the protein +1. RBCs in urine causes false hematuria. Protein content, color, and morphology.
positive proteinuria. Dr. De Vera
Dr. De Vera
A 10-year-old male patient started having headaches and blurring of
vision. He is brought to this clinic by his mother. Initial examination ✔ GUIDE QUESTION
revealed bipedal edema but no symptoms of dysuria, no abdominal The most common cause of gross hematuria in pediatric population is?
pain. The mother recalls that the patient had cough and sore throat 1 A. PSGN C. Malignancy
week ago. BP is 150/90, HR 106, RR 20 T 36.9°C, which among the ff. B. Trauma D. Urinary Tract Infection
is an expected laboratory finding?
A. Increased C3 C. Increased C5
B. Decreased C3 D. Decreased C5
APSGN
• Age group: 5-12yo. Rare before 3yo
You have to understand how to use C3. C3 is a dead giveaway. If
C3 is decreased and you are considering glomerulonephritis in • Clinical spectrum from asymptomatic to renal failure
children, almost always that is PSGN (C3 should normalize after 8 • Documentation of prior strep infection
weeks). Other two choices are MPGN and SLE which presents with o Throat – ASO
mixed nephrotic-nephritic picture. o Skin – antideoxyribonuclease B
Dr. De Vera
• Most commonly sporadic
In relation to the above case, all of the ff. statements are true regarding o Serotype following throat infection: 1, 2, 4 , 12, 18 25
the patient’s condition above except?
o Serotype following skin infection: 49, 55, 57, 60
A. complete recovery occurs in 95% of children
B. early systemic antibiotic therapy does not eliminate the risk of • Clinical manifestation: patient develops nephritic syndrome
this disease o How many weeks after throat infection? 1-2
C. administering antibiotics can alter the natural history of this o How many weeks after skin infection? 3-6
disease • When will the abnormalities resolve and normalize?
D. serum complement level is usually low in this immune-complex o Acute phase : 6-8 weeks
mediated disease o Urinary protein excretion and hypertension: 6 mos
When assessing patient with edema that is related to the kidneys, one o Microscopic hematuria : 1-2 years
approach is to classify the patient as having nephritic VS nephrotic o C3 level : 6-8 weeks
syndrome. • What is the pathogenesis of PSGN?
Dr. De Vera
o Ab is produced & combines with a circulating Ag unrelated to
the kidney → IC accumulate in glomeruli & activate the
SUPPLEMENT: QUICK SHEET complement system → immune injury
Nephritic vs. Nephrotic Mnemonics • What hypothesis is given to explain the predilection to
Nephritic – HOHA damage the glomeruli?
Hypertension o Localized in the glomeruli due to negatively charged capillary
Oliguria wall, mesangial trapping, hydrodynamic forces
Hematuria • Why does serum C3 decrease?
Azotemia o Inflammatory reaction follows immunologic injury due to
Nephrotic – PALE activation of classic and alternative pathways which converge
Proteinuria (nephrotic range) at C3 → lysis of cell membranes → hypocomplementemia
Albumin low • What major noxious products of complement activation are
Lipid High produced after C3 activation?
Edema o anaphylatoxin (increases vascular permeability) & C5a
(release substances that damage cells & basement
membranes)
HEMATURIA
• More than 5 RBCs/hpf (centrifuged) SUPPLEMENT: QUICK SHEET
o RBC morphology IMPORTANT POINT
• Dipstick (based on the peroxidase reaction with hemoglobin) Best single Ab titer to document skin infection is DNAse B
• Pathophysiology: RBC casts formed when they enmesh in protein antigen
matrix
• False negative hematuria What are the only causes of Renal Insufficiency that cause
o Formalin decreased C3?
o High urinary ascorbic acid 1. PSGN
• False positive 2. MPGN
o Alkaline urine (pH>8) 3. Lupus Nephritis
o Hydrogen peroxide
• Presence of RBC is suggested by >10rbc/uL or +1 dipstick in a Indications for Renal Biopsy in APSGN
freshly voided and centrifuged urine • Acute renal failure
• Nephrotic syndrome
IMPORTANT POINTS • Absence of evidence of strep infection
• Grossly red urine with or without blood clots – from the lower • Normal complement level
urinary tract • Hematuria and proteinuria
• Brown-colored / tea-colored urine – glomerular in origin • Low C3 that persists more than 2 months
TOPNOTCH MEDICAL BOARD PREP PEDIATRICS MAIN DIGITAL HANDOUT BY DR. PUNONGBAYAN AND DR. DE VERA Page 34 of 105
For inquiries visit www.topnotchboardprep.com.ph or https://www.facebook.com/topnotchmedicalboardprep/
This handout is only valid for the March 2021 PLE batch. This will be rendered obsolete for the next batch since we update our handouts regularly.
TOPNOTCH MEDICAL BOARD PREP PEDIATRICS MAIN DIGITAL HANDOUT BY DR. PUNONGBAYAN AND DR. DE VERA
For inquiries visit www.topnotchboardprep.com.ph or https://www.facebook.com/topnotchmedicalboardprep/
This handout is only valid for the March 2021 PLE batch. This will be rendered obsolete for the next batch since we update our handouts regularly.
In both nephrotic and nephritic, we do biopsy when the picture does not should be predominantly nephritic, if we have nephrotic features,
perfectly fit the most common causes (i.e. PSGN, MCN). So, for PSGN, this persistently low C3, and other features listed above, we do biopsy.
Dr. De Vera

PHYSICAL EXAM /
HISTORY DIAGNOSIS
DIAGNOSTICS
History of hemoptysis and Goodpasture
severe respiratory distress syndrome
Purpuric rash most
Patient presenting with hematuria, hypertension, With SVI 3 weeks ago prominent at the buttocks HSP
and signs of renal insufficiency Pain in the joints
History of vomiting, bloody
Dehydrated hepato-
diarrhea, and fever HUS
splenomegaly
History of eating hamburger
Patient is febrile with LRTI or
Normal C3 level IgA Nephropathy
GIT infection
Sudden onset of gross hematuria
History of Strep throat or
Severely depressed C3 level APSGN
pyoderma
• Patient wears very thick
Child discovered to have asymptomatic
glasses and wear hearing aid
microscopic hematuria during yearly check-up at Normal labs Alport Syndrome
• Child has uncles with the
school
same condition

ALPORT SYNDROME (HEREDITARY NEPHRITIS) THIN BASEMENT MEMBRANE DISEASE


• Bilateral sensorineural hearing loss (never congenital) • Presence of persistent microscopic hematuria and isolated
• Ocular abnormalities (30-40%) such as anterior lenticonus (its thinning of the GBM on E/M
presence is pathognomonic), macular flecks, & corneal erosions • Often initially observed in childhood and is intermittent
• Critical in the diagnosis: family history, screening urinalysis of • Isolated hematuria in multiple family members without renal
1st-degree relatives, audiogram, ophthalmologic exam dysfunction is termed as benign familial hematuria
• Highly likely: (+) hematuria and at least 2 of the ff: macular • Important to monitor for hypertension, progressive proteinuria,
flecks, recurrent corneal erosions, GBM thickening or thinning, or renal insufficiency
sensorineural deafness
✔ GUIDE QUESTION
PROGNOSIS AND TREATMENT An 8-year-old female was brought to a physician because of swelling of
• NO specific therapy is available the face, arms and legs. A 24-hour collection of urine yielded 5 grams
• ACE inhibitors can slow the rate of progression of protein, rare red cells, and no red cell casts. The child was given
corticosteroids. All symptoms and findings vanished, and the physician
• Manage renal failure complications such as hypertension, concluded, even without renal biopsy, that the child most likely had:
anemia, electrolyte imbalance A. Acute post-streptococcal glomerulonephritis
• ESRD occurs before 30 years old in 75% of hemizygotes with X- B. IgA nephropathy of Berger
linked AS C. Nephritic syndrome
• Patients with ESRD are treated with dialysis and kidney D. Minimal change disease
transplantation

Nelson Textbook of Pediatrics, 20th ed.

TOPNOTCH MEDICAL BOARD PREP PEDIATRICS MAIN DIGITAL HANDOUT BY DR. PUNONGBAYAN AND DR. DE VERA Page 35 of 105
For inquiries visit www.topnotchboardprep.com.ph or https://www.facebook.com/topnotchmedicalboardprep/
This handout is only valid for the March 2021 PLE batch. This will be rendered obsolete for the next batch since we update our handouts regularly.
TOPNOTCH MEDICAL BOARD PREP PEDIATRICS MAIN DIGITAL HANDOUT BY DR. PUNONGBAYAN AND DR. DE VERA
For inquiries visit www.topnotchboardprep.com.ph or https://www.facebook.com/topnotchmedicalboardprep/
This handout is only valid for the March 2021 PLE batch. This will be rendered obsolete for the next batch since we update our handouts regularly.

PROTEINURIA FIXED PROTEINURIA


• charge and size selective properties of the glomerular capillary • First morning void, 3 separate occasion
wall prevent significant amounts of albumin, globulin, and other • Proteinuria ≥ 1+ with SG >1.015
large plasma proteins from entering the urinary space • UPCR ≥ 2
• 3 mechanisms • Further evaluation needed
o Glomerular proteinuria – ineffective filtration • Indicates potential glomerular or tubular disorder
o Tubular Proteinuria – ineffective reabsorption
o Increase plasma protein – ex. rhabdomyolysis ✔ GUIDE QUESTIONS
• Upper limit in healthy children is 150mg/day A 4 year old male patient was brought to the clinic because of swelling.
The mother started to notice it 3 days ago initially on the legs. She also
• Urine dipstick started to notice it that her child had puffy eyelids whenever he wakes
o Negative up in the morning and decreases throughout the day. There was no
o Trace = 10-29mg/dl fever or abdominal pain, (+) anorexia, PE revealed: HR 101, RR 26,
o 1+ = 30mg/dl T>36.7°C, (+) bipedal edema, (-) murmurs, clear bilateral breath
o 2+ = 100mg/dl sounds, urinalysis was done which revealed +4 proteinuria, RBC 0-1,
o 3+ = 300mg/dl bacteria +1, serum creatinine within normal limits, 24-hour urinary
o 4+ = >2g/dL protein is 4g/24hr, what is the next best step for this patient?
A. Treat with steroids
• Non-pathologic (<1000mg/day)
B. Renal biopsy
o Postural C. Renal ultrasound
o Febrile (>38.4°C) D. Urine culture and sensitivity
o Exercise – resolves after 48h In relation to the case above, after 8 weeks of steroid therapy, patient’s
• False positive urinary protein is +3, what is the next best step in the management of
o Very high urine PH (>7) this patient?
o Highly concentrated urine A. Restart treatment with steroids
o Contamination with blood B. Renal biopsy
C. Renal ultrasound
• False negative
D. Abdominal CT scan
o Dilute urine E. Treat with cyclophosphamide
o Urine wherein the predominant protein is not albumin In relation to the case above, what is the most likely diagnosis in this
• Dipstick is POSITIVE if patient?
o > trace in a sample with SG <1.01 A. Membranous glomerulonephritis
o >/= +1 if >1.015 B. Minimal change disease
• 24hr urine protein determination is more accurate C. Mesangial proliferative
D. Focal segmental glomerulosclerosis
o Normal is </= 4mg/m2/hr
o Abnormal 4-40mg/m2/hr
o Nephrotic range > 40mg/m2/hr NEPHROTIC SYNDROME
• Urine protein: creatinine ratio • Idiopathic Nephrotic Syndrome
o <0.5 in <2yo is normal o Minimal change disease
o <0.2 in >2yo is normal o Focal segmental glomerulosclerosis
o >2 is nephrotic range proteinuria o Mesangial proliferation
The cardinal feature that we look for in nephrotic syndrome is • Glomerular diseases
proteinuria. Memorize the cut-off for nephrotic range proteinuria either o Membranous nephropathy
24hr value OR UPCR (urine protein:crea ratio). If PSGN is for nephritic, o Membranoproliferative glomerulonephritis
Minimal change disease is for nephrotic. It is the most common cause,
treatment is steroids. NEPHROTIC SYNDROME (MINIMAL CHANGE DISEASE)
Dr. De Vera
• The urinalysis reveals 3+ or 4+ proteinuria
✔ GUIDE QUESTION • Spot urine protein/creatinine ratio exceeds 2.0 and urinary
A 13yo/ male came in for annual physical examination. The rest of the protein excretion exceeds 3.5 g/24 hr in adults and 40
findings are normal except for a urinalysis with trace proteinuria on mg/m2/hr in children.
dipstick test. Mother is worried. You will advise the following EXCEPT? • The serum albumin level is generally <2.5 g/dL, and the serum
A. This is a benign condition. No evaluation or therapy is cholesterol and triglyceride levels are elevated.
necessary
B. Majority of children will have negative evaluations on repeated
• C3 and C4 levels are normal.
dipstick test and protein quantification • Renal biopsy is not required for diagnosis in most children.
C. This is more common in adolescents and younger children • >95% with MCNS respond to corticosteroid therapy
D. You will request for a UPCR • Pathogenesis
o Increased permeability of glomerular capillary wall →
CAUSES OF TRANSIENT PROTEINURIA proteinuria and hypoalbuminemia
• Fever o Podocytes play a crucial role in filtering proteins
• Exercise § Effacement of foot process, decrease number of podocytes
• Dehydration and altered slit diaphragm integrity = LEAKY Membranes
• Cold exposure • Management of MCNS
• Congestive heart failure o Corticosteroid
• Seizure o Tuberculosis must be first ruled out
• Stress o Uncomplicated nephrotic syndrome between 1-8yo are likely
to have steroid responsive MCNS
✔ GUIDE QUESTION
§ Initiate steroids even without biopsy
Still worried, mother decided to have a second opinion a month later. 4 o Do biopsy if with features less likely of MCNS
Serial determination of urinalysis revealed SG 1.2 and ++ proteinuria. § Gross hematuria, hypertension, renal insufficiency,
The following is true regarding this condition? hypocomplementemia, < 1yo or > 12yo
A. This is still transient proteinuria o Prednisone or Prednisolone (60mg/BSA/day or
B. Further investigation should be done 2mg/kg/day, max 60mg daily) for 4-6 weeks
C. No further investigation should be done o Alternate day (40mg/BSA or 1.5mg/kg) for 8 weeks to 5
D. Refer mother to psych months
o Response is remission attained within initial 4 weeks
o Remission is UPCR < 0.2 or < +1 protein on dipstick for 3
consecutive days
o Relapse is UPCR >2 or ≥ +3 dipstick for 3 consecutive days

TOPNOTCH MEDICAL BOARD PREP PEDIATRICS MAIN DIGITAL HANDOUT BY DR. PUNONGBAYAN AND DR. DE VERA Page 36 of 105
For inquiries visit www.topnotchboardprep.com.ph or https://www.facebook.com/topnotchmedicalboardprep/
This handout is only valid for the March 2021 PLE batch. This will be rendered obsolete for the next batch since we update our handouts regularly.
TOPNOTCH MEDICAL BOARD PREP PEDIATRICS MAIN DIGITAL HANDOUT BY DR. PUNONGBAYAN AND DR. DE VERA
For inquiries visit www.topnotchboardprep.com.ph or https://www.facebook.com/topnotchmedicalboardprep/
This handout is only valid for the March 2021 PLE batch. This will be rendered obsolete for the next batch since we update our handouts regularly.
NONRESPONDERS GLOMERULONEPHRITIS VS NEPHROTIC IN CHILDREN
A. Steroid resistant GLOMERULO- NEPHROTIC
B. Frequent relapser NEPHRITIS SYNDROME
C. Steroid dependent “HOHA” “PALE”
• Hypertension • Proteinuria
__________1. Relapse within 28 days of stopping prednisone Clinical features • Oliguria • Albumin low
therapy • Hematuria • Lipid high
__________2. Patients who respond well to prednisone therapy but • Azotemia • Edema
relapse 4 or more times in a 12 month period Disease
__________3. Children who fail to respond to prednisone therapy • Minimal
commonly
within 8 week • PSGN change
encountered in
__________4. Relapse on alternate-day steroid therapy disease
children
__________5. Children who continue to have proteinuria (+2 or • Decrease C3 that • 24hr urine
greater) after 8 week of steroid therapy Important
normalizes in 8 protein or
diagnostic test
Answers: C B A C A
weeks UPCR
• Supportive
• 95% will resolve
• The major complication of nephrotic syndrome is infection. • 95% will
• Monitor C3
• The most frequent type of infection is spontaneous bacterial Management respond to
• Fluid limit
peritonitis. steroids
• Diuretics
• The most common organism causing peritonitis is
• Anti-hypertensive
pneumococcus.
Management
• Edema – sodium restriction. Cautious Albumin and diuretics UTI IN CHILDREN
• Dyslipidemia – dietary modification • Prevalence varies with age
• Infection – usually 3rd gen cephalosporin o 1st year of life = male predominance
• Thromboembolism – anticoagulation if necessary o Beyond 1-2yo = female predominance
✔ GUIDE QUESTION
• Primarily caused by colonic bacteria
A 13 year old female patient presents to the clinic due to bipedal edema o 75-90% caused by E. coli in girls, followed by Klebsiella and
of 3 days duration, it is gradually progressive, she wakes up in the Proteus
morning with puffy eyelids, urinalysis reveals RBC 8-10 with RBC casts, • 3 basic forms
protein +3, WBC 0-2 no fever or rash noted, patient is HbsAg (+), which o Pyelonephritis
she says she got from her mother at birth. Biopsy reveals thickened o Cystitis
glomerular capillary walls with splitting from the interposition of the o Asymptomatic Bacteuria
mesangium, with C3 lobular deposits in immunofluorescence, what is
• Nearly all are ascending infections
the most likely diagnosis?
A. FSGS
B. Minimal change disease PYELONEPHRITIS
C. Membranous glomerulonephritis • is characterized by any or all of the following: abdominal, back,
D. Membranoproliferative glomerulonephritis
or flank pain; fever; malaise; nausea; vomiting; and, occasionally,
Where is the site of the pathology as seen in the electron microscope in
the condition described above?
diarrhea.
A. Subepithelial • Fever may be the only manifestation.
B. Subendothelial • Newborns - nonspecific symptoms
C. Both A and B • Most common serious bacterial infection in younger than 24mos
D. None of the above
PATHOLOGY CROSS-OVER CYSTITIS
FINDINGS DISEASE • Involvement of bladder
LM: Diffuse thickening of • dysuria, urgency, frequency, suprapubic pain, incontinence and
glomerular capillary walls malodorous urine
IF: Granular IgG and C3 Membranous • Cystitis does not cause fever and does not result in renal
EM: Sub-epithelial deposits of Glomerulonephritis Injury
electron dense material • Acute hemorrhagic cystitis – E. coli
“Spike and Dome” appearance • Eosinophilic cystitis
LM: normal • Interstitial Cystitis – irritative voiding symptoms relieved by
EM: diffuse effacement of Minimal Change voiding. Negative urine culture. Adolescent girls. Idiopathic
epithelial foot process, no Disease
deposits ASYMPTOMATIC BACTERIURIA
LM: Large, hypercellular • Positive urine culture without any manifestation of infection
glomeruli • Most common in girls
Membranoproliferative
Increased mesangial matrix • Does not cause renal injury except in pregnant women (can
Glomerulonephritis
Double Contour or Tram-track result in symptomatic UTI)
appearance
LM: Focal segmental sclerosis RISK FACTORS FOR UTI
and hyalinosis Focal Segmental
EM: Loss of foot process, Glomerulosclerosis
epithelial denudation
✔ GUIDE QUESTION
The following are true regarding UTI in children EXCEPT?
A. It is more common in males during the first year of life
B. E. coli is one of the most common pathogens
C. Pyelonephritis may manifest only with fever
D. Involvement of the renal parenchyma is termed pyelitis

GLOMERULONEPHRITIS
vs. NEPHROTIC SYNDROME Nelson Textbook of Pediatrics, 20th ed.
https://qrs.ly/73bj8bs

TOPNOTCH MEDICAL BOARD PREP PEDIATRICS MAIN DIGITAL HANDOUT BY DR. PUNONGBAYAN AND DR. DE VERA Page 37 of 105
For inquiries visit www.topnotchboardprep.com.ph or https://www.facebook.com/topnotchmedicalboardprep/
This handout is only valid for the March 2021 PLE batch. This will be rendered obsolete for the next batch since we update our handouts regularly.
TOPNOTCH MEDICAL BOARD PREP PEDIATRICS MAIN DIGITAL HANDOUT BY DR. PUNONGBAYAN AND DR. DE VERA
For inquiries visit www.topnotchboardprep.com.ph or https://www.facebook.com/topnotchmedicalboardprep/
This handout is only valid for the March 2021 PLE batch. This will be rendered obsolete for the next batch since we update our handouts regularly.
✔ GUIDE QUESTION MANAGEMENT OF UTI
The following are true regarding the diagnosis of UTI in children NELSON’S
EXCEPT • Co-trimoxazole
A. Urine culture is necessary
• Nitrofurantoin
B. In toilet trained children, midstream urine sample is usually Acute Cystitis
satisfactory • Amoxicillin – high rate of
C. In children who are not toilet trained, a catheterized or resistance
suprapubic aspirate should be obtained • Ceftriaxone
D. Use of adhesive, sealed, sterile collection bag after • Cefotaxime
disinfection is useful only if with a negative culture • Ampicillin + Aminoglycoside
E. NOTA
Acute Pyelonephritis • Oral 3rd generation
cephalosporin like Cefixime is
DEFINITION OF TERMS (PPS GUIDELINES)
as effective but does not cover
• 2 or more acute pseudomonas
pyelonephritis/presumptive UTI OR
Recurrent UTI • Aminoglycoside
• 3 or more cystitis OR
Pseudomonas • Fluoroquinolone
• 1 pyelonephritis + 1-2 cystitis
(ciprofloxacin)
• clinical symptoms + urinalysis findings DOH RECOMMENDATION
Presumptive
suggestive of UTI, NOT supported by urine
UTI • <2mos old: Cefotaxime PLUS
culture.
Amikacin for 10-14 days
E. coli
• >2mos to 18yo. Duration (7-14
LABORATORY TESTS IN UTI (PPS GUIDELINES) Klebsiella
days)
• < 2yo = clean catch urine collection Enterobacter
o Oral co-amoxiclav
• >2yo = mid-stream sample Enterococcus
o Cefuroxime
• Wee bag is only useful if NEGATIVE Group B strep
Collection of o Nitrofurantoin
RESULTS o Ampicillin Sulbactam
Urine
• Suprapubic aspiration or urethral Recurrent UTI,
catheterization if and only if above Catheter-related,
methods cannot be done properly comorbidities
URINE DIPSTICK • Ceftriaxone and/or Amikacin
• Nitrate reductase present in most gram- for 7-14days
Enterobacteriaceae
negative uropathogenic rods Pseudomonas
Nitrite
• Requires sufficient bladder incubation Enterococcus
time (around 4 hours) • If culture shows different
• May be associated with UTI and non- Antibiotic switch sensitivity pattern and/or poor
infectious renal diseases response after 48-72hours
Leukocyes
• False negative: glycosuria, high specific
gravity, contamination with debris ✔ GUIDE QUESTION
Leukocyte + Leukocyte – A 4 year old male was diagnosed with UTI based on positive cultures,
• Send Urine urinalysis, and symptoms. After 10 days of oral Cefixime patient
Nitrite • Urine CS CS improved and was asymptomatic. What will you do next?
A. Request for repeat urine culture C. Request for VCUG
+ Treat as UTI Treat as
B. Request for Ultrasound D. NOTA
Interpretation UTI
• Urine CS IMAGING STUDIES IN UTI
Nitrite Treat IF
• Not UTI • Goal: identify anatomic abnormalities that predispose to UTI,
– WITH determine if there is active renal involvement, asses renal
SYMPTOMS function
URINE MICROSCOPY
• WBC >5/HPF in centrifuged urine Guideline Recommendations for Diagnostic Evaluation
Pyuria
• WBC >10/uL in uncentrifuged Following a Febrile Urinary Tract Infection in Infants
• May be positive from contamination LATE
Bacteria • Not significant unless accompanied by GUIDELINE UTZ VCUG DMSA
presence of WBCs SCAN
Pyuria + Pyuria – National Institute for
• Send Health & Care
• Urine CS Urine Excellence (NICE)
Bacteriuria
Treat as CS American Academy of If abnormal
+ Yes No
UTI Treat Pediatrics UTZ
Interpretation
as UTI If abnormal If
Italian Society for
• Urine CS UTZ or if risk abnormal
Pediatric Nephrology Yes
Bacteriuria Treat IF • Not factors are UTZ or
(ISPN)
– WITH UTI present* VUR
SYMPTOMS *Abnormal antenatal UTZ of fetal urinary tract, family history of reflux,
• Gold standard in a properly collected septicemia, renal failure, age younger than 6mos in a male infant, likely
family noncompliance, incomplete bladder emptying, no clinical response to
sample
appropriate antibiotic therapy within 72 hours or infection with organism
• Indications: other than E. coli.
Urine Culture o Significant urinalysis findings
o Complicated UTI ✔ GUIDE QUESTION
o Seriously ill children A 9 year-old boy presents with a 5-day history of abdominal pain &
o Recurrent UTI diarrhea. One day PTC, he had bloody stools. He is admitted for IVF
KUB • Indications replacement & further workup. After 4 days, his abdominal pain
Ultrasound o Culture proven pyelonephritis subsided but the urine is grossly bloody & he looked pale. His blood
with post void o Febrile presumptive UTI smear showed fragmented RBCs. What is the most common etiologic
study agent for this disease?
o Recurrent UTI
A. Shigella C. C. Difficile
• VCUG to check for reflux B. E. coli D. P. aeruginosa
Other Test • DMSA renal scan
• Acute phase reactants: Procalcitonin

TOPNOTCH MEDICAL BOARD PREP PEDIATRICS MAIN DIGITAL HANDOUT BY DR. PUNONGBAYAN AND DR. DE VERA Page 38 of 105
For inquiries visit www.topnotchboardprep.com.ph or https://www.facebook.com/topnotchmedicalboardprep/
This handout is only valid for the March 2021 PLE batch. This will be rendered obsolete for the next batch since we update our handouts regularly.
TOPNOTCH MEDICAL BOARD PREP PEDIATRICS MAIN DIGITAL HANDOUT BY DR. PUNONGBAYAN AND DR. DE VERA
For inquiries visit www.topnotchboardprep.com.ph or https://www.facebook.com/topnotchmedicalboardprep/
This handout is only valid for the March 2021 PLE batch. This will be rendered obsolete for the next batch since we update our handouts regularly.

HEMOLYTIC-UREMIC SYNDROME • PE: dehydration, petechiae, hepatosplenomegaly, marked


irritability
• One of the most common cause of acute renal failure in young
• Peripheral smear: helmet cells, burr cells, fragmented RBCs
children
• Increased reticulocyte count, negative Coombs test,
• Microangiopathic hemolytic anemia, thrombocytopenia, and
leukocytosis, thrombocytopenia, anemia, hematuria &
renal insufficiency
proteinuria, normal PT & PTT, stool culture is often negative
• Caused by infection, genetics, drugs (calcineurin inhibitors,
• Supportive care: fluids & electrolytes, early institution of
cytotoxic agents, Clopidogrel and Ticlopidine, quinine), and
peritoneal dialysis, BP control, red cell transfusion
other diseases associated with microvascular injury (like SLE
• Platelets NOT generally administered regardless of platelet
and HELLP syndrome – Hemolytic anemia, Elevated Liver
count (almost immediately consumed by active coagulation and
enzymes, Low Platelet count syndrome)
can worsen the clinical course)
• Most common form of HUS is due to toxin-producing E. coli
• NO antibiotic therapy (can lead to increased toxin release)
• Asia and southern Africa: the shiga toxin of Shigella dysenteriae
• Those who survived the acute phase of HUS need long-term
type 1 is causative
follow-up because complications may not be apparent for up to
• Western countries: verotoxin-producing E. coli (VTEC) 0157:H7
20 years (hypertension, chronic renal insufficiency, proteinuria)
How is this different from thrombotic thrombocytopenic
purpura (TTP)? HEMATOLOGY/ONCOLOGY
• TTP has the same triad but can include CNS involvement and ✔ GUIDE QUESTIONS
fever and has a more gradual onset A one-month old male was brought in for consultation. The patient was
• Fewer cases of TTP can occur after diarrhea compared to HUS born full term and the antenatal and perinatal course was
unremarkable. The patient developed jaundice to which the attending
Laboratory criteria for diagnosis: physician requested for CBC and Bilirubin levels on the 10th DOL. A
final diagnosis of breastfeeding jaundice was made, and the patient
The following are both present at some time during the illness:
currently is well. The mother however was concerned that her CBC
1. Anemia (acute onset) with microangiopathic changes showed a Hgb level of 10g/dl (NV: 12-14). Physical examination is
(schistocytes, burr cells, helmet cells) on peripheral blood unremarkable. The following is/are true regarding this condition
smear EXCEPT?
2. Renal injury (acute onset) evidenced by hematuria, A. This condition is expected in the first 8-12 weeks of life
proteinuria, or elevated creatinine level: equal or > 1 mg/dL B. This is explained by the down regulation of EPO production
in < 13 years old or >1.5 mg/dL in > 13 years old or equal or C. No therapy is indicated for both term and pre-term infants
>50% increase over baseline D. Nadir Hgb levels rarely falls between 100mg/dl
The predominant hemoglobin present at birth in an infant is:
A. Hemoglobin A C. Hemoglobin F
B. Hemoglobin Gower D. Hemoglobin A2

PHYSIOLOGIC ANEMIA OF INFANCY


• Normal full-term infants have higher hemoglobin and larger red
blood cells than older children and adults
• 1st week of life – progressive decline in Hgb that persists 6-8
weeks
• Usually lasts 8-12 weeks
• Hb levels around 11g/dL, rarely falls below 10g/dL
Helmet & Burr cells
• No hematologic problem, no therapy required unless
exacerbated by other ongoing processes
CASE CLASSIFICATION • Pathophysiology
PROBABLE o Respiration at birth → more O2 to bind Hb (50% to 95% or
• An acute illness diagnosed as HUS or TTP that meets the lab more)
criteria in a patient who does not have a clear history of acute or o Switch from fetal to adult Hb (lower affinity to O2)
bloody diarrhea in the preceding 3 weeks, or o Net result is down regulation of EPO production
• An acute illness diagnosed as HUS or TTP that: → suppression of erythropoiesis
o Has onset within 3 weeks after onset of an acute or bloody o Hb decreases until tissue O2 needs become greater than O2
diarrhea, and delivery (8-12th week of life) → EPO production increases
o Meets the lab criteria except that microangiopathic changes
are not confirmed PHYSIOLOGIC ANEMIA OF PREMATURITY
• More extreme and rapid decline in Hb
CONFIRMED • Minimal levels up to 7-9g/dL
• An acute illness diagnosed as HUS or TTP that both meet the • Same physiologic factors exacerbated by other factors such as
laboratory criteria and began within 3 weeks after onset of an o Iatrogenic blood loss
episode of acute bloody diarrhea o Shortened RBC life span (40-60days)
o Accelerated expansion of RBC mass due to rapid rate of growth
HUS in preterm babies
• Microvascular injury with endothelial cell damage is • Not considered a benign condition, requires transfusion or EPO
characteristic of all forms of HUS. administration
• Pathogenesis
o Thickening of capillary walls → narrowing of capillary lumen ✔ GUIDE QUESTION
→ widening of mesangium → fibrin thrombi in glomerular A 3 year-old child is brought to you for pica and pallor. He is described
capillaries & arterioles → cortical necrosis → localized clotting as irritable with poor appetite. He prefers purely milk diet. What is the
o Microangiopathic anemia due to mechanical damage to RBCs as most likely diagnosis?
they pass through the altered vasculature
Answers: Iron Deficiency Anemia

• Thrombocytopenia is due to intrarenal platelet adhesion or


damage
• Most common in <4 yrs old
ANEMIA
• Onset is preceded by gastroenteritis (fever, vomiting, abdominal • Anemia is defined as a reduction of the hemoglobin
pain, bloody diarrhea) concentration or red blood cell (RBC) volume below the range of
• Sudden onset of pallor, irritability, weakness, lethargy & oliguria values occurring in healthy persons.
usually occurs 5-10 days after the initial gastroenteritis or
respiratory illness

TOPNOTCH MEDICAL BOARD PREP PEDIATRICS MAIN DIGITAL HANDOUT BY DR. PUNONGBAYAN AND DR. DE VERA Page 39 of 105
For inquiries visit www.topnotchboardprep.com.ph or https://www.facebook.com/topnotchmedicalboardprep/
This handout is only valid for the March 2021 PLE batch. This will be rendered obsolete for the next batch since we update our handouts regularly.
TOPNOTCH MEDICAL BOARD PREP PEDIATRICS MAIN DIGITAL HANDOUT BY DR. PUNONGBAYAN AND DR. DE VERA
For inquiries visit www.topnotchboardprep.com.ph or https://www.facebook.com/topnotchmedicalboardprep/
This handout is only valid for the March 2021 PLE batch. This will be rendered obsolete for the next batch since we update our handouts regularly.
ANEMIA CLASSIFICATION
MICROCYTIC NORMOCYTIC MACROCYTIC
MCV <80 (TAILS) MCV 80-100 MCV >100
• Thalassemia • Anemia of • Folate deficiency
• Iron deficiency chronic disease • Vit B12
• Lead poisoning • Uremia deficiency
• Sideroblastic • Hypothyroidism • Drug and
anemia • Bone marrow alcohol-induced
failure (aplastic anemia
anemia)

IDA vs. THALASSEMIA


FEATURES IDA THALASSEMIA
• Nutritional
Cause • Others: GI Bleed, • Hereditary
Parasitism
RBC Size • Microcytic • Microcytic
RBC Color • Hypochromic • Hypochromic
RDW • Elevated • Normal
CHILD PRESENTING • Low Iron
• Hemoglobin
WITH ANEMIA Other Test • Low Ferritin
Electrophoresis
https://qrs.ly/3ubj8co • High TIBC
Reticulocyte
• Low • Low
Count

Nelson Textbook of Pediatrics, 20th ed.

Nelson Textbook of Pediatrics, 20th ed.

IRON DEFICIENCY ANEMIA • Non-hematologic effects


• The most common hematologic disease of infancy and childhood o Pica
• Most common in 9-24 months of age. o Pagophagia – ICE
o Neurocognitive effects
• Most common cause is poor dietary intake.
• Pallor is the most important sign
DIAGNOSTIC FINDINGS
• At 6-10 mg/dL
• Dx: low reticulocyte count, microcytic, hypochromic RBCs, high
o Compensatory mechanisms
RDW, low serum iron & ferritin, high TIBC
o Almost no symptoms if these mechanisms are efficient
• At 5 mg/dL
o irritable and anorexic
• Tachycardia, cardiac dilation, and systolic murmurs
TOPNOTCH MEDICAL BOARD PREP PEDIATRICS MAIN DIGITAL HANDOUT BY DR. PUNONGBAYAN AND DR. DE VERA Page 40 of 105
For inquiries visit www.topnotchboardprep.com.ph or https://www.facebook.com/topnotchmedicalboardprep/
This handout is only valid for the March 2021 PLE batch. This will be rendered obsolete for the next batch since we update our handouts regularly.
TOPNOTCH MEDICAL BOARD PREP PEDIATRICS MAIN DIGITAL HANDOUT BY DR. PUNONGBAYAN AND DR. DE VERA
For inquiries visit www.topnotchboardprep.com.ph or https://www.facebook.com/topnotchmedicalboardprep/
This handout is only valid for the March 2021 PLE batch. This will be rendered obsolete for the next batch since we update our handouts regularly.
ALPHA THALASSEMIA
• due to a decrease in alpha-globin chain synthesis
• common in Asians and African-Americans
• symptoms depend on how many of the four foci are deleted or
mutated
• 1/4 foci (silent): asymptomatic
• 2/4 foci (thalassemia trait): mild anemia
• 3/4 foci (Hemoglobin H disease): splenomegaly, increased ß4
• 4/4 foci (hydrops fetalis): incompatible with life
• Dx: decreased reticulocyte count (due to ineffective
hematopoiesis), microcytic, hypochromic RBCs, normal RDW,
target cells and Heinz bodies on PBS
o *definitive diagnosis with Hb electrophoresis
• Tx: frequent transfusions, chelation therapy with Deferoxamine
as needed, splenectomy

Increases ↑, Decreases ↓
Decreases ↓, Increases ↑, Increases ↑, Decreases ↓,
Answers: Decreases ↓,

• Why are the measurements of the total iron binding capacity


important?
o TIBC is high in iron deficiency anemia and low in anemia of
chronic disease. Both illnesses have low serum iron levels. McGraw-Hill Education
BETA THALASSEMIA
One of the basic tests for anemia is CBC and RBC indices. IDA and
Thalassemia both present as micro, hypo anemia with low reticulocyte • due to a decrease in beta-globin chain synthesis or absence of beta
count. IDA has high RDW while thalassemia has normal RDW! chain of hemoglobin
Dr. De Vera • common in people of Mediterranean origin, and SEA descent
• types:
MANAGEMENT o Beta-thalassemia major (Cooley anemia)
• Prevention § no beta-globin production
o Premature infants: 7.5mg OD for 3 months § anemia at 6 mos (decline in normal Hgb F & rise in Hgb A),
o 4mo-11mo: 15mg OD for 3 months splenomegaly, extramedullary hematopoiesis)
o 1-5 y/o: 30mg OD for 3 months o Beta-thalassemia minor
o Adolescents: Iron 60mg + Folic Acid 400mcg § asymptomatic carriers, mild or no anemia
• Treatment • What is the pathophysiology of this condition?
o 3-6mg/kg/day elemental iron in 2-3 div doses o ↓ or absent beta chain of Hgb → normal alpha Hgb chains build
o Max 150-200mg per day up → form insoluble aggregates that precipitate within the
o 2-3 months until blood values normalize RBCs → hemolysis or damage to RBCs → susceptible to
• Mild anemia only, repeat CBC after 4 weeks macrophage destruction & splenic sequestration → small &
o Increase in Hb by 1-2g/dL pale RBCs
• Expected rise in Hb = 0.1-0.4g/dL/day depending on the severity • Dx: same as alpha-thalassemia, definitive diagnosis with Hb
of anemia (more severe anemia = more rapid rise in Hb) electrophoresis (increased HbF, HbA2)
• Blood transfusion rarely necessary • Tx: aggressive transfusions, chelation therapy as needed,
hydroxyurea, splenectomy (to treat the resultant hypersplenism)
RESPONSE TO IRON THERAPY o *Beware of Yersinia enterocolitica infection which eats up the
12-24 hours Subjective improvements in patients free iron!
26-28 hours Initial bone marrow response
EVALUATION OF MICROCYTIC ANEMIA
28-72 hours Reticulocytosis Iron Lead Chronic
Thalassemia
4-30 days Increasing hemoglobin level Deficiency poisoning disease
Hemoglobin ↓ N or ↓ ↓ ↓
1-3 months Repletion of iron stores MCV ↓ N or ↓ N or ↓ N or ↓
RDW ↑ N or ↑ N N or ↑
✔ GUIDE QUESTION Number of
An 18-month-old boy was brought to the clinic by his mother when she RBCs
↓ ↓ N or ↑ N or ↓
noted him to be sallow-looking and dyspneic for about 2 weeks. The Serum Iron ↓ N N ↓
mother claims her son has been healthy up until now. PE reveals TIBC ↑ N N ↓
tachycardia, pale mucous membranes. Lab results: Hgb 5.5 g/dL,
decreased reticulocyte count, RBC indices showed microcytic
Ferritin ↓ N N ↑
hypochromic RBCs with normal RDW, normal WBC and platelet. What
is your impression? ✔ GUIDE QUESTION
A. IDA C. Acute Leukemia An 8-month-old male is noted to have jaundice & dyspnea. PE reveals
B. Aplastic Anemia D. Thalassemia tachycardia and splenomegaly. The mother recalls a family history of
“blood disease”. Lab results: Hgb 8.5 g/dL, Hct 29%, MCV 85 fl (NV=
80-100 fl), MCH is normal. Reticulocyte and MCHC increased. WBC and
THALASSEMIAS Platelet counts are normal. What is the most likely diagnosis?
• Inherited microcytic hemolytic anemia due to abnormal A. G6PD deficiency
hemoglobin synthesis B. DIC
• Normal predominant HbA is tetramer made up of 2 alpha globin C. Spherocytosis
chains and 2 beta globin chains D. ITP
• Result when 1 or more of globin genes mutates In the exams, when MCHC is increased, chances are the
• Clinical phenotype determined by which and how many genes examiner is looking for spherocytosis. Among the indices we
are mutated usually look at MCH and MCV. MCHC is important for the
diagnosis of spherocytosis. Normochromic, normocytic RBC
with increased MCHC points to spherocytosis.
Dr. De Vera

TOPNOTCH MEDICAL BOARD PREP PEDIATRICS MAIN DIGITAL HANDOUT BY DR. PUNONGBAYAN AND DR. DE VERA Page 41 of 105
For inquiries visit www.topnotchboardprep.com.ph or https://www.facebook.com/topnotchmedicalboardprep/
This handout is only valid for the March 2021 PLE batch. This will be rendered obsolete for the next batch since we update our handouts regularly.
TOPNOTCH MEDICAL BOARD PREP PEDIATRICS MAIN DIGITAL HANDOUT BY DR. PUNONGBAYAN AND DR. DE VERA
For inquiries visit www.topnotchboardprep.com.ph or https://www.facebook.com/topnotchmedicalboardprep/
This handout is only valid for the March 2021 PLE batch. This will be rendered obsolete for the next batch since we update our handouts regularly.
• Complications of Sickle Cell Disease:
o Painful (vaso-occlusive) crisis
o Aplastic crisis
o Splenic sequestration crisis leading to auto-splenectomy →
↑susceptibility to encapsulated organisms
o Priapism
o Stroke
o Leg ulcers
o Acute chest syndrome (fat emboli & infection)
• Important Facts re: SCD
o How prevalent is this?
§ Autosomal recessive disorder with an 8% carrier rate in
African Americans
§ About 0.2% of African Americans
o What are the typical radiologic findings?
§ Erythropoiesis must increase to compensate for hemolysis →
marrow expansion → resorption of bone → new bone
formation on the external aspect of the skull (“crew cut” or
“hair on end” appearance on skull radiographs)
HEMOLYTIC ANEMIA
• Results from a decrease in RBC survival
• Common findings are pallor, jaundice due to increased indirect
bilirubin, increased LDH,
• Normocytic, normochromic RBCs with increased retic ct.,
characteristic cells on PBS
• Etiologies:
o intrinsic RBC defects (structural or metabolic abnormalities)
o autoimmune destruction
o intravascular destruction

HEMOLYTIC ANEMIA DUE TO INTRINSIC RBC DEFECTS


SPHEROCYTOSIS
• Defect in membrane spectrin or ankyrin (erythrocyte skeletal
proteins) → ↓membrane-volume ratio → causing loss of Radiographic appearance of sickle cell disease
biconcave shape → cells become more fragile → cells trapped in
the spleen & destroyed • Dx: sickle cells and Howell-Jolly bodies on PBS; definitive
• Autosomal dominant diagnosis with Hb electrophoresis (HbS)
• What blood tests would be useful to establish the diagnosis?
o Osmotic fragility test confirms the presence of fragile sphere-
shaped RBCs.
o MCHC is increased due to a reduction in membrane surface
area
o MCV remains normal because the overall volume remains
stable.
o High reticulocyte count
o ↑ indirect bilirubin level • Tx: analgesia and hydration for acute crises, hydroxurea,
• Treatment of spherocytosis: vaccinations against encapsulated organisms, folate
o Splenectomy is curative and should be considered in patients supplementation
with more severe disease. ✔ GUIDE QUESTIONS
o Folate supplementation may also be useful. A 3-year-old child presents with a petechial rash but is otherwise well
and without physical findings. Platelet count is 20,000/cu mm.
✔ GUIDE QUESTION Hemoglobin and WBC count are normal. The most likely diagnosis is:
An 18-year-old African-American female visiting in Manila complains A. Systemic lupus erythematosus
of episodes of extreme pain and discomfort in her legs and lower back. B. Idiopathic Thrombocytopenic Purpura
She has been experiencing these recurrent episodes, with extreme C. Acute Lymphoblastic Leukemia
fatigue, since she was a child. On PE, she is jaundiced with a Hct 23% D. Disseminated Intravascular Coagulopathy
and Hgb of 7 g/dL. She reports she has family members who
experienced the same symptoms. What is your most likely diagnosis? IDIOPATHIC
A. Hereditary Spherocytosis
B. G6PD Deficiency THROMBOCYTOPENIC
C. DIC PURPURA
D. Sickle Cell Disease https://qrs.ly/anbj8ax

SICKLE CELL DISEASE An 18-month-old male child was brought to the ER because of swelling
of bilateral ankles and hematoma of the thighs. No previous bleeding
• What is the pathophysiology?
episodes were noted. PE revealed bilateral ankle swelling, hematoma,
o Develops at around 6 mos old when HbS (result of a single and ecchymosis over the thighs, what is the most likely diagnosis in this
missense mutation in the B-globin gene of Hgb) replaces HbF case?
o Episodes of painful crisis due to hypoxic tissue injury from A. Von Willebrand Disease
microvascular occlusions B. Idiopathic Thrombocytopenic Purpura
o Hgb becomes susceptible to polymerization in conditions of C. Hemophilia
low oxygen or dehydration → reduces flexibility of the RBC D. Henoch Schönlein Purpura
membrane In relation to the above case, what is the expected laboratory finding?
A. Normal platelet count, increased bleeding time, normal PT,
o Any organ can be affected by vascular congestion, thrombosis,
prolonged PTT
and infarction caused by sickling cells B. Normal platelet count, normal bleeding time, normal PT,
prolonged PTT
C. Decreased platelet count, increased bleeding time, normal PT,
normal PTT
D. Decreased platelet count, increased bleeding time, prolonged PT,
prolonged PTT
TOPNOTCH MEDICAL BOARD PREP PEDIATRICS MAIN DIGITAL HANDOUT BY DR. PUNONGBAYAN AND DR. DE VERA Page 42 of 105
For inquiries visit www.topnotchboardprep.com.ph or https://www.facebook.com/topnotchmedicalboardprep/
This handout is only valid for the March 2021 PLE batch. This will be rendered obsolete for the next batch since we update our handouts regularly.
TOPNOTCH MEDICAL BOARD PREP PEDIATRICS MAIN DIGITAL HANDOUT BY DR. PUNONGBAYAN AND DR. DE VERA
For inquiries visit www.topnotchboardprep.com.ph or https://www.facebook.com/topnotchmedicalboardprep/
This handout is only valid for the March 2021 PLE batch. This will be rendered obsolete for the next batch since we update our handouts regularly.

We consider Hemophilia if there is an ISOLATED PTT derangement. What is the most likely diagnosis?
Looking at the pathway below, the most common form of Hemophilia A • Von Willebrand disease - the most common inherited bleeding
(VIII) or B (IX) affects the intrinsic pathway. Thus, PTT derangement. disorder
• Result of a quantitative (type 1 or 3) or qualitative (type 2) defect in
An interesting fact that most students miss out is that Vit K deficiency / vWF
warfarin toxicity will present with BOTH PT and PTT derangement.
• vWF is a large protein made by endothelial cells and
Again, look at the pathway below. In this disease, Vit K dependent factors
megakaryocytes; a carrier for factor VIII and is a cofactor for platelet
IX, X, VII, II are affected. Both intrinsic and extrinsic. Just so happens that
adhesion
VII has the shortest half-life so PT is a better test because it will be
deranged before PTT does.
Dr. De Vera
VON WILLEBRAND DISEASE
• What clinical findings are commonly associated with this
condition?
o Disturbs both primary & secondary hemostasis
o Role in platelet adhesion to exposed subendothelium leads to
increased bleeding time → mucous membrane bleeding,
petechiae, purpura
o Often have a (+) family history
• Dx:
o ↓ vWF levels
o ↓ Factor VIII
o prolonged bleeding time
o abnormal platelet adhesion
o ↑ PTT
o Ristocetin cofactor assay (measures vWF antigen levels and
activity)
TREATMENT OF VON WILLEBRAND DISEASE
• Mild bleeding in type 1: desmopressin (causes release of vWF
from endothelial stores)
• Severe disease: Factor VIII concentrates which contain high vWF
Ag

LABORATORY FINDINGS IN BLEEDING DISORDERS


BLEEDING PLATELET
PT PTT
TIME COUNT
Hemophilia
SpringerLink N ↑ N N
A&B
HEMOPHILIA vWd N ↑ ↑ N or ↓
• X-linked recessive deficiencies of – ITP N N ↑ ↓
o factor VIII (Hemophilia A) – more common
Vitamin K
o factor IX (Hemophilia B or Christmas disease) ↑ N or ↑ N N
deficiency
• severity of bleeding is proportional to factor levels
DIC ↑ ↑ ↑ ↓
o mild (5-25% normal activity): bleeding only after major trauma
or surgery ✔ GUIDE QUESTIONS
o moderate (1-5%): bleeding with mild trauma Most children with leukemia present with:
o severe (<1%): spontaneous bleeding A. Unexplained fever, pallor, and hemorrhage
MISCELLANEOUS B. Hepatomegaly
HEMATOLOGY C. Massive GI bleeding
D. Abdominal pain
What is the most common
Which type of leukemia in childhood will have the best response to
hereditary bleeding von Willebrand Disease chemotherapy?
disorder? A. AML
What is the most common B. ALL
and most serious congenital C. CML
Hemophilia A
coagulation factor D. CLL
deficiencies?
What is the most common ACUTE LYMPHOCYTIC LEUKEMIA
hereditary hypercoagulable Factor V Leiden SIGNS/SYMPTOMS OF ALL
disorder? • acute onset < 4 wks duration of Sx
What is the hallmark of • non-specific (anorexia, irritability, lethargy)
Prolonged bleeding
hemophilia?
• signs of marrow failure (anemia, bleeding, purpuric/petechial
What is the earliest joint
Ankles lesions, low-grade fever)
hemorrhages in children?
• signs of infiltration (bone pain, lymphadenopathy, splenomegaly
✔ GUIDE QUESTIONS > hepatomegaly)
A 2 year-old boy is brought to his doctor by his parents who are
For the board exam, please study ALL. This is the most common childhood
concerned about the multiple bruises on the boy’s shins and hands.
malignancy.
They report the child seems to get large bruises with minimal injury
and bleeds profusely when his teeth are brushed. On questioning, they A similar disease is Aplastic Anemia (AA). Both presents with
reveal the child has a grandmother with a bleeding disorder. Lab tests: PANCYTOPENIA (in CBC affectation of 2 or more cell lines: RBC, WBC
bleeding time 14 mins, PT 12 secs, PTT 41 secs. What is the platelet). The difference is that leukemia is pancytopenia +
management of choice for this disease? hypercellularity (blasts), while AA is pancytopenia + hypocellularity. On
A. Desmopressin PE leukemia presents with SIGNS OF INFILTRATION while AA does not.
B. Platelet Transfusion Dr. De Vera

C. Factor IX Transfusion WHAT IS THE ETIOLOGY OF THESE PE FINDINGS?


D. Steroids or IVIG • Result from leukemic expansion and crowding out of the normal
marrow
• Anemia and thrombocytopenia, bone or joint pain from invasion
VON WILLEBRAND into the periosteum
DISEASE • Fever results from pyrogenic cytokines released from leukemic cells
https://qrs.ly/oibj8ag • Painless enlargement of the scrotum & CNS symptoms -
extensive extramedullary invasion
TOPNOTCH MEDICAL BOARD PREP PEDIATRICS MAIN DIGITAL HANDOUT BY DR. PUNONGBAYAN AND DR. DE VERA Page 43 of 105
For inquiries visit www.topnotchboardprep.com.ph or https://www.facebook.com/topnotchmedicalboardprep/
This handout is only valid for the March 2021 PLE batch. This will be rendered obsolete for the next batch since we update our handouts regularly.
TOPNOTCH MEDICAL BOARD PREP PEDIATRICS MAIN DIGITAL HANDOUT BY DR. PUNONGBAYAN AND DR. DE VERA
For inquiries visit www.topnotchboardprep.com.ph or https://www.facebook.com/topnotchmedicalboardprep/
This handout is only valid for the March 2021 PLE batch. This will be rendered obsolete for the next batch since we update our handouts regularly.
PROGNOSTIC / PREDICTIVE FACTORS NEUROBLASTOMA
• The single most important prognostic factor in ALL is the
• embryonal tumor of neural crest cell origin
treatment.
• 3rd most common pediatric cancer
• 3 of the most important predictive factors:
• 8% of childhood malignancies
1. Age of the patient at the time of diagnosis
2. Initial leukocyte count • Neuroblasts (pluripotent stem cells) invaginate and migrate along
3. Speed of response to treatment the neuroaxis where neural crest cells are present – sympathetic
§ Average risk: age between 1-10 yrs old and leukocyte count ganglia, adrenal glands
of <50,000/uL
SUPPLEMENT: QUICK SHEET
SUPPLEMENT: QUICK SHEET DIFFERENTIAL DIAGNOSIS OF SMALL, ROUND, BLUE-CELL
IMPORTANT FACTS RE: ALL TUMORS:
• sites of relapse: bone marrow, CNS (increased ICP and isolated • Wilms tumor
cranial nerve palsies), testes (painless swelling of one or both • Acute leukemia
testes in 1-2% of males) • Rhabdomyosarcoma
• Where does it spread? Liver, spleen, lymph nodes • Mesothelioma / Medulloblastoma
• poor prognostic factors: • Ewing sarcoma
o < 2 yrs or > 10 yrs • Retinoblastoma
o male
• Primitive neuroectodermal tumor (PNET)
o WBC > 100,000 u/L on presentation
• Neuroblastoma
o presence of CNS leukemia
*** W-A-R-M-E-R-P-N (warmer in the Philippines)
o presence of a mediastinal mass
✔ GUIDE QUESTION NEUROBLASTOMA
An asymptomatic 2-year-old girl is noted to have an abdominal mass on • median age at diagnosis is 2 yrs old & 90% of cases are diagnosed
the left flank by her mother. IVP reveals an enlarged kidney with by 5 yrs old
distorted pelvis and calyces. The px most likely has:
A. Wilms tumor C. Adrenal tumor
• associated with:
B. Malignant lymphoma D. Granulosa cell tumor o N-myc oncogene – tuberous sclerosis
o Neurofibromatosis – Pheochromocytoma
o Hirschsprung disease
WILMS TUMOR • S/Sx: reflect the tumor site & extent of disease; non-tender
• renal tumor of embryonal origin; 2nd most common malignant abdominal mass which may cross the midline, Horner syndrome,
abdominal tumor in childhood hypertension, cord compression (from a paraspinal tumor)
• 2-5 years old • Metastases: periorbital bruising (“raccoon eyes”),
• Incidence of bilateral Wilms tumor is 7% subcutaneous tumor nodules, opsoclonus /myoclonus
• associated with: (“dancing eyes, dancing feet”)
o Neurofibromatosis • most common sites of metastasis: long bones & skull, BM, liver,
o Beckwith-Wiedemann syndrome (hemihypertrophy, lymph nodes, skin
visceromegaly, macroglossia) • Dx: abdominal CT scan (calcification & hemorrhage), 24-hr urine
o WAGR syndrome (Wilms tumor, Aniridia, Genitourinary VMA & HVA (elevated in 95% of cases), BUN/Crea, CXR, bone
abnormalities, mental retardation) scan, LFTs, CBC
• International Neuroblastoma Staging System
SUPPLEMENT: QUICK SHEET • Prognostic factors: age at diagnosis, stage of disease, Shimada
IMPORTANT FACTS RE: WILMS TUMOR histology
• S/Sx: painless abdominal pain with flank mass that does not
cross the midline, hematuria (12-25%), hypertension INSS
• Dx: abdominal UTZ, CT scan or MRI, abdominal X ray, liver & • St 1: tumors confined to the organ or structure of origin
kidney function tests, chest x ray (pulmonary metastasis) • St 2: tumors extend beyond the structure of origin but not across
• Tx: transabdominal nephrectomy & post-surgical chemoTx the midline with (2B) or without (2A) ipsilateral lymph node
• Worse prognosis: large tumor (>500 gms), st III & IV, involvement
unfavorable histologic type (anaplastic) • St 3: extend beyond the midline w/ or w/o bilateral lymph node
• More than 60% of patients generally survive involvement
• St I-III have a cure rate of >90% • St 4: disseminated to distant sites (bone, BM, liver, distant lymph
nodes)
✔ GUIDE QUESTIONS
• St 4S: <1 yr old w/ dissemination to liver, skin, or BM w/o bone
A 9-year-old girl is receiving chemotherapy for ALL. The drug of first
choice for PCP prophylaxis is:
involvement & with a primary tumor
Answers: Co-Trimoxazole
PROGNOSIS OF NEUROBLASTOMA
A 2-year-old male child is brought to the clinic due to an abdominal • With localized disease: regardless of age, 80-90% 5-yr survival
swelling which the mother found out while she was bathing her child. rate
The child has no complaints, but the mother has seen occasional jerky • Overall, at younger age at diagnosis carries a more favorable
movements of his leg. On PE, the mass palpated was found to cross the
prognosis
midline, there were also some bluish subcutaneous nodules palpated.
What is the most likely diagnosis in this case? • 5-yr survival rate:
A. Wilms tumor C. Lymphoma o 83% for infants
B. Neuroblastoma D. Rhabdomyosarcoma o 55-60% for 1-5 yrs old
In relation to the case above, which of the ff. approaches should you o 40% for children >5 yrs old
request to completely diagnose the case?
A. Bone marrow aspirate + urine VMA and HVA TREATMENT
B. Bone marrow aspirate and CT scan • Low risk St 1 & 2: surgery
C. Urine VMA and HVA + CT scan
D. VMA and CT scan • Observation for St 4S (nearly 100% survival)
• St 2: cure rate is >90%
In relation to the above case, which among the ff. is the mainstay in the • Local recurrence: chemotx and radiation
treatment of this condition?
• If with spinal cord compression at Dx: chemotx, surgery, or
A. Surgical Resection C. Chemotherapy
B. Await Spontaneous Resolution D. Radiation Therapy radiation
• Intermediate risk: chemotx (cisplatin, cyclophosphamide,
In relation to the case above, which among the ff. is the most common etoposide & doxorubicin), surgery, or radiation
site of metastasis?
• St 3 & 4: surgery + chemotherapy; 90% survival
A. Lungs C. Brain
B. Bones D. Liver • High-risk: chemotx, stem cell transplantation, surgery, radiation

TOPNOTCH MEDICAL BOARD PREP PEDIATRICS MAIN DIGITAL HANDOUT BY DR. PUNONGBAYAN AND DR. DE VERA Page 44 of 105
For inquiries visit www.topnotchboardprep.com.ph or https://www.facebook.com/topnotchmedicalboardprep/
This handout is only valid for the March 2021 PLE batch. This will be rendered obsolete for the next batch since we update our handouts regularly.
TOPNOTCH MEDICAL BOARD PREP PEDIATRICS MAIN DIGITAL HANDOUT BY DR. PUNONGBAYAN AND DR. DE VERA
For inquiries visit www.topnotchboardprep.com.ph or https://www.facebook.com/topnotchmedicalboardprep/
This handout is only valid for the March 2021 PLE batch. This will be rendered obsolete for the next batch since we update our handouts regularly.
In WILMS vs Neuroblastoma. Please note the ff differences: origin (renal ✔ GUIDE QUESTIONS
vs elsewhere), crosses midline, associated symptoms, CT scan findings. A 16-year-old male patient presents to the clinic due to weight loss over
Dr. De Vera the past 3 months, body malaise, (+) night sweats, and intermittent
SUPPLEMENT: QUICK SHEET febrile episodes of 38-39°C. There was occasional dry cough, no
dyspnea, occasional episodes of epistaxis. The patient had some cough
Wilms Tumor vs. Neuroblastoma and colds 1 month ago, (+) episode of flu 2 weeks ago. No other relevant
Wilms Tumor Neuroblastoma history. Examination revealed: BP 90/60, HR 108, RR 20, T >37.2°C,
Location • Anywhere along the pale palpebral conjunctiva, pale buccal mucosa, (+) multiple nontender
• Kidneys neuroaxis, most cervical and supraclavicular lymphadenopathy, no splenomegaly, clear
commonly adrenals bilateral breath sounds, what is the next best step in the diagnosis in
this px?
Crosses
• No • May cross A. CXR C. urinalysis
midline? B. ECG D. lymph node biopsy
Associated • Many In relation to the above case, what is the most likely diagnosis?
symptoms (hypertension, A. Pulmonary Tuberculosis
• Hypertension, hematuria, B. Infectious Mononucleosis
hematuria opsoclonus, cord C. Hodgkin Lymphoma
compression, D. HIV acute retroviral syndrome
racoon eyes, etc.)
Metastasis • Lungs • Bones LYMPHOMAS
Others • CT scan usually • neoplasms of lymphoid cells (lymphocytes, histiocytes & their

shows calcifications precursors) - 6% of childhood cancers
• Ann Arbor Staging Classification:
o Stage I one lymph node group involved
o Stage II two lymph node groups involved on the same side of
the diaphragm
o Stage III two lymph node groups involved on both sides of the
diaphragm or extra-lymphatic involvement
o Stage IV diffuse extra-lymphatic site involvement (usually
bone marrow, CNS, liver)
HODGKIN NON-HODGKIN
• Painless, non-tender, firm, rubbery cervical or supraclavicular adenopathy
Hx • B symptoms important in staging (weight loss >10% of TBW over 3 months, unexplained high-grade fever,
drenching night sweats)
• systemic adenopathy
PE • regional adenopathy hepatosplenomegaly is rare • more extranodal involvement
• hepatosplenomegaly

• node biopsy, whole body CT Scan, BM


Dx • node biopsy, CXR, BM biopsy, LP
biopsy, LP

• Chemotherapy (COPAD: cyclophosphamide, vincristine, prednisone, • radiation for CNS involvement


Tx doxorubicin) or COMP: cyclophosphamide, vincristine, methotrexate, • chemotherapy for B Sx, St. III & IV
prednisone, 6-mercaptopurine • possible BMT
• bimodal age distribution
• 60% of all lymphomas in children &
Peak age & risk • 20-30 yrs & > 50 yrs; developing countries: early peak before
adolescents; ↑ prevalence with age
factors adolescence; M>F
• EBV (Burkitt lymphoma)
• EBV, CMV, HHV-6
Histology • Reed-Sternberg cell (arises from the germinal center of B cells)
Distribution • contiguous lymph node spread • non-contiguous lymph node spread
• nodular sclerosis
Variants • mixed cellularity lymphocyte-predominant (best prognosis) • low, intermediate & high-grade
• lymphocyte-depleted
✔ GUIDE QUESTIONS
A 10 year old male child presents with pallor, headaches, palpitations PANCYTOPENIAS
and easy fatigability. He also has noted gingival bleeding when
brushing his teeth. He has a history repeated flu like symptoms over 1. FANCONI ANEMIA
the past 3 months. No history of rashes and solvent exposure, no • autosomal recessive
weight loss or anorexia, no muscle or joint pains. PE revealed: pale • aplastic anemia with microcephaly, microphthalmia, hearing
palpebral conjunctiva, pale buccal mucosa, no lymphadenopathy, no loss, limb anomalies (absent radii & thumbs)
splenomegaly. A CBC was done which revealed pancytopenia, what is
• Dx: clinical, cytogenetic analysis
the most likely diagnosis in this case?
A. AML C. Non-Hodgkin Lymphoma • Tx: steroids, BMT, supportive care
B. Myelofibrosis D. Aplastic Anemia
• Aplastic anemia – results from failure or autoimmune destruction of 2. APLASTIC ANEMIA
myeloid stem cells → pancytopenia • associated with ionizing radiation exposure, drugs, viral
• What is the most likely cause? infections
o Most cases are idiopathic. Other possible causes: viruses • Dx: clinical, BMA (hypocellular marrow))
(parvovirus B19, hepatitis viruses, HIV, Epstein-Barr virus), drugs
& chemicals (alkylating & antimetabolite agents, chloramphenicol,
insecticides, benzene), and radiation

APLASTIC ANEMIA
• What are the typical peripheral blood smear findings in this
condition?
o Hypocellularity and pancytopenia
• What other test would be useful in confirming the
diagnosis?
o Bone marrow biopsy (hypocellular bone marrow with a fatty
infiltrate)
TOPNOTCH MEDICAL BOARD PREP PEDIATRICS MAIN DIGITAL HANDOUT BY DR. PUNONGBAYAN AND DR. DE VERA Page 45 of 105
For inquiries visit www.topnotchboardprep.com.ph or https://www.facebook.com/topnotchmedicalboardprep/
This handout is only valid for the March 2021 PLE batch. This will be rendered obsolete for the next batch since we update our handouts regularly.
TOPNOTCH MEDICAL BOARD PREP PEDIATRICS MAIN DIGITAL HANDOUT BY DR. PUNONGBAYAN AND DR. DE VERA
For inquiries visit www.topnotchboardprep.com.ph or https://www.facebook.com/topnotchmedicalboardprep/
This handout is only valid for the March 2021 PLE batch. This will be rendered obsolete for the next batch since we update our handouts regularly.
TREATMENT OF APLASTIC ANEMIA PLEASE READ MORE ON:
• RBC and platelet transfusion • Staging of Tumors
• Allogenic BM transplant (sibling) • Soft Tissue Malignancies
• Granulocyte colony-stimulating factor or granulocyte o Rhabdomyosarcoma
macrophage colony-stimulating factor • Osteosarcoma
• Withdrawal of any toxic causative agent • Ewing Sarcoma
✔ GUIDE QUESTION Chemotherapeutic Regimens
A 2 year old male patient is brought to the clinic by his mother for an
abdominal mass which she incidentally found while bathing the child. NEUROLOGY
There was no other associated symptoms. PE: revealed an abdominal SEIZURES & EPILEPSY
mass from the right quadrant extending past the midline. A CT scan
was done which revealed that the mass had calcifications and • Seizures/convulsions are paroxysmal, time-limited changes
hemorrhage. What is the most likely diagnosis in this case? in motor activity and/or behavior that result from abnormal
A. Wilms tumor electrical activity in the brain
B. Neuroblastoma o Transient occurrence of signs/symptoms resulting from
C. Non-Hodgkin Lymphoma abnormal excessive or synchronous neuronal activity in
D. Rhabdomyosarcoma the brain
MISCELLANEOUS ONCOLOGY • Seizure disorder – a general term to include any one of the ff:
epilepsy, febrile seizures, and seizures secondary to metabolic,
Most common malignancy? Leukemia (ALL) infectious, or other etiologies
• Epilepsy – disorder of the brain characterized by an enduring
Most common solid tumor? Brain Tumors predisposition to generate seizures and by the neurobiological,
cognitive, psychological, and social consequences of this
Most common solid tumor condition; two or more unprovoked seizures occurring in a
Neuroblastoma
outside CNS? time frame of >24 hrs
Most common soft tissue • Epileptic syndrome – disorder that manifests one or more
Rhabdomyosarcoma
tumor? specific seizure types and has a specific age of onset and
Malignancy with highest prognosis
Brain (PNET)
mortality? • Seizure type type is the primary determinant of the type of
medications the patient is likely to respond to
• Epilepsy syndrome determines the type of prognosis one could
expect

TOPNOTCH MEDICAL BOARD PREP PEDIATRICS MAIN DIGITAL HANDOUT BY DR. PUNONGBAYAN AND DR. DE VERA Page 46 of 105
For inquiries visit www.topnotchboardprep.com.ph or https://www.facebook.com/topnotchmedicalboardprep/
This handout is only valid for the March 2021 PLE batch. This will be rendered obsolete for the next batch since we update our handouts regularly.
TOPNOTCH MEDICAL BOARD PREP PEDIATRICS MAIN DIGITAL HANDOUT BY DR. PUNONGBAYAN AND DR. DE VERA
For inquiries visit www.topnotchboardprep.com.ph or https://www.facebook.com/topnotchmedicalboardprep/
This handout is only valid for the March 2021 PLE batch. This will be rendered obsolete for the next batch since we update our handouts regularly.

Nelson Textbook of Pediatrics, 20th ed.


CATEGORY OF EPILEPTIC SEIZURES III. Unknown
• International Classification of Epileptic Seizures A. Motor
• Focal Seizures B. Non-motor
o Formerly known as partial seizure C. Unclassified
o initial activation of a system of neurons limited to part of 1 ✔ GUIDE QUESTION
cerebral hemisphere A 4-year-old male was brought to your clinic for evaluation. Patient
o Subdivided into: was noted to wake up at night with twitching of the right side of face
§ Focal Seizures without impairment of consciousness and drooling. Patient is unable to speak during the episodes but is
(previously known as simple partial seizures) awake and follows command. PE is unremarkable. MRI is normal. ECG
§ Focal Seizures with impairment of consciousness a.k.a. focal showed broad-based centro-temporal spikes. This type of benign
dyscognitive seizures (previously complex partial seizures) epilepsy syndrome with focal seizure is expected to be outgrown at
• Generalized Seizures what age
A. Pre-pubertal
o Clinical and EEG changes indicate synchronous involvement of
B. Adolescence
all or both hemispheres C. Adulthood
• Febrile Seizures D. School age period
NOMENCLATURES FOR SEIZURE It’s called “rolandic” because seizures originate in the Rolandic
• Acute symptomatic seizures – occur secondary to an acute area of the brain, the one that controls the face. Benign Rolandic
problem affecting brain excitability (ex. electrolyte imbalance) epilepsy is also called “benign childhood epilepsy with
centrotemporal strikes.”
• Unprovoked seizure – not an acute symptomatic seizure Dr. Punongbayan
• Remote symptomatic seizure – secondary to a distant brain
injury (ex. an old stroke) BENIGN EPILEPSY SYNDROMES WITH FOCAL SEIZURES
• Reflex seizures - precipitated by a sensory stimulus (ex. flashing
• Most common type is benign childhood epilepsy with
lights)
centrotemporal spike
International League Against Epilepsy (ILAE) classification of o 3-10 y/o
etiology of epilepsy o Outgrown during adolescence
• Genetic epilepsy (previously known as idiopathic) o Focal seizure with buccal and throat tingling and tonic or
o direct result of a known or presumed genetic defect(s) in clonic contractions of 1 side of the face, with drooling and
which the genetic defect is not causative of a brain structural inability to speak but with preserved consciousness and
or metabolic disorder other than the epilepsy comprehension
• Structural/metabolic epilepsy o ECG = broad-based centrotemporal spikes
(previously “symptomatic epilepsy”) o MRI normal
o distinct structural or metabolic entity that causes o Responds well to AED such as carbamazepine
seizure/epilepsy • Benign Epilepsy with occipital spikes
• Unknown Epilepsy (previously “cryptogenic epilepsy”) o Panayiotopoulos type
o Cause is unknown § Early childhood
§ Complex partial seizures with ictal vomiting
ILAE 2017 CLASSIFICATION OF SEIZURE TYPES: o Gastaut Type
FOCAL § Later childhood
(either aware or UNKNOWN § Complex partial seizure, visual auras, migraine headache
GENERALIZED
impaired ONSET o Both are outgrown in few years
awareness)
MOTOR MOTOR MOTOR ✔ GUIDE QUESTION
• Automatisms • Tonic • Tonic-clonic An 8-month-old baby boy was brought to your clinic for evaluation.
• Tonic • Clonic • Epileptic spasms Patient was noted to have episodes of truncal flexion described by
parents “as if doing sit ups” that occur multiple times throughout the
• Clonic • Tonic-clonic (trunk flexion)
day. Patient was also noted to have lost the ability to sit without
• Myoclonic • Atonic support which he was able to do a month ago. EEG shows high voltage,
• Atonic (limp) slow, chaotic background with multifocal spikes. What is your
• Epileptic spasms • Myoclonic diagnosis?
• Hyperkinetic a. Lennox-Gastaut Syndrome
NON-MOTOR NON-MOTOR NON-MOTOR b. West Syndrome
c. Ohtahara Syndrome
• Emotional (Absence) • Behavior arrest
d. Dravet Syndrome
• Sensory • Typical (pausing,
• Autonomic • Atypical freezing, activity West Syndrome
arrest) • Usually between 2 to 12 months
• Behavior arrest • Myoclonic
• Triad:
• Cognitive o Infantile epileptic spasms
UNCLASSIFIED o Developmental regression
o EEG = hypsarrhythmia
SUPPLEMENT: QUICK SHEET Dr. Punongbayan

ILAE (International League Against Epilepsy) 2017


Classification of Seizure Types: SEVERE GENERALIZED EPILEPSIES
I. Focal (either aware or impaired awareness) • Severe generalized epilepsies are associated with intractable
A. Motor onset (automatisms, myoclonic, etc.) seizures and developmental delay
B. Non-motor onset (autonomic, emotional, etc.) • Early myoclonic infantile encephalopathy
II. Generalized o starts during the 1st 2 months of life with severe myoclonic
A. Motor (tonic-clonic, tonic, clonic, etc.) seizures and burst suppression pattern on EEG
B. Non-motor (absence) o usually caused by inborn errors of metabolism
TOPNOTCH MEDICAL BOARD PREP PEDIATRICS MAIN DIGITAL HANDOUT BY DR. PUNONGBAYAN AND DR. DE VERA Page 47 of 105
For inquiries visit www.topnotchboardprep.com.ph or https://www.facebook.com/topnotchmedicalboardprep/
This handout is only valid for the March 2021 PLE batch. This will be rendered obsolete for the next batch since we update our handouts regularly.
TOPNOTCH MEDICAL BOARD PREP PEDIATRICS MAIN DIGITAL HANDOUT BY DR. PUNONGBAYAN AND DR. DE VERA
For inquiries visit www.topnotchboardprep.com.ph or https://www.facebook.com/topnotchmedicalboardprep/
This handout is only valid for the March 2021 PLE batch. This will be rendered obsolete for the next batch since we update our handouts regularly.
• Early infantile epileptic encephalopathy
(Ohtahara syndrome)
o similar age of onset and EEG but manifests tonic seizures and
is usually caused by brain malformations or syntaxin
binding protein 1 mutations.
• Severe myoclonic epilepsy of infancy (Dravet syndrome)
o starts as focal febrile status epilepticus or focal febrile
seizures and later manifests myoclonic and other seizure
types
• West Syndrome
o Usually between 2 to 12 months
o Triad
§ Infantile epileptic spasms
§ Developmental regression
§ EEG = hypsarrhythmia
• Lennox-Gastaut Syndrome
o 2 to 10 years of age
o Triad:
§ Developmental delay
§ Multiple seizure type: absence, myoclonic, astatic and tonic
§ EEG = 1-2hz spike and slow waves, polyspike bursts in sleep
and slow background in wakefulness

✔ GUIDE QUESTION
Which of the following is incorrect regarding absence seizures?
A. Typically starts at 5-8 years of age
B. Usually preceded by aura
C. Lasts for only a few seconds
D. Does not have a post ictal period and is characterized by
immediate resumption of what the patient was doing before the
seizure
Absence seizures (formerly called ‘petit mal’) involve brief Nelson Textbook of Pediatrics, 20th ed.
staring spells that usually last for less than 15 seconds; usually
resolves 2-5 years after its onset, usually at puberty
Dr. Punongbayan ✔ GUIDE QUESTIONS
A 3-year-old male presents to the ED due to seizures. His parents
Generalities in the management of Seizures and Epilepsy: describe the seizures as jerky movements of all of the extremities
lasting for about a minute. The patient has been having cough and
• If the risk of recurrence is low (normal neurodevelopmental
colds for the past 2 days, a sore throat, and a temperature of 39°C. This
status, EEG and MRI), long term therapy is not indicated. is the first time that this has happened and the mother is worried that
• AEDs reduce excitability by: the infection may have spread to the brain. PE reveals a crying anxious
o interfering with the sodium, potassium or calcium ion child, no signs of nuchal rigidity, erythematous pharyngeal mucosa.
channels What is the most likely diagnosis in this case?
o reducing excitatory neurotransmitter release or function A. Benign Febrile Seizures C. Meningitis
o enhancing GABAergic inhibition B. Encephalitis D. Brain abscess
In relation to the case above, what is the next best step in the
• medications acting on sodium channels are effective against
management of this patient?
partial seizures A. EEG D. cranial CT
• medications acting on T-type calcium channels are effective B. blood culture E. none of the above
against absence seizures C. lumbar tap
• MOST MEDICATIONS HAVE MULTIPLE MOA
SUPPLEMENT: QUICK SHEET
• TREATMENT IS INDIVIDUALIZED
Child Neurology Society of the Philippines Guidelines on a
ANTI-EPILEPTIC DRUGS first febrile seizure (2017)
DISEASE 1ST LINE AED ALTERNATIVE • Lumbar puncture should be performed in all patients below
Focal 18 months old for a first febrile seizure.
• Oxcarbazepine and o For those equal or above 18 months old, LP should be done
seizures and -
Carbamazepine in the presence of clinical signs of meningitis.
epilepsies
Absence • Lamotrigine and o Neuroimaging studies should not be routinely done in
• Ethosuximide children for a first simple febrile seizure.
seizures Valproate
Juvenile • The use of continuous anticonvulsants is not recommended
• Valproate and in children after a first febrile seizure.
Myoclonic -
Lamotrigine
Epilepsy • The use of intermittent anticonvulsants (whether Diazepam
• Clobazam, Valproate, or Phenobarbital) is not recommended for the prevention of
Lennox-
Topiramate, recurrent febrile seizures.
Gestaut -
Lamotrigine, ± • EEG should not be routinely requested for in children with a
syndrome
Rufinamide first simple febrile seizure.
Infantile • Adrenocorticotropic
- WHEN TO DO LUMBAR TAP?
spasms hormone (ACTH)
• Benzodiazepines, • all infants younger than 6 mo of age who present with fever and
Benign seizure or if the child is ill-appearing or at any age if there are
Clonazepam,
Myoclonic • Valproate clinical signs or symptoms of concern
Lamotrigine and
Epilepsy • Is an option in a child 6-12 mo of age who is deficient in
Topiramate
Severe • Topiramate, Haemophilus influenzae type b and Streptococcus pneumoniae
Myoclonic Clobazam, Valproate, - immunizations or for whom immunization status is unknown
Epilepsy Zonisamide ✔ GUIDE QUESTIONS
Partial and Which of the ff. if present in the above case is a major factor that can
• Phenobarbital,
secondary • Oxcarbazepine, increase the risk of recurrence of febrile seizures?
Topiramate,
generalized Levetiracetam, A. Fever of 38-39°C
Lamotrigine,
tonic and Carbamazepine, B. Family history of febrile seizures
Clobazam,
clonic Valproic acid C. Male gender
Clonazepam
seizures D. Hyponatremia

TOPNOTCH MEDICAL BOARD PREP PEDIATRICS MAIN DIGITAL HANDOUT BY DR. PUNONGBAYAN AND DR. DE VERA Page 48 of 105
For inquiries visit www.topnotchboardprep.com.ph or https://www.facebook.com/topnotchmedicalboardprep/
This handout is only valid for the March 2021 PLE batch. This will be rendered obsolete for the next batch since we update our handouts regularly.
TOPNOTCH MEDICAL BOARD PREP PEDIATRICS MAIN DIGITAL HANDOUT BY DR. PUNONGBAYAN AND DR. DE VERA
For inquiries visit www.topnotchboardprep.com.ph or https://www.facebook.com/topnotchmedicalboardprep/
This handout is only valid for the March 2021 PLE batch. This will be rendered obsolete for the next batch since we update our handouts regularly.

Nelson Textbook of Pediatrics, 20th ed.


✔ GUIDE QUESTION
Which of the ff. factors if present will give the greatest risk for Nelson Textbook of Pediatrics, 20th ed.
occurrence of subsequent epilepsy in this child? ✔ GUIDE QUESTIONS
A. Focal complex febrile seizure If the above patient subsequently develops a seizure in the emergency
B. Neurodevelopmental abnormalities department lasting for almost 4 minutes now and still ongoing, what is
C. Family history of epilepsy the best approach to the management of this patient?
D. Recurrent febrile seizures A. IV phenobarbital C. IV valproic acid
B. IV diazepam D. Watchful waiting
Which of the ff. patients with a febrile seizure are indicated to have a
lumbar puncture?
A. 8 month-old infant
B. A 4 year-old with otitis media and complaining of ear pain
C. 2 year-old child with no signs of neck stiffness
D. all of the above
Among the choices, letter A poses the highest risk for a possible
CNS infection thus, a lumbar tap is needed to diagnose it. Ear
infections, unless stated to be chronic, do not correlate directly
with a CNS infection. Likewise, a young child with no signs of
neck stiffness or meningeal irritation is unlikely to have
meningitis.
Dr. Punongbayan

Nelson Textbook of Pediatrics, 20th ed

TOPNOTCH MEDICAL BOARD PREP PEDIATRICS MAIN DIGITAL HANDOUT BY DR. PUNONGBAYAN AND DR. DE VERA Page 49 of 105
For inquiries visit www.topnotchboardprep.com.ph or https://www.facebook.com/topnotchmedicalboardprep/
This handout is only valid for the March 2021 PLE batch. This will be rendered obsolete for the next batch since we update our handouts regularly.
TOPNOTCH MEDICAL BOARD PREP PEDIATRICS MAIN DIGITAL HANDOUT BY DR. PUNONGBAYAN AND DR. DE VERA
For inquiries visit www.topnotchboardprep.com.ph or https://www.facebook.com/topnotchmedicalboardprep/
This handout is only valid for the March 2021 PLE batch. This will be rendered obsolete for the next batch since we update our handouts regularly.

SIMPLE FEBRILE SEIZURE APPROACH TO PATIENT WITH FEBRILE SEIZURES


• Most common seizure disorder during childhood
• rare before 9 mo and after 5 yr. of age, peak age of onset is 14–
18 mo of age
• usually associated with a core temperature that increases
rapidly to ≥39°C.
• It is initially generalized and tonic-clonic in nature, lasts a few
seconds and rarely up to 15 min, is followed by a brief postictal
period of drowsiness, and occurs only once in 24 hr.

COMPLEX OR COMPLICATED FS
• when the duration is >15 min
• when repeated convulsions occur within 24 hr
• when focal seizure activity or focal findings are present during
the postictal period.

STATUS EPILEPTICUS
• It is defined as continuous seizure activity or recurrent
seizure activity without regaining of consciousness lasting
for more than 5 min as part of an operational definition put
forth within the past few years.
• In the past, the cutoff time was 30 min, but this has been
reduced to emphasize the risks involved with the longer
duration.
• Convulsive and Non-convulsive types
• Refractory status epilepticus is status epilepticus that has Nelson Textbook of Pediatrics, 20th ed.

failed to respond to therapy, usually with at least 2 medications


(such as a benzodiazepine and another medication). MANAGEMENT OF FEBRILE SEIZURES:
• Management of Status Epilepticus: • general, antiepileptic therapy, continuous or intermittent, is not
o securing airway, breathing, and circulation recommended for children with 1 or more simple febrile seizures
o Determine etiology • Seizure longer than 5 min, acute treatment with diazepam,
o glucose, sodium, calcium, magnesium, complete blood count, lorazepam, or midazolam is needed
basic metabolic panel, CT scan, and continuous EEG, are • Antipyretics can decrease the discomfort of the child but do not
needed for all patients reduce the risk of having a recurrent febrile seizure
o Other tests (ex MRI, LP) depending on suspected clinical
condition Nice to know….
o initial emergent therapy usually involves intravenous EPILEPSY
ASSOCIATED GENES
diazepam, lorazepam, or midazolam SYNDROMES
o If intravenous access is not available, buccal or intranasal Dravet Syndrome • SCN1A
midazolam, intranasal lorazepam, or rectal diazepam are West Syndrome • ARX gene; polymerase G
effective options. Rett Syndrome • MECP2, CDKL5 and protocadherin 19
o “The strongest evidence for initial and emergent therapy is for Ohtahara
diazepam or lorazepam, followed by Phenytoin/Fosphenytoin • Syntaxin binding protein
Syndrome
and phenobarbital, then valproate and levetiracetam” –Nelson
20th ✔ GUIDE QUESTION
A 5 year old male patient presented to the ED with a single episode of
✔ GUIDE QUESTION seizure characterized as generalized tonic clonic lasting less than a
In patients developing febrile status epilepticus, what viral infection is minute. 1 week prior, he was noted to have low grade fever of 38°C, dry
most commonly associated in one third of the cases? cough, headache, irritability, drowsiness, and malaise which was
A. Measles C. HHV-6 and HHV7 dismissed as an episode of flu. At present, patient is lethargic, (+)
B. Rhinovirus D. Enterovirus nuchal rigidity, no other focal neurologic deficits observed. (-)
Due to the high fever and the ability of the virus to cross the vomiting, CXR was normal, lumbar puncture revealed WBC 200 with
blood-brain barrier, 10-15% of children with Roseola will lymphocyte predominance, glucose 30mg/dL, protein is 1000mg/dL.
experience febrile seizure though it is mild and self-limited. What is the most likely diagnosis?
Dr. Punongbayan A. Bacterial meningitis C. Aseptic meningitis
B. Viral meningitis D. TB meningitis
OTHER FACTS ABOUT FEBRILE SEIZURES The given LP findings point to a bacterial cause. Letter A is a
• Simple febrile seizures do not have an increased risk of mortality possible answer. However, the history of chronic cough
• Complex febrile seizures may have an approximately 2-fold increased the possibility of a tuberculous process.
Dr. Punongbayan
long-term increase in mortality (probably secondary to a co-
existing pathology) Triad of Imaging Findings in TB Meningitis?
• Patients with febrile seizures do not have any increase in the • Hydrocephalus
incidence of abnormalities of behavior, scholastic performance, • Basal Enhancements
neurocognitive function, or attention. • Infarcts
• Febrile seizures risk of recurrence
o 30% of those experiencing a first episode HYDROCEPHALUS
o 50% after 2 or more episodes
• How does hydrocephalus develop?
o 50% of infants younger than 1 yr old at febrile seizure onset 1. impaired circulation & absorption of CSF
• Only 2-7% of children who experience febrile seizures proceed 2. from increased production of CSF
to develop epilepsy later in life
TYPE DESCRIPTION CAUSE
• Abnormality of
Obstructive • Obstruction the aqueduct or a
or non- within the lesion in the 4th
communicating ventricular ventricle
type system (aqueductal
stenosis)
TOPNOTCH MEDICAL BOARD PREP PEDIATRICS MAIN DIGITAL HANDOUT BY DR. PUNONGBAYAN AND DR. DE VERA Page 50 of 105
For inquiries visit www.topnotchboardprep.com.ph or https://www.facebook.com/topnotchmedicalboardprep/
This handout is only valid for the March 2021 PLE batch. This will be rendered obsolete for the next batch since we update our handouts regularly.
TOPNOTCH MEDICAL BOARD PREP PEDIATRICS MAIN DIGITAL HANDOUT BY DR. PUNONGBAYAN AND DR. DE VERA
For inquiries visit www.topnotchboardprep.com.ph or https://www.facebook.com/topnotchmedicalboardprep/
This handout is only valid for the March 2021 PLE batch. This will be rendered obsolete for the next batch since we update our handouts regularly.
• Obliteration of
the • Follows a
Nonobstructive
subarachnoid subarachnoid
or
cisterns hemorrhage
communicating
• Malfunction of • Leukemic
type
the arachnoid infiltrates
villi

ALL ABOUT CSF


• Where is CSF produced?
o Choroid plexus epithelium within the cerebral ventricles

SUPPLEMENT: QUICK SHEET http://www.psyweb.com/Brain/brainv0.jsp

CSF Flow Mnemonic


Come Let Me Treat Sisa For Lunch Maybe Somewhere in Ayala
Choroid Plexus
Lateral Ventricle
Foramen of Monro (Interventricular foramen)
Third Ventricle
Aqueduct of Sylvius (Cerebral aqueduct)
Fourth Ventricle
Foramina of Luschka
Foramen of Magendie
Superior Sagittal Sinus
Arachnoid Villi

• How is CSF reabsorbed?


o Arachnoid villus cells, which are located in the superior sagittal
sinus, return CSF to the bloodstream within vacuoles (via a https://emedicine.medscape.com/article/1923254-overview
process called pinocytosis)

Nelson Textbook of Pediatrics, 20th ed.

✔ GUIDE QUESTION The presence of a bulging fontanel increased the probability of


A 5-year-old male patient presented to the ED with confusion and bacterial meningitis by 3.5 times and neck stiffness increased
lethargy. 1 week prior, he was noted to have low grade fever of 38°C, the likelihood of acute bacterial meningitis by eight-fold.
productive cough, headache, irritability, drowsiness, anorexia, and Complex seizures double the risk for bacterial meningitis.
malaise. No treatment was sought initially. At present, patient is A systematic review of the meningeal signs such as neck
lethargic, HR 120, RR 22, T >39°C, (+) nuchal rigidity, no other focal stiffness, Brudzinski’s and Kernig’s signs, as basis for the
neurologic deficits observed. (-) vomiting, lumbar puncture revealed diagnosis of meningitis proved to be variable in sensitivity and
WBC 2000 with neutrophil predominance, glucose 20mg/dL, protein is specificity. Thus, these signs of meningeal irritation were not
500mg/dL. What is the most likely diagnosis? reliably predictive of meningitis if used alone.
A. Bacterial meningitis Dr. Punongbayan
B. Viral meningitis
C. Aseptic meningitis
D. TB meningitis
TOPNOTCH MEDICAL BOARD PREP PEDIATRICS MAIN DIGITAL HANDOUT BY DR. PUNONGBAYAN AND DR. DE VERA Page 51 of 105
For inquiries visit www.topnotchboardprep.com.ph or https://www.facebook.com/topnotchmedicalboardprep/
This handout is only valid for the March 2021 PLE batch. This will be rendered obsolete for the next batch since we update our handouts regularly.
TOPNOTCH MEDICAL BOARD PREP PEDIATRICS MAIN DIGITAL HANDOUT BY DR. PUNONGBAYAN AND DR. DE VERA
For inquiries visit www.topnotchboardprep.com.ph or https://www.facebook.com/topnotchmedicalboardprep/
This handout is only valid for the March 2021 PLE batch. This will be rendered obsolete for the next batch since we update our handouts regularly.
SUPPLEMENT: QUICK SHEET ✔ GUIDE QUESTION
Absolute contraindications to a lumbar puncture are the A 9-year-old patient with Tetralogy of Fallot is admitted due to
following: headache, fever and seizures. Cranial imaging showed multi-loculated
abscess. The patient is being managed empirically with antibiotics and
1. Signs of elevated intracranial pressure (decreased or
is on the second week of treatment. What will be the next appropriate
fluctuating level of consciousness, relative bradycardia and management for this patient?
hypertension, focal neurological signs, abnormal posturing, A. Continue antibiotics for 4-6weeeks
unequal, dilated or poorly responsive pupils, papilledema, B. Serial monitoring of cranial CT scan
and abnormal Doll’s eye movement); C. Surgical management for the abscess
2. Local infection at desired puncture site; and D. Send patient home on oral antibiotics
3. Radiological signs (in cranial scan or MRI) of obstructive
hydrocephalus, cerebral edema or herniation and the BRAIN ABSCESS
presence of an intracranial mass lesion or midline shift
• Most common between 4-8 years old
Relative contraindications (lumbar puncture may be done • Etiology: embolization due to CHD with R->L shunts, meningitis,
but only after appropriate diagnostic and therapeutic chronic OM & mastoiditis, face & scalp infections, orbital
interventions are done): cellulitis, dental infections, penetrating head injuries, VP shunt
1. Signs of shock, sepsis or hypotension infections
2. Coagulation defects [disseminated intravascular • Cerebrum 80%; occipital lobe, cerebellum & brain stem 20%
coagulopathy (DIC), platelet count <50,000/mm3, and • Majority are single abscess; 30% multiple
therapeutic use of warfarin]
3. Focal neurological deficit (especially for suspected IMPORTANT FACTS ABOUT BRAIN ABSCESS:
posterior fossa lesions) • Causes: S. aureus, Streptococci, anaerobes, Gram (-) aerobic
4. Glasgow coma score < 8 bacilli (Proteus, Pseudomonas, Haemophilus, Citrobacter)
5. Epileptic seizures • Early stage: nonspecific symptoms
Philippine CPG on the Diagnosis and Management of Acute Bacterial Meningitis.
PIDSP Technical Working Group. PIDSP Journal vol. 16 no.2 July-December 2015 • Vomiting, severe headache, seizures, focal neurologic signs,
papilledema
MENINGITIS • Dx: (+) blood culture in 10%; CSF not done to avoid herniation;
MORE ON BACTERIAL MENINGITIS cranial CT scan & MRI (most reliable methods)
• What if the child was started on oral antibiotics prior to LP?
MANAGEMENT OF BRAIN ABSCESS
o The presumptive diagnosis of bacterial meningitis can be
made despite negative cultures because pleocytosis, high CSF • Empiric antibiotics depend on the probable pathogenesis &
protein level, and low CSF sugar persist for several days even most likely organism
after antibiotics. • Unknown cause: 3rd generation cephalosporin + Metronidazole
• What do you do next if the LP is traumatic? • Head trauma or neurosurgery: Oxacillin or Vancomycin with 3rd
o It is prudent to rely on bacteriologic results because the generation cephalosporin + Metronidazole
Gram stain, culture & glucose level may not be influenced • Meropenem as monotherapy
by a traumatic LP. • Due to CHD: Penicillin + Metronidazole
Note: a non-traumatic sample is CSF with less than 1,000 red blood cells • Infected VP shunt: Vancomycin + Ceftazidime
per hpf • Immunocompromised: broad spectrum + Amphotericin B
• How reliable is a blood culture in cases of bacterial • Aspiration for encapsulated abscesses
meningitis? • Indications for surgery:
o Blood cultures should be performed in all patients with 1. (+) gas in the abscess
suspected meningitis. It reveals the responsible bacteria in 2. Multiloculated abscesses
up to 80-90% of cases of meningitis. 3. Posterior fossa location
COMPLICATIONS OF BACTERIAL 4. Fungal cause
MNEMONIC 5. Assoc. infections like mastoiditis, periorbital abscess,
MENINGITIS
sinusitis
• H - hydrocephalus
• A - abscess
WHAT TO EXPECT:
• C – cerebritis / cranial nerve involvement
• Duration of antibiotics: 4-6 wks
• T - thrombosis
• High mortality: multiple abscesses, coma
• I - infarct
• Long-term sequela: behavior & learning problems,
• V – ventriculitis / vasculopathy
hydrocephalus, seizures, hemiparesis
• E – extra-axial fluid collection (empyema)
*** MRI > CT scan ✔ GUIDE QUESTIONS
A 16-year-old female patient presents to the emergency department
Patients presenting with CNS infections with fever, altered sensorium, nausea, and vomiting. She started having
• Prefrontal fever 12 hours ago accompanied by sore throat and muscle pains. At
headache Naegleria fowleri present examination, she has multiple purpuric lesions over the lower
• Swimming in extremities, her blood pressure is 80/60, HR 130 RR 20, T>39.5C. Her
• High fever (brain-eating
warm water parents say that she lives in a college dormitory and she doesn’t have a
• Disturbance in amoeba)
boyfriend as far as they know. What is the most likely diagnosis in this
smell patient?
• Systemic disease A. Herpes simplex encephalitis
Borrelia
• Bell’s Palsy • Cutaneous lesions B. Disseminated Gonococcal infection
burgdorferi C. Acute Meningococcemia
• Carditis
D. Henoch Schönlein Purpura
• History of GIT Campylobacter In relation to the case above, which among the ff. diagnostic test
infection jejuni establishes a definitive diagnosis?
• Ascending
• History of A. Culture of the causative agent in the blood
paralysis Haemophilus
Respiratory B. Sputum gram stain and culture
influenza type b C. Cranial CT scan
Infection
D. Viral PCR
• History of viral In relation to the above case, carriage of the causative organism is
• Lethargy and infection usually in the
Reye Syndrome
irritability • Chicken pox A. Nasopharynx C. Bronchi
• Aspirin use B. Gallbladder D. Skin
• Previous history In relation to the case above, if the patient has a documented case of
• Ocular nerve of Tuberculosis severe anaphylaxis to penicillin according to her parents, all of the ff.
TB meningitis antibiotics are appropriate to give except?
palsies • Active A. Chloramphenicol C. Meropenem
Tuberculosis B. Ciprofloxacin D. Doxycycline
TOPNOTCH MEDICAL BOARD PREP PEDIATRICS MAIN DIGITAL HANDOUT BY DR. PUNONGBAYAN AND DR. DE VERA Page 52 of 105
For inquiries visit www.topnotchboardprep.com.ph or https://www.facebook.com/topnotchmedicalboardprep/
This handout is only valid for the March 2021 PLE batch. This will be rendered obsolete for the next batch since we update our handouts regularly.
TOPNOTCH MEDICAL BOARD PREP PEDIATRICS MAIN DIGITAL HANDOUT BY DR. PUNONGBAYAN AND DR. DE VERA
For inquiries visit www.topnotchboardprep.com.ph or https://www.facebook.com/topnotchmedicalboardprep/
This handout is only valid for the March 2021 PLE batch. This will be rendered obsolete for the next batch since we update our handouts regularly.

POOR PROGNOSTICATING FACTORS


• Poor prognostic factors on presentation
o include hypothermia or extreme hyperpyrexia
o hypotension or shock
o purpura fulminans
o seizures
o leukopenia
o thrombocytopenia (including DIC)
o Acidosis
o high circulating levels of endotoxin and tumor necrosis factor-
α.
• The presence of petechiae for <12 hr before admission, absence
of meningitis, and low or normal erythrocyte sedimentation rate
indicate rapid, fulminant progression and poorer prognosis.
Nelson Textbook of Pediatrics, 20th ed

Nelson Textbook of Pediatrics, 20th ed.

MENINGOCOCCEMIA
FACTS ABOUT MENINGOCOCCEMIA:
• Mode of transmission: by aerosol droplets or through contact
with respiratory secretions
• Period of communicability: until 24 hours after initiating
effective treatment
• Incubation period: 1-10 days or less than 4 days

CLINICAL MANIFESTATIONS:
• Asymptomatic colonization to fulminant sepsis
• Infections of the GUT, conjunctiva, pharynx, CNS, heart, skin,
adrenals
• Diffuse adrenal hemorrhage without vasculitis, DIC, coma, and
death → Waterhouse-Friderichsen syndrome (fulminant
cases) These pictures were taken on succeeding days of the same patient who
presented with acute onset of fever, petechiae, and decreased activity and
How is it different from other bacterial meningitis? sensorium. NOTE the quick progression of petechia to purpuric and
• Headache ecchymotic lesions in a span of less than 24 hours
• Photophobia Dr. Punongbayan

• Lethargy
• Vomiting DRUG OF CHOICE
• Nuchal rigidity • Penicillin G 250,000-300,000 U/kg/day IV in 4-6 divided doses
for at least 5-7 days (DOC)
o *seizures & focal neurologic signs occur less frequently than
in patients with meningitis due to H. influenza b & • Ceftriaxone IM or IV (100 mg/kg/day once or twice a day) or
pneumococcus Cefotaxime IM or IV (200-300 mg/kg/day every 6 or 8 hours)
* clinical improvement within 24-72 hrs
FREQUENCY OF SIGNS/SYMPTOMS
• Fever (81%) Who are considered exposed?
• Petechiae and/or purpura (50%) • Household, school or day care contacts during the 7 days
• Hypotension or poor peripheral perfusion (41%) before onset of illness should receive antibiotic prophylaxis
• Vomiting (34%) • Prophylaxis NOT routinely recommended for medical personnel
• Lethargy (30%) except those with intimate exposure (intubation, suctioning,
• Irritability (21%) mouth-to-mouth resuscitation)
• Rhinorrhea (10%)
o * seizures (6%)

TOPNOTCH MEDICAL BOARD PREP PEDIATRICS MAIN DIGITAL HANDOUT BY DR. PUNONGBAYAN AND DR. DE VERA Page 53 of 105
For inquiries visit www.topnotchboardprep.com.ph or https://www.facebook.com/topnotchmedicalboardprep/
This handout is only valid for the March 2021 PLE batch. This will be rendered obsolete for the next batch since we update our handouts regularly.
TOPNOTCH MEDICAL BOARD PREP PEDIATRICS MAIN DIGITAL HANDOUT BY DR. PUNONGBAYAN AND DR. DE VERA
For inquiries visit www.topnotchboardprep.com.ph or https://www.facebook.com/topnotchmedicalboardprep/
This handout is only valid for the March 2021 PLE batch. This will be rendered obsolete for the next batch since we update our handouts regularly.
What do you give to exposed personnel? MYELOMENINGOCELE
• Children: Rifampicin 10 mg/kg po every 12 hrs for a total of 4 • How can this be prevented?
doses (max 600 mg); 5 mg/kg/dose for <1 month old o maternal periconceptual use of folic acid reduces its
• Or Ceftriaxone 125 mg single dose IM for < 12 yrs old incidence by at least 50% (started before conception until at
• > 18 yrs old: Ciprofloxacin 500 mg po as a single dose least the 12th wk of gestation when neurulation is complete)
– 0.4 mg once a day
PATIENT PRESENTING WITH CNS INFECTIONS
• What parts are affected?
Sepsis, seizure, o Dysfunction of the skeleton, skin, GUT, PNS, CNS
irritability, o May be located anywhere along the neuroaxis (lumbosacral
Gram Negative
lethargy, bulging Px is <2 months old 75%)
Bacteria / GBS,
of fontanelles,
rigidity
No mention of
proper vaccination, Hib
<5 y/o
Properly
vaccinated child,
abrupt in onset, Meningococcemia
toxic looking with
Headache, fever, rashes all over
confusion, Young adults Pneumococcus
lethargy, nuchal
rigidity, vomiting Renal transplant
Listeria https://medicaltipsandnews.blogspot.com/2013/07/neural-tube-defects.html
patient
• What are the manifestations?
Gradual onset of o flaccid paralysis of the LE
signs and o absence of DTRs
Enterovirus,
symptoms, not o lack of response to pain & touch
toxic looking o hip subluxation
RBCs in the CSF o clubfeet
HSV
examination o bowel & bladder incontinence
o associated with hydrocephalus (type II Chiari)
SUPPLEMENT: QUICK SHEET
Empiric antibiotics for bacterial meningitis:
1. For neonates: Ampicillin OR Cefotaxime PLUS
aminoglycoside

CEFTRIAXONE
IN THE NEWBORN
https://qrs.ly/gpbj83o

2. 1 month–18 years old: Ceftriaxone OR Chloramphenicol


3. For Hib – Ceftriaxone for 7-10 days
4. For S. pneumoniae – Penicillin for 10-14 days (alternative:
Ceftriaxone)
5. For N. meningitidis – Penicillin for 7 days
6. For E. coli – Cefotaxime for 21 days
7. For GBS – Cefotaxime OR Ceftriaxone for 14 days
Philippine CPG on the Diagnosis and Management of Acute Bacterial Meningitis.
PIDSP Technical Working Group. PIDSP Journal vol. 16 no.2 July-December 2015
✔ GUIDE QUESTION
An 11-year-old boy presents with one-sided headache, on & off,
✔ GUIDE QUESTION associated with nausea for the past 4 days. There was no fever, cough,
colds, or other symptoms. The parent claimed a family history of the
A term baby boy was born to a G1P1 mother via NSD with an APGAR
same nature of the headache. His neurologic examination did not show
score of 8 & 9. He was noted to have a fleshy erythematous mass on the
any deficits. What is the appropriate management for this child?
midline of his lower back. There were no other associated signs.
A. Do Cranial Imaging and EEG
Mother is asking if how she may be able to prevent this in the future?
B. Do cranial imaging alone
A. Folic Acid Supplementation 0.4mg OD
C. Analgesics and re-assurance
B. Folic Acid Supplementation 4mg OD
D. None of the above
C. Vitamin B12 supplementation
D. This cannot be prevented
The recommended dose to prevent NTDs during the first HEADACHE
pregnancy: 400 micrograms or 0.4 mg of folic acid APPROACH TO HEADACHE
In succeeding pregnancies or recurrent cases of NTDs: 4000 • What are the features of migraine?
micrograms or 4 mg of folic acid (although an article in Fetal o Recurrent headache with symptom-free intervals & at least 3
Diagnosis and Therapy 2018; 44:161-165 by Dolin, et al of the ff: (F-R-U-A-N-T)
concluded that the 4 mg dose was too much and that it needed 1. (+) Family history
further studies to reconsider minimizing the dose) 2. Relief following sleep
Dr. Punongbayan
3. Unilateral location
4. Associated aura
MENINGOCELE 5. Abdominal pain
• Meningocele - the most severe form of neural tube defect 6. Nausea & vomiting
(dysraphism) 7. Throbbing in character

WHAT IS THE UNDERLYING CAUSE? EPIDEMIOLOGIC DATA:


• Due to failure of the neural tube to close spontaneously • F > M: adolescents
between the 3rd & 4th week of in utero development • Younger than 10 yrs old: M > F
• Hyperthermia, drugs, malnutrition, chemicals, maternal obesity • More than 50% undergo spontaneous prolonged remission
or diabetes, and genetic determinants adversely affect CNS after the 10th birthday
development from the time of conception

TOPNOTCH MEDICAL BOARD PREP PEDIATRICS MAIN DIGITAL HANDOUT BY DR. PUNONGBAYAN AND DR. DE VERA Page 54 of 105
For inquiries visit www.topnotchboardprep.com.ph or https://www.facebook.com/topnotchmedicalboardprep/
This handout is only valid for the March 2021 PLE batch. This will be rendered obsolete for the next batch since we update our handouts regularly.
TOPNOTCH MEDICAL BOARD PREP PEDIATRICS MAIN DIGITAL HANDOUT BY DR. PUNONGBAYAN AND DR. DE VERA
For inquiries visit www.topnotchboardprep.com.ph or https://www.facebook.com/topnotchmedicalboardprep/
This handout is only valid for the March 2021 PLE batch. This will be rendered obsolete for the next batch since we update our handouts regularly.
PATHOGENESIS:
How do you explain the aura in migraine?
Cortical spreading depression (CSD) – a phenomenon associated
with high CNS hydrogen & potassium ions with the release of
glutamate & nitrous oxide → leads to excitation of trigeminal-
vascular system → activates the release of VIP → vasodilation →
extravasation of plasma proteins from the dural vessels →
localized inflammation of dural vessels → excitation of pain
sensitive receptor → pain

When do you request for imaging studies? CLINICAL MANIFESTATIONS


Indications for cranial CT or MRI: • May present during infancy with infantile spasms
1. abnormal neurologic signs • Typical hypopigmented skin lesions: ash leaf >90% of cases
2. behavioral changes, recent school failure, fall-off in linear • CT scan shows calcified tubers in the periventricular area
growth rate • Childhood: generalized seizure disorder & skin lesions;
3. headache awakens the child during sleep shagreen patch – roughened, raised lesion with an orange-peel
4. migraine & seizure occur in the same episode consistency located primarily in the lumbosacral region
5. focal neurologic signs
• Adolescence: subungual or periungual fibromas from the
6. cluster headaches esp.in <5 yrs old
fingers & toes
TREATMENT OF MIGRAINE MORE ON TUBEROUS SCLEROSIS
• Avoid stimuli: stress, fatigue, anxiety • 2 types of retinal lesions: mulberry tumors or round, flat gray
• Analgesics (Acetaminophen 15 mg/kg or Ibuprofen 7.5-10 lesions
mg/kg) or antiemetics (parenteral Metoclopramide) • Rhabdomyosarcoma of the heart
• What do you do with status migrainosus (persistent headache • Renal manifestations: bilateral angiomyolipomas and cysts
lasting > 3 days)? → Prochlorperazine IV 0.15 mg/kg max.10 mg
• Dx: high index of suspicion when assessing a child with infantile
spasm
When do you give prophylactic therapy?
• Tx: seizure control & baseline studies like 2-D echo, chest X-ray,
• More than 2-4 severe episodes monthly
renal UTZ
• Unable to attend school regularly
• PedMIDAS > 20
• Drugs:
NEUROFIBROMATOSIS
o Propranolol - 10-20 mg tid for > 7 yrs old • Von Recklinghausen disease – AD disorder; every system may
o Flunarizine - 5 mg at bedtime be affected; complications may be delayed for decades
• Result of an abnormality of neural crest differentiation &
✔ GUIDE QUESTION migration during the early stages of embryogenesis
A 6 yr old boy presented with on & off headache & seizures for the past
3 months. His kinder teacher also noted his “learning difficulties”. After SUPPLEMENT: QUICK SHEET
psychological testing, he was found out to have reading and math IMPORTANT!
learning disability with moderate mental retardation. Physical
“Von Recklinghausen” has 17 letters and is located on
examination shows subungual fibromas, a plaque at the lumbosacral
regions with roughened surface and an orange peel appearance, and
chromosome 17!
hypopigmented patches on the extremities. What gene is most likely
involved in this disease? NEUROFIBROMATOSIS 1
A. TSC1 and TSC2 C. Von Recklinghausen • NF-1: most prevalent type: diagnosed when any 2/7 of the ff are
B. BCKD Deficiency D. None of the above present:
1. 6 or more café au lait macules >5mm in diameter in
NEUROCUTANEOUS SYNDROMES prepubertals & >15mm in postpubertal individuals: present
• Tuberous sclerosis at birth & increase in size, number & pigmentation with
• Neurofibromatosis predilection for the trunk & extremities with sparing of the
• Most disorders are familial face
• Arise from a defect in differentiation of the primitive 2. Axillary or inguinal freckling consists of multiple
ectoderm hyperpigmented areas 2-3 mm in diameter
3. 2 or more iris Lisch nodules (hamartomas located within
the iris)
TUBEROUS SCLEROSIS 4. 2 or more neurofibromas (along the skin, PNS, blood
FACTS ABOUT TUBEROUS SCLEROSIS vessels & within viscera) or one plexiform neurofibroma
• Inherited as AD trait 5. Distinctive osseous lesion
• Heterogeneous disease with a wide clinical spectrum varying 6. Optic glioma
from severe MR & incapacitating seizures to normal
intelligence and a lack of seizures, often within the same family
• Affects many organs like skin, brain, heart, kidney, eyes, lungs,
bone

What genes are mutated in this condition?


• Tuberous sclerosis complex gene 1 (TSC1) and gene 2 (TSC2)
are mutated.
• TSC1 encodes for the protein hamartin and TSC2 encodes for
the protein tuberin.
• Both act as tumor suppressor genes → wide expressivity of the
syndrome
• Characteristic brain lesions consist of tubers located in the
convolutions of the cerebral hemispheres → typically present in
the subependymal region → undergo calcification & project
into the ventricular cavity → produce a candle-dripping
appearance

TOPNOTCH MEDICAL BOARD PREP PEDIATRICS MAIN DIGITAL HANDOUT BY DR. PUNONGBAYAN AND DR. DE VERA Page 55 of 105
For inquiries visit www.topnotchboardprep.com.ph or https://www.facebook.com/topnotchmedicalboardprep/
This handout is only valid for the March 2021 PLE batch. This will be rendered obsolete for the next batch since we update our handouts regularly.
TOPNOTCH MEDICAL BOARD PREP PEDIATRICS MAIN DIGITAL HANDOUT BY DR. PUNONGBAYAN AND DR. DE VERA
For inquiries visit www.topnotchboardprep.com.ph or https://www.facebook.com/topnotchmedicalboardprep/
This handout is only valid for the March 2021 PLE batch. This will be rendered obsolete for the next batch since we update our handouts regularly.
• 1st-degree relative with NF-1 whose diagnosis was based on the NEUROFIBROMATOSIS 2
aforementioned criteria • NF-2 is diagnosed when 1 of the ff is present:
• Majority of mutations in NF-1 occur in the paternal germline 1. Bilateral 8th nerve masses (acoustic neuroma)
• High incidence of learning disabilities 2. Parent, sibling, or child with NF-2 & either unilateral 8th
nerve masses or any 2 of the ff: neurofibroma, meningioma,
glioma, schwannoma
o Management: genetic counseling & early detection of treatable
conditions or complications
SUMMARY OF NEUROCUTANEOUS SYNDROMES:
SYNDROME MANIFESTATIONS PE FINDING DIAGNOSTICS TREATMENT
Neurofibro-matosis • Axillary or inguinal • Genetic counseling & early
• Café au lait macules
(Von Reckling- freckling, Lisch • CT scan or MRI detection of treatable
that spare the face
hausen) nodules, optic glioma complications
• CT scan or MRI of the
• Tubers in cerebrum
• multisystemic; brain, heart,
(candle-dripping); • Seizure control;
Tuberous Sclerosis • seizures, abdomen;
• ash leaf, shagreen • multidisciplinary approach
• mental retardation • 2D echo;
patch
• renal UTZ

✔ GUIDE QUESTION MANAGEMENT OF GBS


A 10 year-old boy presented with progressive weakness in both of his • Acute stage: admit for observation because ascending paralysis
legs that started 5 days PTC. He just recovered from a URTI about 2 ½ may occur within 24 hrs
weeks PTC. On the day of consultation, he could not walk • Rapidly progressive ascending paralysis: IVIG (0.4 gms/kg/day
independently. His neuro exam showed absence of patellar and
Achilles tendon reflexes bilaterally. What is your expected CSF finding?
for 5 consecutive days)
A. Increased protein, normal glucose, normal cells • Supportive care; prevention of ulcers
B. Increased protein, increased glucose, increased cells • High-dose pulse methylprednisolone IV for relapses
C. Decreased protein, normal glucose normal cells, ✔ GUIDE QUESTION
D. Decreased protein, decreased glucose, decreased cells
A 5-year-old boy is brought to his doctor by his mother for a follow-up
This is known as the Cytoalbuminocytologic dissociation finding appointment. Two months ago, the patient was seen for a chief
in CSF examination in GBS. complaint of morning headache, vomiting, and decreased activity. At
Dr. Punongbayan
this visit, the mother reports that her son continues to have worsened
symptoms as well as new onset falling and a stumbling gait.
GUILLAIN-BARRE SYNDROME What is your most likely diagnosis?
What is the most likely diagnosis? Answers: Medulloblastoma
• Guillain-Barre syndrome (GBS) or acute demyelinating
Consider a space-occupying lesion in young children who
polyradiculo-neuropathy (symmetric ascending muscle weakness presents with headache, signs of increased intracranial
or paralysis) pressure, and change in gait and/or behavior.
What is the etiology of this condition? In this question, the answer is based on the most common
• Autoimmune reaction that develops in response to a previous malignant brain tumor in children which is medulloblastoma.
infection → leads to aberrant demyelination of peripheral nerves It accounts for about 20% of primary CNS neoplasms and
& ventral motor nerve roots approximately 40% of all posterior fossa tumors.
Dr. Punongbayan

GBS
• Postinfectious polyneuropathy involving mainly motor MEDULLOBLASTOMA
• Not hereditary; affects all ages • Most often found in the cerebellum
• Paralysis usually follows a nonspecific viral infection (GIT or • Most common malignant brain tumor in children
RT) by 10 days (Campylobacter jejuni and herpesvirus) • 4-8 years old; M > F
• Weakness begins in the lower extremities & progressively • MRI is used to visualize the extent of the tumor.
involves the trunk, upper limbs & bulbar muscles (Landry • Medulloblastomas are heterogenous enhancements in the
ascending paralysis) cerebellum, often invading the 4th ventricle, and can cause
obstructive hydrocephalus.
What other physical findings are associated with this • Homer-Wright rosettes - circular patterns of tumor cells
condition? surrounding a center of neutrophils
• Cranial nerve deficits leading to dysphagia, dysarthria, facial INFRATENTORIAL TUMORS
weakness, papilledema, autonomic dysfunction, respiratory • Cerebellar astrocytoma – most common & with the best
muscle paralysis prognosis; cystic; causes hydrocephalus; resection with 90% 5-
• Miller-Fisher syndrome: acute ophthalmoplegia, ataxia, yr survival rate
areflexia • Medulloblastoma – < 7 year old; can spread to extracranial sites;
surgery + irradiation; 80-90% 5-yr survival rate
FACTS ABOUT GBS: • Brain stem glioma
• Benign clinical course with spontaneous recovery within 2-3wks • Ependymoma
• Tendon reflexes usually the last function to recover & lower
extremity weakness last to resolve SUPRATENTORIAL TUMORS
• 3 clinical features are predictive of poor outcome with sequela: • Craniopharyngioma – solid & cystic areas that tend to calcify;
cranial nerve involvement, need for intubation & maximum short stature; pressure to optic chiasm produces bitemporal
disability at the time of presentation visual field defects
• Optic nerve glioma – decreased visual acuity & pallor of the
DIAGNOSTIC TESTS FOR GBS discs; 25% have neurofibromatosis; hyperalert & euphoric
What laboratory finding is likely in this condition? despite being emaciated; invasion of the hypothalamus leads to
• CSF analysis: increased protein, normal glucose, no pleocytosis obesity or DI
• Dissociation between high CSF protein and a lack of cellular • Astrocytoma
response is diagnostic (albuminocytologic dissociation) • Choroid plexus papilloma
• Reduced motor NCVs BRAIN TUMORS
• Cranial MRI – can delineate tumors
• Cranial irradiation + chemotherapy
• Brachytherapy – implantation of radiation seeds

TOPNOTCH MEDICAL BOARD PREP PEDIATRICS MAIN DIGITAL HANDOUT BY DR. PUNONGBAYAN AND DR. DE VERA Page 56 of 105
For inquiries visit www.topnotchboardprep.com.ph or https://www.facebook.com/topnotchmedicalboardprep/
This handout is only valid for the March 2021 PLE batch. This will be rendered obsolete for the next batch since we update our handouts regularly.
TOPNOTCH MEDICAL BOARD PREP PEDIATRICS MAIN DIGITAL HANDOUT BY DR. PUNONGBAYAN AND DR. DE VERA
For inquiries visit www.topnotchboardprep.com.ph or https://www.facebook.com/topnotchmedicalboardprep/
This handout is only valid for the March 2021 PLE batch. This will be rendered obsolete for the next batch since we update our handouts regularly.

MANAGEMENT OF AIS
✔ GUIDE QUESTIONS • Antithrombotic Strategies
The most common cause of arterial ischemic stroke in pediatric o Heparin
population is:
o Aspirin
A. Arteriopathy C. Hematologic
B. Cardiac D. Idiopathic • Neuroprotective strategies
o Glucose control
This type of intracranial hemorrhage is almost always associated with
trauma:
o Temperature control
A. Subdural C. Subgaleal o Seizure prevention
B. Epidural D. Intra-ventricular o Maintenance of cerebral perfusion
A 3 year-old boy, who lives in a farm, was brought to the ER for sudden
• Secondary Stroke Prevention
episode of decrease in sensorium. Lola reported that the patient was • Rehabilitation
active few hours prior and crying and that she was trying to console
him by rocking him back and forth. At the ER, patient arrived in active CEREBRAL SINOVENOUS THROMBOSIS (CSVT)
seizure. Pertinent findings included wasting, bruises on the
extremities, retinal hemorrhages on fundoscopy. What is your • Greatest risk in the neonatal period
impression? • Thrombotic occlusion of venous structures create increased ICP,
A. Abusive head trauma cerebral edema, and venous infarction or hemorrhage
B. Meningococcemia • Clinical presentations are typically gradual, variable, and
C. Organophosphate poisoning nonspecific compared to AIS
D. Cavernous sinus thrombosis • Diagnostic of choice = contrast CT venography or MR
Biomechanics of children less than 2 years old that lead to venography
shaken baby syndrome or abusive head trauma: • Non-contrast CT scan = low sensitivity
1.Large size of the head relative to the body
2.Weakness of cervical muscle
MANAGEMENT OF CSVT
3.High water content of the brain
4.Large subarachnoid space • Anticoagulation therapy – unfractionated or low molecular
Dr. Punongbayan weight heparin

PEDIATRIC STROKE HEMORRHAGIC STROKE (HS)


• Important cause of acquired brain injury in children & newborns • Clinical presentations vary according to location, cause, and rate
• ischemic varieties of arterial ischemic stroke (AIS) and cerebral of bleeding
sinovenous thrombosis (CSVT) are more common than brain • Acute hemorrhagic stroke
malignancy o instantaneous or thunderclap headache
o loss of consciousness
ARTERIAL ISCHEMIC STROKE (AIS) o nuchal rigidity
• Strokes most often involve the middle cerebral artery o focal neurologic deficits
territory but can occur in any cerebral artery of any size o Seizures
• Usually delayed diagnosis o may be rapidly fatal
o Subtle and non-specific signs • CT is highly sensitive
o >50% of initial CT Scans are normal • Abusive head trauma
• The acute onset of a focal neurologic deficit in a child is o Primarily subdural or intraparenchymal
stroke until proven otherwise. • Epidural hemorrhage
• Diagnostic of choice = MRI o Nearly all are caused by trauma
• CT scan demonstrates mature AIS and rules out hemorrhage • Subdural hemorrhage
• 3 main etiologies o May occur spontaneously in children with brain atrophy
o Arteriopathy – disorders of cerebral arteries; leading cause (stretching of the bridging veins)
(>50%) • AV malformation is the MCC of childhood subarachnoid and
o Cardiac – cardioembolic strokes; 25% intraparenchymal hemorrhagic stroke
§ Complex congenital heart disease (ex. TOF)
o Hematologic MANAGEMENT OF HS
§ Sickle cell anemia increases risk for stroke by 400 fold • Emergent neurosurgical intervention for large or rapidly
§ Iron deficiency anemia expanding hemorrhage
§ Coagulation disorders • Neuroprotective measures
For cardioembolic etiology: maximal embolic risk is concurrent with • Reversal of anti-coagulant therapy if indicated
catheterization, surgical repair, or ventricular assistive device use.
Dr. Punongbayan

Nelson Textbook of Pediatrics, 20th ed

TOPNOTCH MEDICAL BOARD PREP PEDIATRICS MAIN DIGITAL HANDOUT BY DR. PUNONGBAYAN AND DR. DE VERA Page 57 of 105
For inquiries visit www.topnotchboardprep.com.ph or https://www.facebook.com/topnotchmedicalboardprep/
This handout is only valid for the March 2021 PLE batch. This will be rendered obsolete for the next batch since we update our handouts regularly.
TOPNOTCH MEDICAL BOARD PREP PEDIATRICS MAIN DIGITAL HANDOUT BY DR. PUNONGBAYAN AND DR. DE VERA
For inquiries visit www.topnotchboardprep.com.ph or https://www.facebook.com/topnotchmedicalboardprep/
This handout is only valid for the March 2021 PLE batch. This will be rendered obsolete for the next batch since we update our handouts regularly.

Nelson Textbook of Pediatrics, 20th ed


PLEASE READ MORE ON: A 4 year old male child is brought to the emergency room due to
difficulty breathing. He has high fever of 39°C. 12 hours ago, he started
• Cerebral Palsy
having sore throat and a mild fever which progressed and prompted
o Different types consult. Patient is seen drooling and the neck is slightly held
• Different types of Migraine hyperextended. What is the most likely diagnosis in this case?
• Myasthenia Gravis A. Acute Laryngotracheobronchitis
• Bell palsy B. Acute Bronchiolitis
C. Acute Epiglottitis
D. Foreign Body Aspiration
PULMONOLOGY The main clinical features of acute epiglottitis are highlighted
✔ GUIDE QUESTIONS in this case. It is one of the common upper airway obstructions
A newborn term baby girl had respiratory distress shortly after birth in a young child.
for which she was intubated. PE reveals a coloboma of the right eye Dr. Punongbayan

and abnormally formed and low-set ears. An NGT cannot be passed Which among the ff. is the best next step in the management of this
although there was no cleft or other mass lesions. The most patient?
appropriate next diagnostic step is: A. Start IV and antibiotics
A. CT scan of the head C. MRI of the head B. Chest x-ray
B. Karyotype D. Flexible bronchoscopy C. Secure airway with intubation
D. Nebulize with racemic epinephrine
The case is about a newborn who had an acute onset of
respiratory distress. There was mention of abnormal physical Lookout for clues that denote respiratory distress in the case
features as well (coloboma and low-set ears) implying a such as drooling and a hyperextended neck which would need
syndrome. Part of this syndrome is the need to insert an NGT immediate stabilization of the airway.
implying an obstruction. CHARGE syndrome would come to Dr. Punongbayan

mind for this patient and one of its features is a choanal atresia, In relation to the above case, if an x-ray was done in this patient, what is
the most common anomaly of the nose occurring in 1/7000 live the expected finding?
births and mostly affecting females. The atretic plate may be A. Consolidation C. Steeple sign
directly seen by a flexible rhinoscopy B. Thumb sign D. Ragged air column sign
Nelson Textbook of Pediatrics 21st ed. 2020
Dr. Punongbayan
The thumb sign is a manifestation of an edematous and
enlarged epiglottis seen on lateral soft tissue X ray of the neck.
SUPPLEMENT: QUICK SHEET It projects as a rounded soft tissue structure into the
CHARGE syndrome: hypopharynx.
• C – coloboma of the eye Dr. Punongbayan

• H – heart defect
• A – atresia choanae VIRAL CROUP EPIGLOTTITIS
• R – retarded growth and development or CNS anomalies Age group • 3 mos to 3 yrs • 3-7 yrs
• G – genital anomalies or hypogonadism Stridor • 88% • 8%
• E – ear anomalies • parainfluenza • H. influenzae
Pathogen
virus type B
✔ GUIDE QUESTIONS • prodrome • rapid (4-12
Onset
Respiratory problems rank high among Filipino children. The nature • (1-7 days) hrs)
of this problem is frequently: Fever severity • low grade • high grade
A. Congenital C. Allergic • barking cough, • muffled voice,
B. Metabolic D. Infectious Associated Sx
• hoarseness • drooling
The principal agent that causes the common colds is the:
A. Respiratory syncytial virus Response to
• stridor
B. Coronavirus racemic • none
improves
C. Parainfluenza virus epinephrine
D. Rhinovirus • “thumbprint/
CXR • “steeple sign”
• leaf sign”

TOPNOTCH MEDICAL BOARD PREP PEDIATRICS MAIN DIGITAL HANDOUT BY DR. PUNONGBAYAN AND DR. DE VERA Page 58 of 105
For inquiries visit www.topnotchboardprep.com.ph or https://www.facebook.com/topnotchmedicalboardprep/
This handout is only valid for the March 2021 PLE batch. This will be rendered obsolete for the next batch since we update our handouts regularly.
TOPNOTCH MEDICAL BOARD PREP PEDIATRICS MAIN DIGITAL HANDOUT BY DR. PUNONGBAYAN AND DR. DE VERA
For inquiries visit www.topnotchboardprep.com.ph or https://www.facebook.com/topnotchmedicalboardprep/
This handout is only valid for the March 2021 PLE batch. This will be rendered obsolete for the next batch since we update our handouts regularly.

SINUSITIS
• Organisms: S. pneumoniae (30%), non-typable H. influenzae
(20%), M. catarrhalis (20%)
• co
Anaerobes are uncommon causes of acute sinusitis in children
• Anything that impairs mucociliary transport or causes nasal
STRIDOR BARKING COUGH obstruction predisposes to sinusitis

§m-
https://qrs.ly/9pbj832 OF CROUP • S/Sx: colds and cough >10-14 days, purulent nasal discharge for
https://qrs.ly/k6bj83w 3-4 consecutive days, headache, tenderness over the sinuses
The succeeding X rays are the typical findings in croup (steeple sign, the • X-ray: air-fluid levels, opacification of the sinuses
one on the left) and in epiglottitis (thumbprint sign, the one on the right). • Tx: antibiotics x 14 days (Co-Amoxiclav)
Dr. Punongbayan • Complications are abscess, meningitis

✔ GUIDE QUESTIONS
Do you remember?
• What are the sinuses present at birth?
Maxillary Ethmoid ,

• What sinuses are pneumatized at 4 years old?


sphenoid
• What sinuses begin to develop at 7-8 years old?
Frontal Frontal
Sphenoid
Answers: Maxillary and Ethmoid

✔ GUIDE QUESTIONS
A 3-year-old male is brought to the ER due to acute onset of noisy Clue: ME present at birth (ME is maxillary and ethmoid)
Dr. Punongbayan
breathing. Patient is coughing from time to time, no cyanosis noted,
and he points to the neck when asked about pain. Mother says that the ✔ GUIDE QUESTIONS
patient was apparently well and playing with his toys before the onset A 3-year-old male child was brought to the clinic due to intermittent
of the symptoms. What is the most likely diagnosis? episodes of continuous coughing until the child turns purple followed
A. Bronchiolitis by a deep loud inspiration, 1 week ago, the mother recalled that he had
B. Asthma episodes of sneezing, rhinorrhea, what is the most likely diagnosis?
C. Foreign body aspiration A. Diphtheria C. Bronchiolitis
D. Bronchitis B. Pertussis D. Pneumonia
Sudden onset of respiratory distress and/or noisy breathing in Buzz phrases for a case of pertussis are: intermittent cough,
a previously well young child (esp. in the 1st 3 years of life) highly paroxysms of cough, usually well and active in between the
signifies foreign body airway obstruction. paroxysms of cough
Dr. Punongbayan
Dr. Punongbayan
Which foreign body is most commonly obtained from respiratory tracts
In relation to the question above, which among the ff. is the best
of children?
treatment for the above condition?
A. Marbles
A. Azithromycin C. Ceftriaxone
B. Beads
B. Penicillin G D. Doxycycline
C. Nuts
D. Berries
In the following table, the main presenting feature is colds with sore
throat and nasal congestion. Additional clinical features will narrow WHOOPING COUGH
down your initial impression to the most likely diagnosis. https://qrs.ly/93bj830
Dr. Punongbayan

CLINICAL
KEY CLUES DIAGNOSIS PERTUSSIS
VIGNETTE
Prominent itching & • 3 stages lasting 2 weeks each
sneezing, nasal Allergic rhinitis o Catarrhal, Paroxysmal, Convalescent
eosinophilia • Purely or predominantly cough. Absent
Unilateral foul o Fever, malaise, exanthem, sore throat, hoarseness, tachypnea,
smelling discharge, Foreign body in wheeze, crackles
bloody nasal the nose • Complications
secretion o Hemorrhage (subconjunctival or intracranial)
Headache, facial pain, o Seizure
Patient presents periorbital edema, Sinusitis o Otitis media
with sore, rhinorrhea for >2wks o Atelectasis
scratchy throat, Persistent rhinorrhea o Pneumonia
Congenital
nasal obstruction w/ onset in the first 3 • Drug of choice: Macrolide (Erythromycin or Clarithromycin)
syphilis
and rhinorrhea mos of life (snuffles) • Bordetella pertussis (gm- coccobacilli)
Hx of prolonged use • Whooping cough
Rhinitis
of topical or oral • Source: secretions from RT of infected persons
medicamentosa
decongestant • Transmitted by close contact via large aerosol drops
Paroxysms of cough • Period of communicability: from 7 days after exposure to 4 wks
leaving the baby after onset of typical paroxysms; most infectious during the
Pertussis or
breathless & catarrhal stage
whooping cough
subconjunctival • Incubation period: 3-12 days
hemorrhages
(snuffles is a catarrhal discharge from the nasal mucous membranes in CARE OF HOUSEHOLD AND OTHER CLOSE CONTACTS:
congenital syphilis) • A macrolide agent should be given promptly to all household
Dr. Punongbayan
contacts and other close contacts regardless of age, history of
immunization, and symptoms.
• The same age-related drugs and doses are used for
treatment.

TOPNOTCH MEDICAL BOARD PREP PEDIATRICS MAIN DIGITAL HANDOUT BY DR. PUNONGBAYAN AND DR. DE VERA Page 59 of 105
For inquiries visit www.topnotchboardprep.com.ph or https://www.facebook.com/topnotchmedicalboardprep/
This handout is only valid for the March 2021 PLE batch. This will be rendered obsolete for the next batch since we update our handouts regularly.
TOPNOTCH MEDICAL BOARD PREP PEDIATRICS MAIN DIGITAL HANDOUT BY DR. PUNONGBAYAN AND DR. DE VERA
For inquiries visit www.topnotchboardprep.com.ph or https://www.facebook.com/topnotchmedicalboardprep/
This handout is only valid for the March 2021 PLE batch. This will be rendered obsolete for the next batch since we update our handouts regularly.
✔ GUIDE QUESTIONS
The most common complication of acute nasopharyngitis is: RECURRENT STREP PHARYNGITIS
A. Otitis media C. Meningitis • Culture (+) strep pharyngitis that has been severe and frequent:
B. Pneumonia D. Tracheitis >7 episodes in the previous year or >5 in each of the preceding 2
To prevent occurrence of glomerulonephritis or rheumatic fever in years → consider tonsillectomy!
children, cases of acute tonsillopharyngitis is best treated with:
A. Bacitracin C. Chloramphenicol
B. Penicillin D. Tetracycline

PATIENTS PRESENTING
WITH UPPER AIRWAY OBSTRUCTION
CLINICAL VIGNETTE CXR FINDINGS DX TX
Patchy infiltrates & ragged
Bacterial tracheitis antibiotics
tracheal column
If vaccinated: Staph. aureus; if
“Thumb’s sign” Acute epiglottitis
unvaccinated: Hib
All levels of respiratory distress:
Croup or Dexamethasone; moderate to severe
Subglottic narrowing
laryngotracheobronchitis distress: steroid and add racemic
epinephrine
Patient presents with s/sx
of upper respiratory
obstruction Air trapping on the right
lung w/ mediastinal shift Foreign body airway obstruction bronchoscopy
towards the right lung

All levels of respiratory distress:


Dexamethasone; moderate to severe
“Steeple sign” Croup of LTB
distress: steroid and add racemic
epinephrine

PE findings of wheezing and a prolonged expiratory phase


reinforces the diagnosis of asthma
ROLE OF X-RAY IN CROUP Dr. Punongbayan

https://qrs.ly/kybj839
PATIENTS PRESENTING WITH WHEEZING
✔ GUIDE QUESTIONS
✔ GUIDE QUESTION Respiratory difficulty in bronchial asthma is due to:
A 4-year-old male was brought to the clinic due to coughing and A. Bronchial muscle spasm C. Mucus hypersecretion
wheezing usually worse at night. The patient has been having these B. Bronchial mucosal edema D. All of the above
symptoms intermittently for the past year. However, his coughing and In the care of asthmatic children, one should be cautious with the
wheezing has progressively become worse with the mother noting prolonged use of this drug:
difficulty of breathing that prompted consult. PE revealed HR 105, RR A. Prednisone C. Salbutamol
46, T 37°C, chest PE presence of wheezing on bilateral lung fields, B. Epinephrine D. Terbutaline
occasional rhonchi, and a prolonged expiratory phase. What is the
most likely diagnosis? Prednisone is recommended for acute mild exacerbations of
A. Atypical pneumonia D. Bronchitis asthma in patients who can be sent home and is given for a
B. Bronchial asthma E. Pertussis duration of 5-7 days only.
Dr. Punongbayan
C. Bronchiolitis
Typical presentation of asthma is coughing and wheezing
usually more often and severe at night that interrupts sleep; the

ASTHMA

TOPNOTCH MEDICAL BOARD PREP PEDIATRICS MAIN DIGITAL HANDOUT BY DR. PUNONGBAYAN AND DR. DE VERA Page 60 of 105
For inquiries visit www.topnotchboardprep.com.ph or https://www.facebook.com/topnotchmedicalboardprep/
This handout is only valid for the March 2021 PLE batch. This will be rendered obsolete for the next batch since we update our handouts regularly.
TOPNOTCH MEDICAL BOARD PREP PEDIATRICS MAIN DIGITAL HANDOUT BY DR. PUNONGBAYAN AND DR. DE VERA
For inquiries visit www.topnotchboardprep.com.ph or https://www.facebook.com/topnotchmedicalboardprep/
This handout is only valid for the March 2021 PLE batch. This will be rendered obsolete for the next batch since we update our handouts regularly.
Potentially modifiable risk factors for exacerbations • Others: sputum/blood eosinophilia
• Medications: ICS not prescribed, poor adherence, incorrect
inhaler technique, high SABA use GINA GUIDELINES 2020:
• Comorbidities: obesity, chronic rhinosinusitis, GERD, confirmed • Every adult and adolescent with asthma should receive ICS-
food allergy, anxiety, depression, pregnancy containing controller medication to reduce their risk of
• Exposures: smoking, air pollution, allergen exposure serious exacerbation, even patients with infrequent symptoms.
• Setting: major socioeconomic problems • Every patient with asthma should have a reliever inhaler, either
• Lung function: low FEV1, esp. if <60% predicted low-dose ICS-formoterol or SABA.
• Prior intubation or in intensive care for asthma; having 1 or
more severe exacerbations in the last 12 months

CLASSIFY ASTHMA BASED ON SEVERITY: (GINA 2002)


PERSISTENT
INTERMITTENT
MILD MODERATE SEVERE
• Daily
Daytime • > 1x/week but less than • Daily
• < 1x/week • Affects daily
Symptoms daily • Limits daily activities
activities
Nighttime Symptoms • < 2x/month • > 2x/month • > 1x/week • > 1x/week
PEFR • > 80% predicted • > 80% predicted • 60-79% • < 60%
PEFR Variability • < 20% • 20-30% • > 30% • > 30%
FEV1 • > 80% predicted • > 80% predicted • 60-79% • < 60%
The worst feature determines the severity classification
LEVELS OF ASTHMA CONTROL (GINA 2006)
CONTROLLED PARTLY CONTROLLED
CHARACTERISTIC UNCONTROLLED
(ALL OF THE FOLLOWING) (ANY PRESENT IN ANY WEEK)
Daytime symptoms • None (2 or less / week) • > twice / week
Limitations of activities • None • Any
• 3 or more features of
Nocturnal symptoms / awakening • None • Any partly controlled
Need for rescue / “reliever” asthma present in
• None (2 or less / week) • > twice / week
treatment any week
Lung function • < 80% predicted or personal best
• Normal
(PEF or FEV1) (if known) on any day
Exacerbation • None • One or more / year 1 in any week

INITIAL ASSESSMENT OF ACUTE ASTHMA


SYMPTOMS MILD SEVERE
Altered consciousness • No • Agitated, confused, or drowsy
Oximetry on presentation (Sa02) • = or >94% • <90%
Talks in sentences/ words • sentences • words
• >200 bpm (0-3 yrs)
Pulse rate • < 100 bpm
• >180 bpm (4-5 yrs)
Central cyanosis • absent • Likely to be present
Wheeze intensity • variable • May be quiet

TOPNOTCH MEDICAL BOARD PREP PEDIATRICS MAIN DIGITAL HANDOUT BY DR. PUNONGBAYAN AND DR. DE VERA Page 61 of 105
For inquiries visit www.topnotchboardprep.com.ph or https://www.facebook.com/topnotchmedicalboardprep/
This handout is only valid for the March 2021 PLE batch. This will be rendered obsolete for the next batch since we update our handouts regularly.
TOPNOTCH MEDICAL BOARD PREP PEDIATRICS MAIN DIGITAL HANDOUT BY DR. PUNONGBAYAN AND DR. DE VERA
For inquiries visit www.topnotchboardprep.com.ph or https://www.facebook.com/topnotchmedicalboardprep/
This handout is only valid for the March 2021 PLE batch. This will be rendered obsolete for the next batch since we update our handouts regularly.

ASSESS FOR DISCHARGE ARRANGE AT DISCHARGE Management of bronchiolitis: American Academy of


• Symptoms improved not • Reliever: continue as Pediatrics 2014 guidelines: SUPPORTIVE MANAGEMENT
needing SABA needed The AAP no longer recommends a trial dose of a bronchodilator,
such as Albuterol or Salbutamol, because evidence to date
• PEF improving and >60- • Controller: start or step up shows that bronchodilators are ineffective in changing the
80% of personal best or • Prednisolone: continue for course of bronchiolitis (evidence quality: B, strong
predicted 5-7 days recommendation). Bronchodilators do not shorten the days of
• O2 sat >94% room air • Follow-up within 2-7 days symptoms, admission rates, or length of stay. Systemic steroids
• Resources at home and racemic epinephrine are also not recommended.
adequate However, if the choices would mention a bronchodilator, that
would still be the most plausible answer as actual local practice
• Management of acute attacks: still include it in the management of bronchiolitis for its dilating
properties.
o short-acting inhaled beta2-agonist Dr. Punongbayan
o oral or IV steroids (Prednisolone/Methylprednisolone)
o anticholinergics (ipratropium bromide) – never used alone PATIENTS PRESENTING WITH WHEEZING
o methylxanthines (theophylline, aminophylline): NOT first line
Hx of viral infection among the family
• Management in between attacks: Bronchiolitis
members. Px is 2 y/o
o inhaled corticosteroids
Hx of atopy in the family. Recurrent
o long-acting inhaled beta2-agonist
wheezing especially after mild viral Bronchial asthma
o leukotriene modifiers (Montelukast)
infection or after exercise
Wheezes are heard loudest over the
✔ GUIDE QUESTIONS
An 8-month-old boy was brought to the ER by his mother due to fast
trachea. Persistent wheezing never Chondromalacia
breathing and wheezing. Mother is known to be a smoker and has a seems to go away
history of bronchial asthma. The mother says that her baby has been Absent breath sounds on right lung. Foreign body
well until 2 days ago when he started having runny nose and a mild Patient is 3 y/o airway obstruction
undocumented fever. PE of the infant reveals wheezing on both lung
Bronchiolitis – again, history of wheezing with exposure to ill family
fields. What is the most likely diagnosis in this case?
members, affecting young children points to this diagnosis
A. Asthma C. Pneumonia
Chondromalacia – think structural abnormality especially when
B. Bronchiolitis D. Laryngotracheobronchitis
wheezing was detected on early infancy
The main features of bronchiolitis are: fever, cough, and Foreign body airway obstruction – age was mentioned to highlight
wheezing in the 1st 2 years of life; there is inflammation of the the increased incidence of FBAO in this age group
lining of epithelial cells of bronchioles causing mucus Dr. Punongbayan

production, inflammation and cellular necrosis. ✔ GUIDE QUESTIONS


Bronchiolitis is caused by obstruction and collapse of the small Management of infectious croup in a hospitalized 15-month-old child:
airways during expiration. It affects the younger children A. Immediate intubation
because bronchiolar wall thickening significantly affects B. Ampicillin and chloramphenicol
airflow. Resistance is inversely proportional to the 4th power of C. Erythromycin
the radius of the bronchiolar passage. D. Aerosolized racemic epinephrine
Bronchiolitis is more common in boys, non-breastfed, those who A hospitalized toddler for infectious croup denotes moderate to
live in crowded conditions, with mothers who smoked during severe respiratory distress, hence the need for additional
pregnancy, racemic epinephrine in the management.
Dr. Punongbayan Racemic epinephrine works by relaxing the muscles in the
If a chest x-ray is requested for this infant, what will be the expected airways and tightening of the blood vessels. Thus, bronchial
finding? and tracheal secretions and airway wall edema decreases.
A. Hyperinflated lungs with patchy atelectasis Racemic epinephrine stimulates both α- and β-adrenergic
B. Steeple sign receptors. It acts on vascular smooth muscle to produce
C. Lobar consolidation vasoconstriction which markedly decreases blood flow at the
D. Diffuse interstitial infiltrates capillary level. This shrinks upper respiratory mucosa and
A general radiographic presentation of obstruction is reduces edema.
hyperaeration or hyperinflation of lungs especially with Dr. Punongbayan
flattening of the diaphragm on a lateral CXR view. Characteristics finding in the CXR of a patient with staphylococcal
Dr. Punongbayan pneumonia include the following, EXCEPT:
In relation to the above case, what is the best step in the management A. Hemothorax C. Pleural effusion
of this patient? B. Pneumatocele D. Pneumothorax
A. Immediate endotracheal intubation
B. Nebulized albuterol and oxygen Hemothorax is most commonly brought about by trauma.
Dr. Punongbayan
C. IV antibiotics
D. Racemic epinephrine

TOPNOTCH MEDICAL BOARD PREP PEDIATRICS MAIN DIGITAL HANDOUT BY DR. PUNONGBAYAN AND DR. DE VERA Page 62 of 105
For inquiries visit www.topnotchboardprep.com.ph or https://www.facebook.com/topnotchmedicalboardprep/
This handout is only valid for the March 2021 PLE batch. This will be rendered obsolete for the next batch since we update our handouts regularly.
TOPNOTCH MEDICAL BOARD PREP PEDIATRICS MAIN DIGITAL HANDOUT BY DR. PUNONGBAYAN AND DR. DE VERA
For inquiries visit www.topnotchboardprep.com.ph or https://www.facebook.com/topnotchmedicalboardprep/
This handout is only valid for the March 2021 PLE batch. This will be rendered obsolete for the next batch since we update our handouts regularly.

PATIENTS PRESENTING WITH PNEUMONIA wheezing, • Prominent areas of


stridor cavitation & multiple Staphylococcus
• Poorly nourished
pneumatoceles
• Unvaccinated Measles
• Right sided hilar Mycobacterium
• w/ onset of rashes all over pneumonia
adenopathy tuberculosis
the body
• Px has CF Pneumococcus is the most common cause of lobar consolidation
Dr. Punongbayan
• Px has CGD
Pseudomonas ✔ GUIDE QUESTIONS
• Px is a burn px
Px with A 3 y/o male comes in with a 2-day history of runny nose and mild dry
• Px is neutropenic
fever, cough with a fever of 38°C. His elder brother was noted to have cough
cough, and • Px is a teen/ young adult and colds 5 days prior. He was brought in for consult today because the
tachycardia • Lives in a dormitory mother noted worsening of cough with a barking character. The child’s
Mycoplasma
• Initial cough is non- voice was also hoarse. PE showed HR 102, RR 43, T 38°C, seen sitting
productive upright, looks anxious whenever examined, slightly erythematous
posterior pharynx, and stridor. What is the most likely diagnosis?
• Px has his own aviarium in
Psittacosis A. Atypical pneumonia D. Epiglottitis
his home B. Bronchiolitis E. Laryngotracheobronchitis
• Hx of eye discharge during C. Bacterial tracheitis
the 1st 5-14 days of neonatal Chlamydia
Buzz words/phrases for LTB: history of URTI, fever, ill family
period member, barking cough, stridor.
Dr. Punongbayan
Mycoplasma – features mentioned above are common presentation of an
atypical infection A 5 y/o male patient comes in with a 2-day history of runny nose and
Psittacosis – also known as parrot fever, and ornithosis—is a zoonotic mild dry cough with a fever of 39°C. He was brought in for consult
infectious disease in humans caused by a bacterium called Chlamydia today because the mother noted worsening of brassy cough, with
psittaci and contracted from infected parrots; fever, chills, dry cough, hoarseness, and with production of copious purulent sputum. PE
weakness, fatigue revealed HR 102, RR 58, T 39°C. The child is seen on semi-recumbent
Chlamydia – history of conjunctivitis, subacute onset, sharp cough, fever position with his mother on the examination bed, looks anxious, with
usually absent, no wheeze; usually affects 3 week-3 month-old infants slightly erythematous posterior pharynx, (+) rhonchi on auscultation.
Dr. Punongbayan What is the most likely diagnosis?
A. Atypical pneumonia D. Epiglottitis
CAUSATIVE B. Bronchiolitis E. Laryngotracheobronchitis
PATHOLOGIC PROCESS C. Bacterial tracheitis
AGENT
• Attaches to respiratory epithelium Buzz words/phrases for Bacterial Tracheitis: high fever,
patient is in distress, acutely ill, brassy cough, copious purulent
• Inhibits cellular destruction
sputum
• Sloughed cellular debris and Mycoplasma Dr. Punongbayan
inflammatory cells and mucus cause If a chest x-ray was done for the patient, what is the most likely finding
airway obstruction seen?
• Extensive areas of hemorrhagic A. Thumb sign
B. Ragged air column sign
necrosis
C. Normal chest x-ray
• Irregular areas of cavitation Staphylococcus D. Steeple sign
• Pneumatoceles, empyema and E. Diffuse interstitial infiltrates
bronchopulmonary fistulas Finding of irregular lining of the trachea due to
• Diffuse infection with interstitial pseudomembranes on lateral neck X ray of patients with
pneumonia bacterial tracheitis
• Necrosis of tracheobronchial mucosa, Dr. Punongbayan

formation of large amounts of exudate, GABHS


edema, and local hemorrhage TYPE OF COUGH
• Involvement of lymphatic vessels and Staccato Chlamydia
pleural Brassy Staphylococcus
• Local edema that aids in the proliferation Barking “Seal” Parainfluenza
of organisms and spread into adjacent Whooping
Pneumococcus Bordetella
areas resulting in focal lobar Post tussive vomiting
involvement Most severe in the morning Asthma
With vigorous exercise Exercise-induced asthma
CLINICAL Disappears with sleep Habit cough
CXR FINDINGS DIAGNOSIS Tight sounding w/ wheezing Asthma
VIGNETTE
Child with • Lobar consolidation Pneumococcus Staccato cough has a sharp quality with sudden bursts
cough, • Hyperinflation w/ bilateral Staphylococcus is the most common cause of bacterial tracheitis) – brassy
colds, interstitial infiltrates & RSV cough is metallic in quality
fever, peribronchial cuffing Parainfluenza – Croup
Bordetella - Pertussis
Dr. Punongbayan

ACUTE INFLAMMATORY UPPER AIRWAY OBSTRUCTION


ACUTE INFECTIOUS
CROUP (LTB) ACUTE EPIGLOTTITIS
LARYNGITIS
• URTI and low-grade fever 1-3 days
• Acute high fever, sore throat, dyspnea,
prior to UAO presentation • Sore throat, cough
and rapidly progressing obstruction
• “barking cough”, hoarseness • generally mild
Signs and (difficult swallowing, labored breathing
• fever may become high-grade • respiratory distress is unusual
symptoms in a few hours)
• symptoms worse at night and agitation • hoarseness out of proportion
• drooling and hyperextended neck;
may aggravate symptoms to symptoms
• no family member is ill
• other family members are ill
• In a tripod position
• Stridor, coryza, normal to moderately • stridor occurs late • Unremarkable except for
PE findings
inflamed pharynx, slight tachypnea • large, cherry red swollen epiglottis by pharyngeal inflammation
laryngoscopy

TOPNOTCH MEDICAL BOARD PREP PEDIATRICS MAIN DIGITAL HANDOUT BY DR. PUNONGBAYAN AND DR. DE VERA Page 63 of 105
For inquiries visit www.topnotchboardprep.com.ph or https://www.facebook.com/topnotchmedicalboardprep/
This handout is only valid for the March 2021 PLE batch. This will be rendered obsolete for the next batch since we update our handouts regularly.
TOPNOTCH MEDICAL BOARD PREP PEDIATRICS MAIN DIGITAL HANDOUT BY DR. PUNONGBAYAN AND DR. DE VERA
For inquiries visit www.topnotchboardprep.com.ph or https://www.facebook.com/topnotchmedicalboardprep/
This handout is only valid for the March 2021 PLE batch. This will be rendered obsolete for the next batch since we update our handouts regularly.
• Mainly clinical;
• Clinical and by laryngoscopy
• X ray of the neck: subglottic narrowing • Lateral X ray of the upper airway: (inflammatory edema of the
or “steeple sign” (may be absent in
• thumb sign vocal cords and subglottic
Diagnosis patients with croup or present as a
• direct or flexible fiberoptic tissue)
normal variant)
laryngoscopy for direct visualization • primary site of obstruction is
• X rays do not correlate well with
the subglottic area
disease severity
• Airway management and treatment of
hypoxia
• Medical emergency: airway;
• nebulized racemic epinephrine for
Treatment Cefotaxime, Ceftriaxone, or Meropenem • Supportive
moderate to severe croup
IV
• single dose of oral dexamethasone 0.6
mg/kg
SPASMODIC CROUP BACTERIAL TRACHEITIS
• 1-3 years old; similar to LTB EXCEPT the history of a viral
prodrome and fever in the patient and family are often • Mean age 5-7 years old; M > F
absent; causes are viral, allergic, psychologic • often follows a viral respiratory infection
Signs and
• mostly nighttime symptoms with mild to moderate coryza • brassy cough
symptoms
and hoarseness • high fever and “toxicity” can occur immediately or
• awakens with a metallic, barking cough, noisy after a few days of apparent improvement
inspiration, appears frightened
• Usually afebrile
• Can lie flat, does not drool, no dysphagia
• severity usually diminishes within several hours and the
PE findings • mucosal swelling at the level of cricoid cartilage with
following day may appear well
copious, thick, purulent secretions
• episodes recur often several times
• Based on bacterial upper airway disease (high fever,
purulent airway secretions, absence of classic findings
Diagnosis • clinical of epiglottitis)
• lateral X ray of the neck: pseudomembranes
detachment in the trachea
• Vancomycin or Clindamycin AND a 3rd generation
Treatment • Same as croup (LTB)
cephalosporin
• Most common is S. aureus
• More of an allergic reaction to viral antigens than direct • MRSA, Strep. pneumoniae, Strep. pyogenes, Moraxella
Etiology
infection catarrhalis, non-typeable H. influenzae, anaerobic
organisms

✔ GUIDE QUESTION
An 8 year-old male comes in to the clinic for checkup but he feels well
and has no symptoms. He has come in contact with his sick grandfather
who was recently diagnosed to have tuberculosis. A skin test was done
which revealed a 10mm induration and 5mm erythema and chest X ray
is negative. What is the next best step in the management of this
patient?
A. supportive, watchful waiting
B. none, negative skin test
C. INH for 9 months
D. INH, Rifampicin, Pyrazinamide and Ethambutol for 4 months
then 2 months Rif
TIP: Approach to a case that appears to be pulmonary TB.
Identify the features mentioned, take note of onset of cough
(acute vs chronic), PPD result (10 mm is positive but 5 mm is
positive too with exposure to adult with PTB and if with
symptoms and abnormal CXR), then classify the patient as to
✔ GUIDE QUESTIONS PTB exposure, infection, disease, reactivation
In most children, the only evidence of primary tuberculosis is: In this case, patient had no symptoms but with exposure to a
A. Cough sick grandfather, a positive PPD of 10 mm, normal chest X ray.
B. Afternoon fever The classification then is PTB infection which is managed with
C. Erythema nodosum INH based on current local guidelines.
D. Recent conversion of PPD to positive Dr. Punongbayan

Tuberculin sensitivity develops _____ after its administration on the


forearm. CLASSIFICATION
A.5 days C. 3 months • TB exposure: (+) exposure to and adult/adolescent with active
B. 72 hours D. 7 months
disease, (-) PPD, no signs/symptoms, negative chest x ray
The most common extrapulmonary form of tuberculosis in children is:
A. Meningitis C. Ileitis findings
B. Scrofula D. TB verrucosa cutis • TB infection: ± exposure, (+) PPD, no signs/symptoms,
A 3 year-old boy has a positive tuberculin skin test. Which of the negative chest x ray findings
following is suggestive of military TB? • TB disease: 3 or more of the ff criteria:
A. Infection of hilar lymph node B. Weight Loss 1. Exposure to an adult/adolescent with active TB disease
C. Hepatosplenomegaly D. Chronic cough 2. (+) PPD
3. Signs/symptoms suggestive of TB
PULMONARY TUBERCULOSIS 4. Abnormal chest x ray findings
• Primary complex (Ghon complex): 5. Laboratory findings (histological, cytological, biochemical,
1. Primary pulmonary focus immunologic, and/or molecular)
2. Regional lymph nodes • TB reactivation / relapse / previously treated
3. Peritracheal lymph nodes
4. Localized pleurisy between the middle & lower lobes
* Time between entry of the tubercle bacilli and tissue hypersensitivity ranges
from 3-12 weeks
TOPNOTCH MEDICAL BOARD PREP PEDIATRICS MAIN DIGITAL HANDOUT BY DR. PUNONGBAYAN AND DR. DE VERA Page 64 of 105
For inquiries visit www.topnotchboardprep.com.ph or https://www.facebook.com/topnotchmedicalboardprep/
This handout is only valid for the March 2021 PLE batch. This will be rendered obsolete for the next batch since we update our handouts regularly.
TOPNOTCH MEDICAL BOARD PREP PEDIATRICS MAIN DIGITAL HANDOUT BY DR. PUNONGBAYAN AND DR. DE VERA
For inquiries visit www.topnotchboardprep.com.ph or https://www.facebook.com/topnotchmedicalboardprep/
This handout is only valid for the March 2021 PLE batch. This will be rendered obsolete for the next batch since we update our handouts regularly.
SIGNS AND SYMPTOMS mentioned in #3 of TB disease MANAGEMENT
(ANY 3 OR MORE): • INH 10 mg/kg/day PO (10-15 mkday; max 300 mg/day)
• Cough w/ or w/o wheezing for > 2 wks • Rif 15 mg/kg/day PO (10-20 mgkday; max 600 mg/day)
• Unexplained fever for > 2 wks • PZA 25 mg/kg/day PO (20-40 mkday; max 2 grams/day)
• Failure to gain weight; weight loss, weight faltering • Ethambutol 15 mg/kg/day PO (15-25 mkday; max 1.2
• Failure to respond to 2 wks appropriate antibiotic therapy for grams/day)
LRTI • Streptomycin IM 20-40 mg IM once daily (max 1 gram/day)
• Failure to regain previous state of health after two weeks of a
viral infection SUPPLEMENT: QUICK SHEET
• Fatigue, reduced playfulness or lethargy Additional notes on PTB drugs:
• INH bactericidal and inhibits mycolic acid synthesis
TB DISEASE: • RIF inhibits DNA-dependent RNA polymerase
• Based on bacteriological status or on the anatomical site: • PZA disrupts membrane energy metabolism
o Bacteriologically-confirmed – biological specimen is (+) by • ETM inhibits transferase enzymes involved in cell wall
smear microscopy, culture, or rapid diagnostic tests synthesis
o Clinically-diagnosed – active TB diagnosed by a clinician and • STREP bactericidal and inhibits protein synthesis
patient is given a full course of meds but does not fulfill the
NOTE: The following table showing TB treatment has its own categories.
criteria for letter (a)
It is different from the classification of pulmonary TB based on whether
• Based on bacteriological status or on the anatomical site: the patient is exposed, infected, or has the disease, or had reactivation.
o Pulmonary disease – involves the lung parenchyma and Dr. Punongbayan
tracheobronchial tree TB TREATMENT REGIMENS (DOH)
o Extrapulmonary TB – involves the organs other than the CATEGORY OF TREATMENT REGIMEN
lungs and tracheobronchial tree • Pulmonary TB, new (bacterio-
logically confirmed or clinically-
PPD INTERPRETATION diagnosed)
• Equal or >10 mm is (+) 2HRZE/
Cat I • Extrapulmonary TB, new
• Equal or >5 mm is (+) in the presence of any or all of the ff: 4HR
(bacteriologically-confirmed or
1. history of close contact with a known or suspected case of TB clinically diagnosed), except CNS /
2. clinical findings suggestive of TB bones or joints
3. chest X ray findings suggestive of TB • Extrapulmonary TB, new 2HRZE/
4. immunocompromised condition Cat Ia
• (CNS / bones or joints) 10HR
• Pulmonary or extrapulmonary,
MOLECULAR DIAGNOSTIC TESTS
previously treated drug-susceptible
• INTERFERON GAMMA RELEASE ASSAY TB (whether bacteriologically-
o Blood sample mixed with antigens specific for MTB complex confirmed or clinically diagnosed) 2HRZES/
strains and absent from the BCG vaccine strain Cat II o relapse 1HRZE
o WBCs from infected persons release IFN-g as a marker of o treatment after failure /5HRE
infection o treatment after lost to follow up
o A (+) result suggests that MTB infection is likely o previous treatment outcome
o Commercially available and FDA-approved: QuantiFERON TB unknown
Gold in tube, ELISPOT-based T-SPOT TB
• Extrapulmonary, previously treated
o Advantages: result available in 24 hours, booster phenomenon
drug susceptible TB (whether 2HRZES/
does not occur, not affected by prior BCG vaccination Cat III 1HRZE
bacteriologically-confirmed or
o Limitations: expensive, needs to be processed immediately,
clinically-diagnosed CNS / bones or /9HRE
paucity of data in <5 year-old children and
joints)
immunocompromised patients Department of Health. Manual of Procedures of the National Tuberculosis Control Program. 5th ed. 2014.

• GENE Xpert MTB/RIF ASSAY SUPPLEMENT: QUICK SHEET


o real time PCR-based molecular test that can simultaneously Notes on anti-TB drugs:
detect TB bacteria and rifampicin resistance in clinical • For INH and RIF: discontinue if ALT/AST is more than 3-5x
specimens in less than 2 hours the normal values; best absorbed on an empty stomach; no
o Meta-analysis on the diagnostic accuracy in children: dose adjustment for renal dysfunction; common adverse
§ Expectorated sputum - 55-90% reactions are hepatitis, allergic skin reactions, peripheral
§ Induced sputum – 40-100% neuropathy
§ Gastric lavage or aspirate – 40-100% • For PZA: Discontinue if AST / ALT is more than 3-5x the
o WHO recommendations on usage of Gene Xpert assay: normal values; requires dose adjustment in renal failure;
§ Used as replacement for conventional microscopy adverse reactions are nausea, vomiting, polyarthralgia,
§ Culture and drug susceptibility testing as the initial hypersensitivity reaction
diagnostic test
• For Ethambutol: Previously omitted from regimens for
§ Suspected at risk for drug-resistant TB or have HIV-
children <6 years old due to difficulty of monitoring optic
associated TB
neuritis; new evidence shows safety in children at
recommended doses in the absence of renal impairment
KEY FACTS TUBERCULIN SKIN TEST AND IGRA
• For Streptomycin: potential for nephrotoxicity and
• TST and IGRA generally should not be tested on persons with electrolyte disorders; can cause eighth cranial nerve damage
low risk of TB infection and TB disease / neuromuscular blockade; contraindicated in pregnancy;
• Neither TST nor IGRA can distinguish active TB from LTBI dose adjustment in renal insufficiency
• IGRA can distinguish LTBI from previous BCG administration
✔ GUIDE QUESTIONS
• Routine testing with TST and IGRA is not generally
A 9 month-old male was brought to the clinic due to cough and difficulty
recommended except when there is suspected active TB in
breathing. The mother noted that he initially had runny nose with clear
immunocompromised patients or indeterminate results from nasal discharge and dry cough 3 days prior. At present, the infant has
either test a fever of 38.5°C, fast breathing, and cough which prompted consult. PE
• TST is preferred over IGRA in children <5 years old. revealed HR of 158, RR 68, T 38.8°C; he is irritable, coughing, (+) nasal
*Tuberculosis in Infancy and Childhood. 4th ed.
PPS Committee on Handbook on Childhood Tuberculosis. 2016
flaring, (+) subcostal retractions, (+) crackles over bilateral lung fields,
and decreased breath sounds over the right lung field. What is the most
likely diagnosis?
A. Pneumonia D. Bronchitis
B. Bronchial asthma E. Pertussis
C. Bronchiolitis

TOPNOTCH MEDICAL BOARD PREP PEDIATRICS MAIN DIGITAL HANDOUT BY DR. PUNONGBAYAN AND DR. DE VERA Page 65 of 105
For inquiries visit www.topnotchboardprep.com.ph or https://www.facebook.com/topnotchmedicalboardprep/
This handout is only valid for the March 2021 PLE batch. This will be rendered obsolete for the next batch since we update our handouts regularly.
TOPNOTCH MEDICAL BOARD PREP PEDIATRICS MAIN DIGITAL HANDOUT BY DR. PUNONGBAYAN AND DR. DE VERA
For inquiries visit www.topnotchboardprep.com.ph or https://www.facebook.com/topnotchmedicalboardprep/
This handout is only valid for the March 2021 PLE batch. This will be rendered obsolete for the next batch since we update our handouts regularly.
Triad of pneumonia: fever, cough, tachypnea (the latter is If a chest X ray was done in the above case which revealed bilateral
the most sensitive indicator); significant finding of crackles in interstitial infiltrates, and the assessment made was pneumonia, what
PE as it implies alveolar-filled inflammatory fluid interacting would be the classification of the above patient?
with air as one inhales A. Minimal risk D. High risk
Dr. Punongbayan B. PCAP B E. Low risk
In relation to the above case, where should the patient be sent? C. PCAP C
A. Home C. Regular ward Which of the ff. is considered the most consistent clinical manifestation
B. OPD D. ICU of pneumonia in children?
A. Fever C. Cough
B. Tachypnea D. Crackles on auscultation

PNEUMONIA
PAPP GUIDELINE ON PCAP 2016

APPROACH TO DIAGNOSIS THERAPEUTIC MANAGEMENT


• For PCAP A or B managed in an ambulatory setting: • For PCAP A and B
o Chest X ray may be requested o Oral amoxicillin (40-50 mg/kg/day in 3 doses for 7 days; max
o Chest X ray, CBC, CRP, ESR, procalcitonin, or blood culture 1500 mg/day)
should not be routinely requested o If with known hypersensitivity to amoxicillin: Azithromycin
• For PCAP C in a hospital setting, the ff procedures should be 10 mg/kg/day OD for 3 days (max dose of 500 mg/day) or
done: Clarithromycin 15 mg/kg/day in 2 doses for 7 days (max dose
o Gram stain and/or CS of pleural fluid when available to 1000 mg/day)
determine etiology • PCAP C
o Pulse oximetry and arterial blood gas to assess gas exchange o No previous antibiotic use, complete HiB vaccination
• For PCAP C in a hospital setting, the ff procedures may be § Penicillin G 100,000 units/kg/day in 4 divided doses
done: o Incomplete or unknown status of HiB vaccination
o chest X ray to confirm clinical suspicion of multilobar § Ampicillin 100 mg/kg/day in 4 divided doses
consolidation, lung abscess, pleural effusion, pneumothorax or DURING THE COURSE OF ILLNESS FOR PCAP C, THE
pneumomediastinum, to determine appropriateness of FOLLOWING:
antibiotic usage, and to predict clinical outcome (Grade B) • May be beneficial
o CRP (Grade A) and WBC (Grade D) to determine o Zinc supplement reduced mortality (Grace B1)
appropriateness of antibiotic usage o Use of either nasal prong or catheter for oxygenation (Grade
o sputum and blood CS (Grade C) to determine etiology A1)
o pulse oximetry to predict clinical outcome (Grade B) • May not be beneficial
o serum electrolytes and glucose (Grade C) to determine o Zinc supplement to reduce treatment failure or LOS (Grade A1)
metabolic derangement o Vitamin D3 to reduce length of hospital stay (Grade A1)
o Parenteral steroid, probiotic, VCO, oral folate, and nebulization
SUPPLEMENT: QUICK SHEET using NSS and or acetylcysteine (Grace C2)
The key to remembering the differences between
ETIOLOGY
classifications of pneumonia:
1. PCAP A and B are both non-severe but the difference • etiologies in young infants:
between the two is the presence of mild dehydration in o 0-28 days: GBS, E. coli, Listeria S. pneumoniae
PCAP B. o 3 wks – 3 mos: RSV*, parainfluenza, Chlamydia, Mycoplasma,
2. PCAP C (severe) and D (very severe) both present with S. pneumoniae*
retractions, head bobbing, and cyanosis. The differences • etiologies in older infants and children:
between the two are the presence of the following in PCAP o 4 mos – 5 yrs: viruses*, S. pneumoniae*, H. influenzae type b,
D: G –A- S (grunting, apnea, sensorium changes) Mycoplasma
o 5-15 yrs: Mycoplasma*, S. pneumoniae
Viruses are the most common over all cause of PCAP worldwide.
S. pneumoniae is the most common bacterial cause of PCAP in children
4mos-5 years of age.
Dr. Punongbayan

TOPNOTCH MEDICAL BOARD PREP PEDIATRICS MAIN DIGITAL HANDOUT BY DR. PUNONGBAYAN AND DR. DE VERA Page 66 of 105
For inquiries visit www.topnotchboardprep.com.ph or https://www.facebook.com/topnotchmedicalboardprep/
This handout is only valid for the March 2021 PLE batch. This will be rendered obsolete for the next batch since we update our handouts regularly.
TOPNOTCH MEDICAL BOARD PREP PEDIATRICS MAIN DIGITAL HANDOUT BY DR. PUNONGBAYAN AND DR. DE VERA
For inquiries visit www.topnotchboardprep.com.ph or https://www.facebook.com/topnotchmedicalboardprep/
This handout is only valid for the March 2021 PLE batch. This will be rendered obsolete for the next batch since we update our handouts regularly.

S/SX CXR, CBC TX


• diffuse streaky infiltrates;
Viral • cough, wheezing, stridor • supportive
lymphocytosis
• (0-2mo) Ampicillin + Aminoglycoside
• cough, high fever, dyspnea, • lobar consolidation
Bacterial • (2mo-5yrs) Ceftriaxone or Cefuroxime
• dullness to percussion • neutrophilia
+ Ampicillin or Amoclav
Mycoplasma
• less ill-looking, • interstitial pattern, usually • (>5 yrs) Erythromycin, Clarithromycin, or
“walking
• non-productive cough lower lobes Azithromycin
pneumonia”
• 6 wks to 6 mos, “staccato” • hyperinflation, “ground-glass”
Chlamydia • Erythromycin PO x 14 days
cough, maternal Hx of infection appearance; eosinophilia

✔ GUIDE QUESTIONS • Group A streptococcus, anaerobes, Staphylococcus aureus;


A 3 y/o male patient presents with a 2-day history of gradual decreased others are Klebsiella and Haemophilus influenzae
oral intake, irritability, and fever of 38.5°C. The child points to the neck • IV antibiotics with or without surgical drainage: 3rd gen
area that seems to bother him, and his voice was a little muffled. PE cephalosporin with Ampicillin-Sulbactam OR Clindamycin
revealed HR 108, RR 33, T 38.7 C with minimal movement and refusal
to move the neck, (+) neck stiffness. On oral examination. there is mild
erythematous posterior pharynx and bulging of the posterior ✔ GUIDE QUESTIONS
pharyngeal wall; chest PE revealed no crackles, occasional rhonchi. A 4 year-old male child was brought to the clinic due to worsening
What is the most likely diagnosis? cough now with purulent sputum. Symptoms started 3 days ago with
A. Retropharyngeal abscess runny nose with clear nasal discharge, after which a frequent
B. Peritonsillar abscess intermittent dry hacking cough followed. Examination revealed HR 90,
C. Bacterial tracheitis RR 33, T 37.2°C, PE revealed: erythematous congested nasal mucosa,
D. Epiglottitis slightly erythematous posterior pharynx, with some occasional coarse
E. Laryngotracheobronchitis crackles and scattered high pitched wheezing, no nasal flaring, no
retractions. What is the most likely diagnosis?
A common differential diagnosis for this is Acute Epiglottitis A. Pneumonia
since they both present acutely and in respiratory distress; B. Bronchial asthma
consider retropharyngeal abscess based on PE finding of C. Bronchiolitis
bulging of the posterior pharyngeal wall; the red, swollen D. Bronchitis
epiglottis can be directly visualized by laryngoscopy (done in E. Pertussis
double set up in anticipation of possible respiratory arrest) in
cases of epiglottitis. The main feature of acute bronchitis is the dry hacking cough;
Dr. Punongbayan most of the patients have low- to moderate-grade fever and do
A 16 year-old female patient presents with a 3-day history of cough and not appear to be acutely ill; most commonly caused by viruses;
sore throat. She sought consult because of worsening sore throat and supportive management
cough, fever, and trismus. There was also some dysphagia. PE revealed Dr. Punongbayan

an asymmetrical right tonsillar bulge with displacement of the uvula


and erythematous posterior pharyngeal wall, BP 100/60, HR 98, RR 22,
T 38.8°C. What is the most likely diagnosis? RHEUMATOLOGY
A. Retropharyngeal abscess SYMPTOMS SUGGESTIVE OF RHEUMATIC DISEASE:
B. Peritonsillar abscess • Joint pain, fever, fatigue, and rash
C. Bacterial tracheitis • Arthralgia without physical findings of arthritis suggests:
D. Epiglottitis
o Infection
E. Laryngotracheobronchitis
What is your management of choice for this patient?
o Malignancy
A. Antibiotics only o orthopedic conditions
B. Surgical management only o benign syndromes
C. Antibiotics and Surgery o pain syndromes like fibromyalgia
D. Antibiotics ± surgical drainage • Pain syndrome (fibromyalgia) – poor sleep, debilitating
generalized joint pain that worsens with activity, school
PERITONSILLAR ABSCESS absences, and normal physical and lab findings in an adolescent
• Growing pains – children aged 3-10 years with a history of
• Bacterial invasion through the capsule of the tonsils
episodic pain that occurs at night after increased daytime
• Adolescents
physical activity and is relieved by rubbing; no limp or
• Group A streptococcus and anaerobes
complaints in the morning
• Fever, sore throat, dysphagia, trismus
• Patellofemoral syndrome – adolescent girl with knee pain
• PE: asymmetric tonsillar bulge with a displaced uvula
aggravated by walking upstairs and on patellar distraction
• CT scan - ideally • Benign hypermobility syndrome – intermittent pain esp. in a
• Surgical drainage (I & D or needle aspiration) and antibiotics girl aged 3-10 years that is increased with activity and is
associated with hyperextensible joints
✔ GUIDE QUESTIONS
A 4 year old female patient presents to the clinic with 5 day history of
fever with temperatures between 38-39°C. There was bilateral redness
of the eyes with no discharge, decreased appetite and intake, physical
exam reveals dry cracked lips, erythematous oral and pharyngeal
mucosal areas, (+) cervical lymphadenopathy, maculopapular and
scarlatiniform rashes most prominent at the groin areas, there was also
noted mild swelling and erythema of the hands and feet, HR 108, RR 28
T > 38.8°C what is the most likely diagnosis?
A. Rubeola D. Kawasaki disease
B. Steven Johnson syndrome E. Scarlet fever
C. Ritter disease
In a patient with known Kawasaki disease, which phase of the illness
RETROPHARYNGEAL ABSCESS gives the highest risk for sudden death?
• 3-4 years old; M>F A. Prodromal C. Subacute
• Retropharyngeal space located between the pharynx & the B. Acute febrile D. Convalescence
cervical vertebrae & extending down into the superior
mediastinum
• PE: drooling, neck held in hyperextension, bulging of the
posterior pharyngeal wall, neck pain, muffled voice,
respiratory distress

TOPNOTCH MEDICAL BOARD PREP PEDIATRICS MAIN DIGITAL HANDOUT BY DR. PUNONGBAYAN AND DR. DE VERA Page 67 of 105
For inquiries visit www.topnotchboardprep.com.ph or https://www.facebook.com/topnotchmedicalboardprep/
This handout is only valid for the March 2021 PLE batch. This will be rendered obsolete for the next batch since we update our handouts regularly.
TOPNOTCH MEDICAL BOARD PREP PEDIATRICS MAIN DIGITAL HANDOUT BY DR. PUNONGBAYAN AND DR. DE VERA
For inquiries visit www.topnotchboardprep.com.ph or https://www.facebook.com/topnotchmedicalboardprep/
This handout is only valid for the March 2021 PLE batch. This will be rendered obsolete for the next batch since we update our handouts regularly.

KAWASAKI DISEASE ✔ GUIDE QUESTIONS


(MUCOCUTANEOUS LYMPH NODE SYNDROME) hich of the ff. is the preferred treatment of Kawasaki in the acute stage
of the disease?
SUPPLEMENT: QUICK SHEET
A. IV Ig C. Warfarin
KAWASAKI DISEASE B. Low dose aspirin D. Both A and B
• FEVER for at least 5 days +4 of the following features: Refer to Table 166-3 seen below for the treatment of Kawasaki
o Conjunctivitis disease lifted from Nelson textbook of Pediatrics 20th ed.
Dr. Punongbayan
§ Bilateral, bulbar injection without exudate
o Rash
§ Any form of rash
o Adenopathy
§ Cervical adenopathy, non-suppurative
o Strawberry tongue and other oropharyngeal changes
§ Red cracked, swollen lips
o Hand changes
§ Brawny edema, induration
§ Peeling around the nail beds
OTHER FEATURES OF KD:
• medium-sized vasculitis that affects more males and is most
common in the 1st 5 years of life
• unknown cause
• aneurysm of the major coronary arteries is the most
characteristic finding
• Perineal desquamation is common in the acute phase
especially in younger than 1 year-old patients
• Periungual desquamation begins 1-3 weeks after the onset of
illness Nelson Textbook of Pediatrics, 20th ed
• Coronary artery aneurysms develop in the 2nd-3rd week of
illness in untreated cases.
ROLE OF IVIG
IN KAWASAKI DISEASE
https://qrs.ly/62bj83g

IVIG RESISTANT KD
• persistent or recrudescent fever 36 hr after completion of the
initial IVIG infusion
• Increased risk for CAA
• Clinical phases of KD: • another dose of IVIG at 2 g/kg is administered to patients with
1. Acute febrile phase: last 1-2 wks; fever and other acute signs IVIG resistance
of illness o Re-treatment of patients with refractory Kawasaki disease -
2. Subacute phase: begins when fever & other acute signs have another IVIG 2g/kg
abated; associated with desquamation, thrombocytosis, o If there is a poor response to the 2nd IVIG, some patients have
development of coronary aneurysms, and highest risk of responded to IV methylprednisolone 30 mg/kg/day for 3 days.
sudden death in those who have developed aneurysms; lasts o Patients with a small solitary aneurysm should continue
until the 4th week aspirin indefinitely.
Nelson Textbook of Pediatrics, 21st ed. 2020
3. Convalescent phase: begins when all clinical signs have
disappeared & continues until ESR & CRP return to normal
about 6-8 wks after the onset MULTISYSTEM INFLAMMATORY SYNDROME
• ATYPICAL OR INCOMPLETE KAWASAKI DISEASE – any infant IN CHILDREN (MIS-C):
or child with prolonged unexplained fever, fewer than 4 of the • A serious, rare condition in children in which the body’s own
principal clinical features, and compatible laboratory or immune system overreacts to a stimulus
echocardiographic findings; most frequent in infants • Results in inflammation of multiple organ systems → leads to
• May have manifestations such as renal failure, unilateral facial impaired organ function and organ failure
nerve palsy, pulmonary infiltrates and/or effusion, testicular • Diagnosed in <21 years of age in mostly Europe and the USA
swelling, acute abdomen, hemophagocytic syndrome • Unknown cause
• INCOMPLETE KAWASAKI DISEASE – considered in all children • Mounting evidence that is linked to COVID-19
with unexplained fever for 5 or more days with 3 or less of the • Still needs more research and evidence
clinical criteria
o More common in young infants
MIS-C CRITERIA BY CDC 2020:
o Perineal erythema and desquamation
o Higher risk to develop coronary artery lesions • patient is 21 years of age or younger and has had a fever of 38°C
o Laboratory evidence of systemic inflammation for at least 24 hours
• blood work shows indications of inflammation
ANCILLARY TESTS IN KD: • requires treatment at a hospital (often in the intensive care unit)
• NO diagnostic test due to severe illness that includes dysfunction of two or more
• WBC is normal to elevated organs, particularly the heart, blood vessels, GI organs, lungs,
• Elevated ESR, CRP for 4-6 weeks kidneys, skin, eyes, or nervous system
• Normocytic, normochromic anemia • absence of other diagnoses that explain inflammatory
• Platelet count usually normal in the 1st wk of illness but rapidly symptoms
increases in the 2nd-3rd wk • A positive test for current or past infection by SARS-CoV-2, or
• Sterile pyuria failing a positive test, evidence of exposure to someone with the
virus within four weeks of the onset of symptoms of MIS-C
• Mildly high liver transaminases
• Evidence of inflammation (including but not limited to 1 or more
• 2D echo should be performed at diagnosis and repeated
of the ff):
after 2-3 weeks of illness
o Elevated CRP
o If both are normal, a repeat study should be done 6-8 wks after
o Elevated ESR
onset of illness.
TOPNOTCH MEDICAL BOARD PREP PEDIATRICS MAIN DIGITAL HANDOUT BY DR. PUNONGBAYAN AND DR. DE VERA Page 68 of 105
For inquiries visit www.topnotchboardprep.com.ph or https://www.facebook.com/topnotchmedicalboardprep/
This handout is only valid for the March 2021 PLE batch. This will be rendered obsolete for the next batch since we update our handouts regularly.
TOPNOTCH MEDICAL BOARD PREP PEDIATRICS MAIN DIGITAL HANDOUT BY DR. PUNONGBAYAN AND DR. DE VERA
For inquiries visit www.topnotchboardprep.com.ph or https://www.facebook.com/topnotchmedicalboardprep/
This handout is only valid for the March 2021 PLE batch. This will be rendered obsolete for the next batch since we update our handouts regularly.
o Elevated fibrinogen • Gottron Papules
o Elevated procalcitonin o Alligator skin like appearance
o Elevated D-dimer o Skin over the metacarpal and proximal IPJ may be
o elevated ferritin hypertrophic and reddish pink
o elevated LDH
o Elevated IL-6
o Elevated neutrophils
o Reduced lymphocytes
o Low albumin

Other evaluations:
• ECG • Heliotrope eyelids / heliotrope rash
• 2d echocardiography o Periorbital violaceous erythema that may cross over the
• Cardiac enzyme or troponin bridge of the nose
• B-type natriuretic peptide

✔ GUIDE QUESTION
An 18-year-old female presents with 3 month history of fever ranging
from 37.8 to 38.5°C, weight loss, irritability, and joint pains. She also
has some muscle weakness and has difficulty climbing the stairs and
combing her hair sometimes. She also started having a violaceous
erythematous rash near the periorbital area that extends to the nose.
Current PE: HR 90, RR18, T37.5°C, characteristic rash over the face, (+)
muscle weakness 4/5 over biceps and hips, (+) reddish pink colored
skin over the dorsal surfaces of the metacarpal joints, what is the most
likely diagnosis? • SHAWL SIGN
A. Dermatomyositis
B. SLE
C. Photosensitive dermatitis
D. Vitamin c deficiency
E. Allergic contact dermatitis
When you encounter a case of a possible inflammatory
myopathy or rheumatic in origin, take note of the onset of
disease (usually chronic) and the involved features such as
presence or absence of fever, myalgia, muscle weakness,
arthralgia, arthritis, and rash. In this case, most notable were
the chronic history, fever, muscle weakness and the pink- CLINICAL MANIFESTATIONS:
colored rash on the dorsum of metacarpophalangeal joints, all • Neck flexor or abdominal muscle weakness
pointing to dermatomyositis. • Unable to sit up or head lag during infancy, and Gower sign (use
Dr. Punongbayan
of hands on thighs to stand from a sitting position)
• Derangement of upper airway function
JUVENILE DERMATOMYOSITIS • Dysphagia is a severe prognostic sign
• The most common of pediatric inflammatory myopathies • Constipation, abdominal pain, or diarrhea
• Systemic vasculopathy with characteristic cutaneous findings • Dilated cardiomyopathy
and focal areas of myositis resulting in progressive proximal
muscle weakness
THE EULAR/ACR CLASSIFICATION CRITERIA FOR ADULT AND JUVENILE IDIOPATHIC INFLAMMATORY MYOPATHIES
When no better explanation for the symptoms and signs exists these classification criteria can be used
SCORE POINTS
WITH
VARIABLE NO MUSCLE DEFINITION
MUSCLE
BIOPSY
BIOPSY
Age of onset
Age of onset of first symptom assumed to
18 ≤ Age (years) at onset of first symptom assumed to be
be related to the disease ≥ 18 years and < 1.3 1.5
related to the disease < 40
40 years
Age of onset of first symptom assumed to Age (years) at onset of first symptom assumed to be related to
2.1 2.2
be related to the disease ≥ 40 years the disease ≥ 40
Muscle weakness
Objective symmetric weakness, usually Weakness of proximal upper extremities as defined by manual
progressive, of the proximal upper 0.7 0.7 muscle testing or other objective strength testing, which is
extremities present on both sides and is usually progressive over time
Objective symmetric weakness, usually Weakness of proximal lower extremities as defined by manual
progressive, of the proximal lower 0.8 0.5 muscle testing or other objective strength testing, which is
extremities present on both sides and is usually progressive over time
Muscle grades for neck flexors are relatively lower than neck
Neck flexors are relatively weaker than
1.9 1.6 extensors as defined by manual muscle testing or other
neck extensors
objective strength testing
Muscle grades for proximal muscles in the legs are relatively
In the legs proximal muscles are relatively
0.9 1.2 lower than distal muscles in the legs as defined by manual
weaker than distal muscles
muscle testing or other objective strength testing
Skin manifestations
Purple, lilac-colored or erythematous patches over the eyelids
Heliotrope rash 3.1 3.2
or in a periorbital distribution, often associated with
periorbital edema
Erythematous to violaceous papules over the extensor
Gottron’s papules 2.1 2.7 surfaces of joints, which are sometimes scaly. May occur over
the finger joints, elbows, knees, malleoli and toes
Erythematous to violaceous macules over the extensor
Gottron’s sign 3.3 3.7
surfaces of joints, which are not palpable
TOPNOTCH MEDICAL BOARD PREP PEDIATRICS MAIN DIGITAL HANDOUT BY DR. PUNONGBAYAN AND DR. DE VERA Page 69 of 105
For inquiries visit www.topnotchboardprep.com.ph or https://www.facebook.com/topnotchmedicalboardprep/
This handout is only valid for the March 2021 PLE batch. This will be rendered obsolete for the next batch since we update our handouts regularly.
TOPNOTCH MEDICAL BOARD PREP PEDIATRICS MAIN DIGITAL HANDOUT BY DR. PUNONGBAYAN AND DR. DE VERA
For inquiries visit www.topnotchboardprep.com.ph or https://www.facebook.com/topnotchmedicalboardprep/
This handout is only valid for the March 2021 PLE batch. This will be rendered obsolete for the next batch since we update our handouts regularly.
Other clinical manifestations
Difficulty in swallowing or objective evidence of abnormal
Dysphagia or esophageal dysmotility 0.7 0.6
motility of the esophagus
Laboratory measurements
Anti-Jo-1 (anti-histidyl-tRNA synthetase) Autoantibody test in serum performed with standardized and
3.9 3.8
autoantibody present validated test, showing positive result
Elevated serum levels of creatine kinase
(CK)* or lactate dehydrogenase The most abnormal test values during the disease course
(LDH)* or aspartate aminotransferase 1.3 1.4 (highest absolute level of enzyme) above the relevant upper
(ASAT/AST/SGOT)* or alanine limit of normal
aminotransferase (ALAT/ALT/SGPT)*
Muscle biopsy features- presence of:
Endomysial infiltration of mononuclear Muscle biopsy reveals endomysial mononuclear cells abutting
cells surrounding, but not invading, 1.7 the sarcolemma of otherwise healthy, non-necrotic muscle
myofibres fibers, but there is no clear invasion of the muscle fibers
Mononuclear cells are located in the perimysium and/or
Perimysial and/or perivascular
1.2 located around blood vessels (in either perimysial or
infiltration of mononuclear cells
endomysial vessels)
Muscle biopsy reveals several rows of muscle fibers which are
Perifascicular atrophy 1.9 smaller in the perifascicular region than fibers more centrally
located
Rimmed vacuoles are bluish by Hematoxylin and Eosin
Rimmed vacuoles 3.1
staining and reddish by modified Gomori- Trichrome stains
*Serum levels above the upper limit of normal
LEVEL OF In relation to the case above, what vessels are commonly affected?
SCORE A. Coronary and other muscular arteries
PROBABILITY
DEFINITE IIM Equal or >7.5 Equal or >90% B. Large arteries
C. Medium arteries
Equal or >5.5 D. Small arteries
PROBABLE IIM Equal or >55 to <90%
to <7.5 Skin biopsies have demonstrated vasculitis of the dermal
Score of 5.3 or capillaries and postcapillary venules.
POSSIBLE IIM Equal or >50 to <55%
5.4 Dr. Punongbayan
In relation to the above case, what is the best approach to treatment?
A. Supportive therapy
DIAGNOSTIC CRITERIA (BOHAN & PETER, 1975) B. Mycophenolate or cyclophosphamide
• Classic rash (heliotrope rash of the eyelids, Gottron papules) C. Oral prednisone
PLUS 3 of the ff: D. Plasmapheresis and IV Ig
1. Weakness (symmetric, proximal) Steroids are most often used to treat significant gastrointestinal
2. Muscle enzyme elevation equal or >1 (CK, aspartate involvement or other life-threatening manifestations
aminotransferase, LDH, aldolase) Dr. Punongbayan

3. Electromyographic changes (myopathy, denervation)


4. Muscle biopsy (necrosis, inflammation) HENOCH-SCHÖNLEIN PURPURA
• IgA-mediated vasculitis of small vessels (dermal capillaries
DIAGNOSIS AND LAB FINDINGS
and postcapillary venules)
• PE of the nailfold capillaries show periungual avascularity with
• Most common cause of nonthrombocytopenic purpura in
capillary dropout and vessel dilatation with terminal bush
children
formation
• Unknown cause; typically follows an URTI
• Alanine aminotransferase – most commonly elevated on
• 3-10 yrs old; male:female ratio 1.2-1.8:1
initial presentation
• Linked with nephritis: HLA-B34 and HLA-DRB1*01
• Antibodies to Pm/Scl identify a small, distinct group with a
protracted disease course with pulmonary and cardiac • Pattern of crops of palpable purpura in the dependent parts
involvement • Lab tests are neither specific nor diagnostic
• MRI either full body or of the thigh and shoulder muscles • ↑ ESR, anemia, 50% have ↑ IgA/IgM, (-) ANA & ANCAs
• Muscle biopsy confirms the diagnosis if the clinical and • Skin biopsy: leukocytoclastic angiitis
laboratory manifestations are inconclusive • Renal biopsy: IgA mesangial deposition

TREATMENT TREATMENT
• Corticosteroids – Methylprednisolone for more severe cases • Symptomatic treatment
• Methotrexate decreases the length of treatment with steroids • Self-limiting
• Folic acid reduces toxicity and S/E of folate inhibition like oral • Steroids for severe abdominal pain and joint pain
ulcers, nausea, and anemia • Regardless of the severity of symptoms, patients require serial
• IVIG for severe cases urinalysis for 6 months after diagnosis especially those who
presented with hypertension or urinary abnormalities
✔ GUIDE QUESTIONS
A 7 year-old male child presents to the clinic with a rash extending from ✔ GUIDE QUESTIONS
the buttocks to the lower extremities. The rash is characterized as A 10 year-old female patient presents with morning stiffness and joint
raised pinkish to erythematous purpuric lesions. The mother recalled pain later in the day particularly in the knees and sometimes in the
that he had just gotten well from the flu 1 week ago. He also has right ankles it has been bothering her for the past 2 months. PE revealed
knee pain with slight swelling but not warm and non-erythematous. He joint swelling with limitation of range of motion of the knees and
is presently afebrile. Urinalysis reveals 6-8 RBC, protein +2, WBC 0-2, ankles. No other complaints noted, HR 90, RR 20, T37.2°C. What is the
glucose (-), bacteria +1. What is the most likely diagnosis in this case? most likely the diagnosis?
A. Hemolytic Uremic Syndrome A. Systemic onset JRA
B. Henoch Schönlein Purpura B. Pauciarticular JRA
C. Dengue Fever C. Polyarticular JRA
D. Meningococcemia D. Juvenile spondyloarthropathy
In relation to the above case, what is the pathophysiologic mechanism Pauciarticular is the old term for oligoarthritis type of juvenile
involved? idiopathic arthritis. Oligoarthritis involves less than 4 joints
A. Deposition of IgA and immune complexes involved in the 1st 6 months of presentation. This is based on
B. Microvascular thrombi leading to hemolysis the 1997 ILAR (International League of Associations for
C. Immune reaction against platelet with increased vascular Rheumatology) classification.
permeability Dr. Punongbayan
D. Endotoxin release resulting to DIC and shock
TOPNOTCH MEDICAL BOARD PREP PEDIATRICS MAIN DIGITAL HANDOUT BY DR. PUNONGBAYAN AND DR. DE VERA Page 70 of 105
For inquiries visit www.topnotchboardprep.com.ph or https://www.facebook.com/topnotchmedicalboardprep/
This handout is only valid for the March 2021 PLE batch. This will be rendered obsolete for the next batch since we update our handouts regularly.
TOPNOTCH MEDICAL BOARD PREP PEDIATRICS MAIN DIGITAL HANDOUT BY DR. PUNONGBAYAN AND DR. DE VERA
For inquiries visit www.topnotchboardprep.com.ph or https://www.facebook.com/topnotchmedicalboardprep/
This handout is only valid for the March 2021 PLE batch. This will be rendered obsolete for the next batch since we update our handouts regularly.
Which of the ff. statements pertain to the above case and condition? A 13 y/o female patient presents to the clinic with complaints of fever,
A. The disease is usually associated with RF positive rash, and joint pains. It started 2 months ago with pain and swelling
B. The disease is usually associated with ANA(+) over her knees and sometimes her ankles. She has also been having
C. It is the most aggressive or disabling form intermittent fever 1-2x/day ranging from 38-39°C returning to normal
D. It is the least common type temperatures in between. She would also has faint, macular, salmon-
colored evanescent rash especially at the height of the fever. What is
About 40-85% of patients with pauciarticular or oligoarticular
the most likely diagnosis?
JIA will have positive ANA
Dr. Punongbayan
A. Systemic onset JRA
B. Pauciarticular JRA
C. Polyarticular JRA
D. Rheumatic fever
Features of fever, rash, and joint pain and swelling for more
than 6 weeks point to a case of juvenile idiopathic arthritis.
Letter D (rheumatic fever) is a close differential diagnosis but
when Jones criteria is utilized, the description of the skin lesion
in this case is more of JIA rather than the erythema
marginatum lesions seen in rheumatic fever.
Dr. Punongbayan

JUVENILE IDIOPATHIC ARTHRITIS


INTERNATIONAL LEAGUE OF ASSOCIATIONS FOR RHEUMATOLOGY (ILAR) CLASSIFICATION OF JUVENILE IDIOPATHIC ARTHRITIS:
CATEGORY DEFINITION
• Arthritis in equal or >1 joint with, or preceded by, fever for at least 2 weeks in duration
documented to be daily for at least 3 days and accompanied by 1 or more of the following:
1. Evanescent erythematous rash
SYSTEMIC
2. Generalized lymph node enlargement
3 .Hepatomegaly or splenomegaly or both
4. Serositis
• Arthritis affecting 1-4 joints during the 1st 6 months of disease:
OLIGOARTHRITIS 1. Persistent – equal or <4 joints affected throughout the course
2. Extended – affecting >4 joints after the 1st 6 months of disease
• RF NEGATIVE – affects 5 or more joints during the 1st 6 months of disease
POLYARTHRITIS • RF POSITIVE – affects 5 or more joints during the 1st 6 months of disease; 2 or more tests for RF at least
3 months apart during the 1st 6 months of disease are positive
• Arthritis and psoriasis, or arthritis and at least 2 of the ff:
1. Dactylitis (swelling of 1 or more digits in an asymmetric distribution that extends beyond the joint
PSORIATIC ARTHRITIS margin)
2. Nail pitting (minimum of 2 pits on any 1 or more nails at any time) and onycholysis
3. Psoriasis in first-degree relative
• Arthritis and enthesitis (tenderness at the insertion of a tendon, ligament joint capsule, or fascia
to the bone), or arthritis or enthesitis with at least 2 of the ff:
1. Presence of or history of sacroiliac joint tenderness or inflammatory lumbosacral pain, or both
ENTHESITIS-RELATED 2. Presence of HLA-B27 antigen
ARTHRITIS 3. Onset of arthritis in a male >6 years old
4. Acute (symptomatic) anterior uveitis
5. History of ankylosing spondylitis, enthesitis-related arthritis, sacroiliitis with IBD, Reiter syndrome, or
acute anterior uveitis in first-degree relative
UNDIFFERENTIATED
• Arthritis that fulfills criteria in no category or in 2 or more of the above categories
ARTHRITIS

AMERICAN COLLEGE OF RHEUMATOLOGY VS INTERNATIONAL LEAGUE OF ASSOCIATIONS FOR RHEUMATOLOGY


CLASSIFICATION OF CHILDHOOD CHRONIC ARTHRITIS
PARAMETER ACR (1977) ILAR (1997)
Term • Juvenile rheumatoid arthritis • Juvenile Idiopathic arthritis
• Oligarthritis
= or <4 joints in 1st 6 months after
• Pauciarticular o Persistent: <4 joints for course of dse
presentation
o Extended: >4 joints after 6 months
>4 joints in 1st 6 months after • Polyarticular RF negative
• Polyarticular
presentation • Polyarticular RF positive
Fever, rash, arthritis • Systemic • Systemic
• Psoriatic arthritis
Other categories included • Exclusion of other forms • Enthesitis-related arthritis
• Undifferentiated
Inclusion of psoriatic arthritis, IBD,
• No • Yes
ankylosing spondylitis

• Goals of treatment:
1. To achieve disease remission
2. To prevent or halt joint damage
3. To foster normal growth and development
• Need individualized treatment plan and tailor management based on:
1. Disease subtype and severity
2. Presence of poor prognostic indicators
3. Response to medications – Monitor potential medication toxicities

TOPNOTCH MEDICAL BOARD PREP PEDIATRICS MAIN DIGITAL HANDOUT BY DR. PUNONGBAYAN AND DR. DE VERA Page 71 of 105
For inquiries visit www.topnotchboardprep.com.ph or https://www.facebook.com/topnotchmedicalboardprep/
This handout is only valid for the March 2021 PLE batch. This will be rendered obsolete for the next batch since we update our handouts regularly.
TOPNOTCH MEDICAL BOARD PREP PEDIATRICS MAIN DIGITAL HANDOUT BY DR. PUNONGBAYAN AND DR. DE VERA
For inquiries visit www.topnotchboardprep.com.ph or https://www.facebook.com/topnotchmedicalboardprep/
This handout is only valid for the March 2021 PLE batch. This will be rendered obsolete for the next batch since we update our handouts regularly.
PHARMACOLOGIC TREATMENT
MEDICATION DOSE SUBTYPE SIDE EFFECTS

NSAIDs
• Naproxen • 15 mg/kg/day po BID Polyarthritis,
Gastritis, renal, and hepatic toxicity
• Ibuprofen • 40 mg/kg/day po TID oligoarthritis, systemic
• Meloxicam • 0.125 mg/kg po OD

Disease modifying anti-


rheumatic drugs Nausea, vomiting, oral ulcers, liver
Polyarthritis,
(DMARDS) toxicity, dyscrasias
oligoarthritis, systemic
• Methotrexate (oldest and • 0.5-1 mg/kg po or SC weekly GI upset, allergic reaction, renal
Polyarthritis
least toxic) • Initial 12.5 mkdose po daily; and liver toxicity, SJS
• Sulfasalazine maintain at 40-50 mg/kg BID

Anti-tumor necrosis factor • 0.8 mg/kg SC weekly (max dose • Polyarthritis, • Immunosuppressant, lupus-like
• Etanercept 50 mg/week) oligoarthritis, systemic reaction, demyelinating disease

Interleukin-1 inhibitor • 15-40 kg: 2 mg/kg/dose SC every • Systemic • Immunosuppressant, headache,


• Canakinumab 8 weeks GI upset, injection reaction
• For >40 kg: 150 mg SC every 8
weeks
Interleukin-6 receptor • Depends on the weight and if • IV: systemic and • Immunosuppressant, liver
antagonist given IV or SC polyarthritis toxicity, GI upset, dyslipidemia,
• Tocilizumab • SC: polyarthritis cytopenia

RECOMMENDED USE OF MEDICATIONS 2019 EULAR / ACR CLASSIFICATION CRITERIA FOR SLE:
• Indications of using DMARDs: • The entry criterion is necessary to classify SLE.
o failure of methotrexate monotherapy o Entry criterion: ANA at a titer of equal or >1:80 on HEp-2
o those with poor prognostic factors cells or an equivalent positive test.
o with severe disease onset • At least 1 clinical criterion required to classify SLE. Additional
• Early aggressive therapy with methotrexate and etanercept may additive (clinical or immunology) criteria are counted toward
result in earlier achievement of clinically inactive disease the total score.
• For sJIA: corticosteroids are started followed by IL-6 antagonist o Additive criteria:
therapy to induce a rapid and dramatic response § Do not count a criterion if there is a more likely explanation
than SLE.
✔ GUIDE QUESTION § Occurrence of a criterion in equal or >1 occasion is sufficient.
An 16 year-old female complains of fatigue, muscle and joint aches, and § Criteria need not occur simultaneously.
fever that have lasted for 2 months. On PE, she had rashes over her § Within each domain, only the highest-weighted criterion is
cheeks and nose as well as friction rub on cardiac auscultation. Lab counted toward the total score if more than 1 is present.
findings: Hgb 10 g/dL, Hct 33%, platelet count of 145,00/mm3, WBC
4,300/mm3, Urinalysis 3+ proteinuria. The following is expected in her
disease except?
• SLE classification requires at least one clinical criterion and
A. Responds to steroids and other immunosuppressant ≥10 points.
B. More common in females than males • Criteria need not occur simultaneously
C. ANA has more than 90% specificity • Within each domain, only the highest weighted criterion is
D. Anti-dsDNA reflects disease activity counted toward the total score
ANA has 95-99% sensitivity but poor specificity to SLE; anti-
Smith is specific for the diagnosis of SLE CLINICAL DOMAINS AND CRITERIA WEIGHT
Dr. Punongbayan
Constitutional
Fever 2
SYSTEMIC LUPUS ERYTHEMATOSUS Hematologic
• Autoantibody production against self-antigens resulting in Leukopenia 3
inflammatory damage to target organs Thrombocytopenia 4
• Unpredictable course of illness Autoimmune hemolysis 4
• Decreased prevalence in whites; female : male 5:1 Neuropsychiatric
• Fibrinoid deposits found in blood vessel walls of affected organs Delirium 2
• Skin, joints, kidneys, blood-forming cells, blood vessels, and Psychosis 3
CNS Seizure 5
Mucocutaneous
CLASSIFICATION CRITERIA Non-scarring alopecia 2
CRITERIA SENSITIVITY SPECIFICITY Oral ulcers 2
Subacute cutaneous OR discoid lupus 4
American College of 82.8% 93.4%
Rheumatology (1997) iMMUNOLOGY DOMAINS AND CRITERIA WEIGHT
Systemic Lupus 96.7% 83.7% Antiphospholipid antibodies
International Anti-cardiolipin antibodies OR Anti-β2GP1 2
Collaborating Clinics antibodies OR Lupus anticoagulant
(2012) Complement proteins
Low C3 OR low C4 3
European League 96.1% 93.4% Low C3 AND C4 4
Against Rheumatism / SLE-specific antibodies
ACR (2019) Anti-dsDNA antibody OR Anti-Smith antibody 6

TOPNOTCH MEDICAL BOARD PREP PEDIATRICS MAIN DIGITAL HANDOUT BY DR. PUNONGBAYAN AND DR. DE VERA Page 72 of 105
For inquiries visit www.topnotchboardprep.com.ph or https://www.facebook.com/topnotchmedicalboardprep/
This handout is only valid for the March 2021 PLE batch. This will be rendered obsolete for the next batch since we update our handouts regularly.
TOPNOTCH MEDICAL BOARD PREP PEDIATRICS MAIN DIGITAL HANDOUT BY DR. PUNONGBAYAN AND DR. DE VERA
For inquiries visit www.topnotchboardprep.com.ph or https://www.facebook.com/topnotchmedicalboardprep/
This handout is only valid for the March 2021 PLE batch. This will be rendered obsolete for the next batch since we update our handouts regularly.

SUPPLEMENT: QUICK SHEET


CRITERIA FOR DIAGNOSIS OF SLE (1997):
(SOAP BRAIN MD)
• Serositis (pleuritis, pericarditis)
Nelson Textbook of Pediatrics, 20th ed
• Oral ulcers (painless) DIAGNOSIS
• Arthritis (2 or more joints) • Confirmed by the combination of clinical & laboratory
• Photosensitivity manifestations
• requires the presence of 4 of 11 criteria serially or
• Blood changes (anemia, leukopenia, low platelet)
simultaneously
• Renal disorder (persistent proteinuria, cellular casts) • (+)ANA result is not required for diagnosis but its absence is rare
• ANA abnormal titer • Renal biopsy to confirm lupus nephritis & to determine
• Immunological changes (anti-DNA Ab, anti-Sm) treatment
• Neurological signs (seizures, frank psychosis) • ANA – excellent screening tool but can be (+) in other rheumatic
conditions; (+) in 95-99% of patients but has poor specificity for
• Malar rash
SLE
• Discoid rash • Anti-double-stranded DNA – more specific for lupus & reflects
the degree of disease activity
• Low total hemolytic complement (CH50), C3, C4 in active disease
• anti-Smith Ab found only in SLE: specific for the diagnosis of
SLE

PLEASE READ MORE ON: SUPPLEMENT: QUICK SHEET


• Ankylosing Spondylosis ANKYLOSING SPONDYLOSIS – commonly affects teen-aged
• ILAR VS ACR Classification of JIA males and presents with low back pain; inflammation of
joints of axial skeleton and limbs
• Juvenile Scleroderma
SCLERODERMA - symmetric thickening and hardening of the
skin with fibrous and degenerative changes of the viscera

TOPNOTCH MEDICAL BOARD PREP PEDIATRICS MAIN DIGITAL HANDOUT BY DR. PUNONGBAYAN AND DR. DE VERA Page 73 of 105
For inquiries visit www.topnotchboardprep.com.ph or https://www.facebook.com/topnotchmedicalboardprep/
This handout is only valid for the March 2021 PLE batch. This will be rendered obsolete for the next batch since we update our handouts regularly.
TOPNOTCH MEDICAL BOARD PREP PEDIATRICS MAIN DIGITAL HANDOUT BY DR. PUNONGBAYAN AND DR. DE VERA
For inquiries visit www.topnotchboardprep.com.ph or https://www.facebook.com/topnotchmedicalboardprep/
This handout is only valid for the March 2021 PLE batch. This will be rendered obsolete for the next batch since we update our handouts regularly.

CARDIOLOGY TRANSITIONAL CIRCULATION


CARDIAC MORPHOGENESIS CHANGES IN CIRCULATION AFTER BIRTH
• Early presomite embryo: the 1st identifiable cardiac precursors • The primary change after birth is a shift of blood flow for gas
are angiogenetic cell clusters arranged on both sides of the exchange from the placenta to the lungs.
embryo’s central axis • Interruption of the umbilical cord results in the following:
• Clusters form paired cardiac tubes by 18 days of gestation o An increase in systemic vascular resistance due to removal of
• Paired tubes fuse in the midline on the ventral surface of the the low-resistance placenta
embryo to form the primitive heart tube by 22 days o Closure of the ductus venosus as a result of lack of blood return
from the placenta
• 20-22 days: embryonic heart begins to contract & exhibit phases • Lung expansion results in the ff:
of the cardiac cycle o Reduction of the PVR → an increase in pulmonary blood flow
• 22-24 days: heart tube begins to bend ventrally & toward the → a fall in PA pressure
right (looping) o Functional closure of the FO occurs due to increased pressure
• 25 days: septation of the ventricles in the LA
• 30 days: septation of the atria o Closure of PDA as a result of increased arterial O2 saturation
• 3 months: AV valve and semilunar valve formation is complete • With expansion of the lungs and the resulting increase in
alveolar O2 tension:
o there is an initial, rapid fall in the PVR (secondary to the
FETAL CIRCULATION
vasodilating effect of O2 on the pulmonary vasculature)
• Oxygenated blood from placenta → 50% of umbilical venous o Between 6-8 weeks of age, there is a slower fall in the PVR and
blood enters hepatic circulation → rest bypasses liver & joins IVC PA pressure.
via DV → RA → FO → LA →LV → ascending aorta (fetal upper o Functional closure of the DA occurs by constriction of the
body and brain) medial, smooth muscle in the ductus within 10-15 hours after
birth.
o Anatomic closure is completed by 2-3 weeks of age by
permanent changes in the endothelium and subintimal layers
of the ductus.

• Fetal SVC blood → RA → TV → RV → PA (only 10% of RV outflow


enters the lungs) → major portion bypasses the lungs and
flows through ductus arteriosus → descending aorta → lower TRANSITIONS OF THE
part of fetal body → placenta via the 2 umbilical arteries
CARDIO-RESPIRATORY SYSTEM AT BIRTH
• Pulmonary vascular resistance decreases
• SVR (systemic vascular resistance) increases
o Removal of the placenta
• Closure of PDA
o Patency dependent on low O2 and high prostaglandins
§ High prostaglandins because of decreased pulmonary
circulation (metabolized in lungs) and increased production
in the placenta
• Functional Closure of PDA (15h)
o ↑ PO2, preferential blood flow from RV to lungs (↓pulmonary
vascular resistance)
• Anatomic Closure
o 2-3 weeks, fibrous proliferation of the intima

NEONATAL CIRCULATION
• A postnatal increase in O2 saturation of the systemic
circulation is the strongest stimulus for constriction of the
Cardiovascular structures unique to the fetus important to ductal smooth muscle which leads to closure of the ductus.
maintain parallel circulation: • The responsiveness of the ductal smooth muscle to 02 is related
1. Ductus venosus: removal of the placenta from the to the gestational age of the newborn; the ductal tissue of a
circulation result in its closure premature infant responds less intensively to oxygen than
2. Foramen ovale: most of the SVC blood goes to the RV; about that of a full-term infant.
1/3 of IVC blood is directed to the LA through the FO whereas
the 2/3 enters the RV and PA Why is there decreased responsiveness of the immature ductus
3. Ductus arteriosus: less oxygenated blood in the PA flows to 02?
through the widely open DA to the descending aorta and then • due to its decreased sensitivity to O2-induced contraction
to the placenta for oxygenation • The DA is more likely to remain open in preterm infants after
• Fetal cardiac output birth because the premature infant’s ductal smooth muscle does
o The fetal heart is unable to increase stroke volume when the not have a fully developed constrictor response to oxygen.
HR falls because it has a low compliance.
o Thus, the fetal cardiac output depends on the HR; when the
HR drops, a serious fall in CO results.
TOPNOTCH MEDICAL BOARD PREP PEDIATRICS MAIN DIGITAL HANDOUT BY DR. PUNONGBAYAN AND DR. DE VERA Page 74 of 105
For inquiries visit www.topnotchboardprep.com.ph or https://www.facebook.com/topnotchmedicalboardprep/
This handout is only valid for the March 2021 PLE batch. This will be rendered obsolete for the next batch since we update our handouts regularly.
TOPNOTCH MEDICAL BOARD PREP PEDIATRICS MAIN DIGITAL HANDOUT BY DR. PUNONGBAYAN AND DR. DE VERA
For inquiries visit www.topnotchboardprep.com.ph or https://www.facebook.com/topnotchmedicalboardprep/
This handout is only valid for the March 2021 PLE batch. This will be rendered obsolete for the next batch since we update our handouts regularly.

The case posted here is that of PDA with the finding of


SUPPLEMENT: QUICK SHEET continuous or machinery-like murmur in the upper left sternal
border or left 2nd ICS plus bounding pulses and a wide pulse
APPROACH TO MURMURS:
pressure.
• All diastolic murmurs are pathologic.
When a term infant is found to have a PDA, the wall of the
• Systolic murmurs are either ejection type or regurgitant type.
ductus is deficient in both the mucoid endothelial layer and the
1. Systolic ejection or blowing murmur – heard best on the muscular media, whereas in the premature infant, the PDA
base or at the 2nd ICS; blood flows through stenotic usually has a normal structure.
structures thus producing a “blowing” sound
Thus, a PDA persisting beyond the 1st few wk of life in a term
2. Systolic regurgitant murmur – heard best on the apex infant rarely closes spontaneously or with pharmacologic
or at the left lower sternal border; blood backflows from intervention; whereas, if early pharmacologic or surgical
one chamber / valve to another because of incompetent intervention is not required in a premature infant, spontaneous
structures closure occurs in most instances.
Dr. Punongbayan
So, lahat ng systolic ejection / blowing murmur – nasa 2nd LICS
A 3 year-old girl was brought to the clinic because of difficulty of
Ang systolic regurgitant murmur – nasa apex or LLSB breathing for the past 2 days. History revealed 5 days cough and colds
Functional or innocent murmurs are usually heard in young infants and with undocumented fever. You learned that she has had recurrent
children, systolic in timing, varies with position, associated with normal respiratory tract infections since she was an infant. She was apparently
diagnostic results (CXR, ECG, 2D echo). diagnosed to have a “heart disease” but was lost to follow-up. On PE,
Dr. Punongbayan she weighs 9 kgs, (+) gr 4/6 systolic regurgitant murmur heard loudest
at the left lower sternal border. Which chambers of the heart are likely
enlarged on chest radiograph?
CONGENITAL HEART DISEASE A. Right side
APPROACH TO CONGENITAL HEART DISEASE B. Left side
C. Both left and right
D. None of the above
Acyanotic Heart This is a case of ventricular septal defect based on the
Disease Cyanotic Heart characteristic murmur which is systolic regurgitant in
Disease character and timing and heard best at the LLSB. The left side
(Left to Right Shunt) of the heart initially enlarges in VSD because of the left to right
direction of the shunt and more blood enters the pulmonary
circulation leading to enlargement of the pulmonary artery and
VSD DECREASED PULMONARY BF: the left chambers of the heart.
ASD Pulmonary Atresia Dr. Punongbayan

PDA Pulmonary Stenosis


COA TOF VENTRICULAR SEPTAL DEFECT
ECD Tricuspid Atresia • Enlargement of the LA, LV and main PA
Ebstein Anomaly • Increase in pulmonary vascular markings
• It is the LV that does volume overwork.
INCREASED PULMONARY BF:
• Because the shunt occurs mainly during systole when the RV
TOGA
also contracts, the shunted blood goes directly to the PA
TAPVR
rather than remaining in the RV cavity.
Truncus Arteriosus

✔ GUIDE QUESTION
Baby Aaron is a full term 14 day-old infant who came in for routine
checkup. Upon auscultation you note a continuous murmur on the left
upper sternal border. Other significant PE findings are the presence of
bounding peripheral pulses and widened pulse pressure. The following
are true regarding this case EXCEPT?
A. the wall of the ductus is deficient in both mucoid endothelial
layer and the muscular media
B. There is a high chance of spontaneous closure
C. This will rarely close despite pharmacologic intervention
D. None of the above

TOPNOTCH MEDICAL BOARD PREP PEDIATRICS MAIN DIGITAL HANDOUT BY DR. PUNONGBAYAN AND DR. DE VERA Page 75 of 105
For inquiries visit www.topnotchboardprep.com.ph or https://www.facebook.com/topnotchmedicalboardprep/
This handout is only valid for the March 2021 PLE batch. This will be rendered obsolete for the next batch since we update our handouts regularly.
TOPNOTCH MEDICAL BOARD PREP PEDIATRICS MAIN DIGITAL HANDOUT BY DR. PUNONGBAYAN AND DR. DE VERA
For inquiries visit www.topnotchboardprep.com.ph or https://www.facebook.com/topnotchmedicalboardprep/
This handout is only valid for the March 2021 PLE batch. This will be rendered obsolete for the next batch since we update our handouts regularly.
CONGENITAL ACYANOTIC HEART DISEASES
DISEASE HEART SOUNDS OTHER PE FINDINGS
• Systolic ejection
• Right sided
ASD murmur at 2nd LICS
enlargement
• widely split S2
• Left sided enlargement;
• Systolic regurgitant
• biventricular
VSD murmur at LLSB
hypertrophy if with
• loud and single S2
Eisenmenger syndrome
• Continuous
• Bounding pulses;
“machinery-like”
• wide pulse pressure;
PDA murmur at the 2 nd
• left-sided enlargement,
left infraclavicular
• enlarged aorta
area

✔ GUIDE QUESTIONS
Which of the following is associated with the presence of an
endocardial cushion defect?
A. Noonan syndrome
B. Marfan syndrome
C. Hunter-Hurler syndrome
D. Down syndrome
SUPPLEMENT: QUICK SHEET Atrioventricular septal defect or ECD is associated with Down
What happens if a large VSD is left untreated? syndrome.
Exposure of pulmonary artery system to high pressure and Marfan syndrome is an inherited disorder of the connective
increased flow → irreversible changes occur in the pulmonary tissue causing abnormalities in the eyes, bone, heart, and blood
arterioles → progressive increase in PVR → when PVR exceeds vessels (mitral valve prolapse and progressive enlargement of
SVR, ductal shunting reverses and becomes R->L → pulmonary the aorta).
vascular obstructive disease (Eisenmenger's syndrome) → PA Hunter syndrome or mucopolysaccharidosis (MPS II) – a rare
is prominent with RVH and pulmonary hypertension → genetic disorder wherein glycosaminoglycans build up in body
bidirectional shunt causes cyanosis tissues; due to a deficiency of iduronate-2-sulfatase causing
heparan sulfate and dermatan sulfate to accumulate in all body
tissues → thickening of cardiac valves resulting in improper
✔ GUIDE QUESTIONS valve closure
A 3 year-old girl was brought to your clinic for a well child visit. Upon
auscultation, you noted a grade 3/6 systolic murmur described as Noonan syndrome – genetic disorder with facial anomalies,
“blowing” on the upper left 2nd ICS and a widely split S2. There is no short stature, webbed neck, chest deformities, undescended
history of cyanosis. The other PE findings are unremarkable. 2D echo testes; pulmonary stenosis is the common cardiac defect.
Dr. Punongbayan
revealed RVH and RAH with a shunt defect measuring 3mm at the site Short of doing a 2D echo, what is one method of distinguishing cyanotic
of the fossa ovalis. What is true about this disease EXCEPT? congenital heart disease from pulmonary disease?
A. Surgery should be attempted immediately
A. Chest x-ray C. Hyperoxia test
B. Spontaneous closure is 87% for lesions < 8mm B. ECG D. ABG
C. Large defects may lead to heart failure
D. LA is not enlarged
HYPEROXIA TEST
This is a case of atrial septal defect. Main features are the ff:
acyanotic, systolic blowing murmur heard best on the left 2nd • 100% FiO2 (O2 hood/rebreather mask) for 10-15 mins
ICS with a widely split S2 and right-sided enlargement. Surgery • Principle: in the absence of fixed cardiac shunt, 100% O2 will
is not needed at this point if the defect is 3mm or less. The most increase alveolar pO2 → increase in pulmonary venous and
common type of ASD is ostium secundum (50-70%) which is systemic arterial pO2
present at the site of fossa ovalis.
The widely split S2 results partially from RBBB which delays CHILD PRESENTING WITH CYANOSIS
both the electrical depolarization of the RV and the ventricular
contraction resulting in delayed closure of the pulmonary valve.
BASIC CASE DIAGNOSIS
Dr. Punongbayan Cyanosis manifesting within
Transposition of great
few hours at birth or within
Arteries
What produces the widely split S2 in ASD? few days of life
• results partially from RBBB which delays both the electrical Cyanosis manifesting after the
depolarization of the RV and the ventricular contraction first year of life, usually in an Tetralogy of Fallot
resulting in delayed closure of the pulmonary valve infant or a toddler

CYANOTIC CONGENITAL HEART DISEASES


I. DECREASED PULMONARY BLOOD FLOW
• Obstruction (TV, RV or pulmonary valve level) & a pathway by
which systemic venous blood can shunt from R->L & enter the
systemic circulation
• Tricuspid atresia
• Tetralogy of Fallot
• Single ventricle with PS
• Degree of cyanosis depends on the degree of obstruction to
pulmonary blood flow

TETRALOGY OF FALLOT
• Occurs in 10% of all CHDs
• Most common cyanotic heart defect beyond infancy
• 4 abnormalities: large VSD, RVOT obstruction, RVH,
overriding of the aorta
• RVOT obstruction is most frequently in the form of infundibular
stenosis (45%)

TOPNOTCH MEDICAL BOARD PREP PEDIATRICS MAIN DIGITAL HANDOUT BY DR. PUNONGBAYAN AND DR. DE VERA Page 76 of 105
For inquiries visit www.topnotchboardprep.com.ph or https://www.facebook.com/topnotchmedicalboardprep/
This handout is only valid for the March 2021 PLE batch. This will be rendered obsolete for the next batch since we update our handouts regularly.
TOPNOTCH MEDICAL BOARD PREP PEDIATRICS MAIN DIGITAL HANDOUT BY DR. PUNONGBAYAN AND DR. DE VERA
For inquiries visit www.topnotchboardprep.com.ph or https://www.facebook.com/topnotchmedicalboardprep/
This handout is only valid for the March 2021 PLE batch. This will be rendered obsolete for the next batch since we update our handouts regularly.
Manifestations of TOF: MANAGEMENT OF HYPOXIC SPELLS
• Ejection click; single S2; gr 3-5/6 systolic ejection murmur at the • Put the child in a knee-chest position (traps the systemic
mid and ULSB with radiation to the upper back (from PS) venous blood in the legs, decreasing the systemic venous return
• CXR: small heart size, decreased pulmonary vascular markings; and helps calm the baby)
concave main PA with an upturned apex (couer en sabot or • Morphine sulfate suppresses the respiratory center &
boot-shaped heart) abolishes hyperpnea (0.2 mg/kg/dose SC)
• NaHCO3 (1 mEq/kg) slow IV
• Oxygen may improve arterial oxygen saturation

How does squatting relieve the spell?


↑ SVR (kinking of major arterial circulation)
in the presence of fixed pulmonary outflow resistance

↓ R->L flow and more of RV blood goes into the PA

↑ oxygenated blood to the LV

↓ stimulation of the respiratory center -- ↓ CO2

↓ systemic venous return

↓ desaturated blood from lower limbs to heart
✔ GUIDE QUESTIONS

An 18 month-old male patient was brought to the clinic because of
intermittent cyanotic episodes more prominent in the lips, mouths,
↑ oxygenated RV blood will be shunted to the LV
fingernails, and toenails which lasts for a few minutes and goes away. ↓
Sometimes the mother notices that the child assumes a squatting this shunted blood has high O2 and low CO2
position. On examination there is a systolic murmur heard loudest over
the left sternal border. What is the most likely diagnosis? • Vasoconstrictors like Phenylephrine raise SVR (0.02 mg/kg IV)
A. VSD C. TGA
• Propranolol may stabilize vascular reactivity of the systemic
B. TOF D. TAPVR
arteries preventing a sudden decrease in SVR (0.01-0.25 mg/kg
Given the age of the patient and features of cyanosis, systolic slow IV)
murmur on the left sternal border, and the relief noted upon
• Ketamine (1-3 mg/kg IV over 60 secs) increases systemic
squatting, these are consistent with Tetralogy of Fallot, the
most common cyanotic congenital heart disease in infants and vascular resistance
young children.
Dr. Punongbayan BLALOCK-TAUSSIG SHUNT
The most common cyanotic heart disease in newborns is: • Palliative systemic-to-pulmonary artery shunt performed to
A. Tetralogy of Fallot C. Truncus arteriosus augment pulmonary artery blood flow
B. Tricuspid atresia D. Transposition of Great Arteries
What is the main pathophysiologic mechanism behind the
• modified Blalock-Taussig shunt
hypercyanotic spells or Tet spells in TOF? o Gore-Tex conduit anastomosed side to side from the
A. due to increased systemic vascular resistance subclavian artery to the homolateral branch of the pulmonary
B. due to overload and pulmonary congestion artery
C. due to decreased pulmonary blood flow
D. due to increased left to right shunting TRICUSPID ATRESIA
MECHANISM OF HYPOXIC SPELL • Atretic (missing) tricuspid valve
• Hypoplastic right ventricle
• Ventricular septal defect
• Atrial septal defect
• Pulmonary stenosis

PATHOPHYSIOLOGY AND MANAGEMENT

HEART OTHER
CONDITION PROCEDURE
SOUNDS FINDINGS
• Systolic • hypoplastic • Glenn
regurgitant RV shunt;
Tricuspid
murmur at • LAD and Fontan
atresia
LLSB LVH in procedure
ECVG
• systolic • boot- • Blalock-
ejection shaped Taussig
murmur at heart on X procedure
TOF
2nd LUSB; ray (couer
loud & en sabot)
single S2
TOPNOTCH MEDICAL BOARD PREP PEDIATRICS MAIN DIGITAL HANDOUT BY DR. PUNONGBAYAN AND DR. DE VERA Page 77 of 105
For inquiries visit www.topnotchboardprep.com.ph or https://www.facebook.com/topnotchmedicalboardprep/
This handout is only valid for the March 2021 PLE batch. This will be rendered obsolete for the next batch since we update our handouts regularly.
TOPNOTCH MEDICAL BOARD PREP PEDIATRICS MAIN DIGITAL HANDOUT BY DR. PUNONGBAYAN AND DR. DE VERA
For inquiries visit www.topnotchboardprep.com.ph or https://www.facebook.com/topnotchmedicalboardprep/
This handout is only valid for the March 2021 PLE batch. This will be rendered obsolete for the next batch since we update our handouts regularly.

II. INCREASED PULMONARY BLOOD FLOW • What is the characteristic anatomy in this condition?
• Not associated with obstruction to pulmonary blood flow • The TA is the embryologic precursor to the aorta and pulmonary
• Cyanosis due to either abnormal ventricular-arterial artery → BOTH ventricles eject blood into a common vessel
connections or total mixing of systemic venous & pulmonary • A VSD is ALWAYS present!
venous blood within the heart • Neural crest cells present in the TA grow in a spiral formation →
• Transposition of the great vessels separation of 2 outflow tracts → forms the aorta & PA
• Total anomalous pulmonary venous return • If this septum fails to form → single outflow tract persists →
• Truncus arteriosus truncus arteriosus
• Systolic ejection murmur with a thrill along the LSB
TRANSPOSITION OF THE GREAT VESSELS • Normal S1 followed by a loud ejection click while S2 is loud and
single
What is the pathophysiology of this condition?
• Apical diastolic low-pitched murmur due to increased flow
• The aorta arises from the RV carrying desaturated blood to the
across the normal mitral valve is audible
body; the PA arises posteriorly from the LV carrying oxygenated
blood to the lungs • Tachypnea, tachycardia, excessive sweating, poor feeding begin
to appear due to increased volume load on the heart produced
• Result: complete separation of pulmonary & systemic
by excessive blood flow through the pulmonary circulation
circulations → hypoxemic blood circulating throughout the body
& hyperoxemic blood circulating in the pulmonary circuit • Mortality by heart failure

TOTAL ANOMALOUS PULMONARY VENOUS RETURN


• All 4 pulmonary veins drain to the RA
• > RV volume overload

• Defects that permit mixing of the 2 circulations (ASD, VSD,


PDA) are needed for survival
• About 50% have PFO or a small PDA
• VSD in 30-40%
• History of cyanosis from birth
• Signs of CHF with dyspnea & feeding difficulties develop during
the newborn period SITE OF CONNECTION % WITH SIGNIFICANT
• Most common cause of cyanotic congenital heart disease in (% OF CASES) OBSTRUCTION
newborns Supracardiac (50) – type I
• Moderate to severe cyanosis esp.in large, male newborns Left SVC (40) 40
• Single & loud S2; no heart murmur is heard in infants with Right SVC (10) 75
an intact ventricular septum Cardiac (25) – type II
• Severe arterial hypoxemia with or without acidosis (hypoxemia Coronary sinus (20) 10
unresponsive to oxygen inhalation) Right atrium (5) 5
• X-ray: egg-shaped cardiac silhouette with a narrow, superior Infracardiac (20) – type III 95-100
mediastinum Mixed (5) – type IV

• Without obstruction: mild cyanosis, tachypnea, feeding


difficulties
• With obstruction: rapid progression to dyspnea, weak pulses,
cardiorespiratory failure
• Chest X ray: prominent PA and RV; increased pulmonary
vascularity; “snowman sign”

Egg on string sign


TRUNCUS ARTERIOSUS
• Pulmonary arteries arise from aorta
• Truncal valve, occasionally quadracuspid, stenotic and/or
insufficient overrides the ventricular septal defect
• Ventricular septal defect, large

TOPNOTCH MEDICAL BOARD PREP PEDIATRICS MAIN DIGITAL HANDOUT BY DR. PUNONGBAYAN AND DR. DE VERA Page 78 of 105
For inquiries visit www.topnotchboardprep.com.ph or https://www.facebook.com/topnotchmedicalboardprep/
This handout is only valid for the March 2021 PLE batch. This will be rendered obsolete for the next batch since we update our handouts regularly.
TOPNOTCH MEDICAL BOARD PREP PEDIATRICS MAIN DIGITAL HANDOUT BY DR. PUNONGBAYAN AND DR. DE VERA
For inquiries visit www.topnotchboardprep.com.ph or https://www.facebook.com/topnotchmedicalboardprep/
This handout is only valid for the March 2021 PLE batch. This will be rendered obsolete for the next batch since we update our handouts regularly.
CONDITION HEART SOUNDS OTHER FINDINGS PROCEDURE
• single & loud S2; no murmur if • egg-shaped heart on X ray • Rashkind, Senning, Mustard,
Transposition of the great
with intact ventricular or Jatene procedure
arteries
septum
• single S2; systolic ejection • minimal cyanosis in • Rastelli procedure
Truncus arteriosus murmur at LSB neonates; older children:
heart failure
• systolic murmur at LSB in • snowman sign or figure-of- • Van Praagh procedure
TAPVR
mild cases 8 on X ray

✔ GUIDE QUESTION CONDITION PE FINDINGS PROCEDURE


A 2 week-old baby boy is seen in the clinic for a well-baby check-up. On • Weak or absent
PE, the baby’s femoral pulses are weak and delayed bilaterally. This femoral pulses; • Primary re-
disease is highly associated with what syndrome? Coarctation
• BP arms > legs; anastomosis or a
A. Down Syndrome C. Klinefelter Syndrome of the aorta
B. Noonan Syndrome D. Turner Syndrome • rib notching in patch aortoplasty
Xray
Congenital heart defects occurs in up to 50% of patients with
• Systolic ejection
Turner syndrome and the most common is left-sided heart • Balloon valvuloplasty
defect such as coarctation of the aorta which was described in Pulmonic murmur at LUSB
• valvotomy (Brock
the case as the baby having weak and delayed femoral pulses. stenosis with radiation to
procedure)
Dr. Punongbayan the upper back
• Balloon valvuloplasty
Aortic • Systolic ejection
COARCTATION OF THE AORTA • Ross procedure
stenosis murmur at RUSB
• Males > females (valve translocation)
• Which part of the aorta is typically affected? CXR FINDINGS DIAGNOSIS
o The lesion is in the descending aorta, distal to the origin of Boot-shaped/Coeur en sabot Tetralogy of Fallot
the left subclavian artery. Egg on string TGA,
o Disease course depends on the degree of obstruction, presence Snowman TAPVR,
of collateral circulation, and associated cardiac anomalies Figure of 8 TAPVR
Rib notching Coarctation of the Aorta
Inverted E Coarctation of the Aorta
3 sign Coarctation of the Aorta

What is the pathophysiology?


• Pressure build-up in the proximal aorta and LV → hypertension in
the upper extremity CXR:
• small heart size,
What are the manifestations? • decreased pulmonary
• Most are asymptomatic but can have CHF if severe vascular markings;
• Weak, delayed, or absent femoral pulses • concave main PA with an
• Blood pressure higher in the arms > legs upturned apex (boot-
• LVH may be seen in CXR or ECG shaped heart)

How does it appear on X ray?


• Rib notching in children
around 7 yrs old PHYSICAL EXAM DIAGNOSIS
• Result of increased blood flow
through the interthoracic and Late systolic murmur
MVP
intercostal vessels which serve with an opening click
as collateral circulation Disparity in pulsation &
BP in the arms & legs
A child is Weak popliteal, CoA
MANAGEMENT presented to posterior tibial, and
• SBE prophylaxis you with dorsalis pedis pulses
• Anti-hypertensive Tx complaints of S2 widely split and fixed
• Balloon angioplasty/stent placement in selected cases (usually exercise in all phases of ASD
recurrent CoA and adolescent/adult) intolerance, respiration
• Neonates with severe CoA: PGE to reopen the ductus easy Loud, harsh, blowing
• Surgical repair – treatment of choice: primary re-anastomosis fatigability VSD
holosystolic murmur
or a patch aortoplasty (area of coarctation is enlarged with a
Wide pulse pressure
roof of prosthesis)
Bounding peripheral
PDA
arterial pulses
Continuous murmur
TOPNOTCH MEDICAL BOARD PREP PEDIATRICS MAIN DIGITAL HANDOUT BY DR. PUNONGBAYAN AND DR. DE VERA Page 79 of 105
For inquiries visit www.topnotchboardprep.com.ph or https://www.facebook.com/topnotchmedicalboardprep/
This handout is only valid for the March 2021 PLE batch. This will be rendered obsolete for the next batch since we update our handouts regularly.
TOPNOTCH MEDICAL BOARD PREP PEDIATRICS MAIN DIGITAL HANDOUT BY DR. PUNONGBAYAN AND DR. DE VERA
For inquiries visit www.topnotchboardprep.com.ph or https://www.facebook.com/topnotchmedicalboardprep/
This handout is only valid for the March 2021 PLE batch. This will be rendered obsolete for the next batch since we update our handouts regularly.
SURGICAL PROCEDURE INDICATION A 13 y/o female patient presents to the clinic with fever and joint pains.
• Blalock-Taussig Shunt with It started 3 days ago when she had fever of 38.8°C with right knee
swelling, which was warm, and very painful. At present, her right knee
GoreTex conduit
pain and swelling has resolved but now her right ankle and left knee is
• Aortopulmonary window shunt TOF swollen and painful. PE revealed BP 90/60, HR 125, RR 24, T 38.7°C,
• Waterson Cooley (+) high pitched apical holosystolic murmur radiating to the axilla.
• Pott shunt What is the most likely dx in this case?
• Rashkind Atrial Septostomy A. Systemic Onset JRA C. Polyarticular JRA
• Jantene Arterial Switch TGA B. Pauciarticular JRA D. Rheumatic Fever
• Senning and Mustard This case mentioned the following main features of rheumatic
• Fontan Procedure Tricuspid Atresia fever: fever, arthralgia, migratory polyarthritis, systolic
regurgitant murmur radiating to the axilla signifying mitral
• Norwood Procedure Hypoplastic Left valve regurgitation. A common differential diagnosis is juvenile
• Glenn anastomosis Heart Syndrome idiopathic arthritis as both conditions usually present with
fever and joint swelling, but the presence of a systolic murmur
HYPOPLASTIC LEFT HEART SYNDROME indicates RF instead.
Dr. Punongbayan
• all of the structures on the left side of the heart are severely
underdeveloped.
• The right ventricle must then do a "double duty" of pumping RHEUMATIC FEVER
blood both to the lungs (via the pulmonary artery) and out to the SUPPLEMENT: QUICK SHEET
body via a patent ductus arteriosus. JONES CRITERIA
• Newborns with HLHS will typically have lower-than-normal • Required : evidence of recent strep infection
oxygen saturations. o ASO, Strep antibodies, Strep A culture, anti-DNase B, anti-
o because all of the blood from the lungs mixes together in the hyaluronidase
single right ventricle before being pumped out of the lungs and • Major (JONES)
body. o J – Joints / arthritis (poly)
• Cyanosis may be the first clue to the presence of a serious o O (©) – Carditis
underlying cardiac condition. o N – Nodules, subcutaneous
• Respiratory distress is often present because the lungs will tend o E – Erythema marginatum
to receive an excessively large amount of blood flow. o S – Sydenham chorea
• There is often no or just a faint murmur present in newborns • Minor (FRAPE)
with HLHS. o F – Fever
• Lethargy, poor feeding, and worsening respiratory distress may o R – Risk Factor (Previous RH or RHD)
be seen as the ductus arteriosus closes. Ultimately, severe shock o A – Arthralgia
resulting in seizures, renal failure, liver failure, and worsening o P – Prolonged PR interval on ECG
cardiac function may develop. o E – Elevated acute phase reactants: ESR / CRP /
leukocytosis
MATERNAL CONDITION FETAL HEART DEFECT
Rubella PDA JONES CRITERIA (1992)
DM TGA MAJOR MANIFESTATIONS
Lupus Complete Heart Block 1. Arthritis (70%)
Persistent Pulmonary
Aspirin • involves large joints simultaneously or in succession
Hypertension
• responds dramatically to salicylates
Alcohol VSD and PS
Lithium Ebstein Anomaly 2. Carditis (50%)
Ebstein anomaly – tricuspid valve is displaced toward the apex) • includes some or all of the following in increasing order of
Dr. Punongbayan
severity:
✔ GUIDE QUESTIONS a. Tachycardia (out of proportion to the fever)
A 16 year-old female develops acute onset of dyspnea characterized as b. Heart murmur of valvulitis – MR or AR
shortness of breath. She also has a high fever of 39°C together with joint c. Pericarditis – friction rub, pericardial effusion, chest pain, ECG
pains over the left knee and right ankle. She has been perfectly well changes
ever since she only recalls a skin infection with purulent discharge over d. Cardiomegaly – seen on chest X-ray
her lower back which resolved on its own. Which among the ff. is the
e. Signs of CHF – gallop rhythm, distant heart sounds,
best single antibody titer to document cutaneous streptococcal skin
infection? cardiomegaly
A. Anti-Streptolysin O Titer
B. Anti-Deoxyribonuclease B SUPPLEMENT:
C. Streptozyme Test AHA Scientific Statement: Revision of Jones Criteria for the
D. Throat Culture And Sensitivity Diagnosis of ARF in the Era of 2D Echo (March 2015):
• Antistreptolysin O (ASO) titer is well standardized; elevated in • Valvulitis is the most consistent feature of ARF.
80% of patients; 333 Todd units in children and 250 Todd units • 2D echocardiography is being used increasingly to diagnose
in adults; a single low titer does not exclude RF carditis.
• Titers usually become elevated 2 weeks after strep infection,
peaks at 4-6 weeks, and decreases after another 2 weeks
• Subclinical carditis refers exclusively to the circumstance in
• A 4-fold rise in titer in 2 samples taken 10 days apart which classic auscultatory findings of valvar dysfunction
Dr. Punongbayan either are not present or are not recognized by the clinician
Which among these findings is considered a minor criteria? but 2d echo reveal mitral or aortic valvulitis.
A. Polyarthralgia D. Subcutaneous Nodules
B. Carditis E. Polyarthritis AHA Statement Concludes the ff: (Mar.2015)
C. Chorea • Echocardiography with Doppler should be performed in all
Tachycardia in rheumatic fever is significant if this is noted during:
cases of confirmed and suspected ARF (Class I; Level of
A. Playing C. Crying
B. Sleeping D. Feeding
Evidence B).
According to the Jones Criteria, which of the following is considered a • Echocardiography/Doppler testing should be performed to
minor manifestation of acute rheumatic fever? assess whether carditis is present in the absence of
A. Saint Vitus dance C. Migratory polyarthritis auscultatory findings, particularly in moderate- to high-risk
B. Elevated ASO titers D. First-degree AV block populations and when ARF is considered likely (Class I; Level
of Evidence B).

TOPNOTCH MEDICAL BOARD PREP PEDIATRICS MAIN DIGITAL HANDOUT BY DR. PUNONGBAYAN AND DR. DE VERA Page 80 of 105
For inquiries visit www.topnotchboardprep.com.ph or https://www.facebook.com/topnotchmedicalboardprep/
This handout is only valid for the March 2021 PLE batch. This will be rendered obsolete for the next batch since we update our handouts regularly.
TOPNOTCH MEDICAL BOARD PREP PEDIATRICS MAIN DIGITAL HANDOUT BY DR. PUNONGBAYAN AND DR. DE VERA
For inquiries visit www.topnotchboardprep.com.ph or https://www.facebook.com/topnotchmedicalboardprep/
This handout is only valid for the March 2021 PLE batch. This will be rendered obsolete for the next batch since we update our handouts regularly.
3. Erythema marginatum EVIDENCE OF ANTECEDENT GROUP A
(<6%) STREPTOCOCCAL INFECTION:
• nonpruritic serpiginous or
1. History of sore throat/scarlet fever unsubstantiated by
annular erythematous
laboratory data is not adequate evidence of recent infection.
evanescent rashes most
2. Streptococcal antibody tests are the most reliable laboratory
prominent on the trunk and
evidence. The onset of the clinical manifestations coincide with
inner proximal portions of
the peak of the streptococcal antibody response.
the extremities;
• Antistreptolysin O (ASO) titer is well standardized; elevated
• never on the face (disappear
in 80% of patients; 333 Todd units in children and 250 Todd
on exposure to cold and
units in adults; a single low titer does not exclude RF
reappear after a hot shower
*Titers usually become elevated 2 weeks after strep infection,
or if covered with a blanket)
peaks at 4-6 weeks, and decreases after another 2 weeks
*a 4-fold rise in titer in 2 samples taken 10 days apart
4. Subcutaneous nodules (2-10%)
• particularly seen in recurrent cases; hard, painless, nonpruritic, Any 1 of the ff can serve as evidence of preceding infection
freely movable swellings 0.2-2 cm in diameter (Jones criteria, March 2015):
• found symmetrically, singly or in clusters on the extensor 1. Increased or rising ASO titer or other streptococcal antibodies
surfaces of large & small joints, over the scalp or along the spine (anti-DNAse B) (Class I; Level of Evidence B). A rise in titer is
• last for weeks better evidence than a single titer result.
2. A positive throat culture for group A-beta hemolytic
streptococci (Class I; Level of Evidence B).
3. A positive rapid group A streptococcal carbohydrate
antigen test in a child whose clinical presentation suggests a
high pretest probability of streptococcal pharyngitis (Class I;
Level of Evidence B).

DIAGNOSIS OF INITIAL ARF (MARCH 2015)


• > 2 major manifestations, or;
• > 1 major plus 2 minor manifestations
• With evidence of previous streptococcal infection

DIAGNOSIS OF RECURRENT RF (2015)


• With a reliable past history of ARF or established RHD, and in the
face of documented GAS infection:
1. 2 major, or;
2. 1 major plus 2 minor, or;
5. Sydenham chorea (10-15%) 3. 3 minor manifestations
• occurs more often in prepubertal girls • may be sufficient for a presumptive diagnosis (Class IIb; Level
• choreic movements (spontaneous purposeless movements of Evidence C)
followed by motor weakness), hypotonia, emotional lability, • When minor manifestations alone are present, the exclusion of
hyperactivity, obsessions & compulsions other more likely causes of the clinical presentation is
recommended before a diagnosis of an ARF recurrence can be
MINOR MANIFESTATIONS made (Class I; Level of Evidence C).
1. Arthralgia – not considered a minor manifestation if arthritis
is present “POSSIBLE” RHEUMATIC FEVER (2015):
2. Fever – exceeds 38.5 C in low risk and >38 C in moderate- and • Where there is genuine uncertainty, it is reasonable to consider
high-risk populations offering 12 months of secondary prophylaxis followed by re-
*In most settings, including all low-risk populations, fever associated with
evaluation to include a careful history and physical examination
ARF usually exceeds 38.5°C orally (Jones criteria March 2015).
in addition to a repeat echocardiogram (Class IIa; Level of
3. Elevated acute phase reactants (CRP >3.0 mg/dL and ESR
>60 mm/hr in low risk and >30 mm/hr in moderate- and high- Evidence C).
risk populations) • In a patient with recurrent symptoms (particularly involving the
4. Prolonged PR interval on the ECG joints) who has been adherent to prophylaxis recommendations
but lacks serological evidence of group A streptococcal infection
and lacks echocardiographic evidence of valvulitis, it is
reasonable to conclude that the recurrent symptoms are not
likely related to ARF, and discontinuation of antibiotic
prophylaxis may be appropriate (Class IIa; Level of Evidence C).

CLINICAL COURSE OF RF:


• Only carditis can cause permanent cardiac damage. Signs of
mild carditis disappear rapidly in weeks but those of severe
carditis may last for 2-6 months.
• Arthritis subsides within a few days to several weeks even
without treatment and does not cause permanent damage.
• Chorea gradually subsides in 6 months or longer and usually
does not cause permanent neurologic sequelae.

DIAGNOSTIC TESTS FOR RF


• CBC
• ESR / CRP
• throat culture
• ASO titer
• chest X-ray
• ECG
• 2-D echo

TOPNOTCH MEDICAL BOARD PREP PEDIATRICS MAIN DIGITAL HANDOUT BY DR. PUNONGBAYAN AND DR. DE VERA Page 81 of 105
For inquiries visit www.topnotchboardprep.com.ph or https://www.facebook.com/topnotchmedicalboardprep/
This handout is only valid for the March 2021 PLE batch. This will be rendered obsolete for the next batch since we update our handouts regularly.
TOPNOTCH MEDICAL BOARD PREP PEDIATRICS MAIN DIGITAL HANDOUT BY DR. PUNONGBAYAN AND DR. DE VERA
For inquiries visit www.topnotchboardprep.com.ph or https://www.facebook.com/topnotchmedicalboardprep/
This handout is only valid for the March 2021 PLE batch. This will be rendered obsolete for the next batch since we update our handouts regularly.
ANTIBIOTIC THERAPY • Anti-inflammatory agents: 6 – 8 weeks
• Once the diagnosis of acute RF has been made and regardless of o ASA - 100 mg/kg/day 4-6 doses (arthritis and mild carditis)
the throat culture results, the patient should receive 10 days of for 3-5 days then 75 mg/kg/day q 6hrs for 4 weeks
oral Penicillin or Erythromycin or a single IM injection of o Prednisone - 2 mg/kg/day (carditis and cardiomegaly or CHF)
benzathine Penicillin to eradicate GAS from the upper q 6 hrs for 2-3 wks & taper
respiratory tract.
• After this initial course of antibiotics, patient should be started PROPHYLAXIS FOR RF
on long-term antibiotic prophylaxis. • Primary prophylaxis prevents 1st episode of RF:
o Treat streptococcal throat infection
MANAGEMENT OF RF • Secondary prophylaxis prevents recurrences of RF:
• Antibiotic: to eradicate Streptococcus o Pen VK 250 mg BID per orem, or:
o Penicillin VK - 200 – 500 mg QID x 10 days, or; o Benzathine PCN 0.6-1.2 MU IM q21 days
o Benzathine PCN - 0.6 – 1.2 MU IM, or;
o Erythromycin - 250 mg TID x 10 days
VALVULAR HEART
SYMPTOMS PE FINDINGS
DISEASE
• loud S1 at the apex and a narrowly split S2 with a loud P2
MITRAL STENOSIS • asymptomatic if mild • narrow pulse pressure
• dyspnea with or without • opening snap with a diastolic rumble
MITRAL exertion • systolic regurgitant murmur with transmission at the L axilla
REGURGITATION • orthopnea, nocturnal dyspnea, • S2 may widely split
AORTIC palpitations • diastolic murmur at the upper and mid-sternal left border with
REGURGITATION radiation to the apex; bounding pulses

✔ GUIDE QUESTION • Clinical Criteria


After the acute episode of rheumatic fever, patient still with heart o 2 major criteria, or
murmur but no other symptoms. The physician tells her that she needs o 1 major criterion and 3 minor criteria, or
secondary antibiotic prophylaxis to prevent recurrence. She asks how o 5 minor criteria
long she needs it.
A. 5 years after the last attack
B. Until she turns 30 y/o POSSIBLE INFECTIVE ENDOCARDITIS
C. 10 years or until 21 years of age • 1 major criterion and 1 minor criterion, or
D. 10 years or until 40y/o • 3 minor criteria

MAJOR
1. Blood culture – viridans Strep or Strep bovis, HACEK
(Haemophilus, Actinobacillus, Cardiobacterium, Eikenella,
Kingella), Staphylococcus, Enterococcus
o 2 separate sites 12 hours apart
o 3 or more 1 hour apart
2. Echocardiographic findings – oscillating mass vegetations,
regurgitant flow near a prosthesis, abscess, partial dehiscence of
prosthetic valves, new wave regurgitant flow

MINOR
• Fever
• Predisposing condition
• Vascular – emboli, pulmonary infarct, aneurysm, Janeway
lesions
• Immunologic – GN, Osler nodes, Roth Spots, RF
• Microbiological evidence
Nelson Textbook of Pediatrics, 20th ed
• Echocardiographic findings
INFECTIVE ENDOCARDITIS
• Causative agents – polymicrobial Pertinent PE findings in Infective Endocarditis (IE)
o Most common: Staphylococcus aureus, viridans Streptococcus, • Osler nodes – tender, pea-sized intradermal nodules in the pads
Enterococcus of fingers & toes
o HACEK – Haemophilus, Actinobacillus, Cardiobacterium, • Janeway lesions – painless small erythematous hemorrhagic
Eikenella, Kingella lesions on the palms and soles
• Vegetations form at the site of endocardial or intimal erosion • Splinter hemorrhages – linear lesions beneath the nails
that result from the turbulent flow
• Develops in previously abnormal or damaged valves
• May occur in children even without abnormal valves or cardiac
malformations
• Patients with CHD where there is turbulent blood flow due to a
hole or a stenotic orifice are most susceptible to develop IE (VSD,
AS, MVP)
DUKE CRITERIA
MODIFIED DUKE CRITERIA 2000:
DEFINITE INFECTIVE ENDOCARDITIS
• Pathologic Criteria
o Microorganisms demonstrated by results of cultures or
histologic examination of a vegetation, a vegetation that has
embolized, or an intracardiac abscess specimen; or
o Pathologic lesions; vegetation, or intracardiac abscess
confirmed by results of histologic examination showing active
endocarditis
TOPNOTCH MEDICAL BOARD PREP PEDIATRICS MAIN DIGITAL HANDOUT BY DR. PUNONGBAYAN AND DR. DE VERA Page 82 of 105
For inquiries visit www.topnotchboardprep.com.ph or https://www.facebook.com/topnotchmedicalboardprep/
This handout is only valid for the March 2021 PLE batch. This will be rendered obsolete for the next batch since we update our handouts regularly.
TOPNOTCH MEDICAL BOARD PREP PEDIATRICS MAIN DIGITAL HANDOUT BY DR. PUNONGBAYAN AND DR. DE VERA
For inquiries visit www.topnotchboardprep.com.ph or https://www.facebook.com/topnotchmedicalboardprep/
This handout is only valid for the March 2021 PLE batch. This will be rendered obsolete for the next batch since we update our handouts regularly.
SUPPLEMENT: INFECTIVE ENDOCARDITIS
2015 National Institute for Health and Care Excellence RISK FACTOR MOST LIKELY ORGANISM
(NICE) Guidelines: Normal person
Staphylococcus
1. Antibiotic prophylaxis against IE is not recommended No underlying disease
routinely for people undergoing dental procedures Underlying heart disease
viridans Streptococci
2. Those at risk of developing IE: acquired valvular heart Dental procedure
disease; valve replacement; structural congenital heart GUT or lower bowel
Group D Streptococcus
diseases except fully repaired ASD, PDA, VSD; hypertrophic manipulation
cardiomyopathy, previous IE After open heart procedure Fungal
SUPPLEMENT: Staphylococcus &
IV drug abusers
2015 European Society of Cardiology (ESC) guidelines on Pseudomonas
high-risk procedures for which antibiotic prophylaxis is + CVP
Coagulase-negative Staph
considered: + Prosthetic valves
--- Consider for dental procedures requiring manipulation of
the gingiva or periapical region of the teeth or perforation of the CASE KEY CLUE ORGANISM
oral mucosa Chills, fever,
Developing a new
1. Amoxicillin 2 grams orally 30-60 minutes prior to the arthralgia, myalgia, Staph aureus
heart murmur
procedure acutely ill looking
2. If allergic to penicillin: Clindamycin 600 mg orally 30-60 Px has poor oral viridans
A child with known
minutes before the procedure hygiene Streptococci
CHD presents w/
3. Empirical therapy: Vancomycin plus Gentamicin in Px just underwent viridans
fever, fatigue,
patients without a prosthetic valve and when there is a high root canal Streptococci
weight loss, painful
risk of S. aureus, enterococcus, and viridans Strep. Recent repair of VSD Fungal
skin lesions on the
4. Native valve endocarditis due to viridans Strep and Strep. Child is an IV drug
fingers Pseudomonas
bovis: aqueous Pen G Na (200,000 U/kg/day IV Q 4 or 6 hrs) user
OR Ceftriaxone (100 mg/kg/day IV od) OR Ceftriaxone plus
Gentamicin (3 mg/kg/day IV od or Q 8 hrs) TREATMENT
5. For Oxacillin-resistant strains due to Staph: Vancomycin • Several weeks are required for a vegetation to organize
(40 mg/kg/day IV Q 8 or 12 hrs) completely therapy must be continued through this period to
avoid recrudescence
MURMURS • Total of 4-6 weeks is recommended
INNOCENT PATHOLOGIC • Surgery for severe aortic or mitral valve involvement with
intractable heart failure, myocardial abscess, recurrent emboli,
• Soft • Diastolic
new heart block, increasing size of vegetations while receiving
• Systolic • Pancystolic
therapy
• Short • Late systolic
• Sounds "musical" • Continuous TREATMENT OF IE
"vibratory" "twangy • Thrill present on ANTIBIOTIC
• Symptomless examination CLINICAL CONDITION
(DOSE AND FREQUENCY)
• Special tests normal (x-ray, • Additional cardiac Patients without a
ECG) abnormalities (e.g. clicks, Vancomycin 40 mg/kg/day IV
prosthetic valve but with
• Standing / Sitting (vary abnormal splitting, in 2-3 equally divided doses
high risk for Staphylococcus
with position) asymmetric pulses plus Gentamicin 3 mg/kg/day
aureus, viridans
Second grade only (up to IV OD or every 8 hours
Streptococcus, enterococcus
Grade II only)
• Sternal border (left or • aqueous Pen G 200,000
midsternal) U/kg/day IV every 4-6 hours
Native valve endocarditis for 4 weeks, or;
GRADE QUALITY
due to viridans • Ceftriaxone 100 mg/kg/day
Streptococcus and IV one dose for 4 weeks, or;
1 Soft, difficult to hear
Streptococcus bovis • Ceftriaxone plus Gentamicin
2 Easily heard
3 mg/kg/day IV in 1 dose or 3
3 Louder but no thrill
equally divided doses
4 Associated with thrill
5 Thrill and audible with edge of stethoscope Oxacillin 200 mg/kg/day IV
6 Thrill and audible with stethoscope just off the chest Endocarditis due to Staph every 4-6 hours for 6 weeks
without prosthetic plus optional Gentamicin 3
• AFTER 3, you have THRILL!
materials mg/kg/day IV every 8 hours
for 3-5 days
CHILD PRESENTING WITH CHF Cefazolin 100 mg/kg/day IV
BASIC CASE KEY CLUES DIAGNOSIS For penicillin-allergic
every 8 hours for 6 weeks plus
patients with endocarditis
Neonate Neonate is 4 days optional Gentamicin 3
Hypoplastic left due to Staph without
presenting with old and is not mg/kg/day IV every 8 hours
heart disease prosthetic materials
systemic cyanotic for 3-5 days
hypoperfusion and Vancomycin 40 mg/kg/day IV
shock with low For Oxacillin-resistant
in 2-3 divided doses for 6
cardiac output and strains due to Staph
weeks
weak peripheral Neonate is 4 days
TAPVR
pulses. Severe old and is cyanotic SYSTOLIC EJECTION MURMURS
respiratory Atrial Septal Defect 2nd LICS with a widely split S2
distress and 2nd LICS with radiation to the upper
grunting Pulmonic Stenosis
back
A 6 week old baby presenting with 2nd RICS with radiation to the upper
Coarctation of the Aortic Stenosis
increasing respiratory distress, back
Aorta
diaphoresis, lethargy Coarctation of the 3rd-4th LICS with radiation to the
aorta interscapular area

TOPNOTCH MEDICAL BOARD PREP PEDIATRICS MAIN DIGITAL HANDOUT BY DR. PUNONGBAYAN AND DR. DE VERA Page 83 of 105
For inquiries visit www.topnotchboardprep.com.ph or https://www.facebook.com/topnotchmedicalboardprep/
This handout is only valid for the March 2021 PLE batch. This will be rendered obsolete for the next batch since we update our handouts regularly.
TOPNOTCH MEDICAL BOARD PREP PEDIATRICS MAIN DIGITAL HANDOUT BY DR. PUNONGBAYAN AND DR. DE VERA
For inquiries visit www.topnotchboardprep.com.ph or https://www.facebook.com/topnotchmedicalboardprep/
This handout is only valid for the March 2021 PLE batch. This will be rendered obsolete for the next batch since we update our handouts regularly.
SYSTOLIC REGURGITANT MURMURS INFECTIOUS DISEASES
Ventricular Septal
Defect
Left lower sternal border APPROACH TO RED EYE IN NEONATES
Left lower sternal border with CAUSATIVE DRUG OF
Mitral PND, KEY CLUES
radiation to the left anterior axillary AGENT CHOICE
Regurgitation red eye with purulent
line
discharge, tense edema
Here is my tip on how to approach a case that seem to describe a valvular Ceftriaxone 50
heart disease: Determine if a certain valvular heart disease (MS, MR, and
of eyelids with marked Neisseria
mg/kg/day for
AR), when suspected, is in failure or not in failure. Just think of the chemosis; incubation gonorrhoeae
1 dose
location of the mitral and aortic valves (which chambers they are period 2-5 days; onset at
situated). These conditions primarily would have DYSPNEA as the birth or until >5 DOL
patient’s chief complaint when they are in failure. Of course, a 2D echo mild to severe swelling of Chlamydia Erythromycin
would be the most appropriate diagnostic tool to identify the specific eyelids with copious trachomatis 50 mg/kg/day
problem. But there are certain PE findings (esecially the character and purulent discharge; (inclusion every 6 hours
location of the murmurs) that would help you come up with the most
incubation period 5-14 d blenorrhea) for 14 days
likely diagnosis.
Days 5-18: edema and
Dr. Punongbayan
Systemic
erythema of eyelids,
Pseudomonas antibiotics;
purulent discharge,
Mitral valve involvement in ¾ of cases of valvular heart disease; ¼ of aeruginosa Gentamicin eye
cases involve the aortic valve; tricuspid valve involvement is rare;
pannus formation,
ointment
pulmonary valve involvement almost never occurs. endophthalmitis, sepsis
Pediatric Cardiology for Practitioners by Myung Park
For Chlamydia – Giemsa stain epithelial cells scraped from the tarsal
conjunctiva for intracytoplasmic inclusions
Dr. Punongbayan
MITRAL VALVE STENOSIS
• thickening of leaflets and fusion of commissures leading to EYE INFECTIONS
calcification and immobility of the valves • Red itchy eyes
• LA and right-sided heart chambers are hypertrophied • Thin exudate
• valve is described as “fish mouth buttonhole deformity” Adenovirus
• Pain & photophobia
Imagine this is the heart with its 4 chambers: (MV between the LA and • ± Cough & colds
CONJUNCTIVITIS

LV; imagine it is stenotic or narrowed --- here are the consequences):


Right atrium Left atrium • Red eyes
Staphylococcus
• RA enlargement • LA enlargement • Presence of pus
• Right-sided failure • Increased LA pressure • Red eyes
• Hepatic congestion • Pulmonary congestion • Pus
• Jugular vein distention • Pulmonary hypertension Chlamydia
• Inclusion bodies in
• Atrial fibrillation
scrapings
Right ventricle Left ventricle
• RVH • LV filling • Inturned eye lashes
• RV pressure overload • Corneal scarring Chlamydia
• RV failure • Blindness
• dyspnea with or without exertion, orthopnea, PND, weak peripheral
pulses with narrowed pulse pressure
• Stye Staphylococcus
• loud S1 at the apex and a narrowly split S2 with a loud P2 indicating • Bilateral eye lid
EYELID INFECTIONS

pulmonary hypertension; mitral diastolic rumble at the apex swelling,


Trichinella
Dr. Punongbayan eosinophils, muscle
pain, Hx of infection
MITRAL REGURGITATION • Unilateral
• shortened leaflets due to fibrosis; backflow of blood from LV to inflammation at bite
LA; dilated LA and LV site around eye or
Trypanosoma cruzi
• hallmark finding is a systolic regurgitant murmurs at the apex mouth, Hx of travel
with radiation to the left anterior axillary line to Mexico or South
How can you differentiate the murmur from that of VSD since they both America
have a systolic regurgitant type of murmur? VSD – no transmission to the Adenovirus – most common cause of viral conjunctivitis and
LAAL; MR – with transmission to the LAAL pharyngoconjunctival fever (red eye, red throat, fever)
Dr. Punongbayan
Trichinella – nematode; pork worm
Trypanosoma cruzi (Chagas disease or American trypanosomiasis) –
AORTIC REGURGITATION vector is kissing bug in Latin America
Trypanosoma brucei (African trypanosomiasis) – vector is tsetse fly;
• semilunar cusps are deformed and shortened; aortic valve is sleeping sickness)
dilated so that the cusps fail to appose tightly; backflow of blood Dr. Punongbayan
from aortic valve to LV
SUPPLEMENT: QUICK SHEET
• hallmark PE finding is a high-pitched diastolic murmur loudest
at 3rd-4th LICS more audible when sitting and leaning forward • Cestodes (flatworms) – all are treated with Praziquantel
• other findings: diastolic thrill at 3rd LICS; hyperdynamic EXCEPT Echinococcus granulosus (Albendazole)
precordium, bounding water hammer pulse or Corrigan pulse, • All TREMATODES (flukes) are treated with Praziquantel
wide pulse pressure • NEMATODES (roundworm, hookworm, whipworms)
• Albendazole for : Ascaris, Ancylostoma, Necator, Capillaria
• Mebendazole: Trichuris
• Pyrantel pamoate: Enterobius
• Ivermectin: Strongyloides
• DEC: Wuchereria and Brugia
• Thiabendazole: Trichinella

EYE INFECTIONS: ON FUNDOSCOPY


CHORIORETINITIS NEONATE
Retinopathy w/
keratitis in a Mother has hx of
Cytomegalovirus
newborn or a young drug abuse
child

TOPNOTCH MEDICAL BOARD PREP PEDIATRICS MAIN DIGITAL HANDOUT BY DR. PUNONGBAYAN AND DR. DE VERA Page 84 of 105
For inquiries visit www.topnotchboardprep.com.ph or https://www.facebook.com/topnotchmedicalboardprep/
This handout is only valid for the March 2021 PLE batch. This will be rendered obsolete for the next batch since we update our handouts regularly.
TOPNOTCH MEDICAL BOARD PREP PEDIATRICS MAIN DIGITAL HANDOUT BY DR. PUNONGBAYAN AND DR. DE VERA
For inquiries visit www.topnotchboardprep.com.ph or https://www.facebook.com/topnotchmedicalboardprep/
This handout is only valid for the March 2021 PLE batch. This will be rendered obsolete for the next batch since we update our handouts regularly.
✔ GUIDE QUESTION A. Varicella
Which drug will you give topically to eradicate the nasopharyngeal B. Steven Johnson Syndrome
carriage of Staphylococcus aureus in an adolescent with recurrent C. Ritter Disease
carbunculosis? D. Kawasaki
A. Mupirocin C. Bacitracin E. Scarlet Fever
B. Neomycin D. Polymyxin Key phrase for varicella: simultaneous appearance of all stages
SKIN INFECTIONS of the lesions
Dr. Punongbayan
INFECTION KEY CLUE CAUSATIVE AGENT
Neck, face, axillae,
Staphylococcus
buttocks
From neck down
Furuncle,
Has bath tub at
Carbuncle Pseudomonas
home
(hot tub folliculitis)
Loves to play in In relation to the case above, if the patient has a 4 year old young
the bath tub brother what can be given to prevent or modify the disease in the
Inflammation of younger sibling
sebaceous glands Teenager Propionibacterium A. Immunoglobulin within 96 hours
B. Live vaccine + immunoglobulin within 96 hours
& follicles
C. Live vaccine within 3-5 days
PATIENT PRESENTING WITH CUTANEOUS LESIONS D. None of the above
Mother brings her Issue of POST-EXPOSURE PROPHYLAXIS for Varicella in a
child with multiple younger child: the live varicella vaccine itself can be given
infected wounds within 3-5 days of exposure to modify the course.
on the lower On PE, you see Passive immunization with Varicella Zoster IG (VZIG) is for
Ecthyma
extremities. multiply dry, (1) immunocompromised,
(ulcerative form
Mother states that heaped up tightly (2) pregnant, and
of impetigo) (3) newborns whose mother had chickenpox 5 days before up to
the lesions started adherent crusts
as mosquito bites 2 days after delivery
VZIG 125 U/10 kg IM (max. 625 units) to be given within 96
and child kept on
hours after exposure
scratching them. Dr. Punongbayan
Initially vesicular
Honey crusted VARICELLA
lesion
Mother brings a Catalase negative GABHS ISSUES IN THE MANAGEMENT OF VARICELLA:
child with Issue #1: Post-exposure Prophylaxis
No pain, no
cutaneous lesion. systemic • active vaccine can be given within 5 days of exposure to
Started from a bite symptoms modify course
of mosquito • Anti-VZV Ig for immunocompromised, pregnant & newborns
Initially vesicular
Child kept on exposed to maternal varicella: 125 units/10 kg IM given within
Longer lasting and 96 hrs after exposure (max. 625 units)
scratching
bigger bullae Staphylococcus
***newborns whose mother had varicella 5 days before to 2 days
formation
after delivery
Catalase positive
Issue #2: Is Acyclovir routinely given?
✔ GUIDE QUESTIONS • It is not recommended routinely for treatment of
A 1 year-old male patient presents to the clinic with 2-day history of uncomplicated cases in the otherwise healthy child because
fever and rash. Patient was noted to be irritable, weak, with poor
of:
appetite, and fever of 38°C. One day PTC, an erythematous rash
appeared which started in the face and around the oral mucosa, axilla, 1. the marginal benefit
and groin areas. It proceeded to blistering and peeling off of the skin 2. the cost of the drug
easily with accompanying pain. Examination revealed conjunctival 3. the low risk for complications of varicella
erythema with fissuring and crusting around the nose and mouth with • Oral Acyclovir 20 mg/kg/dose - max. 800 mg/dose – given as 4
some superficial erosions on the lips, slightly erythematous posterior doses/day for 5 days
pharynx, diffuse erythema of the skin with denuded areas over the
axillae, antecubital area, and groin. HR 110, RR 30, T 39°C. What is the Issue #3: Can we give Acyclovir at any point in the course of
most likely diagnosis?
A. Rubeola
the illness?
B. Steven Johnson Syndrome • It is MOST effective if given within 24 hours of the onset of
C. Ritter Disease the rash.
D. Kawasaki
E. Scarlet Fever When is intravenous Acyclovir indicated?
Ritter disease is an old name for staphylococcal scalded skin • For severe disease and for immunocompromised patients
syndrome; exfoliatins A and B are distinct proteins that produce • For patients with disseminated VZV: Acyclovir 500 mg/m2 every
localized or generalized skin manifestations; produce skin 8 hrs IV initiated within 72 hrs of development of symptoms →
separation by splitting the desmosome changing the IC matrix continued for 7 days or until no new lesions have appeared or 48
in the stratum granulosum hrs
Nikolsky sign – gentle stroking of the skin causes it to separate
at the epidermal level APPROACH TO PATIENTS PRESENTING
SSSS caused by group 2 phage S. aureus leading to the release WITH VESICULAR LESIONS
of exotoxins that causes separation of the epidermis beneath the Herpes zoster
granular cell layer. Preceded with neurological pain
(Shingles)
Drug of choice: Oxacillin or if MRSA is suspected, start Very large bullous lesions
Vancomycin Progressing rapidly
Dr. Punongbayan Staphylococcal Scalded
An 8 year-old male patient comes to the clinic for fever and rash. Two Preceded by fever, fatigue, malaise
Skin Syndrome
days prior, the patient had fever of 38°C, malaise, anorexia, headache, Toxic looking patient
and mild abdominal pain. After a day, pruritic erythematous macules Denuded areas
started appearing over the face and trunk. These rashes eventually Multiple vesicular lesions, some
became vesicles some of which started to erupt, then new lesions papules, some crusted all at Varicella
started to appear over the extremities as well. Persistence of symptoms different ages
prompted consult. PE reveals some vesicles on a red base over the
extremities, some crusted lesions over the face and trunk. What is the
most likely diagnosis?
TOPNOTCH MEDICAL BOARD PREP PEDIATRICS MAIN DIGITAL HANDOUT BY DR. PUNONGBAYAN AND DR. DE VERA Page 85 of 105
For inquiries visit www.topnotchboardprep.com.ph or https://www.facebook.com/topnotchmedicalboardprep/
This handout is only valid for the March 2021 PLE batch. This will be rendered obsolete for the next batch since we update our handouts regularly.
TOPNOTCH MEDICAL BOARD PREP PEDIATRICS MAIN DIGITAL HANDOUT BY DR. PUNONGBAYAN AND DR. DE VERA
For inquiries visit www.topnotchboardprep.com.ph or https://www.facebook.com/topnotchmedicalboardprep/
This handout is only valid for the March 2021 PLE batch. This will be rendered obsolete for the next batch since we update our handouts regularly.
PATIENTS PRESENTING WITH CELLULITS OR • Actinomyces is a facultatively anaerobic gram-positive bacteria. It is
SUBCUTANEOUS LESIONS commonly found in the nose and throat. Conventional therapy for
Red, raised, actinomycosis is high-dose intravenous Penicillin at a dosage of 18-24
million U daily for 2-6 weeks, followed by oral Penicillin or Amoxicillin
butterfly rash in Erysipelas
for 6-12 months.
appearance with (GAS – Strep • Nocardiosis is an acute, subacute, or chronic infectious disease that
derma pain & rapid pyogenes) occurs in cutaneous, pulmonary, or disseminated forms. Nocardia are
Inflamed spread obligate aerobic, partially acid fast, beaded, branching, gram-positive
erythematous Following contact bacilli. Trimethoprim-Sulfamethoxazole is the first-line treatment but
skin, tender, and with saltwater or Vibrio vulnificus in patients with a sulfa allergy, Imipenem, Ceftriaxone, or Linezolid are
warm oysters options for first-line therapy.
Dr. Punongbayan
Burn px
Blue green pus Pseudomonas
OTHER CUTANEOUS LESIONS
Grape-like odor
Pustule to dark red fluid
Erysipelas appears to be like a red plaque with a well-defined margin Malignant Bacillus anthracis
filled to necrosis to
and has involved the upper dermis extending into superficial lymphatics; pustule (Anthrax)
black eschar
also called St. Anthony’s fire; most common cause is Streptococcus
Target Fever, headache Borrelia
Vibrio vulnificus is a gram- negative bacterium that causes serious lesion Rash border burgdorferi
wound infections, septicemia, and diarrhea and the leading cause of
Bull’s eye Bite site (Lyme disease)
shellfish-associated deaths in the US. From cellulitis to myositis and
necrotizing fasciitis, the infection may spread rapidly especially in high- Bacillus anthracis
risk individuals (those with liver disease, hemochromatosis, and chronic • gram positive rod-shaped bacteria found naturally in soil
illness). 1. Cutaneous – most common form and occurs in exposed areas
2. Inhalation – occurs when spores are aerosolized or as
Presumptive diagnosis: fever, hypotension, bullous skin lesions, rapid
weaponized spore preparations; cause hemorrhagic of thoracic
deterioration to shock, exposure to saltwater / shellfish / oysters
lymph nodes leading to hemorrhagic mediastinitis; rapidly
Blood culture and stool culture (TCBS media)
fulminant bacteremic phase
Treatment: Doxycycline (100 mg orally 2x daily) PLUS either Cefotaxime
3. Gastrointestinal syndrome – occur following the consumption
or Ceftriaxone
Dr. Punongbayan
of undercooked infected meat from animals infected with
anthrax; necrotic ulcers and GI hemorrhage
DOC: bactericidal agent plus protein synthesis inhibitor:
PATIENTS PRESENTING WITH LYMPHATIC OBSTRUCTION Ciprofloxacin 30 mg/kg/day every 8 hrs (not to exceed 400 mg per dose
OR LYMPHOCUTANEOUS LESIONS PLUS Clindamycin 40 mg/kg every/day every 8 hrs (not to exceed 900
Solitary or lymphocutaneous mg/dose)
lesions Sporothrix schenckii Borrelia burgdorferi
Rose gardeners (Sporotrichosis) • Lyme disease due to a tick bite; Northeast and upper Midwest;
Likes lying in the garden erythema migrans, arthritis, and facial palsy
With hx of painful genital ulcer Chlamydia trachomatis o Erythema migrans – lesion that appears at the site of tick bite 7-14
and inguinal and femoral (Lymphogranuloma days after; occurs most often on the head, neck, arms, legs, back
lymphadenopathy venereum) • Clinical stages: early localized, early disseminated, late Lyme disease
• DOC: one of these 3 drugs which have equivalent efficacy: Doxycycline,
Fever, headache, elephantiasis of Wuchereria bancrofti Amoxicillin, or Cefuroxime (Doxycycline has the best penetration into
limbs or genitalia (Lymphatic filariasis) the CNS and can treat potential coinfecting agents; approved in the US
to be given to <8 year-old children if given for <21 days)
Painless, discrete solitary genital Treponema pallidum
ulcer (Syphilis) • Doxycycline 2.2 mg/kg twice daily (max. 100 mg/dose) for 10-14 days
Dr. Punongbayan

• Management for Sporotrichosis: Itraconazole; Amphotericin B for the


PATIENTS PRESENTING WITH INFECTED WOUND
severely ill
• Empiric treatment for LGV – Doxycycline 100 mg orally 2x a day for 21 Hx of animal bite
Pasteurella
days Cellulitis with lymphadenitis
• Drug of choice for Syphilis: single dose of benzathine Penicillin G 2.4 Hx of cat bite or cat scratches
million units IM Or no hx of bite but with pet kitty Bartonella
• Drug of choice for W. bancrofti: single dose Diethylcarbamazine Lymphadenopathy with stellate henslae
(contraindicated in patients coinfected with onchocerciasis) regardless granulomas
whether symptoms or microfilaremia are present; with the addition of Hx of shallow puncture wound through
Doxycycline 200 mg/day for 4-6 weeks for its macrofilaricidal activity Pseudomonas
Dr. Punongbayan
tsinelas or rubber shoes
Surgical wound clean S. aureus
PATIENTS PRESENTING WITH GRANULOMATOUS LESIONS Surgical wounds dirty S. aureus
& DRAINING SINUS TRACT Trauma Clostridium
Jaw area swelling
Sinus tract formation Pasteurella multocida
Yellow exudate • Cats are the source of infection in 60 to 80 percent of human P.
Carious teeth Actinomyces multocida infections.
• Gram-negative nonmotile coccobacillus
Dental procedure
• Commensals in the upper respiratory tracts of >90 percent of felines
Yellow granules in and are the major pathogen causing infection as a result of a cat bite
Px w/ exudates or scratch
granulomatous Mycobacterium
Tropical fish enthusiasts • Spreading edema, erythema, and tenderness at the site of bite or
lesions, marinum scratch
draining sinus Subcutaneous swelling of • First line drug: Amoxicillin/Clavulanic Acid
tracts shoulder
Nocardia Bartonella henselae
Sinus Tract formations • Etiologic agent of Cat Scratch Disease (CSD) can cause disease in both
Granules normal and immunocompromised hosts
Usually in the cervical • 80 percent of cases occur in children with a peak incidence between
Mycobacterium ages 2 and 14 years
area
tuberculosis • CSD – low-grade fever, papule or pustule at the site of scratch, tender
Child has chronic cough enlarged lymph node that develop 1-3 weeks after exposure; 3-10 days
• M. marinum infection commonly develops as a complication of skin incubation period
and soft tissue injuries exposed to aquatic equipment such as fish lines • Gram-negative coccobacillus, aerobic, non-spore forming, facultatively
and fishhooks. After an incubation period of 2 to 3 weeks, solitary intracellular
erythematous to violaceous, hyperkeratotic papules form on the • Most cases of CSD are self-limited; may give Azithromycin
Dr. Punongbayan
elbows, hands, feet, or knees. The gold standard of treatment is
Clarithromycin with Ethambutol. Rifampicin is typically added if
deeper organ infection is present such as osteomyelitis.

TOPNOTCH MEDICAL BOARD PREP PEDIATRICS MAIN DIGITAL HANDOUT BY DR. PUNONGBAYAN AND DR. DE VERA Page 86 of 105
For inquiries visit www.topnotchboardprep.com.ph or https://www.facebook.com/topnotchmedicalboardprep/
This handout is only valid for the March 2021 PLE batch. This will be rendered obsolete for the next batch since we update our handouts regularly.
TOPNOTCH MEDICAL BOARD PREP PEDIATRICS MAIN DIGITAL HANDOUT BY DR. PUNONGBAYAN AND DR. DE VERA
For inquiries visit www.topnotchboardprep.com.ph or https://www.facebook.com/topnotchmedicalboardprep/
This handout is only valid for the March 2021 PLE batch. This will be rendered obsolete for the next batch since we update our handouts regularly.
PATIENTS WITH ENT INFECTION How do you determine treatment?
Acute Otitis Red, bulging tympanic Streptococcus. • Elimination of the organism documented by at least 2
Media membrane, fever pneumoniae, successive cultures from the nose & throat taken 24 hrs apart
Moraxella after completion of therapy
Low grade fever,
catarrhalis, • Asymptomatic carrier: antimicrobial prophylaxis given for 7-
Sinusitis headache, prolonged
Haemophilus 10 days and diphtheria toxoid
colds
influenzae type b
Sneezing ✔ GUIDE QUESTION
Common cold Scratching throat Rhinovirus A 5 year-old male child presents with a 2-day history of sore throat
Runny nose progressively getting worse, and fever of 38-39°C. PE revealed HR 108,
RR 28, T39°C. There is erythematous posterior pharyngeal wall,
S. pneumoniae is the most common organism causing Acute Otitis Media bilateral tonsils are enlarged and with yellow exudates, and with (+)
followed by Hib and Moraxella catarrhalis) tender enlarged anterior cervical lymph nodes. What is the most likely
Dr. Punongbayan
diagnosis?
✔ GUIDE QUESTIONS A. retropharyngeal abscess
Pharyngoconjuctival fever in children is most commonly caused by: B. peritonsillar abscess
A. Rhinovirus C. Respiratory syncytial virus C. diphtheria
B. Coronavirus D. Adenovirus D. Epiglottitis
A 4 year-old female child presents with a 3-day history of E. Streptococcal tonsillopharyngitis
undocumented fever with sore throat. She is brought to the clinic today
due to fever of 38°C, hoarseness, worsening of sore throat, cough, and PATIENT PRESENTING WITH SORE THROAT
difficulty swallowing. The pharynx is erythematous with an adherent Inflamed tonsils or pharynx
gray membrane. She has also a bullneck appearance. What is the most Streptococcal
Purulent
likely diagnosis? Fever tonsillopharyngitis
A. Diphtheria White papules with red base on posterior
B. Streptococcal pharyngitis palate & pharynx
Herpangina
C. viral pharyngitis Purulent tonsils
D. adenovirus related pharyngitis Infectious
Fatigue
In relation to the case above, the patient has a brother who recently had Lymphadenopathy mononucleosis
his 3rd dose of DTaP 2 weeks ago. What is your plan of management on Low grade fever with gradual onset of
the patient’s brother? membranous nasopharyngitis Diphtheria
A. Give a 4th dose as soon as possible Bull neck from lymphadenopathy
B. No further interventions necessary
Finely popular, erythematous eruption
C. Give IVIg
producing a bright red discoloration of the
D. Repeat the primary series and isolate the patient from the
skin, which blanches on pressure, more
brother
intense along the creases of the elbows,
In this case, the patient is a 4 year-old girl whose brother had axillae, and groin. The skin has a goose-
Scarlet Fever
just received the 3rd dose of DTaP vaccine 2 weeks prior. The pimple appearance & feels rough. After 4
brother then is adequately protected and would need no further days, the rash begins to fade & is followed
intervention at the moment. by desquamation
Dr. Punongbayan

Diphtheria antitoxin, produced in horses, was used for treatment of PATIENTS PRESENTING WITH INFECTIOUS ARTHRITIS
diphtheria in the US since the 1890s. It is not indicated for prophylaxis of
contacts of diphtheria patients. • Pain • All ages except between
S. aureus
• Redness 15-40
The antitoxin will neutralize circulating (unbound) toxin and prevent
progression of disease. The patient must be tested for sensitivity before • Low grade • 15-40 y/o
Neisseria
antitoxin is given. fever • Sexually active
gonorrhoeae
For close contacts, especially household contacts, a diphtheria booster, • Tenderness • Promiscuous
appropriate for age, should be given. (Take note: single antigen joints • Prosthetic joint S. epidermidis
diphtheria toxoid is not available → give DTaP or TDaP, whichever is • Reduced • Following GIT infection Reiter’s
appropriate) ability few weeks ago syndrome
Contacts should also receive antibiotics—benzathine Penicillin G
(600,000 units for persons younger than 6 years old and 1.2 million units Reiter’s syndrome (reactive arthritis):
for those 6 years old and older) or a 7- to 10-day course of oral Can’t see Conjunctivitis), Can’t pee (Urethritis),
erythromycin (40 mg/kg/day for children and 1 g/day for adults). Can’t climb a tree (Arthritis)
Dr. Punongbayan • Develops in response to an infection and characterized by a triad of
arthritis, conjunctivitis, and non-specific urethritis
• Considered an autoimmune disease marked by inflammatory synovitis
DIPHTHERIA and erosion at the insertion sites of ligaments and tendons
HOW IS DIPHTHERIA DIFFERENT FROM STREP THROAT? Dr. Punongbayan

• Relative lack of fever


• Non-exudative throat PATIENTS PRESENTING WITH OSTEOMYELITIS
Adults, children,
DIAGNOSIS/TREATMENT OF DIPHTHERIA infants w/o major
FEVER Staphylococcus
• Culture from nose & throat obtained from beneath the trauma or special
membrane including a portion of the membrane itself; negative Bone pain with conditions
results do not rule it out swelling, Neonates, <1 mo old Staphylococcus
• Specific antitoxin – mainstay of therapy & given based on redness
Salmonella
clinical dx; administered once at empirical dose based on the Associated
Sickle cell anemia cholerasuis or
degree of toxicity, site & size; single empirical dose of 20,000- cellulitis
enterica,
100,000 units
Hx of trauma Pseudomonas
TREATMENT OF DIPHTHERIA: Salmonella enterica
• Why give antimicrobials? • rod-shaped, flagellate, facultatively aerobic, gram-negative bacteria
To halt toxin production, treat localized infection & prevent • Salmonella osteomyelitis – rare sequelae of typhoid which may occur
transmission to contacts as a result of hematogenous seeding of the bone or joints due to a
bacteremic episode; symptoms may include pain in a specific bone with
• Procaine Penicillin G IM or IV (300,000 units/day for those overlying redness, fever, and weakness
weighing 10 kgs or less and 600,000 units/day for those • Capillary occlusion secondary to intravascular sickling may devitalize
weighing more than 10 kgs) for 14 days, OR; and infarct the gut, permitting Salmonella invasion.
• Erythromycin: eradicates nasopharyngeal carriage 40-50 Dr. Punongbayan

mg/kg/day orally or IV every 6 hours for 14 days

TOPNOTCH MEDICAL BOARD PREP PEDIATRICS MAIN DIGITAL HANDOUT BY DR. PUNONGBAYAN AND DR. DE VERA Page 87 of 105
For inquiries visit www.topnotchboardprep.com.ph or https://www.facebook.com/topnotchmedicalboardprep/
This handout is only valid for the March 2021 PLE batch. This will be rendered obsolete for the next batch since we update our handouts regularly.
TOPNOTCH MEDICAL BOARD PREP PEDIATRICS MAIN DIGITAL HANDOUT BY DR. PUNONGBAYAN AND DR. DE VERA
For inquiries visit www.topnotchboardprep.com.ph or https://www.facebook.com/topnotchmedicalboardprep/
This handout is only valid for the March 2021 PLE batch. This will be rendered obsolete for the next batch since we update our handouts regularly.
PATIENTS PRESENTING WITH TOXIC LOOKING RASHES MOST CONFUSION BETWEEN RUBELLA, RUBEOLA & ROSEOLA
Generalized • TIPS: when maculopapular rash with fever presents, take note of
Neisseria the following:
Abrupt onset with
meningitidis 1. appearance of rash in relation to onset of fever
fever, chills, and
(Meningococcemia) 2. whether rash desquamates or not
shock 3. accompanying symptoms of fever and rash
Petechiae to
Fever, headache • Rubella – post-auricular lymphadenopathy + rash
purpuric
Rash that includes Rickettsia rickettsii • Rubeola/MEASLES – 3 C’s, fever disappears once rash has moved
lesions
palms and soles down to feet
Fever, headache • Roseola – Fever disappears then RASH
Rickettsia Dr. Punongbayan
Rash that spares the
prowazekii ✔ GUIDE QUESTIONS
palms and soles
A 5 year-old male presents with fever and rashes. 6 days prior, he was
Erythematous Trunk & neck and noted to have a fever of 37.9°C, dry cough, runny nose and redness of
maculo- then extremities GABHS (Scarlet the eye with tearing. Then 3 days prior, the mother noted flat red spots
papular rash, associated with sore fever) starting on the hairline, behind the ears and upper neck spreading to
Sand paper like throat and fever the entire face, neck, upper arms, and chest. Today, the rash spread
Trunk & neck and over the back, abdomen, and thighs which prompted them to seek
then extremities consult. PE: HR 110, RR 32 T39.5°C, generalized maculopapular rash
Diffuse with areas of confluence. What is the most likely diagnosis?
with desquamation
erythematous S. aureus (Toxic A. Rubeola D. Kawasaki disease
on palm and soles
macular Shock Syndrome) B. Rubella E. Scarlet Fever
Acute onset of fever, C. Roseola
sunburn-like
pharyngitis In relation to the case above, which among the ff interventions is said
rash
Diarrhea, to decrease mortality in the above condition?
hypotension A. Immediate antibiotic therapy
B. Ribavirin therapy
Rickettsia prowazekii – typhus or epidemic fever;
C. Vitamin A supplementation
• human body louse vector;
D. IV Ig
• reservoir – humans, flying squirrel;
• Central Asia and Africa, North and South America;
• poor communities and refugee populations; MEASLES
• pink macules on trunk and axillae sparing the face, palms, and soles SUPPLEMENT: QUICK SHEET
becoming purpuric
MEASLES MNEMONIC
Rickettsia rickettsii – Rocky Mountain Spotted Fever;
• "MEASLES COMP" (complications):
• tick vector
o Myocarditis
• reservoir – rodents;
• North, Central, and South America; pink macules on wrists, forearms,
o Encephalitis
trunk, palms and soles becoming purpuric o Appendicitis
• DOC for rickettsia – Doxycycline o Subacute sclerosing panencephalitis
• Mnemonic for RMSF – R-R → Rocky mountain spotted fever, rickettsii o Laryngitis
TSST- toxic shock syndrome toxin o Early death
• Staph aureus, Strep pyogenes; o pooh poohs (diarrhea)
• major criteria: fever 38.8 or greater, hypotension. rash o Corneal ulcer
Dr. Punongbayan o Otis media
o Mesenteric lymphadenitis
✔ GUIDE QUESTIONS o Pneumonia and related (bronchiolitis-bronchitis-croup)
An 8 year-old male presents to the clinic with a history of low-grade
fever, headache, and cough and colds for the past 3 days. He then What do you give for postexposure prophylaxis?
develops a reddish facial flushing which then spread into the trunk and • Measles Ig for prevention & attenuation within 6 days of
the upper arms. On close examination, the rash is erythematous exposure (0.25 mL/kg max. of 15 mL IM)
macular with central clearing and does not affect the palms and soles.
What is the most likely diagnosis? • Measles active vaccine can be given for susceptible children > 1
A. Rubeola D. Kawasaki Disease yr old within 72 hours
B. Rubella E. Erythema Infectiosum o Infants <6 months old, pregnant & immunocompromised
C. Roseola persons should receive Ig but not the active vaccine.
A 1 year-old male came for consult for rashes. Four days prior, patient
started having runny nose with clear nasal discharge with mild ✔ GUIDE QUESTIONS
conjunctival redness. Three days ago, patient had a fever of 39C which A 5 year-old boy with measles lesions of five days duration was brought
resolved on the 3rd day but a few hours after there was appearance of a to the ER because of continuing high-grade fever and progressive
rash starting on the trunk which eventually spread to the face and difficulty of breathing. Despite optimal supportive treatment, however,
extremities. PE revealed small pinkish maculopapular lesions over the the patient succumbed. On autopsy, one should look for:
trunk, face and extremities, HR: 120, RR: 33, T37.2°C. What is the most A. Negri bodies C. Cowdry type A inclusions
likely diagnosis? B. Warthin-Finkeldey cells D. Councilman bodies
A. Rubeola D. Kawasaki A 13 year-old female presents to the clinic due to generalized rash of 2
B. Rubella E. Scarlet Fever days duration. It initially started in the face and spread rapidly up to
C. Roseola the legs within 1 day. There was accompanying fever of 38°C, (+) joint
In relation to the condition above, what is the causative organism? pains over the knees and ankles, persistence of symptoms prompted
A. Paramyxovirus D. HHV-6 consult. PE revealed: slightly hyperemic conjunctivae and posterior
B. Coxsackie Virus E. Parvovirus B19 pharynx, (+) generalized maculopapular rash over the whole body with
C. Streptococcus pyogenes some evidence of clearing in the face, (+) tender enlarged posterior
auricular and cervical lymph nodes. What is the most likely diagnosis?
CHILD PRESENTING WITH MACULOPAPULAR RASHES A. Rubeola D. Kawasaki
Cough, coryza and conjunctivitis, B. Rubella E. Scarlet Fever
Measles or Rubeola C. Roseola
and fever
In relation to the condition above, what is its associated enanthem?
Posterior cervical, cervical or German Measles or A. Koplik spots C. Forchheimer spots
auricular nodes Rubella B. Nagayama spots D. Strawberry tongue
Slapped cheek looking then spread Erythema infectiosum Strawberry tongue may be seen in Kawasaki disease or in scarlet fever.
to trunk with central clearing or 5th disease Dr. Punongbayan

Abrupt onset of fever with URTI for A 23 year-old pregnant woman decided to continue with her pregnancy
even when she tested positive for anti-rubella IgM on the 8th week of
3-4 days, fever disappears & rashes Roseola or 6th disease
her gestation. Which of the following will indicate congenital rubella if
appears on trunk and spread the newborn was infected during pregnancy?
Preceded by rapid onset of fever, A. Anti-Rubella IgG in the mother
myalgia, ocular pain, hypesthesia. Dengue Fever B. Anti-Rubella IgM in the mother
Rashes blanch on pressure C. Anti-Rubella IgG in the infant
D. Anti-Rubella IgM in the infant
TOPNOTCH MEDICAL BOARD PREP PEDIATRICS MAIN DIGITAL HANDOUT BY DR. PUNONGBAYAN AND DR. DE VERA Page 88 of 105
For inquiries visit www.topnotchboardprep.com.ph or https://www.facebook.com/topnotchmedicalboardprep/
This handout is only valid for the March 2021 PLE batch. This will be rendered obsolete for the next batch since we update our handouts regularly.
TOPNOTCH MEDICAL BOARD PREP PEDIATRICS MAIN DIGITAL HANDOUT BY DR. PUNONGBAYAN AND DR. DE VERA
For inquiries visit www.topnotchboardprep.com.ph or https://www.facebook.com/topnotchmedicalboardprep/
This handout is only valid for the March 2021 PLE batch. This will be rendered obsolete for the next batch since we update our handouts regularly.
What to expect in rubella & what can you give? Mother has passed
• Infection usually confers permanent immunity although re- antibodies to baby
infection may occur in 3-10% among those with a history of Newborn Positive during pregnancy; this
previous rubella passive immunity may
• Active vaccine can theoretically prevent illness if given within last for 6-12 months
72 hours of exposure No current or prior
• Use of immune globulin for post exposure prophylaxis is not infection; not immune;
Any age Negative Negative
routine but may be considered if termination of pregnancy is not no or low immune
an option (0.55 mL/kg IM) response

AGE IgM IgG interpretation SUPPLEMENT: APPROACH TO RASH


Positive • Identify the lesion… primary lesion? secondary lesion?
Adult /
Positive or Recent infection • Distribution
Child
negative • Timing of appearance in relation to fever
Adult / Prior infection or • Associated symptoms
Positive
Child vaccination; immune • Identify if there is absence or presence of desquamation
Recent postnatal or • Incubation period and period of communicability
Newborn Positive -
congenital infection

INCUBATI
CONDITION ETIOLOGY CHARACTERISTIC COMMUNICABILITY
ON
• Coryza, Cough, Conjunctivitis
• High grade fever
• Maculopapular ashes appear at
peak of fever
Measles RNA virus, 4 days before and 4 days after onset of
• Photophobia 8-12
(Rubeola) Paramyxoviridae rash
• Cephalocaudal progression
• Branny desquamation when rash
reaches soles
• Koplik Spot
• Low grade fever.
Rubella • No photophobia
Togaviridae,
(German measles • Cephalo-caudal rash. 14-21 7 days before 7 days after rash
RNA
or 3 day measles) • POSTERIOR AURICULAR LN
• Forchheimer Spots
• Fever 3-5 days
• High grade fever
Roseola
• Fussiness
(Exanthem
HHV 6 • Seizures 9-10 days unknown
Subitum, Sixth
Disease) • RASHES APPEAR WHEN FEVER
ABATES
• Nagayama spots
• Fever, malaise 1-2 days before
rash
• Maculopapular, vesicular,
1-2 days before rash, 7 days after rash
Varicella VZV pustular lesions present 14-16 days
and ALL LESIONS HAVE CRUSTED
simultaneously
• Appear first on the trunk (similar
to roseola)
Coxsackie Virus
Hand, foot and • Ulcers in tongue, buccal mucosa Respiratory tract shedding usually
A16 3-6 days
mouth disease • Tender ulcers on hands and feet limited to a week or less
Enterovirus 71
• Slapped cheek appearance
Erythema • Lacy pattern
Before onset of the rash until after the
Infectiosum Parvovirus B19 • Spread to trunk 4-14
onset of rash
5th disease • Spares palms and soles
• Complication: Aplastic Crisis
✔ GUIDE QUESTIONS A. Paramyxovirus D. HHV-6
A 5 year-old male patient presents to the clinic with mild fever of 38°C, B. Coxsackie A virus E. Parvovirus B19
sore throat and rash which started 2 days ago. On careful examination, C. Streptococcus pyogenes
there were noted scattered vesicles over the buccal mucosa, palate and
posterior pharynx. There was also an erythematous maculopapular SPOTS OF VIRAL EXANTHEMS
and vesicular type rash over the hands, fingers, feet, buttocks, and groin
• Koplik Spot
areas. HR 90, RR 28, T 38.3°C. What is the most likely causative
organism for this condition? o 2-3 days before rashes
A. Paramyxovirus D. HHV-6 o grains of salt on a wet background
B. Coxsackie A virus E. parvovirus B19 o Clustered white lesions of buccal mucosa, adjacent 2nd-3rd
C. Streptococcus pyogenes lower molar
• Forchheimer Spot
A 7 y/o male patient comes to the clinic with fever and rash. Two days o Petechiae on soft palate
prior, patient had runny nose, headache, and a fever of 37.9°C. Then he o Rubella
started having an erythematous rash initially characterized as facial o 20% of cases only
flushing, which rapidly spread to the trunk and upper extremities
o Not diagnostic of rubella
characterized as maculopapular with central clearing giving a reticular
lacy appearance. There was also noted arthritis over the right knee and
§ Can be seen in measles or other viral exanthem
ankle. PE: reveals characteristic rash as described, slightly • Nagayama Spot
erythematous posterior pharynx, HR 98, RR 22 Temp: 37.8°C What is o Roseola
the most likely causative agent in this case? o Reddish papules of soft palate
TOPNOTCH MEDICAL BOARD PREP PEDIATRICS MAIN DIGITAL HANDOUT BY DR. PUNONGBAYAN AND DR. DE VERA Page 89 of 105
For inquiries visit www.topnotchboardprep.com.ph or https://www.facebook.com/topnotchmedicalboardprep/
This handout is only valid for the March 2021 PLE batch. This will be rendered obsolete for the next batch since we update our handouts regularly.
TOPNOTCH MEDICAL BOARD PREP PEDIATRICS MAIN DIGITAL HANDOUT BY DR. PUNONGBAYAN AND DR. DE VERA
For inquiries visit www.topnotchboardprep.com.ph or https://www.facebook.com/topnotchmedicalboardprep/
This handout is only valid for the March 2021 PLE batch. This will be rendered obsolete for the next batch since we update our handouts regularly.
✔ GUIDE QUESTIONS LEPTOSPIROSIS
A 3-year-old male patient presents to the ER with fever 40°C and sore
throat. Upon inspection, there are multiple vesicles and ulcers over the
PATHOGENESIS
tonsillar pillars and posterior pharyngeal wall, no rashes on the body • Leptospires enter humans through moist & abraded skin or
or extremities. What is the most likely diagnosis? mucous membranes → circulate in the bloodstream → primary
A. Herpes labialis lesion is damage to the endothelial lining of small blood vessels
B. Hand Foot and Mouth Disease with ischemic damage to the liver, kidneys, meninges & muscles
C. Herpangina • Incubation period: 7-12 days
D. Streptococcal Pharyngitis
In relation to the above case, what is the most likely causative agent?
CLINICAL MANIFESTATIONS
A. Human Herpes Virus
B. Coxsackie A Virus • Anicteric leptospirosis
C. GABHS o Initial or septicemic phase is abrupt, with fever, chills, severe
D. Adenovirus headache, malaise, nausea, vomiting, severe muscular pain &
In relation to the above case, which among the ff. is the best treatment tenderness (lower extremities)
for this condition? o Conjunctival suffusion with photophobia & orbital pain
A. Acyclovir without chemosis & purulent exudate
B. Supportive o Hepatosplenomegaly, generalized lymphadenopathy
C. Amoxicillin-Clavulanate
o Truncal red maculopapular rash
D. Ribavirin
o Second or immune phase follows a brief asymptomatic
interlude with recurrence of fever (biphasic)
What viral illness has these features?
o Aseptic meningitis
• Due to RNA virus of the family paramyxoviridae and the genus
• Icteric (Weil syndrome)
Rubulavirus; only 1 serotype is known
o Severe form affecting <10% of children
• Painful enlargement of the parotid glands: usual presenting sign
o Hemorrhage & cardiovascular collapse
• Spread from human reservoir by direct contact, airborne o RUQ pain, hepatomegaly, increased liver enzymes,
droplets, fomites contaminated by saliva hyperbilirubinemia
o Azotemia → oliguria → anuria
MUMPS
FACTS TO REMEMBER ABOUT MUMPS COURSE OF LEPTOSPIROSIS:
• Incubation period: 16-18 days • initial or septicemic phase lasting 2-7 days, during which
• Period of communicability: 1-2 days before the onset of parotid leptospires can be isolated from the blood, cerebrospinal fluid
swelling until 5 days after the onset of swelling (CSF), and other tissues.
• About 30-40% are subclinical • This phase may be followed by a brief period of well-being
• Initially unilateral but becomes bilateral in 70% of cases before onset of a second symptomatic immune or
leptospiruric phase – associated with the appearance of
DISTINCT FEATURES OF MUMPS circulating IgM antibody, disappearance of organisms from the
• Pain & swelling in one or both parotid glands (peaks in 1-3 days) blood and CSF, and appearance of signs and symptoms
• Swollen glands push the ear lobe upward & outward & the associated with localization of leptospires in the tissues.
angle of mandible is no longer visible
• Swelling subsides within 3-7 days DIAGNOSIS
• Presumptive dx: symptomatic children with titers of 1:100 or
ALARMING MANIFESTATIONS OF MUMPS!!! greater in 2 or more specimens & asymptomatic children with
• Edema of the homolateral pharynx & soft palate evidence of exposure & seroconversion (fourfold rise in Ab
accompanies the parotid swelling & displaces the tonsil medially titer in specimens obtained 2 or more weeks apart)
• Acute edema of the larynx • ELISA and dot-ELISA
• Edema over the manubrium & upper chest wall may be due • Microscopic slide-agglutination test using killed Ags – most
to lymphatic obstruction useful screening test
• Clinical evidence and serologic test establishes the diagnosis
COMPLICATIONS OF MUMPS
1. Meningoencephalitis TREATMENT
o Most frequent; • Initiation of treatment before the 7th day probably shortens the
o enters the CNS via the choroid plexus and infect the choroidal clinical course & decrease the severity of infection
epithelium; occurs 5 days after the parotitis; M >F • Penicillin or Tetracycline
2. Orchitis & epididymitis • Parenteral Penicillin G 6-8 M U/m2/day in 6 divided doses for 1
o Occurs in 30-40% of males after puberty; extremely rare in week
prepubescent boys
o Begins within days after onset of parotitis ✔ GUIDE QUESTIONS
o Infertility is rare even with bilateral orchitis A 7 year-old female was brought in for consult due to fever for 3 days,
3. Oophoritis headache, arthralgia, and myalgia. Patient was noted to have decreased
o Pelvic pain & tenderness noted in 7% of post pubertal females; appetite, abdominal pain, and vomiting. Vital signs: HR 140 RR 30 BP
90/60. Pertinent physical examination finding shows flushed skin,
fertility is unimpaired
with full pulses, and cold distal extremities. You will advise the
4. Pancreatitis following EXCEPT?
5. Arthritis A. This patient can be managed as outpatient
o occurs within 3 weeks of onset of parotitis; mild and self- B. Cornerstone in management is fluid hydration and CBC
limiting monitoring
C. WBC and platelet counts are expected to be decreased
✔ GUIDE QUESTION D. Hemoconcentration is expected
A 12 year-old male was brought in for fever, abdominal pain, and In relation to the above case, which test will you request for this patient
decreased urine output. Physical examination revealed jaundice, to document infection?
hepatomegaly, calf tenderness, and fever. BUN and creatinine levels are A. Dengue IgM
elevated. Upon further questioning there was a history of wading in B. Dengue IgG
flood waters 2 weeks ago. What is your drug of choice for this patient? C. Dengue NS1
A. Penicillin D. All of the above
B. Vancomycin
C. Meropenem
D. Azithromycin
This is a case of Leptospirosis.
Dr. Punongbayan

TOPNOTCH MEDICAL BOARD PREP PEDIATRICS MAIN DIGITAL HANDOUT BY DR. PUNONGBAYAN AND DR. DE VERA Page 90 of 105
For inquiries visit www.topnotchboardprep.com.ph or https://www.facebook.com/topnotchmedicalboardprep/
This handout is only valid for the March 2021 PLE batch. This will be rendered obsolete for the next batch since we update our handouts regularly.
TOPNOTCH MEDICAL BOARD PREP PEDIATRICS MAIN DIGITAL HANDOUT BY DR. PUNONGBAYAN AND DR. DE VERA
For inquiries visit www.topnotchboardprep.com.ph or https://www.facebook.com/topnotchmedicalboardprep/
This handout is only valid for the March 2021 PLE batch. This will be rendered obsolete for the next batch since we update our handouts regularly.

DENGUE FEVER 1. Severe plasma leakage


§ Shock (DSS)
• 4 serotypes from the family Flaviridae
§ Fluid accumulation with respiratory distress
• Vector: Aedes aegypti 2. Severe bleeding
• Incubation period: 4-6 days § As evaluated by clinician
• Virus replication → viremia → Ag-Ab complex → complement 3. Severe organ involvement
cascade & activation of Hageman factor → ávascular § liver: AST or ALT >1,000
permeability → shock § CNS: impaired consciousness
§ Heart and other organs
DENGUE FEVER PPS/DOH 2012 CLASSIFICATION
• Dengue without warning signs: DIAGNOSTIC TESTS FOR DENGUE FEVER:
o Live in/travel to endemic areas Confirmed dengue:
o Fever and 2 of the ff criteria: • PCR
• Nausea, vomiting • Retroorbital pain • Viral culture isolation
• Rash • Anorexia • Detection of the virus
• Headache • Diarrhea • Detection of the viral nucleic acid
• Body malaise • Flushed skin • Detection of antigens and antibodies
• Myalgia • (+) tourniquet test * Or a combination of these techniques
• Arthralgia Dengue IgM and IgG:
AND • Primary infection: anti-dengue serum IgG is detectable in low
§ Lab test, at least CBC (leukopenia with or without titers at the end of 1st wk of illness → increases slowly after →
thrombocytopenia); IgG detectable after several months
§ And/or Dengue NS1 antigen test or dengue IgM antibody test • Secondary infection: IgG detected even in the acute phase &
(optional tests) persists from 10 months to life
• Dengue with warning signs: • IgM is detected in 80% of patients by day 5 and 99% by day 10
1. Abdominal pain or tenderness • IgM peaks in 2 weeks after onset of symptoms and decline in 2-
2. Persistent vomiting 3 months
3. Clinical fluid accumulation Timing of diagnostic tests of dengue fever
4. Mucosal bleeding • IgM detected in 80% of patients by day 5 and 99% by day 10
5. Lethargy, restlessness • IgM peaks in 2 wks after onset of symptoms & decline in 2-3
6. Liver enlargement >2 cms months
7. Decreased or no urine within 6 hours o Primary infection: anti-dengue serum IgG is detectable in low
8. LAB: increase in Hct with decrease in platelet titers at the end of 1st wk of illness → increases slowly after
• Severe dengue: → IgG detectable after several months
o Lives in or travels to an endemic area with fever of 2-7 days o Secondary infection: IgG detected even in the acute phase &
and any of the above clinical manifestations for dengue with or persists from 10 months to life
without warning signs, plus ANY of the following:

WHO Dengue Guidelines for Diagnosis, Treatment, and Prevention Control 2009
2017 PPS-PIDSP Clinical Practice Guideline on Dengue Fever

APPROACH TO GENITAL ULCERS


SIGNS / SYMPTOMS SYPHILIS (PRIMARY) CHANCROID HERPES SIMPLEX TYPE 2
• Well-demarcated indurated borders • Unindurated borders and a • Vesicles rupture to form shallow
Ulcers
and a clean base (chancre) purulent base ulcers
Pain • painless • painful • painful
Number of lesions • Usually single • multiple • Usually multiple
Inguinal • Usually mild and minimally tender • Painful nodes >50%; inguinal • Constitutional symptoms and
lymphadenopathy bubo nodes
• Early: chancre plus reactive • Exclusion of other causes of
nontreponemal test and no history of ulcers in the presence of:
syphilis • Ttypical ulcers and
• 4-fold increase in quantitative lymphadenopathy • Typical lesions
Clinical suspicion nontreponemal test with history of • Typical Gram stain • Positive HSV-2 type-specific
syphilis • History of contact with high-risk serology test
• (+) treponemal EIA with reactive individual or living in an
nontreponemal test and no prior endemic area
history of tx
• Identification of T. pallidum from a • Detection of Haemophilus • Detection of HSV by culture or
Definitive
chancre or lymph node aspirate on ducreyi by culture PCR from ulcer scraping or
diagnosis
dark field microscopy aspiration of vesicle fluid
TOPNOTCH MEDICAL BOARD PREP PEDIATRICS MAIN DIGITAL HANDOUT BY DR. PUNONGBAYAN AND DR. DE VERA Page 91 of 105
For inquiries visit www.topnotchboardprep.com.ph or https://www.facebook.com/topnotchmedicalboardprep/
This handout is only valid for the March 2021 PLE batch. This will be rendered obsolete for the next batch since we update our handouts regularly.
TOPNOTCH MEDICAL BOARD PREP PEDIATRICS MAIN DIGITAL HANDOUT BY DR. PUNONGBAYAN AND DR. DE VERA
For inquiries visit www.topnotchboardprep.com.ph or https://www.facebook.com/topnotchmedicalboardprep/
This handout is only valid for the March 2021 PLE batch. This will be rendered obsolete for the next batch since we update our handouts regularly.
DIAGNOSIS OF SYPHILIS: TREATMENT FOR GONORRHEA
• 2-step screening process with a nontreponemal test followed • all patients with presumed or proven gonorrhea should be
by a confirmatory treponemal test evaluated for concurrent syphilis, HIV, and C. trachomatis
• Nontreponemal tests: VDRL and RPR – sensitive tests that infection (15-25% coinfection in males and 35-50% in females)
detect Abs against phospholipid Ags that cross react with • 45 kgs or less: Ceftriaxone 50 mg/kg IV or IM once for
cardiolipin-lecithin-cholesterol Ags of damaged host cells pharyngeal, anorectal, urogenital infection, or conjunctivitis;
o Quantitative results useful in screening and in monitoring DGI or septic arthritis for 7 days; meningitis 50 mg/kg IV or IM
therapy every 12-24 hrs 10-14 days
o Usually become nonreactive within 1 year of adequate tx for • more than 45 kgs: Ceftriaxone 250 mg IM plus Azithromycin
primary form and within 2 yrs for secondary form 1 g PO once for pharyngeal, anorectal, and urogenital infection;
• Treponemal tests confirm diagnosis and measure specific T. Ceftriaxone 1 g IM plus Azithromycin 1 g PO once for
pallidum Abs conjunctivitis
o T. pallidum hemagglutination assay (TPHA), particle o for DGI and septic arthritis: Ceftriaxone 1g IV or IM OD for 7
agglutination test, fluorescent treponemal Ab absorption test days plus Azithromycin 1 g PO once
o Do not correlate with disease activity; become (+) soon after o meningitis: Ceftriaxone 1-2 g IV every 12-24 hrs 10-14 days
initial infection plus Azithromycin 1 g PO once

TREATMENT OF SYPHILIS: TREATMENT FOR CHANCROID


STATUS TREATMENT • Most are resistant to penicillin and ampicillin due to plasmid-
Aqueous crystalline penicillin G, mediated B-lactamase production
200,000-300,000 U/kg/ day, IV, • Easy to treat if recognized early
Congenital syphilis
administered as 50,000 U/kg, every • Azithromycin 1 g PO single dose OR Ceftriaxone 250 mg IM
4-6 hrs for 10 days single dose
Primary, secondary, • Alternative: erythromycin 500 mg PO tid 7 days and
and early latent Penicillin G benzathine, 50,000 ciprofloxacin 500 mg bid 3 days for 18 years old and older
syphilis (ELI - U/kg, IM, up to the adult dose of 2.4 • Symptoms usually resolve in 3-7 days
acquired within the million U in a single dose • Evaluate for other STIs as well as the sexual contacts
preceding year)
Penicillin G benzathine, 50,000 TREATMENT OF HERPES SIMPLEX INFECTION
U/kg, IM, up to the adult dose of 2.4 • First episode genital herpes reduces severity and duration of
Late latent syphilis
million U, administered as 3 single illness
(beyond 1 year
doses at 1-wk interval (total o Acyclovir 400 mg tid PO for 7-10 days (for smaller children
duration)
150,000 U/kg, up to the adult dose 10-20 mg/kg/dose 4x a day not to exceed the adult dose),
of 7.2 million U) Valacyclovir 1 g bid PO for 7-10 days, or Famciclovir 250 mg
Aqueous crystalline penicillin G, tid PO for 7-10 days
200,000-300,000 U/kg/ day, IV, o 3 options for recurrent infections:
Neurosyphilis
every 4-6 hrs for 10-14 days, in § Option 1: no therapy
doses not to exceed the adult dose § Option 2: episodic tx with acyclovir 800 mg tid PO for 2 days
or valacyclovir 500 mg bid PO for 3 days
OTHER SEXUALLY TRANSMITTED INFECTIONS: § Option 3: long-term suppressive tx with acyclovir 400 mg bid
CAUSATIVE PO; prevents outbreaks and improves quality of life
MANIFESTATIONS
AGENT § CNS infection: IV acyclovir 10 mg/kg every 8 hours given as
• infection of the GUT mucous membranes 1 hr infusion for 14-21 days
and of the mucosa of the rectum,
oropharynx, and conjunctiva DRUG Route of administration
Bioavailability
• postpubertal males: urethritis and Oral (the only one with
Poorest; requires
epididymitis Acyclovir suspension form); more frequent
• postpubertal females: endometritis, topical; IV form dosing
salpingitis, and peritonitis (PID) Very good; once
Neisseria Valacyclovir oral
• asymptomatic: about 80% of mature or bid dosing
gonorrhea females are asymptomatic; 10% in males Very good; once
(non-motile, Famciclovir oral
• localized: incubation period of 2-5 days in or bid dosing
aerobic, gram-
negative men and 5-10 days in women • early initiation of therapy results in maximal therapeutic
diplococcus) o urethritis: purulent discharge and benefits
dysuria without urgency or frequency • all 3 drugs have exceptional safety profiles
o vulvovaginitis: purulent discharge with • doses should be modified in patients with renal impairment
swollen, tender, and excoriated vulva; • foscarnet and cidofovir used for acyclovir-resistant mutants
dysuria
• disseminated: asymmetric arthralgia,
petechiae, tenosynovitis, suppurative
arthritis

OTHER SEXUALLY TRANSMITTED INFECTIONS:


CAUSATIVE AGENT MANIFESTATIONS TREATMENT
• major cause of epididymitis and a cause of • males and nonpregnant females: Azithromycin (1 g PO
23-55% of nongonococcal urethritis single dose) and Doxycycline (100 mg PO bid for 7
• sexually abused children usually days)
asymptomatic • pregnant: Azithromycin 1 g PO OR Amoxicillin 500 mg
Chlamydia trachomatis • perinatally acquired rectal and vaginal tid 7 days
(gram negative obligate infections can persist for 3 years or longer • treat sex partners of patients with nongonococcal
intracellular pathogen): 2 • urethritis, epididymitis, cervicitis, urethritis if they had contact for the last 60 days
biovars LGV and trachoma salpingitis, proctitis, PID
• autoinoculation from the genital tract to eye
• less acute symptoms with mucoid discharge
• diagnosis by NAAT like PCR (cervical,
vaginal, or urethral swab)

TOPNOTCH MEDICAL BOARD PREP PEDIATRICS MAIN DIGITAL HANDOUT BY DR. PUNONGBAYAN AND DR. DE VERA Page 92 of 105
For inquiries visit www.topnotchboardprep.com.ph or https://www.facebook.com/topnotchmedicalboardprep/
This handout is only valid for the March 2021 PLE batch. This will be rendered obsolete for the next batch since we update our handouts regularly.
TOPNOTCH MEDICAL BOARD PREP PEDIATRICS MAIN DIGITAL HANDOUT BY DR. PUNONGBAYAN AND DR. DE VERA
For inquiries visit www.topnotchboardprep.com.ph or https://www.facebook.com/topnotchmedicalboardprep/
This handout is only valid for the March 2021 PLE batch. This will be rendered obsolete for the next batch since we update our handouts regularly.
• moist, fleshy, papillomatous lesions on the • genital warts are benign and usually remit
perianal mucosa, labia, vaginal introitus, on • weekly application of 25% podophyllin in tincture of
the shaft, corona, and glans of the penis benzoin left on the warts for 4-6 hours
Human papilloma virus
• squamous intraepithelial lesions (cervical) • topical podofilox (0.5% solution or gel bid for 3 days
(genital; types 6 and 11) –
• occur after inoculation during birth, result of followed by 4 days of no tx repeated for up to 4 cycles)
condyloma acuminata
sexual abuse, or from incidental spread from or imiquimod 5% cream applied at bedtime 3x/week
cutaneous warts up to 16 wks
• laser surgery, surgical excision

ACQUIRED IMMUNODEFICIENCY SYNDROME DIAGNOSIS


• Human immunodeficiency virus from the Retroviridae family • Any child >18 months: demonstration of IgG Ab to HIV by a
• Transmission of HIV occurs via sexual contact, parenteral repeatedly reactive enzyme immunoassay & confirmatory test
exposure to blood, or vertical transmission from mother to child (Western immunoblot or IF assay) establishes the dx of HIV
infection
• Vertical transmission can occur before (intrauterine), during
(intrapartum), or after (through breastfeeding) delivery • Some viral detection assays: HIV DNA or RNA by PCR, HIV
culture, HIV p24 Ag, immune complex-dissociated p24 Ag
PATHOGENESIS OF AIDS
TIMING OF DIAGNOSTIC TESTS
• HIV in circulation → intense viremia → widespread seeding of
• For infants born to HIV-infected mothers: viral diagnostic
virus to various organs → HIV selectively binds to cells
testing should be done within the 1st 2 days of life, at 1-2 months
expressing CD4+ molecules & cells of monocyte-macrophage
old & at 4-6 months old
lineage → CD4+ cells migrate to lymph nodes
• CD4+ & CD8+ lymphocyte counts done at 1 & 3 months old &
• CD8+ cells/suppressor cells play an important role in containing
repeated every 3 months starting at 6 months of age
the infection
TREATMENT
3 DISTINCT PATTERNS OF AIDS IN CHILDREN
1. HIV-infected newborns 15-25% - rapid course; median • Basis of initiating antiretroviral tx for pediatric HIV-infected
survival of 6-9 months if untreated patients are based on:
2. Perinatally infected newborns 60-80% - much slower 1. the magnitude of viral replication
progression with median survival time of 6 yrs. 2. CD4+ count & percentage
3. Long-term survivors <5% of perinatally infected children – 3. clinical condition
minimal or no progression of disease with normal CD4+ • Antiretrovirals able to inhibit the HIV reverse transcriptase
counts & very low viral loads for >8 yrs or protease enzymes
• Reverse transcriptase inhibitors: nucleoside or non-
CLINICAL MANIFESTATIONS nucleoside RTIs
• Generalized lymphadenopathy • NRTIs: Didanosine, Abacavir, Stavudine, Lamivudine,
Zidovudine
• Hepatosplenomegaly
• NNRTIs: Delaviridine, Efavirenz, Nevirapine
• Failure to thrive
• Protease inhibitors are potent agents that prevent packaging
• Chronic or recurrent diarrhea
of infectious virions before they leave the infected cells;
• Interstitial pneumonia
Indinavir, Amprenavir, Nelfinavir, Ritonavir, Saquinavir
• Oral thrush
• HIV-infected children with symptoms (clinical category A, B, C)
• Recurrent bacterial infections
or with evidence of immune dysfunction (immune category 2 or
• Lymphocytic interstitial pneumonitis (LIP) 3) should be treated with antiretrovirals regardless of age or
• Early onset of progressive neurologic deterioration viral load.
• In general, the best single prognostic indicator is the plasma
HIV CLASSIFICATION SYSTEM TO CATEGORIZE viral load.
THE STAGE OF PEDIATRIC DISEASE:
CLINICAL STATUS SUPPLEMENT: SPIDERMAN DROPLET
• Category A (mild symptoms)
* Separate 3 feet between patients
o At least 2 mild symptoms:
* Wear regular mask / close curtain or varifold
1. Lymphadenopathy
• S – scarlet fever
2. Parotitis
• P - Parvovirus B19 / Pneumonia / Pertussis
3. Hepatosplenomegaly
• I – Influenza
4. Dermatitis
5. persistent or recurrent sinusitis or otitis media • D – Diphtheria
• Category B (moderate symptoms) • E – Epiglottitis
o Any of the ff: • R – Rubella
1. oropharyngeal thrush persisting for >2 months • M – Mumps / Meningitis / Mycoplasma infection
2. recurrent or chronic diarrhea • AN- Adenovirus
3. persistent fever for >1 month SUPPLEMENT: MRS. WEE CONTACT
4. hepatitis *private room
5. recurrent herpetic gingivostomatitis *wear procedural gown and gloves
6. pneumonitis • M – Multi-drug resistant organisms (MRSA)
7. disseminated varicella with visceral involvement • R – Respiratory infections (RSV)
8. cardiomegaly
• S- Skin infections (scabies)
• Category C (severe symptoms):
• W – Wound infection (open wounds)
o 2 serious bacterial infections:
• E – Eye infection (conjunctivitis)
1. Sepsis
2. Meningitis SUPPLEMENT: METV AIRBORNE
3. pneumonia in a 2-yr period *negative pressure room
4. lower RT candidiasis *wear N95 mask
5. Cryptococcosis • M – Measles
6. Encephalopathy • E – emerging and re-emerging infections (SARS, MERS CoV,
7. Malignancies Ebola)
8. disseminated mycobacterial infection • T – Tuberculosis
9. Pneumocystis carinii pneumonia • V – Varicella
10. severe weight loss

TOPNOTCH MEDICAL BOARD PREP PEDIATRICS MAIN DIGITAL HANDOUT BY DR. PUNONGBAYAN AND DR. DE VERA Page 93 of 105
For inquiries visit www.topnotchboardprep.com.ph or https://www.facebook.com/topnotchmedicalboardprep/
This handout is only valid for the March 2021 PLE batch. This will be rendered obsolete for the next batch since we update our handouts regularly.
TOPNOTCH MEDICAL BOARD PREP PEDIATRICS MAIN DIGITAL HANDOUT BY DR. PUNONGBAYAN AND DR. DE VERA
For inquiries visit www.topnotchboardprep.com.ph or https://www.facebook.com/topnotchmedicalboardprep/
This handout is only valid for the March 2021 PLE batch. This will be rendered obsolete for the next batch since we update our handouts regularly.
PLEASE READ MORE ON:
• Polio
• EBV/Infectious Mononucleosis
• Cytomegalovirus
• Influenza

ENDOCRINOLOGY

https://learn.pediatrics.ubc.ca/body-systems/endocrine-system/normal-sexual-maturity-rating/

STAGING OF MALE GENITALIA DEVELOPMENT

✔ GUIDE QUESTION
A 15-year-old female was being examined in the clinic as part of her
wellness checkup. Her breast areolae and papilla were found to form
secondary mounds and pubic hair was coarse and curly. What is the
most likely SMR stage of this patient?
A. 1
B. 2
C. 3
D. 4
E. 5
Primary mound of breast – SMR 2 – thelarche
Secondary mound of breast – SMR 4
Dr. Punongbayan

SMR LANDMARKS
STAGE BOY GIRL
1 • Prepubertal • Prepubertal https://learn.pediatrics.ubc.ca/body-systems/endocrine-system/normal-sexual-maturity-rating/

2 • Testes enlarge • Thelarche


STAGING FOR PUBIC HAIR
• Penis enlarge, pubic • Enlarged breasts and
3
hair begins areola
• Curly hair and papilla
4 • Curly pubic hair
form secondary mound
• Adult testes, pubic
• Adult triangle. Medial
5 hair to medial
surface of thigh
surface of thigh

STAGING OF BREAST DEVELOPMENT

https://learn.pediatrics.ubc.ca/body-systems/endocrine-system/normal-sexual-maturity-rating/

SEXUAL DEVELOPMENT
• GIRLS: thelarche (10-11 years) → pubarche → growth spurt →
menarche (average age 12.5 years old, 9-15 years)
• BOYS: gonadarche (growth of testes ≥4mL volume or 2.5cm
longest diameter, 11-12yo) → pubarche → adrenarche → growth
spurt
Menstruation start 2-2.5 years after thelarche
Girls’ peak height velocity (PHV) coincides with SMR 2-3 while boys’ PHV
coincides with SMR 3-4
Dr. Punongbayan

VARIANTS:
• GIRLS: Precocious puberty → 8 years
Delayed puberty → 13 years
• BOYS: Precocious puberty → 9 years
Delayed puberty → 14 years

TOPNOTCH MEDICAL BOARD PREP PEDIATRICS MAIN DIGITAL HANDOUT BY DR. PUNONGBAYAN AND DR. DE VERA Page 94 of 105
For inquiries visit www.topnotchboardprep.com.ph or https://www.facebook.com/topnotchmedicalboardprep/
This handout is only valid for the March 2021 PLE batch. This will be rendered obsolete for the next batch since we update our handouts regularly.
TOPNOTCH MEDICAL BOARD PREP PEDIATRICS MAIN DIGITAL HANDOUT BY DR. PUNONGBAYAN AND DR. DE VERA
For inquiries visit www.topnotchboardprep.com.ph or https://www.facebook.com/topnotchmedicalboardprep/
This handout is only valid for the March 2021 PLE batch. This will be rendered obsolete for the next batch since we update our handouts regularly.
INCOMPLETE (PARTIAL) PRECOCIOUS PUBERTY A 14 year-old female comes to the clinic for a complaint of short stature.
• Premature thelarche – breast development in the first 2 yrs of Her mother expressed concern because the father was 5’10 in height
life, regress after 2 yrs & rarely progressive and the mother is 5’4 in height while the daughter was only 5 feet tall.
The daughter has not had her menarche as well. The mother had her
• Premature adrenarche – pubic hair; early maturational event of
menarche at age 15. Physical examination revealed essentially normal
adrenal androgen production findings, however, a bone age X ray shows a bone age of 12 years old.
• Premature menarche What is the most likely diagnosis?
A. Familial short stature
WHEN TO SUSPECT AN ABNORMALITY B. Constitutional growth delay
• If 13: No thelarche C. Pathological growth delay
D. XO (Turner syndrome)
• If 14: no menses, hematocolpos, no vagina
• If 16: with menses but other secondary sex characteristics not Delayed growth but final adult height is achieved; normal birth
present (gonadal failure or pituitary) and growth history
Dr. Punongbayan
• Precocious puberty
GROWTH DELAY
CHILD WITH SUSPECTED SEXUAL PRECOCITY:
• Short stature is defined as 2 or more standard deviations below
URGENT REASONS FOR
INITIAL WORK UP: the mean height for children of that gender and chronological
REFERRAL:
age. The height growth pattern over a period of time is more
• Bone age (X ray) • CNS abnormality like
important than a single measurement at one point in time.
• Serum LH, FSH headache, seizure, visual
• Testosterone and estradiol changes
FAMILIAL SHORT STATURE
serum levels • Very high 17-OH
• Normal birth history and gestational weight
• DHEAS progesterone level
• Family history of short stature
• TSH • Rapidly progressing
• Short Stature With
• Free T4 puberty
o Normal linear growth velocity for age
• Pelvic ultrasound • Very young age
o Bone age consistent with chronological age
• Initial presentation is o Normal age for onset of puberty
vaginal bleeding o Absence of physical or psychological disease
• Bone age >2 SD above CA
• Signs of rapid virilization CONSTITUTIONAL SHORT STATURE
• Delayed growth in one parent but average final stature
CHILD WITH SUSPECTED DELAYED PUBERTY: • Normal birth history and growth for first few months
SUGGESTIVE • Chronologic age is greater than bone age (CA>BA)
WHEN TO
HISTORY & PE WORK UP
REFER
FINDINGS
GIRLS: No breast • Bone age • If with CNS
enlargement by age • Serum LH and symptoms
13 years FSH • If with features
BOYS: No testicular • Testosterone and of Turner
enlargement by age estradiol levels syndrome
14 years (equal or • TSH, FT4
>4 mL or 2.5 cm in • prolactin
diameter)
BOTH: delay in
adrenarche; look
for CNS symptoms,
family history

✔ GUIDE QUESTIONS
A 15-year-old female presents to the clinic with short stature and is
lagging behind in height amongst her peers. She was born via normal
vaginal delivery with normal weight and height. On further history
taking, she does not have menses yet but with some evidence of breast
development. Which among the ff. is the best next step in the
management?
A. Xray of the wrist
B. FSH
C. GnRH
D. LH
The case mentioned an adolescent who had features that
appeared to be consistent with a syndrome, most probably
Turner syndrome because of the following features: teenaged
girl, short stature, with primary amenorrhea, hypothyroidism,
and liver function abnormalities.
The most common clinical feature of Turner syndrome (45, X) is
short stature hence, bone aging through X ray of the wrist can
be prioritized. Prompt recognition and diagnosis is important
to provide optimal benefits from growth hormone treatment. MIDPARENTAL HEIGHT:
Primary hypogonadism is one of the common features and an (G/*ℎ&. ' 1 ℎ&'(ℎ* − 13) + 03*ℎ&. ' 1 ℎ&'(ℎ*
elevated serum FSH can be expected because of the ovarian J#KLM =
failure. 2
G/*ℎ&. ' 1 ℎ&'(ℎ* + (TUVW<>' ? W<X:WV + 13)
Dr. Punongbayan
!QRS =
Y
• Parents height in cm
Z/.(&* ℎ&'(ℎ* = 0'[\/.&-*/] ℎ&'(ℎ* ± 2M_ (10B0)

FAILURE TO THRIVE
• Persistent weight less than 5th percentile for age
• Growth curve: crossing 2 major percentile lines on growth chart

TOPNOTCH MEDICAL BOARD PREP PEDIATRICS MAIN DIGITAL HANDOUT BY DR. PUNONGBAYAN AND DR. DE VERA Page 95 of 105
For inquiries visit www.topnotchboardprep.com.ph or https://www.facebook.com/topnotchmedicalboardprep/
This handout is only valid for the March 2021 PLE batch. This will be rendered obsolete for the next batch since we update our handouts regularly.
TOPNOTCH MEDICAL BOARD PREP PEDIATRICS MAIN DIGITAL HANDOUT BY DR. PUNONGBAYAN AND DR. DE VERA
For inquiries visit www.topnotchboardprep.com.ph or https://www.facebook.com/topnotchmedicalboardprep/
This handout is only valid for the March 2021 PLE batch. This will be rendered obsolete for the next batch since we update our handouts regularly.
✔ GUIDE QUESTIONS o the principal regulator of tonicity
A 10 year-old male with pneumonia was found to have serum sodium o has both antidiuretic & vascular pressor activity
of 130, patient had moist mucous membranes, BP 90/60, HR 88, RR20, o synthesized in the paraventricular & supraoptic nuclei of the
T > 36.5°C, serum osmolality was 265 mOsm/kg, urinary osmolality hypothalamus
200 mOsm/kg, urinary sodium 40mmol/liter, low serum uric acid.
What is the most likely diagnosis in this case?
A. Diabetes Insipidus Urine osmolality Urine osmolality
B. Cerebral salt wasting (mOsm/kg) (mOsm/kg)
Diagnosis
C. SIADH After fluid
After desmopressin
D. Psychogenic polydipsia deprivation
Normal urine osmolality 500-800 mOsm/kg water Central DI >750 <300
Random urine osmolality 300-900 mOsm/kg water Nephrogenic
<300 <300
Serum osmolalty <280 and serum sodium <135 DI
Random urine Na 40-220 mEqs/L/day
Dr. Punongbayan

APPROACH TO POLYURIA AND POLYDIPSIA:


SIADH • Serum osmolality
DIANOSTIC CRITERIA OF SIADH • Serum Na, K, Ca, glucose
• Absence of: renal, adrenal, or thyroid insufficiency, CHF, • BUN, serum creatinine
nephrotic • Urinalysis (SG and sugar)
• syndrome, or cirrhosis, diuretic ingestion, dehydration • Urine osmolality
• Urine Osmolality > 100 (usually>plasma)
• Serum Osmolalty < 280 and serum sodium < 135 DIAGNOSIS OF DI
• Urine Na >25 • DI is established if the serum osmolality is >300 mOsm/kg and
• Hyponatremia and hypoosmolality resulting from inappropriate the urine osmolality is <300 mOsm/kg
continued secretion or action of vasopressin despite normal or • DI is unlikely if the serum osmolality is <270 mOsm/kg or the
increased plasma volume → impaired water excretion urine osmolality is >600 mOsm/kg.
• Hyponatremia results from an excess of water
NEPHROGENIC DIABETES INSIPIDUS
SUPPLEMENT: QUICK SHEET • If the serum osmolality value is 290 mOsm/kg or higher with a
SIADH Bartter-Schwartz criteria: simultaneous urine osmolality value of <290 mOsm/kg, a formal
• Continued renal excretion of Na water deprivation test is not necessary.
• Hyponatremia with hypo-osmolality • inability to respond to ADH (and thus the presence of NDI)
• Urine less than maximally dilute should then be confirmed by the administration of vasopressin
• Absence of clinical evidence of volume depletion (10-20 μg intranasally) followed by serial urine and serum
• Absence of other causes of hyponatremia osmolality measurements hourly for 4 hr.
• Correction of hyponatremia by fluid restriction
Type of DI: Etiology Management
Congenital, trauma, Fluid therapy; long-
Serum Serum Urine osmolality
Disorder tumors, autoimmune, acting vasopressin
Na osmolality / urine Na
Central infection, drugs analog dDAVP
SIADH low low high
(ethanol, phenytoin,
Diabetes
high high low etc.)
insipidus
Congenital (more Treat underlying
severe), disorder; thiazides
✔ GUIDE QUESTIONS
hypercalcemia, (decrease urine
A 13 year-old male who had a recent head trauma 5 months ago from
hypokalemia, renal flow to DCT, induce
an accident suddenly presented with polyuria and polydipsia. Serum Nephrogenic
osmolality is 285 mOsm/kg and the urine osmolality is 100mOsm/kg, disease (PCKD, CRF), formation of
urine specific gravity 1.005. What is the next best step for this patient? drugs (lithium, functional
A. Water Deprivation Test amphotericin, receptors)
B. Urine Sodium Excretion rifampicin, etc.)
C. Serum Vasopressin Levels
D. 24 Hour Urine Protein CLNICAL PARAMETERS
Water Deprivation – fluid restriction for 12 hours results in CLINICAL
>850 mOsm/kg SIADH CSW CENTRAL DI
PARAMETER
Normal Person = concentrate urine
DI = still have diluted urine
Serum Na ↓ ↓ ↑
Normal urine osmolality is 300-900 mOsm/kg Urine Output N or ↓ ↑ ↑
Dr. Punongbayan

If after water deprivation test the urine osmolality is 200mOsm/kg and Urine Na ↑ ↑↑↑ ↓
with administration of vasopressin, urine osmolality is still Intravascular
N or ↑ ↓ ↓
200mOsm/kg, what is the most likely diagnosis? Volume status
A. SIADH Serum Uric acid ↓ N or ↑ ↑
B. Central Diabetes Insipidus
C. Nephrogenic Diabetes Insipidus Vasopressin
↑ ↓ ↓
D. Psychogenic Polydipsia Level

DIABETES INSIPIDUS ✔ GUIDE QUESTION


A 17 yr-old female complains of anxiety attacks with palpitations and
• What are the cardinal features of diabetes insipidus? sweating which started about 6 months ago but are increasing in
o Polyuria and polydipsia frequency. PE reveals marked proptosis and fine tremors were noted
• What are the two types of DI? as well. She reveals that her feelings of restlessness have begun to
o 1. Vasopressin Deficiency (Central DI) adversely affect her studies. Among the ff, which is expected?
o 2. Vasopressin Insensitivity (Nephrogenic DI) A. Increase TSH, Decreased T4 and T3
• How will you distinguish between these conditions? B. Decreased TSH, Decreased T4 and T3
C. Decreased TSH, Increased T3 and T4
o Water deprivation test and ADH administration
D. Increased TSH, Increased T3 and T4
FACTS ABOUT DIABETES INSIPIDUS
• What is the hormone involved in DI and what is its role?
o Vasopressin – secreted from the posterior pituitary

TOPNOTCH MEDICAL BOARD PREP PEDIATRICS MAIN DIGITAL HANDOUT BY DR. PUNONGBAYAN AND DR. DE VERA Page 96 of 105
For inquiries visit www.topnotchboardprep.com.ph or https://www.facebook.com/topnotchmedicalboardprep/
This handout is only valid for the March 2021 PLE batch. This will be rendered obsolete for the next batch since we update our handouts regularly.
TOPNOTCH MEDICAL BOARD PREP PEDIATRICS MAIN DIGITAL HANDOUT BY DR. PUNONGBAYAN AND DR. DE VERA
For inquiries visit www.topnotchboardprep.com.ph or https://www.facebook.com/topnotchmedicalboardprep/
This handout is only valid for the March 2021 PLE batch. This will be rendered obsolete for the next batch since we update our handouts regularly.

HYPERTHYROIDISM (GRAVES DISEASE) • HLA-DR4, HLA-DR5 associated with an increased risk of goiter
& thyroiditis
• What is the most likely diagnosis?
• Thyroid antiperoxidase antibodies (TPOAbs) seen in the sera
o Hyperthyroidism (Graves’ disease)
of 90% of children; inhibit enzyme activity & stimulate natural
• Who are affected?
killer cell cytotoxicity
o F > M; rarely occurs before adolescence
• What is the pathophysiology of this condition? MANIFESTATIONS, DIAGNOSTICS, AND TREATMENT
o autoimmune induced (autoantibodies stimulate • Thyroid is diffusely enlarged, firm & nontender in most patients
hypersecretion of thyroid hormones) • Most of the affected children are euthyroid & asymptomatic
• What is thyroid storm? • Thyroid scan: 50% reveal irregular & patchy distribution of the
o acute-life-threatening surge of thyroid hormone in the blood radioisotope
usually precipitated by surgery, trauma, infection, acute iodine • Thyroid UTZ: scattered hypoechogenicity
load or long-standing hyperthyroidism • Tx: if with evidence of hypothyroidism, give sodium-L-thyroxine
o Tachycardia (>140/min), heart failure, fever, agitation, (50-150 ug/day)
delirium, psychosis, stupor, and/or coma • May be self-limited; periodic re-evaluation
o Treated with Methimazole • (+)nodules – biopsy (identify CA)

DIAGNOSTIC TESTS
ADRENAL GLANDS
• ↑ T4, T3; ↓ TSH
• (+) TRSAb; its disappearance predicts remission of the disease • Adrenal gland: 2 endocrine systems: medullary gland &
• Radionuclide study: palpable nodule and increased T3 cortical system
• Adrenal cortex:
TREATMENT FOR GRAVES DISEASE: 1. Zona glomerulosa – aldosterone
• Can remit and recur 2. Zona fasciculata – cortisol & androgens
3. Zona reticularis – androgens
• Definitive Tx: radioactive iodine ablation or thyroidectomy
• PTU and Methimazole inhibit iodine organification and coupling
in the thyroid CONGENITAL ADRENAL HYPERPLASIA
• Clinical response is apparent in 2-3 weeks • AR disorders of adrenal steroidogenesis leading to a deficiency
• Adequate control evident in 1-3 months of cortisol → increased secretion of corticotropin →
adrenocortical hyperplasia & overproduction of intermediary
DRUG PROPERTIES ADVERSE EFFECT metabolites
• Inhibits • Deficiency of 21-hydroxylase accounts for 90% of affected
extrathyroidal patients
conversion of T4-T3 • Lack of 21-hydroxylase (about 90% of cases) → excess
PTU substrates which are shunted towards synthesis of sex
• Protein-bound
• Less able to cross hormones → decreased cortisol → loss of feedback inhibition →
placenta increased ACTH → further stimulation of the conversion of
• 10x more potent than cholesterol into sex hormone precursors (17-OH progesterone
• Transient and androstenedione)
PTU
Methimazole • leukopenia • low serum Na & Cl; increased K; low serum cortisol
• Longer serum half-
• (asymptomatic) • Inc. plasma renin; serum aldosterone
life
• 21-hydroxylase deficiency: increased serum 17-OHP
• Pelvic ultrasound to visualize presence of uterus in female
HYPOTHYROIDISM pseudohermaphroditism
• Due to deficient production of hormone or a defect in • Glucocorticoids inhibit excessive production of androgens &
hormonal receptor activity prevents progressive virilization
• congenital or acquired • Tx: Hydrocortisone 10-20 mg/m2/day orally in 2-3 divided
• Etiologies of congenital hypothyroidism: doses
o Thyroid dysgenesis
o Thyrotropin receptor-blocking antibody
o Defective synthesis of thyroxine
o Defect of iodide transport
o Thyroid peroxidase defects of organification & coupling
• Birthweight & length are normal
• Prolonged physiologic jaundice
• Feeding difficulties, sluggishness, lack of interest, somnolence
• Respiratory difficulties due to large tongue
• Frequent constipation
• Large abdomen; usually with umbilical hernia
• Hypothermic; cold & mottled skin; dry & scaly
• Edema of the genitals & extremities
• Retardation of physical & mental development progresses

DIAGNOSTICS & THERAPEUTICS:


• Low serum T4 & T3; elevated serum TSH
• X-ray: epiphyseal dysgenesis, deformity of T12 or L1-2; large TYPE MANIFESTATIONS
fontanels & wide sutures; large sella turcica; cardiomegaly
• Normal at birth but signs of sexual precocity
• Tx: Sodium-L-thyroxine 10-15 ug/kg/day
appear w/in 1st 6 mos of life
• Monitor hormone levels & maintain Non-salt
• Usually normal mental development
losing
• Small testes & enlarged penis
THYROIDITIS • Females: enlarged clitoris or labial fusion
• Lymphocytic / Hashimoto / Autoimmune • Symptoms begin shortly after birth
• Most common cause of thyroid disease in children & • progressive weight loss, prominent vomiting,
adolescents dehydration
Salt-losing
• Etiology: organ-specific autoimmune disease is characterized • Females: more pronounced virilization of
histologically by lymphocytic infiltration between the thyroid external genitals
follicles • Males: genitals appear normal
TOPNOTCH MEDICAL BOARD PREP PEDIATRICS MAIN DIGITAL HANDOUT BY DR. PUNONGBAYAN AND DR. DE VERA Page 97 of 105
For inquiries visit www.topnotchboardprep.com.ph or https://www.facebook.com/topnotchmedicalboardprep/
This handout is only valid for the March 2021 PLE batch. This will be rendered obsolete for the next batch since we update our handouts regularly.
TOPNOTCH MEDICAL BOARD PREP PEDIATRICS MAIN DIGITAL HANDOUT BY DR. PUNONGBAYAN AND DR. DE VERA
For inquiries visit www.topnotchboardprep.com.ph or https://www.facebook.com/topnotchmedicalboardprep/
This handout is only valid for the March 2021 PLE batch. This will be rendered obsolete for the next batch since we update our handouts regularly.
CAH DUE TO 21-OH DEFICIENCY

Pediatrics Endocrinology, Mechanisms, Manifestations and Management, Ora H. Pescovitz, Erica A. Eugster, 2004 by Lippincott Williams & Wilkins
MANIFESTATIONS INDICATING ADRENAL CRISIS CUSHING SYNDROME
• Vomiting, diarrhea
• Characteristic pattern of obesity with associated
• Abdominal pain
hypertension which is the result of abnormally high blood
• Unexplained fever
levels of cortisol resulting from hyperfunction of the adrenal
• Weight loss cortex
• anorexia • Etiology: functioning adrenocortical tumor (infants);
• Hypoglycemia pituitary adenomas; hyperplasia of adrenals
• Micropenis • Rounded face, prominent cheeks, moon facies, buffalo
• Bilateral cryptorchidism hump, generalized obesity, abnormal masculinization,
• Hyperpigmentation of the skin impaired growth, hypertension, increased susceptibility to
infection
When to consider AI crisis: • Older children: purplish striae on hips, abdomen & thighs,
• Neonates with atypical genitalia delayed puberty, emotional lability, weakness, headache
• Electrolyte abnormalities • Serum cortisol levels are normally elevated at 8 am & decrease
• Lethargy to <50% by 8pm → diurnal rhythm is lost
• Dehydration • Urinary excretion of free cortisol & 17-hydroxycorticosteroids
• Hypotension or shock are increased
• Failure to thrive • Unilateral adrenalectomy for benign cortical adenomas
• Critically ill patients with septic shock unresponsive to fluid • Bilateral tumors: subtotal adrenalectomy
resuscitation and inotropes • Trans-sphenoidal pituitary microsurgery for children

LAB FINDINGS IN CAH ✔ GUIDE QUESTION


• low serum Na & Cl; inc. K; low serum cortisol An adolescent female was referred to you due to episodes of headaches
• Increased plasma renin; serum aldosterone; that comes and goes. Upon physical examination you noted the patient
• Decreased serum glucose to be tachycardic, hypertensive with excessive sweating. Imaging was
requested by the referring physician which showed an abdominal mass.
• 21-hydroxylase deficiency: increased serum 17-OHP
The following is expected for this condition EXCEPT?
• Pelvic ultrasound to visualize presence of uterus in female A. Management is primarily surgical
pseudohermaphroditism B. Urine VMA is expected to be elevated
C. This is a catecholamine secreting tumor from chromaffin cells
MANAGEMENT OF ADRENAL CRISIS D. Most common site of the tumor is the adrenal cortex
• Shock: NSS bolus 20 mL/kg
o Repeat up to a total of 60 mL/kg within 1 hour PHEOCHROMOCYTOMA
• Hypoglycemia: initial bolus of D10W 0.5-1 g/kg IV
• Catecholamine-secreting tumor arising from the chromaffin
• Hyperkalemia: ECG (tall, peaked T waves with shortened QT
cells
interval, progressive lengthening of PR interval and QRS
• Most common site of origin is the adrenal medulla
duration)
• Tumors may develop anywhere along the abdominal
o Insulin- glucose solution
sympathetic chain, likely to be located near the aorta at the level
• Stress glucocorticoids
of the IMA or at its bifurcation.
o Given as IV bolus over several minutes started ASAP
• Periadrenal area, urinary bladder, ureteral walls, thoracic cavity,
• Hydrocortisone succinate sodium
cervical region
o 50 mg/m2 for children
o Age-based dosing may be used
o 0-3 years old: 25 mg IV
o 3-12 years old: 50 mg IV
o Older than 12 years: 100 mg IV

• Glucocorticoids inhibit excessive production of androgens


& prevents progressive virilization
• Hydrocortisone 10-20 mg/m2/day orally in 2-3 divided doses
• Monitor growth & hormonal levels
• Flurohydrocortisone (0.05-0.3 mg daily) & NaCl 1-3 gms given
to normalize plasma renin activity
• Hydrocortisone continued indefinitely in all patients with
classic forms of CAH https://doctorlib.info/physiology/physiology-2/86.html

TOPNOTCH MEDICAL BOARD PREP PEDIATRICS MAIN DIGITAL HANDOUT BY DR. PUNONGBAYAN AND DR. DE VERA Page 98 of 105
For inquiries visit www.topnotchboardprep.com.ph or https://www.facebook.com/topnotchmedicalboardprep/
This handout is only valid for the March 2021 PLE batch. This will be rendered obsolete for the next batch since we update our handouts regularly.
TOPNOTCH MEDICAL BOARD PREP PEDIATRICS MAIN DIGITAL HANDOUT BY DR. PUNONGBAYAN AND DR. DE VERA
For inquiries visit www.topnotchboardprep.com.ph or https://www.facebook.com/topnotchmedicalboardprep/
This handout is only valid for the March 2021 PLE batch. This will be rendered obsolete for the next batch since we update our handouts regularly.
FACTS RE: PHEOCHROMOCYTOMA LAB FINDINGS
• Occur in children 6-14 yrs old • Dx: demonstration of elevated blood or urinary levels of
• Tumors found more often on the right side, about 1-10 cm in catecholamines & their metabolites
diameter • Predominant catecholamine in children is norepinephrine
• Bilateral in >20% affected children derived from the adrenal gland & adrenergic nerve endings
• Inherited as AD trait • Urinary excretion of vanillylmandelic acid (major metabolite
• May be associated with other syndromes such as of epi-, norepi- & metanephrine) is increased
neurofibromatosis, part of MEN syndromes, tuberous sclerosis, • Ultrasound, CT scan, MRI: tumors
Sturge-Weber syndrome, ataxia-telangiectasia
MANAGEMENT
• Surgical removal of tumors
• Preoperative `- & a-adrenergic blockers
• Thorough transabdominal exploration of all the usual sites
• Accurate indicators of malignancy – presence of metastatic
disease or local invasiveness that precluded complete resection
or both

✔ GUIDE QUESTION
A 7-year-old girl was brought to her doctor because she was noted to
be less active and wetting her bed, which she had stopped doing 3 years
prior. There was significant weight loss noted too. Her mother noticed
her to be eating and drinking more than usual. Few days ago, patient
developed diarrhea and vomiting. Patient had decreased oral intake
and was then rushed to the ER. The patient was drowsy and weak
looking. HR 140 RR 30 O2 sat 98% BP 100/70. CBG 450. What should
be the initial management for this patient?
A. Hydration
B. Insulin Administration
C. Biguanides
D. Intubation

DIABETES MELLITUS
• Syndrome of metabolic disease characterized by
hyperglycemia due to deficiency of insulin secretion or
insulin action or both resulting in abnormal metabolism of
CHO, CHON & fat
• Most common endocrine-metabolic disorder of childhood &
adolescence
• Girls=boys; peaks at 5-7 yrs old & puberty
• Basic cause of the initial clinical findings is the sharply
diminished insulin secretion
• Mechanisms that lead to failure of pancreatic ß-cell function
point to the likelihood of autoimmune destruction of pancreatic
islets in predisposed individuals
• About 80-90% of newly diagnosed patients have islet-cell
antibodies (ICAs) directed at cell surface.
• Tissue damage of pancreatic a cells is mediated by T
lymphocytes → produce cytokines → induce destruction of islet
cells
• Type I – absolute insulin deficiency
• Type II – insulin resistance and increased insulin levels
CATECHOLAMINE BIOSYNTHESIS
• Both types can result in retinopathy, nephropathy, and
neuropathy.

EFFECTS OF INSULIN DEFICIENCY:


• With progressive deficiency → excessive glucose production &
impairment of its utilization → hyperglycemia with glucosuria →
resultant osmotic diuresis produces polyuria, urinary losses of
electrolytes, dehydration, polydipsia → hypersecretion of
epinephrine, glucagon, cortisol, & GH which amplifies &
perpetuates metabolic derangements & accelerates metabolic
decompensation
• Combination of insulin deficiency & ↑ counterregulatory
hormones is responsible for accelerated lipolysis & impaired
lipid synthesis → ↑ plasma total lipids, cholesterol, TG, FFA →
ketone body formation which exceeds the capacity for
peripheral utilization & renal excretion → metabolic acidosis &
rapid deep breathing

CLINICAL MANIFESTATIONS:
• Classic: polyuria, polydipsia, polyphagia, weight loss (often in a
CLINICAL MANIFESTATIONS less than a month)
• S/Sx result from excessive secretion of epinephrine & • Clue to polyuria: onset of enuresis in a previously toilet-trained
norepinephrine child
• May be symptom-free in between attacks of hypertension • Pyogenic skin infections & monilial vaginitis in adolescent
• Headache, palpitations, abdominal pain, dizziness, pallor, females
vomiting, sweating, convulsions
• Severe: precordial pain radiates into the arms, pulmonary
edema, cardio- & hepatomegaly
TOPNOTCH MEDICAL BOARD PREP PEDIATRICS MAIN DIGITAL HANDOUT BY DR. PUNONGBAYAN AND DR. DE VERA Page 99 of 105
For inquiries visit www.topnotchboardprep.com.ph or https://www.facebook.com/topnotchmedicalboardprep/
This handout is only valid for the March 2021 PLE batch. This will be rendered obsolete for the next batch since we update our handouts regularly.
TOPNOTCH MEDICAL BOARD PREP PEDIATRICS MAIN DIGITAL HANDOUT BY DR. PUNONGBAYAN AND DR. DE VERA
For inquiries visit www.topnotchboardprep.com.ph or https://www.facebook.com/topnotchmedicalboardprep/
This handout is only valid for the March 2021 PLE batch. This will be rendered obsolete for the next batch since we update our handouts regularly.
DIAGNOSTIC CRITERIA: PLEASE READ MORE ON:
• Symptoms of diabetes plus a • Hypoparathyroidism
1. random plasma glucose >200 mg/dL, or; • Hyperparathyroidism
2. fasting plasma glucose >126 mg/dL • Klinefelter syndrome
• Polyuria, polydipsia, & unexplained weight loss with glucosuria • Noonan syndrome
& ketonuria • Turner syndrome
DIABETIC KETOACIDOSIS (ISPAD) criteria:
• glucose >200 IMMUNOLOGY/ALLERGOLOGY
• Venous pH < 7.3 or bicarbonate <15mmol/L GENETIC BASIS OF ATOPY & COMMON MANIFESTATIONS
• ketonemia (B-hydroxybutyrate 3 or more mmol/L) or • Atopic diseases have a strong familial predisposition with about
moderate or large ketonuria 60% heritability
• Precipitating factors: trauma, infections, vomiting, psychologic • The 5q23-35 region comprises several genes implicated in
disturbances allergic disease pathogenesis
• Clinical signs: tachycardia, tachypnea, dehydration, deep, • The risk of allergic disease in a child approaches 50% when one
sighing (Kussmaul) respiration with acetone-like breath, parent is allergic and 66% when both parents are allergic
nausea, vomiting, abdominal pain, confusion, drowsiness • Allergic salute, nasal crease,
allergic cluck, allergic shiners
CLASSIFICATION OF DKA: with Dennie-Morgan folds
NORMAL MILD MODERATE SEVERE
Nasal crease – a horizontal
HC03 20-28 16-20 10-15 <10
wrinkle over the nasal bridge
7.35-
pH 7.25-7.35 7.15-7.25 <7.15
7.45
Kussmaul or
Kussmaul
depressed
Oriented; respirations,
No respirations;
Clinical alert but oriented but
change depressed
fatigued sleepy;
sensorium to
arousable
coma

CLASSIFICATION OF DKA (ISPAD):


MILD MODERATE SEVERE
HCO3 <15 <10 <5

pH <7.3 <7.2 <7.1


✔ GUIDE QUESTION
A 9-year-old male was brought to you for chronic sneezing and cough
MANAGEMENT more prominent in the morning. Father and mother were known
• Ketoacidosis: expansion of intravascular volume, correction of asthmatics. Upon physical examination, patient had watery nasal
discharge with boggy turbinates. (+) allergic shiners. For the past 2
deficits in fluid, electrolyte & acid-base status; initiation of months, patient has been having almost daily attacks with difficulty
insulin therapy sleeping. What is the classification of this patient’s disease?
• Initial hydrating fluid is isotonic saline (hypotonic relative to the A. Mild Persistent
patient’s serum osmolality) B. Moderate-Severe Persistent
• Administration of glucose (5% solution in 0.2 N saline) is C. Mild Intermittent
initiated when blood glucose approaches 300 mg/dL to limit the D. Moderate-Severe Intermittent
decline of serum osmolality & reduce cerebral edema Based on ARIA guidelines, given the chronic history of sneezing
• Give potassium added after the initial 20 ml/kg if UO is adequate. and coughing for 2 months with difficulty of sleeping, this
• Bicarbonate only if pH <7.2 given slowly patient falls under the clinical classification moderate-severe
persistent AR. JUST REMEMBER THE NUMBER 4!.....The cutoff
• Anticipate cerebral edema – limit rate of fluid to 4 L/m2/day or
duration to distinguish intermittent from persistent is less than
less 4 weeks for the former while more than 4 weeks for the latter.
• Insulin 0.1 U/kg of regular insulin followed by constant The cutoff frequency of symptoms to differentiate intermittent
infusion of 0.1 U/kg/hr from persistent is less than 4 days for the former while more
than 4 days for the latter.
MONITORING Dr. Punongbayan

• Reliable index of long-term glycemic control – measure


glycosylated Hgb ALLERGIC RHINITIS
• Glycohemoglobin (HbA1c) represents the fraction of Hgb to • The classification of Allergic Rhinitis takes into account the
which glucose has been nonenzymatically attached in the following criteria:
bloodstream o Based on allergen causing symptoms (etiological
• reflects the average blood glucose concentration of the classification)
preceding 2-3 months o Duration of symptoms (clinical classification)
• Glycated Hgb predicts risk of progression of diabetic o Severity of clinical symptoms (depending on impact of disease
complications on quality of life measures)
o Disease pathophysiology (limited use)
HbA1c
• The more consistently lower the level, the better the metabolic I. BASED ON ALLERGEN CAUSING SYMPTOMS
control, the more likely it is that microvascular complications
(ETIOLOGICAL CLASSIFICATION)
will be less severe, delayed in appearance, or avoided.
• In known diabetics: • Seasonal – occurs only during specific periods of the year; e.g.
o 7% indicates good diabetic control pollination, mold sporulation
o 10% indicates fair diabetic control • Perennial – triggered by allergens formed in the patient’s
o 13-20% indicates poor diabetic control environment at concentrations sufficient to induce symptoms all
year round; dust mites, pet fur, cockroach
• Episodic – exposure to a specific airborne allergen on a
sporadic and short-term basis

TOPNOTCH MEDICAL BOARD PREP PEDIATRICS MAIN DIGITAL HANDOUT BY DR. PUNONGBAYAN AND DR. DE VERA Page 100 of 105
For inquiries visit www.topnotchboardprep.com.ph or https://www.facebook.com/topnotchmedicalboardprep/
This handout is only valid for the March 2021 PLE batch. This will be rendered obsolete for the next batch since we update our handouts regularly.
TOPNOTCH MEDICAL BOARD PREP PEDIATRICS MAIN DIGITAL HANDOUT BY DR. PUNONGBAYAN AND DR. DE VERA
For inquiries visit www.topnotchboardprep.com.ph or https://www.facebook.com/topnotchmedicalboardprep/
This handout is only valid for the March 2021 PLE batch. This will be rendered obsolete for the next batch since we update our handouts regularly.

II. DURATION OF SYMPTOMS (CLINICAL • Diagnosis generally established by 6 yrs old


CLASSIFICATION) • 20% are seasonal, 40% perennial, 40% mixed
• Exposure of an atopic host to an allergen leads to specific IgE
• Intermittent – less than 4 days/week or less than a month/year
production
• Persistent – equal or more than 4 days/week or equal or more
• Early-phase reaction: degranulation of mast cells & release of
than a month/year
inflammatory mediators
• Late-phase reaction: arises 4-8 hours following allergen
III. SEVERITY OF CLINICAL SYMPTOMS exposure
(DEPENDING ON IMPACT OF DISEASE ON QUALITY OF • Sneezing, rhinorrhea, nasal obstruction, itching of the nose,
LIFE MEASURES: DAILY ACTIVITIES, SCHOOL OR WORK palate, pharynx & ears, itching, redness & tearing of the eyes
ATTENDANCE, SLEEP, NEED OF THERAPY) • Pale mucosa, clear mucoid nasal discharge, allergic salute, rabbit
• Mild nose
• Moderate / Severe – either 1, 2, or 3 out of 4 elements or all 4 • Avoidance of exposure to suspected allergens and irritants
elements affected
✔ GUIDE QUESTION
IV. DISEASE PATHOPHYSIOLOGY (LIMITED USE) In line with the previous case, what is the drug of choice for this patient?
A. Intranasal steroids
• IgE-mediated – about >90% B. Oral antihistamine
• Non-IgE-mediated – IgG, T lymphocytes, eosinophils C. Topical antihistamine
D. Oral steroids
The 9-year-old boy in the aforementioned case had moderate-
severe persistent allergic rhinitis for which he would need
intranasal steroids to control his symptoms.
Dr. Punongbayan

ALLERGIC RHINITIS
• Immunotherapy for those who cannot avoid inhalant allergens
and drug therapy
• Oral antihistamines; inhaled budesonide, fluticasone,
mometasone
ARIA at-a-Glance Pocket Reference 2007

ARIA at-a-Glance Pocket Reference 2007


✔ GUIDE QUESTION A. Selective IgA Deficiency
A 3 year-old male patient comes to the clinic for right ear pain without B. Bruton Agammaglobulinemia
discharge and fever that started 1 day ago which prompted consult. C. Common Variable Immunodeficiency
Patient had cough and colds 3 days ago. The mother says that the D. Severe Combined Immunodeficiency
patient has been having repeated ear infections and colds since infancy. This is a case of selective IgA deficiency based on the cluster of
He was also hospitalized twice for pneumonia at ages 9 months and 18 repeated symptoms involving the respiratory and urinary tract
months with concomitant urinary tract infection. He is also known to systems with a less than normal IgA level. This deficiency has
have allergies to certain milk products. Further work up revealed IgA characteristically recurrent R-U-G infections!!! (respiratory –
<10mg/dL, IgG, IgM and IgE within normal limits, what is the most urogenital – gastrointestinal)
likely condition? Dr. Punongbayan

TOPNOTCH MEDICAL BOARD PREP PEDIATRICS MAIN DIGITAL HANDOUT BY DR. PUNONGBAYAN AND DR. DE VERA Page 101 of 105
For inquiries visit www.topnotchboardprep.com.ph or https://www.facebook.com/topnotchmedicalboardprep/
This handout is only valid for the March 2021 PLE batch. This will be rendered obsolete for the next batch since we update our handouts regularly.
TOPNOTCH MEDICAL BOARD PREP PEDIATRICS MAIN DIGITAL HANDOUT BY DR. PUNONGBAYAN AND DR. DE VERA
For inquiries visit www.topnotchboardprep.com.ph or https://www.facebook.com/topnotchmedicalboardprep/
This handout is only valid for the March 2021 PLE batch. This will be rendered obsolete for the next batch since we update our handouts regularly.

IMMUNODEFICIENCY ✔ GUIDE QUESTION


A 6 month-old male infant was seen for persistent diarrhea for the past
• Diagnosis: suspected in every patient, regardless of age, who has 2 weeks. The patient has visible wasting, underweight, and appears
recurrent, persistent, and unusual or opportunistic infections weak and irritable. The mother said that the patient has been having
• Persistent infection in the context of proper antibiotic therapy is repeated bouts of diarrhea since 4 months of age. He also was admitted
significant! for pneumonia at 3 months of age. The infant on examination has oral
• Infections in more than one organ system and severe thrush, no visible tonsillar tissue, no palpable lymphadenopathies.
Previous X ray at 3 months of age show no thymic shadow. Further
• Consanguinity of parents, early deaths of relatives, unusual
workup was done which revealed depressed levels of IgG, IgA, IgM and
reaction to vaccines IgE below that of appropriately aged matched controls. What is the
most likely diagnosis?
A. DiGeorge Syndrome
B. Bruton Agammaglobulinemia
C. Common Variable Immunodeficiency
D. Severe Combined Immunodeficiency
This is a case of SCID – severe combined immunodeficiency. The
main features that were mentioned in the case are the
following: presence of a fungal infection (oral thrush),
much earlier onset of symptoms, in this case, at 4 months
of age, and the low levels of immunoglobulins.
The other choice was CVID – presence of low, but not absent, T
cell function; recurrent or chronic pulmonary infections, FTT,
oral or cutaneous candidiasis, chronic diarrhea, recurrent skin
infections, UTIs, gram (-) bacterial sepsis, severe varicella in
infancy; neutrophils and eosinophilia
Dr. Punongbayan

COMBINED IMMUNODEFICIENCY DISORDER:


SEVERE CID (SCID)
• Profound system immunodeficiency
• Selective defect in cell-mediated immunity with susceptibility to
chronic candida infection of the skin and & mucous membranes
w/ or w/o endocrinopathy
• 50% are due to deficiency of adenosine deaminase due to
deletion or mutation of the encoding gene → deoxyadenosine &
TIPS IN APPROACHING A CASE OF PROBABLE IMMUNODEFICIENCY: deoxy-ATP accumulate inside lymphocytes → cell death
1. Consider the age of onset of symptoms • Infants usually succumb within the first 2 years of life
o for less than 6 months of age in onset: consider a severe form of
immunodeficiency
• Failure to thrive, chronic otitis media, chronic diarrhea,
o for more than 6 months of age in onset: consider a B-cell oropharyngeal candidiasis, recurrent pneumonia
immunodeficiency as it is known to exhibit its manifestations • Varicella, Candida, Pneumocystis carinii, CMV
beyond 6 months old when protective maternal IgG levels • Profound lymphopenia, unable to reject transplant, short-
transferred to a newborn start to go down limbed dwarfism
2. Identify the systems involved and whether the infections are • Bone marrow transplantation
recurrent, persistent, or unusual
3. Ask for the response to usual / conventional treatment
Dr. Punongbayan T-CELL IMMUNODEFICIENCY DISORDER:
✔ GUIDE QUESTION CONGENITAL THYMIC APLASIA
A 1 year-old male patient comes to the clinic due to cough, fast • (DiGeorge syndrome)
breathing, and fever. It started 1 day ago which prompted consult. • Increased susceptibility to viral, fungal & protozoal infections
Patient had cough and colds 3 days ago. There was history of repeated • Aberrant thymus or its absence occurs in the 12th week of
ear infections at 8 months and 11 months. He was also admitted for
gestation
pneumonia at 8 months of age. At present, HR 120, RR 55, T 38.5°C, (+)
crackles bilateral lung fields with decreased breath sounds over the • Symptoms at birth: abnormal facies, hypothyroidism, congenital
right lung, (-) cervical lymphadenopathies, (-) tonsillar tissue. After heart disease
antibiotic treatment, further workup was done which revealed • Low lymphocyte count
depressed levels of IgG, IgA, IgM, and IgE below that of appropriately • Fetal thymus transplant as early as possible results in
aged matched controls. What is the most likely diagnosis in this case? reconstitution of T-cell immunity
A. Selective IgA Deficiency
B. Bruton Agammaglobulinemia
C. Common Variable Immunodeficiency
D. Severe Combined Immunodeficiency

B-CELL IMMUNODEFICIENCY DISORDER:


X-LINKED AGAMMAGLOBULINEMIA
• Bruton disease
• A block in the development of pre-B cell to B-cell due to a defect
in heavy chain gene rearrangement
• Healthy infants until 6 months old → recurrent ear infections,
bronchitis or pneumococcal pneumonia, and/or dermatitis ✔ GUIDE QUESTION
• Low or absent Igs; few or absent B-cells A 10 month-old male patient is brought to the clinic by his mother
• Total Ig < 250 mg/dL because of a rash on his face. This started 1 week ago and he was seen
• Gamma globulin IM once a month at interval of 2-3 weeks by his mother to be scratching or touching the facial area. She also
• Maintenance of trough level > 400 mg/dL noticed rashes in the area over the upper extremities. There was no use
of new soaps/shampoos, no substance was applied to the skin; mother
Remember B-B-B!!! is a known asthmatic. On close examination, there is an eczematous
(Bruton disease, B-cell deficiency, bacterial infections are recurrent) rash over the face particularly the cheeks and the extensor surfaces of
Dr. Punongbayan
the upper extremities. What is the most likely diagnosis?
A. Irritant Contact Dermatitis
B. Allergic Contact Dermatitis
O
D. Atopic Dermatitis
E. Infantile SLE
C. Photosensitive Dermatitis

TOPNOTCH MEDICAL BOARD PREP PEDIATRICS MAIN DIGITAL HANDOUT BY DR. PUNONGBAYAN AND DR. DE VERA Page 102 of 105
For inquiries visit www.topnotchboardprep.com.ph or https://www.facebook.com/topnotchmedicalboardprep/
This handout is only valid for the March 2021 PLE batch. This will be rendered obsolete for the next batch since we update our handouts regularly.
TOPNOTCH MEDICAL BOARD PREP PEDIATRICS MAIN DIGITAL HANDOUT BY DR. PUNONGBAYAN AND DR. DE VERA
For inquiries visit www.topnotchboardprep.com.ph or https://www.facebook.com/topnotchmedicalboardprep/
This handout is only valid for the March 2021 PLE batch. This will be rendered obsolete for the next batch since we update our handouts regularly.

WISKOTT-ALDRICH SYNDROME
✔ GUIDE QUESTION
• Immunodeficiency with thrombocytopenia and eczema
Which of the following is the gold standard in the diagnosis of food
• X-linked recessive syndrome allergy?
• With normal appearing megakaryocytes but small defective A. Patch Test
platelets B. Skin Prick Test
• Often have prolonged bleeding from the circumcision site or C. RAST (radioallergosorbent test)
bloody diarrhea during infancy D. Double Blind Placebo Controlled Food Challenge
• Treatment: nutrition, routine IVIG, use of killed vaccines, platelet
transfusion, splenectomy TREATMENT OF B CELL DEFECTS
• Treatment of choice: bone marrow or cord blood • Stem cell transplant recommended for XLP and Hyper IgM type 3
transplantation • Judicious use of antibiotics for infections and regular IVIG (or
subcutaneous IG) are the only effective treatment for primary
B cell defects
• Dose of 400 mg/kg per month

PRIMARY DEFECTS OF ANTIBODY PRODUCTION


CHROMOSOME &
DISORDER FUNCTIONAL DEFICIENCY MAIN CLINICAL FEATURES
REGION
• Bacterial infections (resp. tract) and esp.
• No antibody production
XLA or Bruton enteroviruses;
(total Ig <100 mg/dL)
agammaglobulinemia • small to absent tonsils;
• lack of B cells
• no palpable lymph nodes
• Later age of onset of infections;
• Low IgG with normal B • echovirus meningitis is rare;
CVID
cells • normal nodes & tonsils; cancer;
• ↑ risk of granuloma
• Low or absent IgA (<10 • resp., GI (giardiasis),
Selective IgA deficiency • 6p21.3
mg/dL) • urogenital tract infections
• Become symptomatic during the 1st or 2nd
• Failure to produce IgG, IgA, year of life with recurrent pyogenic
• 12p13,
Hyper IgM syndrome IgE antibodies with infections;
• Xq26
normal or ↑ IgM • profoundly neutropenic;
• P. jiroveci pneumonia
• Inadequate immune response to EBV;
• low Ig; • affected males are usually healthy until they
X-linked lympho-proliferative
• Xq25 • lack of anti-EBNA and long- acquire EBV infection;
disease or Duncan disease
lived T cell immunity • fatal infectious mononucleosis, lymphomas,
acquired hypogammaglobulinemia

PRIMARY DEFECTS OF CELLULAR IMMUNITY


CHROMOSOME FUNCTIONAL
DISORDER MAIN CLINICAL FEATURES
& REGION DEFICIENCY
Thymic hypoplasia or DiGeorge • Low number of T cells • Partial or complete
syndrome – hypoplastic or aplastic and impaired T cell • CATCH 22 syndrome
thymus and parathyroid function • susceptible to fungi, viruses, and P. jiroveci
• chronic and severe Candida skin & mucous
• poor response to
membrane infections in the 1st mo or until
Chronic mucocutaneous candidiasis • 21q22.3 Candida antigen;
2nd decade of life;
• autoimmune responses
• APECED
TREATMENT OF T CELL IMMUNODEFICIENCY • For SCID: bone marrow transplant (92% have survived after T-
• Thymic tissue transplant cell-depleted parental marrow is given soon after birth without
• Some have been given nonirradiated unfractionated BM or pretransplant chemotherapy)
peripheral blood transplant from HLA-identical sibling PLEASE READ MORE ON:
• Treatment of choice for fatal combined T- and B-cell defects:
• Organs of immune system
transplantation of MHC-compatible sibling or T-cell depleted
• 4 types of immunopathologic Reactions
parental hematopoietic stem cells
PRIMARY COMBINED ANTIBODY AND CELLULAR IMMUNODEFICIENCIES
CHROMOSOME
DISORDER FUNCTIONAL DEFICIENCY MAIN CLINICAL FEATURES
& REGION
• present w/in the 1st few months of life with
• 5p13,
SCID recurrent or persistent diarrhea, pneumonia,
• 10p13, • Absence of T- and B-cell
(most severe OM, sepsis, skin infections;
• 11p13, functions
immunodeficiency) • persistent infections with opportunistic agents;
• 1q31-q32
• unable to reject foreign tissue
• Immunodeficiency with thrombo-cytopenia and
• T cell deficiency; eczema;
Wiskott-Aldrich
• Xp11.4-11.21 • poor Ab production to • prolonged bleeding from circumcision site or
syndrome
polysaccharides bloody diarrhea during infancy;
• survival beyond the teens is rare
TOPNOTCH MEDICAL BOARD PREP PEDIATRICS MAIN DIGITAL HANDOUT BY DR. PUNONGBAYAN AND DR. DE VERA Page 103 of 105
For inquiries visit www.topnotchboardprep.com.ph or https://www.facebook.com/topnotchmedicalboardprep/
This handout is only valid for the March 2021 PLE batch. This will be rendered obsolete for the next batch since we update our handouts regularly.
TOPNOTCH MEDICAL BOARD PREP PEDIATRICS MAIN DIGITAL HANDOUT BY DR. PUNONGBAYAN AND DR. DE VERA
For inquiries visit www.topnotchboardprep.com.ph or https://www.facebook.com/topnotchmedicalboardprep/
This handout is only valid for the March 2021 PLE batch. This will be rendered obsolete for the next batch since we update our handouts regularly.
• Coarse facies, coronary artery
Hyper IgE syndrome (AD • Recurrent bacterial infections (S. aureus,
aneurysm, mortality in
or sporadic or Job • 17q21.3 pneumococcus, H. influenzae),
adulthood;
syndrome) • Aspergillus, mucocutaneous candidiasis
• pathologic fractures
• Higher incidence of lymphomas;
• Abscesses, HPV, HSV, VZV, MCV, mucocutaneous
Hyper IgE syndrome (AR) • 21q22.3 • skin cancer;
candidiasis; asthma, allergies / anaphylaxis
• mortality in childhood

TREATMENT:
• Treatment of choice for fatal combined T- and B-cell defects: transplantation of MHC-compatible sibling or T-cell depleted parental
hematopoietic stem cells
• For SCID: bone marrow transplant (92% have survived after T-cell-depleted parental marrow is given soon after birth without pre-
transplant chemotherapy)

DISORDERS OF NEUTROPHIL FUNCTION


DISORDER ETIOLOGY IMPAIRED FUNCTION CLINICAL CONSEQUENCE
• Recurrent pyogenic infections;
• neutropenia;
• hepatosplenomegaly;
Degranulation • mild bleeding diathesis;
• Decreased chemotaxis,
abnormality: • AR; • partial oculo-cutaneous albinism
degranulation and bactericidal
• disordered coalescence of • progressive peripheral neuropathy;
activity;
Chediak-Higashi lysosomal granules • light skin and silvery hair;
• impaired NK function
syndrome • photophobia;
• rotary nystagmus;
• Gram +, Gram -, fungi;
• prolonged BT with normal platelet
• Neutrophilia;
• recurrent bacterial infection
associated with a lack of pus
Adhesion abnormality • Impaired neutrophil adhesion and
formation;
platelet activation;
• AR • bleeding tendency; delayed umbilical
Leukocyte adhesion • decreased binding of C3bi to
cord separation with significant
deficiency (1-3) neutrophils
omphalitis;
• slow healing ulcers;
• S. aureus, E. coli, Candida, Aspergillus
• Recurrent pyogenic infections with
catalase-positive microorganisms;
• onset in early infancy;
Microbicidal activity
• X-linked and AR; • S. aureus, Serratia, B. cepacia,
• Failure to activate neutrophil
• due to a defect in the Candida; pneumonia,
Chronic respiratory burst leading to failure
generation of microbicidal • osteomyelitis,
granulomatous to kill catalase-positive microbes
disease
oxygen metabolites • skin infections,
• lymphadenitis;
• hallmark is granuloma formation and
inflammatory process

PLEASE READ MORE ON:


• Organs of immune system
How the Culture of Bullying in the Hospital Hurts Patients
• 4 type of immunopathologic reactions
Essay first published in Health and Lifestyle Magazine
by Topnotch MD Thad Hinunangan, MD

END OF PEDIATRICS I remember the first time I was fighting back tears while examining a
patient as a medical intern. It was early morning when I received a referral for
a motor vehicle accident patient in my Ophthalmology rotation. I did the eye
exam and noted my findings on the chart, since the patient was unable to
LOST ambulate and there was no available stretcher in the crowded Emergency Ro
by David Wagoner om, I used some printed material instead of the Snellen chart.
"Stand still. The trees ahead and bushes beside you I notified the resident physician and endorsed my findings. This particular
Are not lost. Wherever you are is called Here, resident was an intern’s monitor and projected himself as “pro-student” and
And you must treat it as a powerful stranger, having oriented us during the beginning of the rotation, I thought he, of all
Must ask permission to know it and be known. people would be more tolerant.
The forest breathes. Listen. It answers,
I have made this place around you. “Ano ‘to?” (What’s this?) He said as he looked at my chart entry. I explained
If you leave it, you may come back again, saying Here. how I did my physical exam, and since I did my clerkship in another institution,
No two trees are the same to Raven. the order of my eye exam was not the standard one in the current institution.
No two branches are the same to Wren.
If what a tree of a bush does is lost on you, I apologized and explained that I was confused with the order of the eye
You are surely lost. Stand still. The forest know exam, but I thought as long as it was complete, it would not be that of an issue.
Where you are. You must let it find you." I believed that we settled the matter then, but when he learned I used a fine
print instead of the Snellen chart to check the visual acuity, he launched into a
full tirade.
DEAR TOPNOTCH FRIENDS:
Change the System. If you get bullied as a junior doctor, tanggap lang at konting “Saang school ka ba graduate?” (What school did you graduate from?) He
tiis. But do not become the bully when you become a senior doctor. Be part of the asked loudly, while we were in front of the patient and his family. He crushed
generation who will break the tradition of abuse. Stop bullying in the medical out my entries.
profession. Be kind to everyone.
Here’s an article written by a Topnotch MD regarding this issue. Read and reflect. I was aware of the particular culture in certain institutions that medical
Enrico Paolo Banzuela school of origin was a big deal, such that interns are judged based on where they
came from. It would have not been a problem if I came from one of the top
TOPNOTCH MEDICAL BOARD PREP PEDIATRICS MAIN DIGITAL HANDOUT BY DR. PUNONGBAYAN AND DR. DE VERA Page 104 of 105
For inquiries visit www.topnotchboardprep.com.ph or https://www.facebook.com/topnotchmedicalboardprep/
This handout is only valid for the March 2021 PLE batch. This will be rendered obsolete for the next batch since we update our handouts regularly.
TOPNOTCH MEDICAL BOARD PREP PEDIATRICS MAIN DIGITAL HANDOUT BY DR. PUNONGBAYAN AND DR. DE VERA
For inquiries visit www.topnotchboardprep.com.ph or https://www.facebook.com/topnotchmedicalboardprep/
This handout is only valid for the March 2021 PLE batch. This will be rendered obsolete for the next batch since we update our handouts regularly.
schools like University of Santo Tomas or Cebu Institute of Medicine, but I came by policy but such is a rule that some surgeons choose to ignore. Several times,
from a modest medical school Remedios Trinidad Romualdez Medical residents of other specialties would barge in the cutting room while we are
Foundation in Tacloban. I have gotten used to people replying with, “Saan yun?” dissecting to demand that we prioritize their particular case as if we were their
(Where’s that?). And then I had to explain where my Alma Mater was. I thought employees or that other cases deserved any less of our full attention. Newsflash:
it was offensive to me why I needed to explain where I came from, I mean it is we are all Medical Officers- same position, different department.
not my fault I was born in Leyte and I chose to go to school there, in fact I am
proud that I am Waraynon. It was certainly nobody’s business to embarrass me Even the menial task of providing complete information on the Surgical
by looking down on my credentials while in front of the patient and his family. Pathology form has a lot of benefit for the patient, which unfortunately some
The family member looked away, sensing the tension. fail to give. I once received a modified radical mastectomy specimen which had
only one phrase written on the history. I processed the specimen as usual but
“Mas magaling pa sa iyo ang mga third year dito!” (Third year students here when I examined it microscopically, there were only fibrous tissue and very
are better than you!) He said. scant foci of tumor cells. My consultant had me repeat the sections of the breast
thinking that I just missed the lesion but the other possibility was that the
I was trying to hold my composure, and still tried to assist him while he was patient probably had chemotherapy beforehand which could help explain the
examining the patient. My vision was starting to cloud. I have very shallow altered morphology.
tears, I tried not to blink because I was afraid my tears would fall directly to the
patient’s face. I tried contacting the surgeon to no avail. I visited the General Surgery office
with a written letter inquiring about the patient’s history, but they provided no
Before we were done, I curiously checked the findings he scribbled on the response. I went to the ward to try to see the patient and the chart but she was
chart particularly the visual acuity. It turned out my findings which he crushed already discharged. Finally, I personally went to the Medical Records and dug
out were exactly the ones he wrote on the chart. It was one of those instances through the bulk of files to retrieve the chart and found what I was looking for,
when I experienced bullying in the medical setting as a novice. It would not be the line that said: “History of Invasive Ductal Carcinoma, status post 6 cycles of
the last. chemotherapy (2016).” It was all I needed and was able to sign-out the case.
The morphology was altered because she had already undergone treatment
Different forms and had responded well- there was only microscopic tumor foci left and all the
Abuse comes in different forms. It could be verbal, negligence, or unjust margins were clear. Just like that, our final diagnosis hit the mark.
treatment or unreasonably long hours. It is as old as the medical profession
itself. People have gotten so used to it that it sometimes feels like the “norm.” I was disappointed that because of the lack of information given, the
patient had to wait for a longer period before we could sign out the case. Had I
Some of my friends who took their internship in different institutions in not investigated further, the case would have dragged on. Perhaps the surgeon
Manila experienced being “runners” and personal servants of some of their thought it was only their job that mattered, when it is the pathologist who will
seniors. Some interns were assigned to make coffee, some clerks designated as give the final anatomic diagnosis on what the lesion is and ascertain if they had
the “takeout person” or ones assigned to bring the bag/s of their senior while adequately treated the patient.
the senior strutted like a fashion model.
No doubt doctors are intelligent people, but what escapes me is how we
There is a fine line between doing a few favors and being a compete never realize how much of a disservice we are doing to patients by bullying
personal slave. I don’t mind holding the door open or making coffee for juniors or fellow doctors, thinking that one specialty is more superior than
someone now and then, but when the tasks are not training related or nothing another.
remotely geared towards improving one’s medical skills, then we have a
problem. The lack of respect ultimately results in delay of patient diagnosis or
inadequate treatment, something all of us should avoid at all costs. It also goes
As a clerk, a male surgeon once refused to be assisted by me and wanted a against the principle of teamwork of all members of the health care team. Lastly,
“pretty female clerk” (his words) who could do surgery with him all day in the let me echo the words of one of our mentors: “We all stand on the shoulders of
Operating Room. At the end of the day, we felt sorry for our petite female those who came before us.” Let that be a reminder of the responsibility we have
colleague who had to retract all day at three nephrectomies just because she to train the younger generation of doctors and uplift the medical profession.
was a pretty female. The jokes and vague sexual innuendos are another thing.
Hospitals have advocacies on gender sensitivity and sexual harassment, but
somehow the practice is still thriving.

When Tables Turn


In the private institution where I had my clerkship, we clerks did majority
of the work because interns have protected time of five hours for their board
review, and when we get patients in the Emergency Room, we clerks do the
history and primary intervention first before we endorse to our interns.

Since I had my internship in a large public teaching hospital, I never had


the chance to experience being a “senior” because no such distinction existed
between roles of clerks and interns. Only during the time when I became a
resident did I experience being in a position of authority.

I remember that incident in the Emergency Room of my Ophtha rotation,


and I realized every resident physician who work closely with interns and
clerks actually have a choice: they can continue to perpetuate that culture or
they can choose to stop the culture of bullying.

Being in a training institution meant we had an obligation to teach, rather


than berate juniors when they make mistakes which is expected in the learning
curve.

Far-Reaching Consequences
The consequences are not limited to the aspiring doctors who, sadly take
the brunt of the abuse. These would-be doctors experience depression, feelings
of inferiority, and possibly lead to more serious consequences like quitting the
training program and even suicides in severe cases.

Patients also become the unwitting ultimate victims. The quality of care
becomes compromised as clerks, interns, and residents focus more on avoiding
the wrath of their seniors, rather than providing the utmost care. I remember a
certain Emergency Medicine resident who gets severely agitated and rude
when there are many patients for admission, that the interns would spend time
convincing the relatives of patients to just transfer to another hospital. Sadly,
some of these patients had nowhere else to go because of poverty, and in the
end some do not make it at all.

As a Pathology resident now, I was surprised how other specialties also


bully their way in the laboratory. Some choose not to follow the policy of 24
hours’ notice for frozen sections, which require Pathology consultants to read
the slides and thus need to be properly scheduled. Some just operate on their
patient and decide at a moment’s notice, which requires a written explanation
TOPNOTCH MEDICAL BOARD PREP PEDIATRICS MAIN DIGITAL HANDOUT BY DR. PUNONGBAYAN AND DR. DE VERA Page 105 of 105
For inquiries visit www.topnotchboardprep.com.ph or https://www.facebook.com/topnotchmedicalboardprep/
This handout is only valid for the March 2021 PLE batch. This will be rendered obsolete for the next batch since we update our handouts regularly.

You might also like